Nuclear Cardiology: Practical Applications [4 ed.] 9781264257218, 126425721X, 9781264257201, 1264257201

Complete with review questions, detailed answers, and new and emerging techniques—the go-to guide for acing certificatio

108 95 24MB

English Pages [608] Year 2022

Report DMCA / Copyright

DOWNLOAD PDF FILE

Table of contents :
Cover
Title Page
Copyright Page
Dedication
Contents
Contributors
Preface
Section 1. Fundamentals of Nuclear Cardiology
1. Fundamentals of Nuclear Cardiology Physics
2. Radiation Safety and Protection in Nuclear Cardiology
3. Radiopharmaceuticals for Cardiovascular Imaging
4. Cardiac SPECT and PET Instrumentation
5. Quality Control in SPECT, Dedicated PET, and Hybrid CT Imaging
6. Radiation Exposure and Reduction Strategies in Myocardial Perfusion Imaging
7. Physician Certification and Laboratory Accreditation
Section 2. Radionuclide Myocardial Perfusion Imaging
8. Exercise and Pharmacologic Stress Testing
9. SPECT Myocardial Perfusion Imaging Protocols
10. Cardiovascular Positron Emission Tomographic Imaging
11. Myocardial Blood Flow Quantitation in Clinical Practice
12. Ventricular Function
13. Non-Cardiac Findings
14. Interpretation and Reporting of SPECT and PET Myocardial Perfusion Imaging
Section 3. Indications and Applications
15. Clinical Applications of Quantitation and Artificial Intelligence in Nuclear Cardiology
16. Appropriate Use of Nuclear Cardiology Techniques
17. Evaluation of Patients with Suspected Coronary Artery Disease
18. Evaluation of Patients with Known Coronary Artery Disease
19. Risk Stratification with Myocardial Perfusion Imaging
20. Nuclear Cardiovascular Imaging in Special Populations
21. Preoperative Risk Assessment for Noncardiac Surgery
22. Radionuclide Imaging in Heart Failure
Section 4. Beyond Perfusion Imaging
23. Nuclear Cardiology Procedures in the Evaluation of Myocardial Viability
24. Radionuclide Imaging of Cardiac Innervation
25. Imaging Cardiac Amyloidosis
26. 18 F-FDG PET/CT for Imaging Cardiac Sarcoidosis and Inflammation
27. Hybrid Imaging: SPECT–CT and PET–CT
Section 5. Review Questions
Answers and Explanations for Review Questions
Index
Recommend Papers

Nuclear Cardiology: Practical Applications [4 ed.]
 9781264257218, 126425721X, 9781264257201, 1264257201

  • Commentary
  • TRUE PDF
  • 0 0 0
  • Like this paper and download? You can publish your own PDF file online for free in a few minutes! Sign Up
File loading please wait...
Citation preview

Nuclear Cardiology

FM.indd 1

17-02-2022 12:14:09

Notice Medicine is an ever-changing science. As new research and clinical experience broaden our knowledge, changes in treatment and drug therapy are required. The authors and the publisher of this work have checked with sources believed to be reliable in their efforts to provide information that is complete and generally in accord with the standards accepted at the time of publication. However, in view of the possibility of human error or changes in medical sciences, neither the authors nor the publisher nor any other party who has been involved in the preparation or publication of this work warrants that the information contained herein is in every respect accurate or complete, and they disclaim all responsibility for any errors or omissions or for the results obtained from use of the information contained in this work. Readers are encouraged to confirm the information contained herein with other sources. For example, and in particular, readers are advised to check the product information sheet included in the package of each drug they plan to administer to be certain that the information contained in this work is accurate and that changes have not been made in the recommended dose or in the contraindications for administration. This recommendation is of particular importance in connection with new or infrequently used drugs.

FM.indd 2

17-02-2022 12:14:09

Nuclear Cardiology Practical Applications Fourth Edition Gary V. Heller, MD, PhD, MASNC, FACC Gagnon Cardiovascular Institute Morristown Medical Center Morristown, New Jersey

Robert C. Hendel, MD, MACC, MASNC, FAHA, FSCCT Sidney W. and Marilyn S. Lassen Chair in Cardiovascular Medicine Professor of Medicine and Radiology Tulane University School of Medicine New Orleans, Louisiana

New York  Chicago  San Francisco  Athens  London  Madrid  Mexico City Milan  New Delhi  Singapore  Sydney  Toronto

FM.indd 3

17-02-2022 12:14:09

Copyright © 2022 by McGraw Hill LLC. All rights reserved. Except as permitted under the United States Copyright Act of 1976, no part of this publication may be reproduced or distributed in any form or by any means, or stored in a database or retrieval system, without the prior written permission of the publisher. ISBN: 978-1-26-425721-8 MHID: 1-26-425721-X The material in this eBook also appears in the print version of this title: ISBN: 978-1-26-425720-1, MHID: 1-26-425720-1. eBook conversion by codeMantra Version 1.0 All trademarks are trademarks of their respective owners. Rather than put a trademark symbol after every occurrence of a trademarked name, we use names in an editorial fashion only, and to the benefit of the trademark owner, with no intention of infringement of the trademark. Where such designations appear in this book, they have been printed with initial caps. McGraw-Hill Education eBooks are available at special quantity discounts to use as premiums and sales promotions or for use in corporate training programs. To contact a representative, please visit the Contact Us page at www.mhprofessional.com. TERMS OF USE This is a copyrighted work and McGraw-Hill Education and its licensors reserve all rights in and to the work. Use of this work is subject to these terms. Except as permitted under the Copyright Act of 1976 and the right to store and retrieve one copy of the work, you may not decompile, disassemble, reverse engineer, reproduce, modify, create derivative works based upon, transmit, distribute, disseminate, sell, publish or sublicense the work or any part of it without McGraw-Hill Education’s prior consent. You may use the work for your own noncommercial and personal use; any other use of the work is strictly prohibited. Your right to use the work may be terminated if you fail to comply with these terms. THE WORK IS PROVIDED “AS IS.” McGRAW-HILL EDUCATION AND ITS LICENSORS MAKE NO GUARANTEES OR WARRANTIES AS TO THE ACCURACY, ADEQUACY OR COMPLETENESS OF OR RESULTS TO BE OBTAINED FROM USING THE WORK, INCLUDING ANY INFORMATION THAT CAN BE ACCESSED THROUGH THE WORK VIA HYPERLINK OR OTHERWISE, AND EXPRESSLY DISCLAIM ANY WARRANTY, EXPRESS OR IMPLIED, INCLUDING BUT NOT LIMITED TO IMPLIED WARRANTIES OF MERCHANTABILITY OR FITNESS FOR A PARTICULAR PURPOSE. McGraw-Hill Education and its licensors do not warrant or guarantee that the functions contained in the work will meet your requirements or that its operation will be uninterrupted or error free. Neither McGraw-Hill Education nor its licensors shall be liable to you or anyone else for any inaccuracy, error or omission, regardless of cause, in the work or for any damages resulting therefrom. McGraw-Hill Education has no responsibility for the content of any information accessed through the work. Under no circumstances shall McGraw-Hill Education and/or its licensors be liable for any indirect, incidental, special, punitive, consequential or similar damages that result from the use of or inability to use the work, even if any of them has been advised of the possibility of such damages. This limitation of liability shall apply to any claim or cause whatsoever whether such claim or cause arises in contract, tort or otherwise.

This edition is dedicated to the readers who will find this edition useful as this has been the goal of our efforts to complete this revision, especially an era of COVID where everything has been more difficult, but new important knowledge has been emerging and important. I primarily dedicate this to my co-editor, friend and colleague, Bob Hendel, who has had such an important role in all four editions bringing important comments, revisions, and humor to the process. G.V.H. For my colleagues-past, present, and future, who have provided me with inspiration throughout my career. Additionally, this book is also dedicated to those “behind the scenes,” such as our dedicated and passionate technologists, thoughtful administrators, and societal/regulatory personnel who help optimize the value of nuclear cardiology. And of course, to Gary Heller, the instigator and inspirator for Nuclear Cardiology: Practical Applications and kayaker extraordinaire. R.C.H.

FM.indd 5

17-02-2022 12:14:09

This page intentionally left blank

READ_THINK_ITALIAN.indd 2

11/19/21 1:12 PM

CONTENTS

Contributors.. Preface. .

ix

. . . . . . . . . . . . . . . . . . . . . . . . . .

. . . . . . . . . . . . . . . . . . . . . . . . . . . . .

9. SPECT Myocardial Perfusion Imaging Protocols������������������ 141

xiii

Milena J. Henzlova, Cole B. Hirschfeld and Andrew J. Einstein

Section 1. F undamentals of Nuclear Cardiology��������������� 1

10. Cardiovascular Positron Emission Tomographic Imaging���������������� 155 Matthew J. Memmott, Parthiban Arumugam and Gary V. Heller

1. Fundamentals of Nuclear Cardiology Physics�������������������� 3 C. David Cooke, James R. Galt and E. Lindsey Tauxe

11. Myocardial Blood Flow Quantitation in Clinical Practice������������������ 175 Krishna K. Patel, Gary V. Heller and Timothy M. Bateman

2. Radiation Safety and Protection in Nuclear Cardiology������������������� 15 James R. Galt, C. David Cooke and Jason S. Tavel

12. Ventricular Function���������������� 195 Prem Soman and Saurabh Malhotra

3. Radiopharmaceuticals for Cardiovascular Imaging��������������� 29

13. Non-Cardiac Findings���������������� 217

James A. Case and Gary V. Heller

Rupa M. Sanghani and Jamario Skeete

4. Cardiac SPECT and PET Instrumentation�� 49

14. Interpretation and Reporting of SPECT and PET Myocardial Perfusion Imaging��� 233

James A. Case

Robert C. Hendel and Gary V. Heller

5. Quality Control in SPECT, Dedicated PET, and Hybrid CT Imaging������������ 73

Section 3. Indications and Applications������ 271

Sue Miller, Sunil Selvin and Joey Stevens

15. Clinical Applications of Quantitation and Artificial Intelligence in Nuclear Cardiology������������������������ 273

6. Radiation Exposure and Reduction Strategies in Myocardial Perfusion Imaging��������������������������� 97

Robert J.H. Miller and Piotr J. Slomka

Michael C. Desiderio and Gary V. Heller

16. Appropriate Use of Nuclear Cardiology Techniques������������������������ 289

7. Physician Certification and Laboratory Accreditation���������������������� 109

Gursukhman Deep S. Sidhu and Robert C. Hendel

Robert C. Hendel and Gursukhman Deep S. Sidhu

17. Evaluation of Patients with Suspected Coronary Artery Disease�������������� 299

Section 2. R  adionuclide Myocardial Perfusion Imaging�������������� 119

Sanjeev U. Nair and Gary V. Heller

8. Exercise and Pharmacologic Stress Testing���������������������� 121

18. Evaluation of Patients with Known Coronary Artery Disease�������� 323

Seyed Mehdi Khalafi, Archana Ramireddy and Robert C. Hendel

Javier Gomez and Rami Doukky

vii

FM.indd 7

17-02-2022 12:14:09

viii

Contents

19. Risk Stratification with Myocardial Perfusion Imaging������������������ 339

24. Radionuclide Imaging of Cardiac Innervation����������������������� 457

Javier Gomez and Rami Doukky

20. Nuclear Cardiovascular Imaging in Special Populations��������������� 371 Robert C. Hendel, Michael C. Desiderio and Gary V. Heller

21. Preoperative Risk Assessment for Noncardiac Surgery����������������� 405 Muhammad Siyab Panhwar, Sumeet S. Mitter and Robert C. Hendel

22. Radionuclide Imaging in Heart Failure��� 419 Gautam V. Ramani and Prem Soman

Mark I. Travin and Ana Valdivia

25. Imaging Cardiac Amyloidosis���������� 479 Cory Henderson, Dillenia Rosica and Sharmila Dorbala

26.

18

F-FDG PET/CT for Imaging Cardiac Sarcoidosis and Inflammation��������� 495 Cesia Gallegos, Bryan D. Young and Edward J. Miller

27. Hybrid Imaging: SPECT–CT and PET–CT�� 517 Cory Henderson, Patrycja Galazka and Sharmila Dorbala

Section 5. Review Questions�������������� 533 Section 4. Beyond Perfusion Imaging�������� 429 23. Nuclear Cardiology Procedures in the Evaluation of Myocardial Viability������ 431 Christiane Wiefels, Fernanda Erthal, Benjamin Chow, Gary V. Heller and Rob S.B. Beanlands

FM.indd 8

Answers and Explanations for Review Questions���������������������� 563 Index������������������������������� 585

17-02-2022 12:14:09

CONTRIBUTORS

Parthiban Arumugam, MD

Sharmila Dorbala, MD, MPH, FACC, MASNC

Consultant Nuclear Medicine Physician and Clinical Director Nuclear Medicine Centre Manchester University NHS Foundation Trust Manchester Royal Infirmary Manchester, United Kingdom

Director of Nuclear Cardiology Professor, Department of Radiology Division of Nuclear Medicine and Molecular Imaging and the Noninvasive Cardiovascular Imaging Program Heart and Vascular Center Departments of Radiology and Medicine (Cardiology) Brigham and Women’s Hospital Harvard Medical School Boston, Massachusetts

Timothy M. Bateman, MD

Co-Director, Cardiovascular Radiologic Imaging Saint-Lukes Health System Professor of Medicine University of Missouri-Kansas City Kansas City, Missouri

Rami Doukky, MD, MSc, MBA, FACC, FASNC Professor of Medicine and Radiology Chairman, Division of Cardiology Cook County Health Chicago, Illinois

Rob S.B. Beanlands, MD, FRCPC, FCCS, FACC, FASNC Vered Chair and Head, Division of Cardiology Professor, Medicine (Cardiology)/Radiology Distinguished Research Chair University of Ottawa Director, National Cardiac PET Centre University of Ottawa Heart Institute Ottawa, Ontario, Canada

Andrew J. Einstein, MD, PhD, FACC, FAHA, MASNC, MSCCT, FSCMR Associate Professor of Medicine (in Radiology) Director, Nuclear Cardiology, Cardiac CT, and Cardiac MRI Director, Advanced Cardiac Imaging Fellowship Seymour, Paul and Gloria Milstein Division of Cardiology, Department of Medicine, and Department of Radiology Columbia University Irving Medical Center/New YorkPresbyterian Hospital New York, New York

James A. Case, PhD, MASNC

University of Missouri, Columbia Chief Scientific Officer, Cardiovascular Imaging Technologies Kansas City, Missouri

Benjamin Chow, MD, FRCPC, FACC, FESC, FASNC, MSCCT

Fernanda Erthal, MD

Cardiac Imaging Staff Department of Cardiac Imaging Diagnosticos da America SA (DASA) Rio de Janeiro, Brazil

Director of Cardiac Imaging Saul & Edna Goldfarb Chair in Cardiac Imaging Director of Cardiac Imaging Fellowship Training Co-Director of Cardiac Radiology Professor, Departments of Medicine (Cardiology) and Radiology University of Ottawa Heart Institute Ottawa, Ontario, Canada

Patrycja Galazka, MD

Fellow, Noninvasive Cardiovascular Imaging The Noninvasive Cardiovascular Imaging Program Heart and Vascular Center Departments of Radiology and Medicine (Cardiology) Brigham and Women’s Hospital Harvard Medical School Boston, Massachusetts

C. David Cooke, MSEE

Lead Applications Developer/Analyst Department of Radiology and Imaging Sciences Emory University School of Medicine Atlanta, Georgia

Cesia Gallegos, MD, MHS

Michael C. Desiderio, DO, FACC

Assistant Professor of Medicine (Cardiology) Section of Cardiovascular Medicine Yale University School of Medicine New Haven, Connecticut

Medical Director, Cardiology UPMC North Central Pennsylvania Region Williamsport, Pennsylvania

ix

FM.indd 9

17-02-2022 12:14:09

x

Contributors

James R. Galt, PhD

Matthew J. Memmott, MSc

Javier Gomez, MD, FACC, FASNC

Edward J. Miller, MD, PhD, FASNC, FACC

Gary V. Heller, MD, PhD, FACC, MASNC

Robert J.H. Miller, MD, FRCPC, FACC

Robert C. Hendel, MD, FAHA, FSCCT, MACC, MASNC

Sue Miller, CNMT

Director of Nuclear Medicine Physics Professor, Department of Radiology and Imaging Sciences Emory University School of Medicine Atlanta, Georgia Assistant Professor of Medicine Director and Cardio-Oncology Services Division of Cardiology Cook County Health Chicago, Illinois Gagnon Cardiovascular Institute Morristown Medical Center Morristown, New Jersey

Sidney W. and Marilyn S. Lassen Chair in Cardiovascular Medicine Professor of Medicine and Radiology Tulane University School of Medicine New Orleans, Louisiana

Cory Henderson, MD

Assistant Professor of Medicine and Radiology Division of Cardiovascular Medicine, Department of Medicine Boston University School of Medicine Boston, Massachusetts

Milena J. Henzlova, MD

Professor of Medicine (retired) Mount Sinai School of Medicine New York, New York

Cole B. Hirschfeld

Fellow, Cardiovascular Disease Weill Cornell Medicine NewYork-Presbyterian Hospital New York, New York

Consultant Medical Physicist Nuclear Medicine Centre Manchester University NHS Foundation Trust Manchester, United Kingdom Associate Professor of Medicine (Cardiology) and Radiology & Biomedical Imaging Yale University School of Medicine Section of Cardiovascular Medicine New Haven, Connecticut Clinical Assistant Professor Department of Cardiac Sciences University of Calgary and Libin Cardiovascular Institute Calgary, Alberta, Canada Chief Operating Officer Molecular Imaging Services, Inc. Newark, Delaware

Sumeet S. Mitter, MD, MSc

Assistant Professor Department of Medicine, Division of Cardiology Icahn School of Medicine at Mount Sinai New York, New York

Sanjeev U. Nair, MBBS, MD, FACP, FACC, FSCAI Interventional Cardiologist SN Cardiovascular Associates Fort Worth, Texas

Muhammad Siyab Panhwar, MD Cardiovascular Medicine Fellow Tulane University Medical Center New Orleans, Louisiana

Krishna K. Patel, MD, MSc

Seyed Mehdi Khalafi, MD

Cardiovascular Medicine Fellow Tulane University Medical Center New Orleans, Louisiana

Assistant Professor of Medicine (Cardiology) and Population Health and Policy The Zena and Michael A. Wiener Cardiovascular Institute Blavatnik Women’s Health Research Institute Institute for Transformative Clinical Trials Icahn School of Medicine at Mount Sinai New York, New York

Saurabh Malhotra, MD, MPH, FACC, FASNC

Gautam V. Ramani, MD

Director of Advanced Cardiac Imaging Cook County Health Associate Professor of Medicine Rush Medical College Chicago, Illinois

FM.indd 10

Associate Professor of Medicine Division of Cardiovascular Medicine University of Maryland Baltimore, Maryland

17-02-2022 12:14:09

Contributors 

Archana Ramireddy

Joey Stevens, CNMT

Dillenia Rosica, MD

E. Lindsey Tauxe, MEd, CNMT, FASNC

Rupa M. Sanghani, MD, FACC, FASNC

Jason S. Tavel, PhD, DABR

Clinical Cardiac Electrophysiologist Kaiser Permanente Northern California Santa Clara, California Clinical Assistant Professor of Radiology Department of Radiology Geisinger Health System Danville, Pennsylvania Associate Professor of Medicine Division of Cardiology Rush University Medical Center Chicago, Illinois

Sunil Selvin, CNMT

Vice President, Operations & Clinical Education Molecular Imaging Service, Inc. Newark, Delaware

Senior Clinical Accounts Manager Molecular Imaging Services, Inc. Newark, Delaware Operations Director (Retired) Medicine-Cardiovascular Disease University of Alabama at Birmingham Birmingham, Alabama Medical Physicist Astarita Associates, Inc. Smithtown, New York Adjunct Assistant Professor Molloy College Rockville Centre, New York

Mark I. Travin, MD, FACC, MASNC

Cardiologist Cardiovascular Institute of the South Lafayette, Louisiana

Department of Radiology/Division of Nuclear Medicine Director of Cardiovascular Nuclear Medicine Montefiore Medical Center Professor of Clinical Radiology and Clinical Medicine Albert Einstein College of Medicine Bronx, New York

Jamario Skeete, MD

Ana Valdivia, MD

Gursukhman Deep S. Sidhu, MD

Cardiovascular Disease Fellow Division of Cardiology, Department of Medicine Rush University Medical Center Chicago, Illinois

Piotr J. Slomka, PhD

Director of Innovations in Imaging, Cedars-Sinai Professor of Medicine and Cardiology Division of Artificial Intelligence in Medicine, Cedars-Sinai Professor of Medicine, UCLA School of Medicine Los Angeles, California

Prem Soman, MD, PhD

Professor of Medicine, and Clinical & Translational Science University of Pittsburgh Associate Chief, Cardiology Director, Nuclear Cardiology and the Cardiac Imaging Fellowship Director, Cardiac Amyloidosis Center University of Pittsburgh Medical Center Pittsburgh, Pennsylvania

FM.indd 11

xi

Department of Radiology/Division of Nuclear Medicine Montefiore Medical Center Albert Einstein College of Medicine Bronx, New York

Christiane Wiefels, MD, MSc

Assistant Professor of Medicine Division of Nuclear Medicine, Department of Medicine University of Ottawa Ottawa, Ontario, Canada PhD Candidate Federal Fluminense University Brazil

Bryan D. Young, MD PhD, FACC

Assistant Professor of Medicine Division of Cardiovascular Medicine, Department of Internal Medicine Yale School of Medicine; Yale New-Haven Health System New Haven, Connecticut

17-02-2022 12:14:09

This page intentionally left blank

READ_THINK_ITALIAN.indd 2

11/19/21 1:12 PM

PREFACE

questions with detailed answers related to each chapter, found at the end of the book. This fourth edition is ideally suited for trainees and early-career professionals both radiologists and cardiologists. Additionally, this book should be very useful for technologists and healthcare professionals involved in decision-making for testing procedures. We are grateful to the contributors who have done an outstanding job updating and expanding the book and its value. We hope that you find the fourth edition of Nuclear Cardiology: Practical Applications useful and that it will be a focal point for your nuclear cardiology education. To this end, it is our goal to assist in the improvement of nuclear cardiology practice and to benefit the patients for which we care.

We are pleased to present this fourth edition of Nuclear Cardiology: Practical Applications. We have undertaken substantial revisions, emphasizing recent changes in technology as well as the contemporary clinical applications of nuclear cardiology. We have provided insight as to future directions of the field, delineating where this important imaging modality is positioned in current-day clinical practice, especially in the setting of multi-modality imaging. We have greatly expanded information regarding positron emission tomography, including an entire chapter on the assessment of myocardial blood flow. Each chapter now features a table of key points and many of the tables and figures have been updated and expanded. We believe these help in the learning process as well as providing easy referencing key pieces of information. For your personal knowledge assessment, especially for preparation for credentialing examinations, we have provided a multitude of

Gary V. Heller Robert C. Hendel

xiii

FM.indd 13

17-02-2022 12:14:09

This page intentionally left blank

READ_THINK_ITALIAN.indd 2

11/19/21 1:12 PM

SECTION

1

FUNDAMENTALS OF NUCLEAR CARDIOLOGY

sec01.indd 1

15-02-2022 12:34:58

This page intentionally left blank

READ_THINK_ITALIAN.indd 2

11/19/21 1:12 PM

Fundamentals of Nuclear Cardiology Physics C. David Cooke, James R. Galt and E. Lindsey Tauxe

KEY POINTS ■■

■■ ■■

■■

■■

■■

Elements are defined by the number of protons in the nucleus. Nuclides are defined by the number of protons and the number of neutrons. The ratio of protons to neutrons determines the stability of a nucleus. Unstable nuclei decay to a more stable state through several different mechanisms: α decay, β− decay, β+ (positron) decay, electron capture, and isomeric transition. The rate at which unstable nuclei decay can be described by the decay constant. It is often more convenient to describe the rate of decay by the half-life. The interaction of radiation with matter is dependent on the energy and type of the radiation, as well as the atomic number (Z number) of the matter. Attenuation is the loss of radiation as it passes through matter and is absorbed or deflected.

INTRODUCTION A practical review of basic atomic and nuclear physics is essential to understand the origins of radiations, as well as their interactions with matter. The nature and type of emissions are determined by the

ch01.indd 3

CHAPTER

1

structural character of the atom and nucleus. The ways in which radiation interacts with matter have a direct relationship with imaging and radiation safety. The types of radiations and the ways in which they interact with matter are the foundation of radionuclide imaging and radiation safety. This chapter will focus on atomic and nuclear structure and the interaction of radiations with matter as they relate to radionuclide imaging.

ATOMIC AND NUCLEAR STRUCTURE Matter is composed of atoms and the characteristics of a specific form of matter are determined by the number and type of atoms that make it up. How atoms combine is a function of their electron structure. The electron structure is determined by the nuclear architecture. As we have yet to image the atom, its structure is based on a “most-­probable” model that fits physical behaviors we observe. The probabilistic approach is based on the model of the atom proposed by Niels Bohr in 1913. The Bohr atom proposed a positively charged nucleus, ­surrounded by negatively charged electrons. A neutral atom is one in which the positive and negative charges are matched. A mismatch in these charges determines the ionic character of the atom, which is the basis for its chemistry. The electron configuration is also a source for emissions used in radionuclide imaging. These emissions, or radiations, will be in one of two forms: particulate or electromagnetic. The

15-02-2022 09:36:29

4

Section 1  Fundamentals of Nuclear Cardiology

origins of either type of radiation may be from the nucleus or the electron structure.

▶▶Electron Configuration Electrons are arranged around the nucleus in shells. The number of shells is determined by the number of electrons, which is, in turn, determined by the number of protons in the nucleus. The force exerted on these shells, called binding energy, is determined by the proximity of the shell to the nucleus. Higher binding energies are exerted on shells closest to the nucleus and conversely, lower binding energies for those more distant from the nucleus. The innermost shell is named the “K” shell and electrons in this shell are subject to the highest binding energy. The magnitude of that energy is dependent on the positive forces, which is determined by the number of protons in the nucleus. The shells more distant from the nucleus are named L, M, N, and so on. Each of these shells has lower binding energies as a result of their distance from the nucleus (Fig. 1-1). The radii of each of these shells increase as a function of their distance from the nucleus. An ­ ­expression of this is given by assigning an integer value (1, 2, 3, …) to each shell. The lower values represent smaller radii. These integer values are called quantum numbers. Therefore, the K shell has a quantum number of 1, L = 2, M = 3, etc. This pattern continues until all available electrons are bound to a shell. The innermost shells are filled with electrons preferentially. The maximum number of electrons is specific to each shell and is calculated by 2n2, where

n is the quantum number. Therefore, the maximum number of electrons for each shell is: K = quantum #1 = 2(1)2 = 2 electrons L = quantum #2 = 2(2)2 = 8 electrons M = quantum #3 = 2(3)2 = 18 electrons These shells are further subdivided into substates. The number of substates for each shell can be calculated by 2n − 1; therefore: K shell = 2(1) − 1 = 1 substate L shell = 2(2) − 1 = 3 substates M shell = 2(3) − 1 = 5 substates Each substate for a given shell will have a unique binding energy. For instance, the L shell has three substates, LI, LII, and LIII.1 Each of these has slightly different distances from the nucleus, and therefore slightly different binding energies (Fig. 1-2).2

Atomic Radiations Electrons in inner shells being under high binding energy and thus tightly bound to the nucleus are in an inherently low-energy state. Outer shell and free electrons are in an inherently higher-energy state. Therefore, to move an inner shell electron to an outer shell requires energy. The amount of energy required is simply the difference between binding energies. Example: Binding energy for a hypothetical “K” shell = 100 keV and “L” = 50 keV. K100 − L50 = 50 keV

L shell

K shell K shell • High binding energy

LI substate LII substate

LIII substate

L shell • Low binding energy M shell • Lower binding energy

FIGURE 1-1  Atomic structure. The nucleus is surrounded by electron shells. The binding energy decreases as the distance from the nucleus increases (K > L > M).

ch01.indd 4

Unique binding energies LIII>LII>LI

FIGURE 1-2  Electron configuration. Electrons are arranged in subshells, as illustrated for the L shell. Each subshell has a unique binding energy.

15-02-2022 09:36:32

Chapter 1  Fundamentals of Nuclear Cardiology Physics

of energy input to move the electron from the “K” shell to the “L” shell. Conversely, the movement of an electron from an outer shell to an inner shell, L → K, yields energy. This energy yield results in the emission of radiation. The energy of the radiation is equal to the differences in binding energies of the shells. The radiation may take on two different forms: characteristic x-ray or Auger (oh-zhay) effect. Example: Binding energy for a hypothetical “K” shell = 100 keV and “L” = 50 keV. K100 − L50 = 50 keV of energy released as the electron moves from the “L” shell to the “K” shell. Characteristic x-rays are electromagnetic radiations (photons) that are created when an outer shell electron moves to fill an inner shell vacancy. This vacancy may occur for several reasons—to be discussed later. The energy of this photon is equal to the difference between binding energies. Since binding energies are determined by, or characteristic of, the number of protons in the nucleus, and it is the number of protons that determines an element’s identity, the characteristic x-ray energies are specific to each element and the electron shells from which they originate. X-radiation is defined as an electromagnetic radiation originating outside the nucleus, therefore the term characteristic x-ray. The Auger effect occurs under the same conditions as characteristic x-ray, that is, an inner shell vacancy being filled by an outer shell electron. The difference is that the excess energy from the cascading electron is radiated to another electron. This ejects that electron from its shell. This free electron will have kinetic energy equal to the difference in the binding energies less the binding energy of the shell of the free electron. The Auger effect is more common in elements with lower numbers of protons (Z number).1–3

▶▶Nuclear Structure The nucleus is composed mainly of neutrons and protons. Any particle contributing to the structure of the nucleus is called a nucleon. The conventional nomenclature to describe the nucleons is: AZ X N . where: X = Symbol of the chemical element A (Atomic mass number) = Total number of nucle­ ons = # Protons + # Neutrons

ch01.indd 5

5

Z (Atomic number) = # Protons N = # Neutrons Since the number of neutrons (N) can be derived from the atomic mass number (A) and the number of protons (Z), it is usually omitted (N = A − Z). In addition, since the number of protons (Z) defines an element, as does its chemical symbol (X), only one is necessary; hence, Z is often omitted as well. The total mass of an atom is essentially the combined masses of the nucleons. Electrons contribute less than 1% to the total mass.1 Nuclides having the same number of protons (Z) are called isotopes. Isotopes are the same element, but have different atomic masses (A) and therefore have different numbers of neutrons (N); for example: 125 53I72 , 127 131 I , and I . Nuclides with the same number of 53 74 53 78 neutrons (N) are called isotones and will be different elements, since the number of protons (Z) will 132 133 be different; for example: 131 53I78 , 54 Xe78 , and 55Cs 78. Nuclides with the same atomic mass number (A) are called isobars and are different elements as well, since they will have different numbers of protons (Z) 99 and neutrons (N); for example: 99 42 Mo57 and 43Tc56. Finally, nuclides with the same number of protons (Z) and neutrons (N), but in different energy states 99 are called isomers; for example: 99m 43Tc and 43Tc. An easy mnemonic for remembering this is that isotopes (with a p) have the same number of protons, isotones (with an n) have the same number of neutrons, isobars (with an a) have the same atomic number, and isomers (with an e) are the same nuclide with different energies. Isotopes having different N numbers are of particular interest to imagers because they have the same chemistry, since their Z numbers and, therefore, electron numbers are the same.1–5 Some isotopes exhibit the emission of radiations, which is due to the differences in the number of neutrons. These isotopes are called unstable. If all the stable isotopes of all elements are plotted, comparing proton number to neutron number, a pattern emerges as illustrated in Figure 1-3. Elements with low Z numbers have proton to neutron ratios that are 1:1. As Z numbers increase, this ratio increases to as high as 1.5. This distribution of stable elements is called the line of stability. By definition, an element with a proton to neutron ratio that falls to either the left or right of the line of

15-02-2022 09:37:29

Section 1  Fundamentals of Nuclear Cardiology

6

and dosimetry perspectives, due to their lower probability of creating potentially damaging interactions as compared to particles. With these considerations, it is important to understand the modes of decay of 99mTechnetium, 201Thallium, and 82Rubidium— the most commonly used radionuclides in nuclear cardiology.1,2

Line of stability 100 Neutron-rich zone

Neutron number (N)

80 60

N

40

Z

β− Decay Proton-rich zone

20 0

=

0

20

40 60 Atomic number (Z)

80

100

FIGURE 1-3  Line of stability. All naturally occurring stable nuclides fall along a distribution known as the line of stability (LOS). As illustrated, for light elements (Z < 20) N ~ Z and for heavier elements N ~ 1.5Z. Unstable elements, lying to the left of the LOS, are neutron rich; those lying to the right of the LOS are proton rich.

­stability is unstable. The unstable isotopes, radioisotopes, are unstable because their nuclear configurations are either proton rich or neutron rich relative to stable configurations. These radioactive elements seek stability by undergoing transformations in their nuclear configurations to a more stable P ↔ N ratio. The type of transformation will be a result of the P ↔ N ratio, that is, proton rich versus neutron rich. This type of transition is called the mode of decay.1–4

Modes of Decay The goal of nuclear decay is to equate the balance of forces in the nucleus. The repelling forces originating from the positive charge (coulombic forces) of the protons, when matched by the attractive forces from within the nucleus (exchange forces), define ­stability. When these forces are mismatched, nuclear transformations (radioactive decay) result. The mode of decay will produce unique emissions and lead to a more stable nuclear configuration. In radionuclide imaging, the ideal mode of decay would result in a high yield of photons, at an energy that is efficiently detected by our imaging instrumentation. Photon emission is also desirable from the radiation safety

ch01.indd 6

In an unstable nuclear configuration where the nucleus is neutron rich, β− decay occurs. To decrease the neutron–proton ratio, a neutron is converted to a proton and an energized electron is emitted. The expression of this nuclear transition is: n → p + e− + ν + energy where n is the neutron, p the proton, e the electron, and ν is the neutrino. The neutrino (ν) behaves like a particle with no mass and is not critical to imaging considerations. The primary emission is the energized electron (e−). The nuclear configuration that results from β− decay is a daughter with a stable or more stable energy state and an additional proton in its nucleus. Example: 146C 8 → 147 N7 Since the number of protons is changed, the elemental identity changes. This is called a transmutation. The daughter atomic mass (A) remains the same as the parent nucleus, and the energy carried off by the ejected electron is called transition energy. This leads to a more balanced relationship of coulombic force (repelling forces due to the protons) and exchange force (attractive nuclear forces). The resulting emission of the energized electron, a β− particle, is of no use in imaging and contributes to an increase in radiation dose in a biologic system. This decay process may lead to a daughter that is not fully stable, but more stable than the parent.1,2 The change in nuclear configuration is an increase in Z and a decrease in N. β+ Decay In nuclear configurations where the parent is proton rich, β+ decay may occur. In this mode of decay, a proton is converted to a neutron and the emission of

15-02-2022 09:37:33

Chapter 1  Fundamentals of Nuclear Cardiology Physics

an energized, positively charged electron (β+) results. The nuclear equation is: p → n + e+ + ν + energy The energy of the β+ particle contributes to resolving the transition energy between the unstable parent and more stable daughter, as in β− decay. An important secondary emission will result from the formation of the β+ particle. Since there is an abundance of negatively charged electrons in nature, the resulting positively charged electron (β+) will be attracted to, and collide with, a free negatively charged electron. This collision results in the annihilation of both particles. The annihilation leads to the conversion of the mass of these particles to their equivalent energy state. This is expressed by Einstein’s equation E = mc 2, where E is energy, m the mass, and c is the speed of light. This essentially states that energy and mass are simply two physical forms of the same thing. Therefore, two photons (E) are emitted, each with the energy equivalent to the mass (m) of an electron, which is 511 keV. Unique to this annihilation is that these photons are emitted in a 180-degree trajectory from each other. It is these photons that are detected and registered into an image in positron imaging, such as with 82Rb. The change in nuclear configuration is a decrease in Z and an increase in N. 82 Rb45 → Example: 37

82 36 Kr46

Electron Capture An alternative to β+ decay in proton-rich nuclear configuration is electron capture. This mode of decay is defined as the capture of a K-shell electron by the nucleus, the subsequent combination with a proton, and creation of a neutron. The nuclear expression is therefore: p + e- → n + ν + energy The vacancy left by the captured electron would then be filled by an outer shell electron. A cascade of an electron, filling subsequent vacancies, creates secondary emissions called characteristic x-rays and Auger electrons. The energies of these emissions will be characteristic of the binding energy of the

ch01.indd 7

7

daughter, since the nuclear transition occurred prior to the production of the x-rays and Auger electrons. It is the characteristic x-rays that are imaged in 201Tl myocardial perfusion imaging. The energy of the x-rays is determined by the binding energy of 201Hg, the daughter of the decay of 201Tl. Electron capture decreases the proton–neutron ratio.1 Example: 20181Tl120 →

201 80 Hg121

Isomeric Transitions and Internal Conversions The daughter of the decay of a radioactive parent will ideally be in its most stable energy configuration or ground state. This does not always occur, leading to either of the two unstable states: excited state or metastable state. Excited states are very unstable and exist for very short time periods, usually less than 10−12 seconds. Metastable states, however, may exist for several hours. These metastable states lead to the release of energy in the form of electromagnetic emissions, without changing the proton–neutron ratios. The daughter nucleus has the same nuclear structure as the parent has, but in a more stable energy configuration. This form of decay is called an isomeric transition and results in electromagnetic emissions called γ-rays. These emissions are the same as x-rays, differing only by their location of origin, that is, the nucleus. As noted with the production of characteristic x-rays, there is a competing process, resulting in a particulate radiation. This process is called internal conversion. For any given metastable state, there is a specific ratio of isomeric transitions to internal conversions. In imaging, the higher percentage of isomeric transitions compared to internal conversions is preferred due to the resulting higher yield of photons. The decay of 99mTc to 99Tc is an example of an isomeric transition of the metastable state (99mTc). The percent occurrence of isomeric transitions of a population of 99mTc nuclei is approximately 87%. For example, for every 100 decays of 99mTc nuclei, there is a yield of 87 γ-photons and 13 internal conversion electrons. For any given mode of decay, should the daughter be metastable, there will be the emission of γ-photons and internal conversion electrons as secondary emissions. This will be indicated as [B−, γ], [B+, γ], [EC, γ], and so forth. The internal conversion electron

15-02-2022 09:37:41

8

Section 1  Fundamentals of Nuclear Cardiology

yield, in ratio to γ-photon yield, is specific to a given radionuclide.1,2,5

P Z

In unstable nuclei with very high atomic masses, the most probable mode of decay is α decay. An alpha particle consists of two protons and two neurons, which is essentially a helium nucleus. Alpha decay results in a daughter with a Z number of 2 less than the parent and an atomic mass less by 4 relative to the parent. 231 90Th141

Due to its high charges and heavy mass, the alpha particle has a very short travel distance in matter and deposits its energy very quickly. It has no application in diagnostic imaging and induces significant potential for biologic damage.1,3

Decay Schemes The modes of decay may be expressed graphically, called decay schemes. Decay schemes graphically illustrate all possible nuclear transitions that unstable nuclei undergo. They are often accompanied by tables with detailed information about the transitions, such as the percentage occurrence, isomeric transitions, internal conversions, characteristic x-rays, Auger electrons, and biologic dose information. In decay schemes, the nuclear energy levels are expressed as horizontal lines. The space between these lines represents the transition energy (Q). The types of emissions are depicted by a unique direction of a line (Fig. 1-4). Note that the arrows may be angled to either the right or left. In neutron-rich parents, the mode of decay “shifts” the daughter to the right, corresponding to the shift on to the line of stability graph. Conversely, a mode of decay for a proton-rich parent moves to the left, toward the line of stability. The tables that accompany decay schemes provide additional detail including the secondary emissions, as mentioned earlier. Since many of the secondary emissions are particulate, that is, electrons, these data are of particular interest in radiation dosimetry. In the decay scheme for 201Tl, the data regarding the characteristic x-rays of 201Hg are in these tables.

ch01.indd 8

Energy

Alpha (α) Decay

Example: 235 92 U143 →

P = Parent D = Daughter

A

Alpha

A−4 D1 Z−2

β−

β+

A D2 Z−1 A

EC (electron capture)

D3 Z−1 Z−2

Z−1

Z Atomic number

A mD4 Z+1 IT D4 (isomeric Z + 1 transition) A

Z+1

FIGURE 1-4  Decay schemes. This figure illustrates the configurations of decay schemes for the different modes of decay. The schemes move to the left for proton-rich radionuclides and to the right for neutron-rich radionuclides.

Parent–Daughter Equilibrium Not all nuclear transitions lead to a stable daughter. The β− decay of 99Mo yields 99mTc, which then decays to 99Tc by isomeric transitions and internal conversions. 99m Tc decays to 99Tc with an 87% frequency through isomeric transitions. Therefore, for every 100 decays of 99mTc, we observe 87 γ-rays and 13 internal conversion electrons, as stated earlier. This higher yield of photons makes 99mTc a very desirable radionuclide for imaging. A sample of 99Mo would always contain some proportion of 99mTc and 99Tc. Since both parent and daughter are decaying, the relative activities would reach equilibrium, based on their half-lives. These states of equilibrium are employed when using both technetium and rubidium generators. When the parent half-life is marginally longer than that of the daughter, the amount of the daughter in the mixture will reach a maximum over a period of time. That elapsed time will be a multiple of half-lives of the daughter. If the daughter radionuclide is removed from the mixture, the same multiple of half-lives will have to occur, before the maximum amount of the daughter is subsequently reached. This equilibrium state is called transient equilibrium.2,4 It is this transient state that is the basis of 99mTc production from 99 Mo–99mTc generators (Fig. 1-5).

15-02-2022 09:37:46

Chapter 1  Fundamentals of Nuclear Cardiology Physics

9

Parent

Daughter

1

2

3 4 Time (Number of daughter half-lives)

FIGURE 1-5  Transient equilibrium. When the parent half-life is marginally longer than that of the daughter, the amount of daughter activity will reach a maximum after relatively few daughter half-lives have passed. 99Mo and 99mTc typically reach transient equilibrium after approximately four 99mTc half-lives.

In parent–daughter mixtures where the halflife of the parent is markedly longer than that of the daughter, secular equilibrium is reached. In this state of equilibrium, the concentration of the parent is decreasing so slowly relative to the daughter that the mixture appears to have the half-life of the parent. It is this equilibrium that is the basis for the 82Sr–82Rb generators used in 82Rb positron emission tomography (PET) imaging (Fig. 1-6).1

RADIOACTIVITY The specific time that an unstable nucleus will undergo a transition cannot be determined, only predicted. Nuclear transitions are spontaneous and random, so the mathematics of radioactive decay is based on probabilities and rates, not specific nuclear events. If a population of radioactive atoms, N, is considered, the rate of nuclear transitions would be expressed as ∆N/∆t. The rate implies that a constant would express the average number of transitions that occur per unit time. This constant is called the decay constant; which is specific to a given radionuclide and is expressed as λ. The mathematical relationship is: ∆N/∆t = −λN, where N is the total number of

ch01.indd 9

Daughter

Activity

Activity

Parent

1

2 3 4 Time 7 8 9 10 (Number of daughter half-lives)

FIGURE 1-6  Secular equilibrium. When the parent half-life is considerably longer than that of the daughter, the amount of parent activity will decrease very little over time. Therefore, many more daughter half-lives must pass before the equilibrium is reached. An example is 82Sr with a half-life of 25 days and 82 Rb with a 1.2-minute half-life.

radioactive nuclei and λ the decay constant. Since the total N decreases with time (t), the decay constant (λ) is a negative value.1,2,4 The number of transitions per unit time (∆N/∆t) is called activity. Activity is measured in curies (Ci), which is defined as 3.7 × 1010 disintegrations per second (dps). The International System of Units (SI) unit equivalent is the becquerel (Bq), which is defined as 1 Bq = 1 dps. So 1 Ci = 3.7 × 1010 Bq. The most commonly used units are in the mCi (MBq) range for nuclear cardiology procedures. In nuclear decay, the number of radioactive nuclei (N) is always decreasing as time passes at an average rate defined by the decay constant (λ). Decay is expressed, therefore, as an exponential function; that is, the number of radioactive nuclei available is affected by both the number of unstable nuclei and its average rate of decay. To calculate the specific number of decays for a given time, we have the following expression: N (t) = N (0) e−λt where N(t) is the number of unstable nuclei remaining after the elapsed time (t) has passed. The expression

15-02-2022 09:37:56

10

Section 1  Fundamentals of Nuclear Cardiology

e−λt is called the decay factor (DF) and is unique to the time (t) and the decay constant (λ). The “e” is the base of natural logs, which is 2.718, so, when raised to the power of −λt, it determines the specific fraction of remaining radioactive nuclei after time (t) has elapsed. When the elapsed time (t) is the time required for half of the total number of radioactive nuclei to decay, it is termed the half-life. Half-life (T1/2) and the decay constant (λ) are related as: ln 2 T1/2 = λ Since ln 2 is equal to 0.693, we have: T1/2 =

0.693 λ

The activity (Ci) is dependent on the number of unstable nuclei and the decay constant. Therefore, units of activity (A) can be substituted for the number of radioactive nuclei (N), in the decay equation, yielding the following expression: A(t) = A(0) e−λt where A(t) is the activity at elapsed time t, A(0) the activity at t = 0, and e−λt is the decay factor for the specific time (t). As an example of practical use, consider a 30-mCi syringe of a 99mTc-labeled myocardial perfusion agent. The calibration time for the 30 mCi is 8:00 am. What will be the activity at 9:45 am? A(0) = 30 mCi A(t) = ? t = 1 h, 45 min or 1.75 h 0.693 0.693 = = 0.1155 λ= T1/2 6h A(t) = 30 e−(0.1155)(1.75) A(t) = 30 × 0.817 A(t) = 24.51 mCi The same principles apply in the situation where an activity determination is required for a time (t) in

ch01.indd 10

the past, the precalibrated value. In this case, the DF would be the reciprocal of the elapsed time DF1,2: 1 0.817 DF = 1.22 DF =

So, we have: A(0) = (24.51)(1.22) = 30 mCi

INTERACTIONS OF RADIATION WITH MATTER As discussed earlier, there are distinct types of radiations: particulate and electromagnetic. These radiations have distinct interactions with matter. Interactions of radiations with matter are processes that absorb or degrade the energy of the radiation and, in some cases, change its fundamental characteristics. The energy of the radiation and the Z number of the matter determine the type of interaction or multiple interactions that occur. Depending on the energy, radiations may be nonionizing; that is, the energy is not sufficient to free electrons from their orbit. If the energy is sufficient to free electrons from their binding energy, the radiation is ionizing. The energy of the ultrasound radiations in echocardiography does not cause ionization in tissues, and thus is labeled “nonionizing.” The photons used in cardiac catheterization and nuclear cardiology are classified as ionizing, implying that their interaction with tissue creates ion pairs. This interaction in tissue forms the basis for radiation biology as applied in radiation safety practices. Further, these interactions form the physical bases for detecting radiation and creating images in our instrumentation.

▶▶Particle Interactions with Matter As we have stated, radionuclide images are created through the detection of photons and γ-rays for 99m Tc, characteristic x-rays for 201Tl, and annihilation photons for 82Rb and other PET radiopharmaceuticals. When photons undergo interactions in matter, either tissue or imaging detectors, energized charged

15-02-2022 09:38:38

Chapter 1  Fundamentals of Nuclear Cardiology Physics

ch01.indd 11

and mass, will lose its energy over a longer path length, and is thus characterized as a low LET particle. The LET may also be expressed in terms of the relative number of ionizations that is produced. This is termed specific ionization. Particles with a high LET have high SI values; particles with a low LET have low SI values. LET and SI characteristics have profound impact on the type of damage done to tissue.

▶▶Photon Interactions with Matter The interactions of photons in matter are energydegrading processes, just as in particle interactions. There are three mechanisms of photon interaction in matter: photoelectric absorption, Compton scatter, and pair production. The probability of occurrence for each of these mechanisms is a function of the energy of the photon and the atomic number of the matter (Fig. 1-7). Photoelectric absorption occurs when the photon transfers all of its energy to an inner shell electron. The photon is completely absorbed and the electron, called a photoelectron, is ejected from its shell. The energy of the photoelectron is equal to the incident photon energy, minus the binding energy of its shell. It is key to remember that the photon no longer exists (it has been absorbed) and its energy is converted 100

Photoelectric absorption

75 Atomic number

particles that are electrons, are produced. Particles, either β or α, interact with matter through electrical collisions. These collisions result in excitation, ionization, or bremsstrahlung. In excitation, energy from the incident particle is transferred to an outer shell electron. The electron is energized but does not exceed its binding energy. That increased energy is dissipated generally as heat radiation. Excitation is a low-energy interaction. A higher-energy interaction occurs when the incident particle transfers enough energy to exceed the binding energy of the electron. An ion pair, an energized free electron and a positive ion, results. These ionizing interactions represent higher-energy level interactions and contribute to the specific exposure rate when applied to tissue, discussed in detail in a later chapter. The third type of particulate interaction results when the incident particle penetrates the electron cloud and interacts with the charge field of the nucleus. The trajectory of the incident particle is markedly changed, resulting in a decrease in velocity. The decrease in velocity represents an energy loss. That energy loss produces photons of x-rays called bremsstrahlung, German for breaking radiation. Bremsstrahlung interactions are high-energy interactions more typical of high-energy particles interacting with high Z number matter. Since any of these interactions may lead to partial dissipation of the energy of the incident particle, multiple interactions are required for the full energy of the particle to be absorbed in matter. These multiple interactions occur along a path that is determined by the energy, charge, and mass of the particle, as well as the Z number of the matter. A highly charged massive particle will undergo many high-energy interactions over a very short path length. Alternately, a lower-energy, less massive particle will undergo several low-energy interactions over a long path length. The difference in the degree of penetrability of the particle is called linear energy transfer (LET). A heavy highly charged particle would have a high LET, since the density of interactions is high over a short path length. An alpha particle because of its high charge (two protons) and heavy mass (Z = 2; A = 4) has a high LET where dense levels of excitation, ionization, and bremsstrahlung are created in a very short distance. In contrast, a β particle, having little charge

11

Pair production

50

25 Tissue 0

Tl & Tc 0

Compton scatter

0.1 1.0 10.0 Photon energy (MeV)

100.0

FIGURE 1-7  Probability of interaction. The probability of the interaction of a photon in matter as a function of photon energy and Z number is illustrated. Note the most probable interaction of photon from either 99mTc or 201Tl, in tissue, is Compton scatter.

15-02-2022 09:38:38

12

Section 1  Fundamentals of Nuclear Cardiology

from electromagnetic energy to kinetic energy of the photoelectron. The kinetic energy of the photoelectron is dissipated by the mechanisms previously discussed: excitation, ionization, and bremsstrahlung. The vacancy left by the ejected electron is quickly filled by an outer shell electron. Characteristic x-rays and Auger electrons are then emitted, as described earlier. Photoelectric absorption is most probable in interactions of low- to medium-energy photons in matter with high atomic numbers. Photoelectric absorption is what makes gamma camera photomultiplier tubes (PMTs) possible. The second mode of interaction is Compton scatter. It occurs when a photon interacts with an outer shell electron, that is, an electron with low binding energy. Contrary to photoelectric absorption, the photon is not completely absorbed. It transfers a portion of its energy to the electron, which is subsequently ejected, and called a Compton electron. The resultant Compton scattered photon is in an energydegraded state and in an altered trajectory relative to the incident photon. The angle of the scattered photon is related to the amount of energy transferred to the electron. The greater the amount of energy transferred to the electron, the greater the angle of scatter. Compton scatter is most probable in low- to

medium-energy photons, interacting in matter with low atomic masses. This is of particular interest in imaging, since it is the most probable interaction of 201 Tl (Hg x-rays) and 99mTc γ-rays in tissues. As the photons are scattered, not absorbed, they still exist, but have lost their association with the point of origin through the scattering process. These photons are the source of image degradation, but can be identified by their lower-energy values. The identification of these photons and their rejection from an image are critical functions of imaging equipment.1–5 The third interaction of photons in matter is pair production. High-energy photons may completely avoid interacting with orbital electrons and interact in the magnetic field of the nucleus. This interaction results in the creation of a pair of electrons, one positive and one negative. The positive electron immediately combines with a negative electron, creating two 511-keV annihilation photons. The energy of the incident photon must be at least two times the mass energy equivalency of an electron (511 keV) or 1.022 MeV. Since energies in this range are not used in imaging, pair production is not relevant to this discussion.2 These three mechanisms are illustrated in Figure 1-8.

γ Photon Compton electron

γ Photon

γ Photon

Photoelectron Annihilation photon θ e+

e– Annihilation photon A

B

C

FIGURE 1-8  (A) Photoelectric absorption. All photon energy is transferred to the ejected photoelectron. (B) Compton scatter. Partial energy transfer, incident photon scattered by angle θ. (C) Pair production. Photon converts to an electron pair (β−, β+); annihilation photon results.

ch01.indd 12

15-02-2022 09:38:39

Chapter 1  Fundamentals of Nuclear Cardiology Physics

13

Table 1-1 Half-Value Thickness for Tl, Tc, and Rb Half-Value Thickness (mm) Matter Pb Tissue (H2O) Nal

Z Number

201

Tl (80 keV)

82

Tc (140 keV)

Rb (511 keV)

82

0.25

0.30

3.80

8

38.0

46.0

73.0

53

0.67

2.40

21.0

▶▶Attenuation These interactive processes, in addition to photon energy and atomic number, are affected by the thickness of the absorber. For a given thickness, a number of photons will not interact and therefore be transmitted. As the thickness of the absorber increases, the fraction of transmitted photons will decrease. The fraction of absorbed photons is a function of photon energy and atomic number, but the total number of photons absorbed is a function of the thickness of the absorber.1 For any given relationship of photon energy and atomic number, there is an average rate of absorption. The rate is called the linear attenuation coefficient and is symbolized by µ. Since the linear attenuation coefficient is a fixed rate, the rate of transmitted photons is also fixed. If the thickness of a given absorber is doubled, those transmitted photons will be subject to further absorption. The mathematical relationship of incident beam intensity, transmitted beam intensity, and linear attenuation is exponential. The mathematical expression is: I(x) = I(0) e−µx where I(x) is the beam intensity after interacting with an absorber of thickness x and a linear attenuation coefficient of µ, from an initial intensity I(0).6 This mathematical expression is the same as used in predicting the reduction in activity over time. Therefore, the expression e−µx is the specific fraction of absorbed photons by a specific thickness of matter. The thickness of a given absorber that results in a reduction of initial beam intensity of 50% is called the half-value

ch01.indd 13

99m

thickness (HVT) or half-value layer (HVL). The HVT can be calculated by: HVT =

0.693 µ

where µ is the linear attenuation coefficient and 0.693 is the ln 2. Note that this is the same equation as used in calculating the half-life of a radionuclide. The absorption characteristics of tissue are essentially the same as those of water, since the linear attenuation values are similar. Table 1-1 illustrates the differences in HVTs for lead, a common material used in shielding, water (tissue), and NaI, a common detector material in imaging equipment. Note that the HVTs in tissue for 201Tl (80 keV) and 99mTc (140 keV) are 38.0 and 46.0 mm, respectively. It is typical to have these thicknesses of tissue overlaying the heart. If these thicknesses reduce the beam intensity by 50%, the imaging effects would be a reduction in count density of 50%, as well. Also note that 511 keV annihilation photons (from PET) are attenuated less than either 201Tl or 99mTc (because of the higher energy); however, since both photons must be detected for the event to be counted, the effects of attenuation on PET images may be more pronounced. It is critical, therefore, to differentiate these tissue attenuation effects, from reductions in biodistributions of tracers when interpreting radionuclide images.

CONCLUSION In summary, the photons we use to construct cardiac radionuclide images come from both nuclear and atomic sources, as demonstrated by the emissions

15-02-2022 09:38:52

14

Section 1  Fundamentals of Nuclear Cardiology

from 99mTc and 201Tl, respectively. The emissions used may also be the result of processes that are secondary to the primary emission, as seen with the annihilation photons from the decay of 82Rb. The principles of the detectors used in imaging equipment, as well as those used in radiation safety, are based on the characteristics of photons and the matter in which they interact. Those characteristics are defined by the physics of nuclear structure, mass–energy relationships, and radiation interactions in matter. As a practical consideration, it is important to note that radionuclide image quality is directly related to the type, energy, and the behavior in matter of photons. The three mainstream radionuclides used in nuclear cardiology, 99mTc, 201Tl, and 82Rb, exhibit all of these physical differences; therefore, familiarity with the physical basis of radiation physics is essential.

ch01.indd 14

REFERENCES 1. Cherry SR, Sorenson JA, Phelps ME. Physics in Nuclear Medicine. 4th ed. Philadelphia, PA: Saunders/Elsevier Science; 2012:7–18, 19–30, 31–42. 2. Chandra R, Rahmim A. Nuclear Medicine Physics: The Basics. 8th ed. Philadelphia, PA: Wolters Kluwer; 2018:1–8, 9–20, 21–32. 3. Cook SE. Moderate Atomic and Nuclear Physics. Princeton, NJ: D. Van Nostrand Company, Inc.; 1961. 4. Murphy PH, Galt JR. Radiation physics and radiation safety. In: Iskandrian AE, Garcia EV, eds. Nuclear Cardiac Imaging. 5th ed. New York, NY: Oxford University Press; 2016:11–35. 5. Jelley NA. Fundamentals of Nuclear Physics. Cambridge, United Kingdom: Cambridge University Press; 1990:26–31, 140–158. 6. Saha GB. Basics of Pet Imaging. New York: Springer Science + Business Media; 2005:1–14, 111–121.

15-02-2022 09:38:52

Radiation Safety and Protection in Nuclear Cardiology

CHAPTER

James R. Galt, C. David Cooke and Jason S. Tavel

KEY POINTS ■■

■■

■■

■■

■■ ■■

ch02.indd 15

Important concepts used to describe radiation include exposure, absorbed dose, and dose equivalent. A relatable way of explaining the radiation dose from medical procedures to patients is to compare the dose to the reference dose, the yearly radiation exposure from natural sources. Institutions must maintain radiation levels consistent with the As Low As Reasonably Achievable (ALARA) philosophy. Deleterious effects of radiation are classified as either stochastic, where the probability of occurrence increases with dose, or deterministic, in which the severity increases with dose after a threshold is exceeded. Major factors in limiting dose are time, distance, and shielding. Regulatory agencies, such as the Nuclear Regulatory Commission (or state agencies in Agreement States), regulate the use of radioactive materials including transportation, handling, disposal, personnel monitoring, and medical use.

2

INTRODUCTION The Health Physics Society defines radiation as “energy that comes from a source and travels through space.”1 Radiation can be atomic particles such as alpha and beta emissions, as well as electromagnetic energy associated with radio, microwaves, radar, visible light, ultraviolet light, x-rays, and gamma rays. If the radiation has enough energy to remove an electron from an atom, creating an ion, it is termed ionizing radiation.2 Ionizing radiation includes x-rays, gamma rays, and alpha and beta particles and can lead to biological damage by depositing energy in living tissue, breaking molecular bonds. This chapter will introduce units that describe radiation, sources of radiation exposure, radiation dose limits and introduce radiation biology. The chapter will further focus on radiation safety and protection regulations pertinent to the practice of nuclear cardiology. In the United States of America (USA), these regulations are governed by the Nuclear Regulatory Commission (NRC) and are found in Title 10, Parts 19, 20, and 35 of the Code of Federal Regulations (CFR). Most states in the USA are agreement states, meaning that they have an agreement with the NRC to regulate the use of radioactive materials within the state. Agreement state regulations have to be at least as strict as NRC regulations. NRC NUREG 1556 Volume 9, Revision 3 provides

15-02-2022 16:37:19

16

Section 1  Fundamentals of Nuclear Cardiology

guidance specific to radioactive materials licensing for medical use and offers suggested policies and procedures for radiation safety compliance.3

MEDICAL USE OF RADIOACTIVE MATERIALS Medical use of radioactive materials is defined in Part 35 of the CFR as “the intentional internal or external administration of byproduct material or the radiation from byproduct material to patients or human research subjects under the supervision of an authorized user.” The authorized users for nuclear cardiology are physicians who meet certain training and experience requirements and are identified on the facility’s radioactive materials license.4 Also important to the license is the Radiation Safety Officer (RSO), a person who is responsible for implementing the radiation protection program. The regulations specify that the RSO must have independent authority to stop unsafe operations and sufficient time to oversee radiation safety at the facility.3 Duties and training requirements for authorized users and RSOs are specified in the CFR, but it should be noted that some agreement states have more stringent requirements than the NRC.

RADIOLOGICAL UNITS There are several terms used when describing radiation. These include exposure, absorbed dose, and dose equivalent. Named after Wilhelm Roentgen, the scientist who discovered x-rays in 1895, the Roentgen (R) is the common unit of radiation exposure in air.2 One Roentgen corresponds to the amount of radiation required to produce a charge of 2.58 × 10-4 Coulombs per kilogram (C/kg) in air. The Rad, or Radiation Absorbed Dose, is the measure of the amount of energy absorbed by an object as radiation passes through.2 The amount of energy absorbed is dependent on the energy of the incident photon and the composition of the material. The f-factor is a tissue weighting factor used to convert exposure in air (R) to absorbed dose (rad) in tissue taking into account the x-ray or gamma ray energy and effective atomic number of the tissue exposed.5 For example, a 100-keV gamma photon incident on fat will transfer

ch02.indd 16

Table 2-1 Quality Factors for Different Types of Ionizing Radiation

Type of Radiation

Radiation Quality Factor (for Converting Absorbed Dose to Dose Equivalent)

X-ray and gamma rays Beta particles

1 1

Neutrons

10

High-energy protons

10

Alpha particles

20

91% of its energy, whereas the same photon will deliver 96% of its energy to muscle tissue. Therefore, a source of radiation exposing a point in air to 100 R will deliver a dose of 91 rad to fat tissue and 96 rad to muscle tissue at the same reference point. Dose equivalent is a term used to quantify the amount of energy deposited in tissue along with the associated biological risk from the type of radiation.2 The common unit for dose equivalent is Roentgen Equivalent Man (rem), which is calculated by multiplying the radiation absorbed dose (rad) by a radiation quality factor (QF).2 Table 2-1 illustrates that the QF for x-rays, gamma rays, and beta particles is equal to 1, whereas the QF for alpha particles is 20.6 This means that an absorbed dose of 10 rads from an x-ray, gamma ray, or beta particle equates to 10 rem (10 rad × 1), whereas the dose equivalent would be 200 rem (10 rad × 20) if originating from alpha particulates. Although the common units to describe the absorbed dose and dose equivalent are the rad and rem, the international community now uses the Système International (SI) units of Gray (Gy) and Sievert (Sv), respectively. 1 Gy = 100 rad and 1 Sv = 100 rem. Radiation exposure is simply expressed as C/kg. Table 2-2 provides a summary of the units of radiation exposure, absorbed dose, and dose equivalent.

SOURCES OF IONIZING RADIATION EXPOSURE Ionizing radiation occurs naturally in the earth’s soil and rock, in the cosmic rays descending from the sun

15-02-2022 16:37:19

Chapter 2  Radiation Safety and Protection in Nuclear Cardiology

17

Table 2-2 Units of Radiation Exposure and Dose Quantity

Classical Unit

International Unit (SI)

Relationship

Exposure (in air)

Roentgen (R)

C/kg

1 R = 2.58 × 10−4 C/kg

Dose (in tissue)

Radiation absorbed dose (Rad = R × f)

Gray (Gy)

1 Gy = 100 rad 1 mGy = 0.1 rad

Dose equivalent (risk adjusted dose)

Roentgen equivalent in man (Rem = Rad × QF)

Sievert (Sv)

1 Sv = 100 rem 1 mSv = 0.1 rem

SOURCES OF RADIATION EXPOSURE IN THE UNITED STATES

Internal 5.0%

Terrestrial (Soil) 3.0%

Consumer Products 2.0%

Industrial and Occupational 0.1%

Cosmic (Space) 5.0% Radon and Thoron 37.0%

Nuclear Medicine 12.0%

Medical Procedures 36.0%

FIGURE 2-1  Radiation exposure sources in the United States. The U.S. population receives approximately 620 mrem (6.2 mSv) annually, with ~50% from manmade sources (medical procedures, nuclear medicine, and consumer products) and ~50% from background radiation. Data from NCRP Publications. NCRP Report No 160 (2015). Copyright © 2021 National Council on Radiation Protection and Measurements. All Rights Reserved. https://ncrponline.org/publications/reports/ncrp-report-160-2/.

and stars, as well as in our own body.7 This ubiquitous background radiation in the United States totals approximately 3.1 mSv/yr across the population.8 Exposure to Radon and its decay products accounts for the majority of the natural background radiation. In addition to natural background, people are exposed to radiation in manufactured products such as smoke detectors, ceramics, and building materials, as well as from medical sources including diagnostic

ch02.indd 17

x-rays and nuclear medicine procedures.9 The effective radiation dose from these sources is about equal to that of natural sources and accounts for an additional 3.1 mSv/yr (Fig. 2-1). It is important to note that these estimates are population based and not all individuals are exposed to radiation from manufactured products and radiological examinations. A convenient and relatable way of explaining the radiation dose from medical procedures to patients

15-02-2022 16:37:20

18

Section 1  Fundamentals of Nuclear Cardiology

Table 2-3 Effective Radiation Dose from Common Examinations in Terms of Background Radiation Single Procedure Examination

Administered Activity

Stress/Rest

Reference Level

Estimated Dose

Estimated Dose

Reference Level

201

148 MBq (4 mCi)

20.72 mSv (2.07 rem)

6.91

99m

Tc-tetrofosmin stress only

296 MBq (8 mCi)

2.04 mSv (0.20 rem)

0.68

99m

Tc-tetrofosmin stress

888 MBq (24 mCi)

6.13 mSv (0.61 rem)

2.04

99m

Tc-tetrofosmin rest

296 MBq (8 mCi)

2.04 mSv (0.20 rem)

0.68

99m Tc-sestamibi stress only

296 MBq (8 mCi)

2.66 mSv (0.27 rem)

0.89

99m

Tc-sestamibi stress

888 MBq (24 mCi)

7.99 mSv (0.80 rem)

2.66

99m

Tc-sestamibi rest

296 MBq (8 mCi)

2.34 mSv (0.23 rem)

0.78

82

1480 MBq (40 mCi)

2.52 mSv (0.25 rem)

0.84

5.03 mSv (0.50 rem)

1.68

13

740 MBq (20 mCi)

2.00 mSv (0.20 rem)

0.67

4.00 mSv (0.40 rem)

1.33

Tl

Rb rest or stress

N-ammonia rest or stress

20.72 mSv (2.07 rem)

6.91

8.17 mSv (0.82 rem)

2.72

10.33 mSv (1.03 rem)

3.44

Notes: Reference level = Effective dose/ Yearly background dose Dose levels for procedures data from Dorbala S, Blankstein R, Skali H, et al. Approaches to reducing radiation dose from radionuclide myocardial perfusion imaging. J Nucl Med. 2015;56(4):592–599.

is to express the dose in terms of the yearly exposure from natural sources.10 This is often called the reference dose but may also be called by other expressions, such as BERT (Background Equivalent Radiation Time).11,12 Table 2-3 shows the estimated radiation doses to patients from common nuclear cardiology procedures and their associated reference levels. The dose levels in Table 2-3 also illustrate the radiation dose differences in procedures that have led the leadership of the nuclear cardiology community to recommend dose reduction by utilization of shorter half-life radiopharmaceuticals for singlephoton emission computed tomography myocardial perfusion imaging (MPI) (99mTc preferred over 201 Tl). Where positron emission tomography MPI imaging is available, either 13N-ammonia or 82Rb will result in a lower radiation dose to the patient. Use

ch02.indd 18

of stress-only protocols may also be used to reduce radiation dose. In this scenario, the stress imaging is done first and rest imaging is only performed if the stress image is read as abnormal.13,14

RADIATION DOSE LIMITS AND INVESTIGATIONAL LEVELS The regulations for dose limits in the United States can be found in Title 10, Part 20 of the CFR (10 CFR 20). Table 2-4 illustrates these limits, which include maximum radiation dose to occupational workers, the embryo/fetus of an occupational worker, and the public. Both occupational and public dose limits exclude exposure to natural background radiation and radiation from an individual’s own medical procedures.

15-02-2022 16:37:20

Chapter 2  Radiation Safety and Protection in Nuclear Cardiology

19

Table 2-4 Radiation Dose Limits in the United States Area

Dose Limit

Occupational worker Whole-body total effective dose

0.05 Sv (5 rem) per year

Occupational worker Lens of the eye

0.15 Sv (15 rem) per year

Occupational worker Shallow dose to skin or extremity

0.5 Sv (50 rem) per year

Embryo/fetus of a declared pregnant Occupational worker

5 mSv (0.5 rem) over the gestation period

Individual member of the public

1 mSv (0.1 rem) per year

Individual member of the public—special circumstance

5 mSv (0.5 rem) per year

Data from Standards for Protection Against Radiation, 10 CFR §20.1201–1208, 1301. (2020).

Table 2-5 ALARA Investigational Levels3 ALARA

ALARA

Area

Investigational Level I (Per Calendar Quarter)

Investigational Level II (Per Calendar Quarter)

Whole body

1.25 mSv (125 mrem)

3.75 mSv (375 mrem)

Extremity and skin

12.5 mSv (1250 mrem)

37.5 mSv (3750 mrem)

Lens of the eye

3.75 mSv (375 mrem)

11.25 mSv (1125 mrem)

While the dose limit to the public from sources of ionizing radiation is 1 mSv (0.1 rem) per year, this limit may be increased to 5 mSv (0.5 rem) from an infrequent exposure related to another person’s medical procedure providing the authorized user determines the exposure is appropriate. Because it is impractical to set occupational worker dose limits to that of the public, regulators rely on the Linear Non-Threshold risk model to set limits at a point below which the threshold for radiation effects are known and at a point to minimize the theoretical effects from low levels of ionizing radiation exposure. The dose limits are established at levels of risk already assumed by occupational workers.15 To this point, radiation dose limits are established at a level where the risk of a fatal cancer from occupational exposure to ionizing radiation is similar to the assumed risk of a fatal work accident, which is 1 in 10,000 annually.16 In addition to ensuring dose limits are not exceeded, an institution must adopt investigational levels in order

ch02.indd 19

to maintain radiation levels consistent with the As Low As Reasonably Achievable (ALARA) philosophy. These quarterly investigational levels may be established by the institution, or the facility may adopt those recommended by the NRC. As illustrated in Table 2-5, there are two investigational levels associated with various monitoring points on the body. An ALARA Level I is a dose equal to 10% of the annual dose limit, whereas an ALARA Level II is triggered at 30% of the annual dose limit. To help ensure annual dose limits are not exceeded, these ALARA investigational levels are routinely checked on a quarterly basis.3 The NRC recommends that the following actions must be taken when an employee exceeds ALARA investigational levels:

▶▶ALARA I Exceeded The RSO or designee should investigate and review actions that might be taken to reduce a recurrence.

15-02-2022 16:37:20

20

Section 1  Fundamentals of Nuclear Cardiology

No further action is necessary unless determined otherwise by the RSO.

The RSO should perform a timely investigation into the cause, take action to reduce the recurrence, and submit a report to the institution’s Radiation Safety Committee.

nuclear cardiology are not addressed in the regulatory guide, the recommendation for most Tc-99mbased radiopharmaceuticals is to stop breast feeding for 24 hours. No interruption is recommended for N-13, O-15, and Rb-82, while a 4-hour interruption is specified for F-18 FDG. Tl-201, on the other hand, calls for a 4-day suspension of breast feeding and clinical doses mandate that written instructions be given to the mother.18

RELEASE OF PATIENTS ADMINISTERED WITH RADIOACTIVE MATERIAL

RADIATION BIOLOGY

Patients administered with radioactive material may be released into the public provided they are not likely to expose any individual to a point where their dose equivalent could exceed 5 mSv (0.5 rem).17 Furthermore, the patient or patient’s guardian must be provided with a set of instructions to maintain doses to ALARA if the total effective dose equivalent to any other individual is likely to exceed 1 mSv (0.1 rem). Breast-feeding cessation guidelines must also be provided if the dose equivalent to a nursing child could exceed 1 mSv (0.1 rem). NRC Radiation Guide 8.39, “Release of Patients Administered Radioactive Material,”18 provides guidance for the release of patients, administered activities of radiopharmaceuticals requiring instructions, and breast-feeding. Diagnostic doses of all radiopharmaceuticals used in nuclear cardiology fall well below the limit that would prohibit release of the patient. For example, the patient would have to have 28 GBq (760 mCi) of Tc-99m on-board to prevent release. While recommendations for cessation of breast feeding for the Tc-99m radiopharmaceuticals used in

The deleterious risks from ionizing radiation exposure are similar to exposure from other hazards encountered by employees in the workplace, which include both chemical and biological agents.19 Moreover, the resulting damage to cells from ionizing radiation cannot be distinguished from cellular injury due to these other harmful factors.20 The killing of cells directly and/or damage to the DNA are dependent on several variables, including the caustic agent source and strength, duration of exposure, and stage of cell growth.21,22 The effects are classified as either stochastic, where the probability of occurrence increases with dose, or deterministic, in which the severity increases with dose, after a threshold is exceeded (Fig. 2-2).23 Stochastic effects include cancer and genetic manifestations, whereas deterministic effects include reddening of the skin (erythema) and development of cataracts. The source and intensity of radiation play a particular role in assessing risks from exposure. While both stochastic and deterministic risk effects are dose dependent, meaning risk increases with radiation energy and duration of exposure, these effects

▶▶ALARA II Exceeded

Probability

Severity Stochastic effects

FIGURE 2-2  Stochastic and deterministic effects from exposure to radiation. The effects from radiation exposure are classified as either (A) stochastic—the probability for the effect to appear increases with radiation dose or (B) deterministic—the severity of the effect increases with radiation dose only after a dose threshold is exceeded.

ch02.indd 20

A

Radiation dose

Deterministic effects

B

Radiation dose

15-02-2022 16:37:21

Chapter 2  Radiation Safety and Protection in Nuclear Cardiology

cellular DNA. This may leave the cell modified yet still viable.16

are heightened by the type of radiation. X-rays and gamma rays are high-energy quanta of electromagnetic radiation characterized by short wavelengths and high frequencies that have the ability to ionize an exposed atom. The biological effectiveness, or the risk adjusted for radiation quality, for x-rays, gamma rays, and beta particles is the same (Table 2-1). On the other hand, alpha emissions are high-energy particulate radiation that deposits significantly more energy as it passes through tissue. Because of this increased transfer of energy, alpha particles bear a higher biological risk than x-rays, gamma rays, and beta particles. At equivalent doses of radiation, the associated biological risk from alpha emissions is 20 times that of x-rays, gamma rays, and beta particles. When exposed to radiation, cells can be affected directly by energy deposited into the atom/molecule or indirectly through ionization of the surrounding medium, which then interacts with the target cells. Whether directly or indirectly via free radical production in cellular water, the toxic effects from radiation exposure include the removal of an electron from DNA molecules causing single- or doublestrand breaks. Double DNA strand breaks are overly traumatic and often lead to cellular death. Although single-strand breaks frequently repair themselves correctly, incorrect or mutagenic repairs do occur. A mutagenic repair represents a change in the

Spending less time near source lessens radiation exposure

A

21

RADIATION PROTECTION PRINCIPLES Occupational workers exposed to ionizing radiation must practice radiation safety techniques on a daily basis to maintain levels of exposure below federal dose limits. To assess the levels of cumulative exposure, occupational workers are issued personal radiation monitors. When a pregnant worker declares her pregnancy status, an additional monitor is provided to be worn at the waist level for assessing the radiation dose to the fetus. Over and above the universal precautions common to the healthcare environment, there are three cardinal rules in the protection against ionizing radiation. These are time, distance, and shielding (Fig. 2-3).24

▶▶Time Radiation is emitted as a rate. A radiation source emitting photons or particles at a rate of 80 mGy/hr would expose a particular area to 80 mGy in 1 hour. Understanding this principle, occupational workers limit their time around known radiation sources, which in turn limits their cumulative exposure. In

Increasing distance from source lessens radiation exposure

B

Increased shielding lessens radiation exposure

C

FIGURE 2-3  Principles of radiation protection. From http://www.nrc.gov/about-nrc/radiation/protects-you/protection-principles.html. The three rules for protection from radiation are (A) reduce your time around a source, (B) increase your distance from the source, and (C) use appropriate shielding devices.

ch02.indd 21

15-02-2022 16:37:21

22

Section 1  Fundamentals of Nuclear Cardiology

the example given above, a worker exposed to an 80-mGy/hr source would only receive 40 mGy of exposure if his or her time around the source were limited to 30 minutes.

▶▶Distance Like a source of visible light that appears dimmer at increased distances, ionizing radiation decreases in intensity as well. The inverse square law states that the intensity of radiation decreases with the inverse of the square of the distance, or I1(d1)2 = I2(d2)2, where I = intensity and d = distance. Radiation is emitted in an isotropic manner. As distance increases, the same flux of radiation covers a larger surface area, thereby decreasing the exposure to a finite object in the path (Fig. 2-4). Continuing the previous example, a worker exposed to 40 mGy at 1 meter from a radiation source will reduce their exposure to 10 mGy at a distance of 2 meter. Example: I1 × (d1)2 = I2 × (d 2)2 40 mGy × (1 m)2 = I2 × (2 m)2 40 mGy = I2 × 4 I2 = 10 mGy

▶▶Shielding Shielding is often used to provide a protective barrier between a worker and a radiation source. The shielding material and thickness are dependent upon the

radiation source and strength. Concrete and lead of various thicknesses are commonly used in barriers to provide structural protection to occupational workers and the general public. Partial barriers, lead-lined transport containers, and lead-lined syringe shields are also used to protect radioisotope workers during the preparation and administration of radiopharmaceuticals.

PERSONNEL MONITORING Occupational radiation workers are required to be monitored for radiation exposure if they are likely to enter a high or very high radiation area, are likely to exceed 10% of the annual dose limits, or are a declared pregnant woman likely to receive an annual dose in excess of 1 mSv (0.1 rem).25 There are several types of personnel monitors used in nuclear cardiology facilities. These include the film badge, a thermo luminescent dosimeter (TLD), and an optically stimulated luminescence (OSL) device. The NRC requires evaluation of these devices by a laboratory that is accredited by the National Voluntary Laboratory Accreditation Program. Film badges utilize a strip of photographic film contained in a plastic holder. Exposure to radiation will increase the darkening in the developed film. The degree of darkening is proportional to the radiation exposure received. Incorporated into the badge holder are filters such as lead, copper, aluminum, and plastic. The degree of darkening behind each filter helps determine the quality of the radiation received, such as highenergy photons, low-energy photons, and particulate radiation. Film badges are commonly used to measure

I1(d1)2 = I2(d2)2

FIGURE 2-4  The Inverse Square Law. Radiation intensity is reduced with the inverse of the square of the distance from a source. A radiation source exposing material to 80 mGy/hr at 1 meter results in only 20 mGy/hr at a distance of 2 meter. (80) / (2)2 = 20 mGy/hr = 10 mGy per 30 minutes.

ch02.indd 22

Distance Exposure

1 meter 80 mGy/hr

2 meters 20 mGy/hr

15-02-2022 16:37:22

Chapter 2  Radiation Safety and Protection in Nuclear Cardiology

whole-body irradiation and are comparatively inexpensive. However, they are affected by heat, moisture, and sunlight and must be utilized with care. TLDs use inorganic crystals such as lithium fluoride to measure radiation exposure. When exposed to ionizing radiation, excitation of atoms causes electrons to elevate from their valence band to become trapped in the forbidden band. When heated with high temperatures (annealing), the electrons are released back into the valence band emitting light proportional to the radiation received. By returning to their stable state, the annealing process allows the TLD to be re-used. TLDs do not have the ability to distinguish radiation quality and are commonly used as extremity monitors, such as the ring badge. OSL combines the benefits of film badge and TLD technology. An aluminum oxide crystalline structure is manufactured into a thin strip and placed between a filter pack. Irradiated atoms in the structure liberate electrons that become trapped, and these electrons release light when stimulated by a laser. The release of light is proportional to the radiation received. Similar to that in film badge technology, plastic and metal filters are used to assess the incident radiation quality. OSL devices are commonly used for whole-body radiation monitoring. The devices are sealed in a blister pack and are unaffected by light, heat, and moisture.

MEDICAL EVENTS Once known as misadministrations, medical events are defined by the NRC in Part 35 of the CFR. The NRC must be notified by telephone of a reportable medical event no later than the next calendar day after occurrence. The telephone notification must be followed by a written report within 15 days. Pertinent to nuclear cardiology, the following events must be reported to the NRC if they result in a dose equivalent to the patient that exceeds 0.05 Sv (5 rem) or a dose to the skin or an organ that exceeds 0.5 Sv (50 rem). Events meeting these definitions but not exceeding the reportable dose limits need only be documented and reported to the facility’s administrators. a. An administration of a dose that is greater than 20% difference from the prescribed dose b. An administration of the wrong radiopharma­ ceutical

ch02.indd 23

23

c. An administration of a radiopharmaceutical to the wrong person d. An administration of a radiopharmaceutical by the wrong route (i.e., oral vs. intravenous)

POSTINGS There are several postings required in a nuclear cardiology laboratory. These include the “Notice To Employees” (NRC Form 3) and appropriate radiation caution signage.26–28 NRC Form 3 outlines employee/ employer responsibilities for protection against radiation and is required to be posted in a conspicuous manner and replaced if defaced or altered.29 This form should be posted in an area frequented by workers either on their way to or from the area where radiation is used or stored. This form is commonly found in employee lounges and entrance doors to radiation areas. A “Caution Radioactive Materials” sign (Fig. 2-5) is required to be posted in any area where radioactive materials are used or stored that exceed 10 times the quantity listed in Appendix C of Title 10, Part 20 of the Code of Federal Regulation. This equates to 1 mCi of 57Co, 0.1 mCi of 137Cs, and 10 mCi of 99mTc and 201 Tl. A “Caution Radiation Area” sign is required to be posted in each area where radiation levels could result in an individual receiving 0.05 mSv (5 mrem) in 1 hour at 30 cm from a radiation source. Caution signs are commonly located on the entrance doors to the hot lab, imaging room, and injection areas such as the treadmill room. If a radioactive material is used in a room for less than 8 hours per day and is under constant supervision, the area is not required to be posted with the caution sign. Furthermore, a

FIGURE 2-5  Radiation caution signs found in nuclear cardiology laboratories.

15-02-2022 16:37:23

24

Section 1  Fundamentals of Nuclear Cardiology

radiation area sign is not needed if the source of the exposure is from a patient meeting the criteria to be released into the public.

AREA SURVEYS AND SPILL PROCEDURES Ambient radiation level surveys must be performed at the end of each working day in areas where radioactive materials are used and stored. The most common instrument for performing ambient radiation surveys in the Nuclear Cardiology department is the Geiger–Müller (GM) meter, a gas-filled detector that produces and collects ion pairs when ionizing radiation passes through the detector and deposits energy. The NRC recommends an ambient exposure trigger limit of 5 mR/hr in restricted areas and 0.1 mR/hr in unrestricted areas.3 Removable contamination surveys (wipe tests) should be performed weekly using an appropriate gamma-counting device such as a sodium iodide (NaI) well counter. While Appendix R of NRC NUREG 1556 Volume 93 suggests an action level of 20,000 disintegrations per minute (dpm)/100 cm2 for 99mTc and 201Tl, many nuclear cardiology facilities use the 2000 dpm/100 cm2 action level required for other isotopes common in general nuclear medicine departments. It is also important to verify ambient radiation and contamination limits in your individual area of practice as many states have more restrictive requirements. When ambient radiation levels or contamination surveys exceed limits, a radiation spill has occurred requiring action. The NRC recommends establishing an activity threshold for a major and minor spill and provides guidance in Appendix N of NRC NUREG 1556 Volume 9.3 The suggested procedures are as follows: Minor Spill (less than 100 mCi of 99mTc/201Tl) 1 . Notify persons in the area that a spill has occurred. 2. Prevent the spread of contamination by covering the spill with absorbent paper. 3. Wear gloves and protective clothing and clean up the spill using absorbent paper. 4. Survey the area with appropriate radiation detection instruments. 5. Report the incident to the RSO. Major Spill (greater than 100 mCi of

99m

201

Tc/ Tl)

1. Clear the area. Notify all persons not involved in the spill to vacate the room.

ch02.indd 24

2. Prevent the spread of contamination by covering the spill with absorbent paper labeled “caution radioactive material,” but do not attempt to clean it up. 3. Shield the source if possible. Do this only if it can be done without further contamination or a significant increase in radiation exposure. 4. Close the room and lock or otherwise secure the area to prevent entry. 5. Notify the RSO immediately. 6. Decontaminate personnel by removing contaminated clothing and flushing contaminated skin with lukewarm water, and then washing with mild soap. If contamination remains, the RSO may consider inducing perspiration. Then wash the affected area again to remove any contamination that was released by the perspiration.

WASTE DISPOSAL Radioactive waste generated in the Nuclear Cardio­ logy department may be disposed by various methods. The two most common are decay-in-storage and return to an authorized recipient. Decay-in-storage is allowed for isotopes with a half-life of 120 days or less.30 Storage of waste for decay requires that the waste to be held for a minimum of 10 half-lives in a shielded container and prior to final disposal the waste must be surveyed to ensure it is consistent with background radiation. All radioactive labels must be removed or defaced, and a record of the waste disposal must be maintained for 3 years. A summary of the minimum length of time radioactive waste must be held in storage for isotopes common to nuclear cardiology is summarized in Table 2-6.

Table 2-6 Minimum Time Required for Decay-in-Storage Isotope

Half-Life

Minimum Time in Storage (10 half-lives)

99m

Tc

6.01 hours

60.1 hours

201

Tl

72.9 hours

30.4 days

18

F

1.83 hours

18.3 hours

13

9.96 minutes

1.7 hours

82

1.3 minute

13 minutes

N R

15-02-2022 16:37:23

Chapter 2  Radiation Safety and Protection in Nuclear Cardiology

25

FIGURE 2-6  DOT labels for shipping radioactive packages. From left to right: The Radioactive I “White,” Radioactive II “Yellow,” Radioactive III “Yellow,” and the Excepted Package Limited Quantity.

Radioactive waste, including used and unused doses, may also be transferred to an authorized recipient. Transfer of the waste requires verification that the recipient is licensed to receive the specified radioactive material. Furthermore, when transferring this radioactive material, the facility must follow Department of Transportation (DOT) shipping guidelines. A summary of these requirements is outlined in the “Shipping of Radioactive Packages” section.

RECEIVING Receipt of radioactive packages is outlined in Title 10, Part 20.1906 of the CFR. Radioactive packages must be monitored within 3 hours of delivery if received during normal working hours and no later than 3 hours from the beginning of the next working day if received after normal hours. Proper monitoring requires surveying the package for external radiation exposure and wipe testing for removable contamination. Radiation surveys must be less than 10 mR/hr at 1 meter and less than 200 mR/hr at the package surface. Contamination wipe testing must be performed if there is suspicion that the package is damaged or the integrity is degraded in any manner. The wipe test results must be less than 6600 dpm/300 cm2. Should the wipe test or survey results exceed limits, the delivery carrier and the NRC must be notified immediately.31

SHIPPING OF RADIOACTIVE PACKAGES In addition to receiving radioactive materials, the Nuclear Cardiology department must routinely ship radioactive materials. These shipments include the return of spent and unused patient radiopharmaceutical doses, as well as the return of sealed calibration

ch02.indd 25

sources. Shipping of radioactive materials is regulated by the DOT under Title 49 of the CFR. There are four DOT labels used for shipping radioactive materials (Fig. 2-6). The use of each label is determined by the quantity of material in the package and the radiation exposure at the surface and at 1 meter from the package. The most common radioactive package shipped from the Nuclear Cardiology department is the “UN2910—Excepted Package, Limited Quantity.” For a package to ship under the specifications of UN2910, the maximum quantity in the package cannot exceed the limits derived from the table in 49CFR173.425. Furthermore, the following criteria must be met: a. The surface of the package must not exceed 0.5 mR/hr. b. A wipe test of the package must not exceed 6600 dpm/300 cm2. The limited quantity package limits for common isotopes derived from 49CFR173.425 are illustrated in Table 2-7. The limits apply to radioactive material Table 2-7 “Excepted Package Limited Quantity” Limits for Common Isotopes Used in Nuclear Cardiology Isotope

Form

Limit

99m

Tc

Liquid

11 mCi

201

Tl

Liquid

11 mCi

18

F

Liquid

1.6 mCi

57

Liquid

27 mCi

Co

137

Cs

Liquid

1.6 mCi

68

Ge

Solid

14 mCi

15-02-2022 16:37:24

26

Section 1  Fundamentals of Nuclear Cardiology

Table 2-8 DOT Label Radiation Exposure Limits DOT Label

Surface Exposure (mR/hr)

1 M Exposure (mR/hr)

Wipe Test Limit

Excepted Package Limited Quantity

≤0.5

Background

6600 dpm/300 cm2

White I

≤0.5

Background

6600 dpm/300 cm2

Yellow II

>0.5 ≤50

1.0

6600 dpm/300 cm2

Yellow III

>50 ≤200

10

6600 dpm/300 cm2

a

a

The Excepted Package Limited Quantity label conforms to White I limits and the contents of the package meet the specifications of 49CFR173.425.

in both solid and liquid forms. For packages containing more than one isotope, the lower limit applies to the entire contents of the package. A radioactive package that exceeds the excepted package quantities has a surface exposure of less than 0.5 mR/hr, an exposure at 1 meter that is indistinguishable from background, and a wipe test of less than 6600 dpm/300 cm2 receives a Radioactive I “White” label. A Radioactive II “Yellow” label is used for packages with a surface exposure of 0.5 to 50 mR/hr, a 1-meter exposure of less than 1 mR/hr, and a wipe test of less than 6600 dpm/300 cm2. The Radioactive II label is commonly used when shipping calibration sources back to the vendor for disposal. Packages with a surface exposure of 50 to 200 mR/hr, a 1-meter exposure of less than 10 mR/hr, and a wipe test of less than 6600 dpm/300 cm2 must be labeled with a Radioactive III “Yellow” label. Radioactive labels must be placed onto two sides of the package and packages requiring the Radioactive I, II, or III label must be classified as a “Type A” shipping container. A “Type A” shipping container is a strong package that has met certain criteria including a water spray test, drop test, compression test, and a penetration test. A summary of the radiation exposure limits for each DOT label is illustrated in Table 2-8.

SAFE USE OF RADIOACTIVE MATERIAL The NRC requires licensees to develop a policy for the safe use of radioactive materials. Appendix T of

ch02.indd 26

NRC NUREG 1556 Volume 9 Revision 3 provides a sample policy that facilities may choose to adopt in lieu of developing their own.3 The NRC policy for the safe use of radioactive materials provides a basis for establishing a safe working environment. The model policy covers proper protective clothing, shielding and handling requirements, safe storage procedures, procedures for area monitoring, and, most importantly, patient safety.

REFERENCES 1. The Health Physics Society. 2014. http://hps.org/publicinfor mation/ate/faqs/whatisradiation.html. Accessed August 23, 2020. 2. United States Nuclear Regulatory Commission. 2014. http:// www.nrc.gov/reading-rm/basic-ref/glossary.html. Accessed August 23, 2020. 3. Burkhart M, Cockerham A, Cook J, et al. Consolidated Guidance About Materials Licenses Program-Specific Guidance About Medical Use Licenses. US NRC NUREG-1556, volume 9 revision 3) 2019. 4. Baldwin JA, Bag AK, White SL, Palot-Manzil FF, O’Malley JP. AJR Am J Roentgenol. 2015;205(2):251–258. 5. Cherry SR, Sorenson JA, Phelps ME. Radiation safety and health physics. In: Cherry SR, Sorenson JA, Phelps ME, eds. Physics in Nuclear Medicine. 4th ed. Philadelphia, PA: Saunders/Elsevier Science; 6. Standards for Protection Against Radiation, 10 CFR § 20.1004 (2018). 7. United States Nuclear Regulatory Commission. 2017. http:// www.nrc.gov/about-nrc/radiation/around-us/sources/ nat-bg-sources.html. Accessed August 23, 2020. 8. United States Nuclear Regulatory Commission. 2017. http:// www.nrc.gov/about-nrc/radiation/around-us/sources.html. Accessed August 23, 2020. 9. United States Nuclear Regulatory Commission. 2017. http:// www.nrc.gov/about-nrc/radiation/around-us/sources/manmade-sources.html. Accessed August 23, 2020.

15-02-2022 16:37:24

Chapter 2  Radiation Safety and Protection in Nuclear Cardiology 10. Peck D, Samei E. How to Understand and Communicate Radiation Risk. Image Wisely, 2017. https://www.imagewisely. org/-/media/Image-Wisely/Files/CT/IW-Peck-Samei-Radia tion-Risk.pdf. Accessed August 23, 2020. 11. Cameron JR. Are X-rays safe? http://www.angelfire.com/mo/ radioadaptive/jcameron1.html. Accessed August 23, 2020. 12. Nickoloff EL, Lu ZF, Dutta AK, So JC. Radiation dose descriptors: BERT, COD, DAP, and other strange creatures. Radiographics. 2008;28(5):1439–1450. 13. Dorbala S, Blankstein R, Skali H, et al. Approaches to reducing radiation dose from radionuclide myocardial perfusion imaging. J Nucl Med. 2015;56(4):592–599. 14. Cerqueira MD, Allman KC, Ficaro EP, et al. Recommendations for reducing radiation exposure in myocardial perfusion imaging. J Nucl Cardiol. 2010;17(4):709–718. 15. Kase KR. Radiation protection principles of NCRP. Health Phys. 2004;87(3):251–257. 16. NCRP 116. 1993. National Council on Radiation Protec tion and Measurements. Limitation of Exposure to Ionizing Radiation, NCRP Report No. 116. http://app.knovel.com/ hotlink/toc/id:kpLEIRRN04/limitation-exposure-ionizing/ limitation-exposure-ionizing. Accessed November 26, 2014. 17. Standards for Protection Against Radiation, 10 CFR § 35.75 (2004). 18. U.S. Nuclear Regulatory Commission. Release of patients administered radioactive materials. Washington, DC: U.S. Nuclear Regulatory Commission; Regulatory Guide 8.39 Revision 1; 2020. 19. Salihu HM, Myers J, August EM. Pregnancy in the workplace. Occup Med (Lond). 2012;62(2):88–97.

ch02.indd 27

27

20. Travis EL. Basic biologic interactions of radiation. In: Kelly KM, ed. Primer of Medical Radiobiology. St. Louis, MO: Mosby-Yearbook, Inc.; 1989:25–44. 21. Bushberg JT, Boone JM. Radiation biology. In: Mitchell CW, ed. The Essential Physics of Medical Imaging. Baltimore, MD: Lippincott Williams & Wilkins; 2011:600, 751–836. 22. Valentin J. ICRP publication 84: Pregnancy and medical radiation. Ann ICRP. 2000;30, 7–67. 23. Adriaens I, Smitz J, Jacquet P. The current knowledge on radiosensitivity of ovarian follicle development stages. Hum Reprod Update. 2009;15(3):359–377. 24. United States Nuclear Regulatory Commission. 2017. http:// www.nrc.gov/about-nrc/radiation/protects-you/protectionprinciples.html. Accessed August 23, 2020. 25. Standards for Protection Against Radiation, 10 CFR § 20.1502 (2018). 26. Standards for Protection Against Radiation, 10 CFR § 19.11 (2018). 27. Standards for Protection Against Radiation, 10 CFR § 20.1901 (2018). 28. Standards for Protection Against Radiation, 10 CFR § 20.1902 (2018). 29. USNRC Notice to Employees. 2017. https://www.nrc.gov/ re a d i ng - r m / d o c - c ol l e c t i ons / for ms / n rc 3 i n fo. ht m l. Accessed August 23, 2020 30. Standards for Protection Against Radiation, 10 CFR § 35.92 (2017). 31. Standards for Protection Against Radiation, 10 CFR § 20.1906 (2018).

15-02-2022 16:37:24

This page intentionally left blank

READ_THINK_ITALIAN.indd 2

11/19/21 1:12 PM

Radiopharmaceuticals for Cardiovascular Imaging James A. Case and Gary V. Heller

KEY POINTS ■■

■■

■■

■■

Ideal tracer kinetics is important in understanding the strengths and limitations of current and future tracers. Thallium-201 has excellent tracer kinetics but is limited by low-energy emission level and long half-life, and therefore is not recommended for routine clinical use. Technetium-based tracers offer improved image quality due to higher emission level and shorter half-life and are the radiopharmaceuticals of choice for single-photon emission computerized tomographic nuclear cardiology. The most common positron emission tomography tracer, 82Rb, is generator-derived radiopharmaceutical, with excellent blood flow characteristics and low radiation, making it suitable for most clinical practices.

INTRODUCTION Myocardial perfusion scintigraphy has significantly evolved since its introduction more than four decades ago. In 1972, the only radiotracer available for nuclear cardiology was 201Tl.1–3 Although this agent had impressive characteristics for the assessment of myocardial blood flow, its long half-life (72 hours) and the multiple emission energy photons (numerous Auger

ch03.indd 29

CHAPTER

3

photons center at 72 keV and photons at 137 keV and 167 keV) made scatter and attenuation correction difficult.4,5 By the early 1990s, investigations were underway with using 99mTc.6–12 This isotope has the advantage of a single-emission photon at 141 keV and a more ideal half-life of 6 hours. Thus image quality would be improved and radiation exposure reduced over thallium.12 This agent could be prepared at a centralized radiopharmacy and delivered for use to nearby nuclear cardiology laboratories. Alternatively, 99mTc could be produced from a molybdenum generator located at the radiopharmacy. 99mTc sestamibi, 99mTc tetrofosmin, and 99mTc teboroxime were subsequently developed to assess myocardial blood flow.13,14 Sestamibi and tetrofosmin are still in widespread use today, while teboroxime is not, due to clinical limitations. The development of technetiumbased tracers for nuclear cardiology led to a more widespread use of myocardial perfusion imaging (MPI) that has continued through the present time. Positron emission tomography (PET) radiotracers were also undergoing revolutionary changes. Positron emitting isotopes of oxygen, carbon, nitrogen, and fluorine could all be used for producing metabolically useful molecules. In 1996, Medicare approved reimbursement for glucose analog, 18Ffluorodeoxyglucose (FDG), for the assessment of metabolically active tumors. This stimulated the development of cyclotron networks for producing the 18F and marked improvements in PET scanners opened the door to high-quality PET MPI using a generator-based potassium analog, 82Rb.15–18 This

15-02-2022 13:51:55

30

Section 1  Fundamentals of Nuclear Cardiology

allowed for on-site production of a short-lived positron emission agent without the need for an expensive cyclotron. This chapter will review important characteristics of the various single-photon emission computerized tomographic (SPECT) and PET MPI radioactive agents used in clinical practice, with an increasing focus on quantitative myocardial blood flow measurement. The emphasis of this chapter is to provide the nuclear cardiologists with practical knowledge of these agents and a high-level understanding of the direction of radiopharmaceutical development.

IDEAL PHYSIOLOGIC CHARACTERISTICS OF PERFUSION IMAGING AGENTS The commercially available radionuclides for MPI have advantages and disadvantages. To understand these advantages and dispensers, it is important to enumerate the characteristics of an “ideal” myocardial blood flow tracer:19–21

• Myocardial uptake of an ideal perfusion radio-



• •

• • • •

ch03.indd 30

tracer should be proportional to the myocardial blood supply over a wide range of myocardial blood flow values including both exercise and pharmacologic stress. Uptake of the radiotracer should allow for delineation of regional changes in uptake and sufficient signal-to-noise to make confident image interpretation decisions. Uptake in noncardiac structures should not interfere with the reconstruction and interpretation of the myocardium. Uptake of the radiotracer should be rapid enough to allow complete absorption of the radiotracer within the peak vasodilatation timing window of either exercise and/or pharmacologic stress. Uptake of the radiotracer should be stable throughout the course of perfusion imaging acquisition. Rest/stress patient radiation doses should be less than 10 mSv while maintaining a high-count highresolution tomographic images of the heart. The complete rest stress protocol should be completed in a single visit to the nuclear laboratory for the majority of patients. The radiotracer should be either sufficiently stable to allow for same-day delivery from the

radiopharmacy to the nuclear laboratory or its production would be sufficiently simple to allow on-site production.

KINETIC MODELING FOR MYOCARDIAL BLOOD FLOW A recent development of nuclear cardiology has been the ability to quantitate myocardial blood flow in conjunction with perfusion imaging. This has added considerably to the diagnostic accuracy, as well as risk stratification in nuclear imaging. Myocardial blood flow assessment is becoming commonplace in clinical cardiac PET laboratories, and work is continuing to make it practical at least for some SPECT systems (Chapter 11). Calculating myocardial blood flow from measurements of radiotracer uptake and blood pool concentration requires a model for tracer transport into myocytes. The most common model for tracer transport is a multi-compartment model. Each compartment of the model represents a location the tracer can be in during the acquisition. In the simplest model, the tracer would begin in the blood (compartment one) and then be transported into the cell (compartment two) (Fig. 3-1). More complicated models would include parameters for washout from the myocyte, compartments for the interstitial layer, metabolism of the tracer, and different subcomponents within the cell. The choice of a compartmental model for calculating myocardial blood flow is based on the underlying kinetics of the tracer and the parameters of the acquisition. In addition, the choice of a compartmental model should take into consideration common imaging artifacts, such as image noise, patient motion, and noncardiac uptake. Most myocardial perfusion tracers are modeled accurately using a two-tissue compartment model (2TCM), where one compartment represents the blood and the two-tissue compartments represent the cell, an interstitial layer, or subcomponent in the cell:22 Cm =

t K1k3 K1k2 −(k2 + k3)t ⊗ Ca(t) + C (τ)dτ e ∫ k2 + k3 k2 + k3 0 a

where Cm is the myocardial uptake, K1 is the uptake into the first tissue compartment, k2 is the washout

15-02-2022 13:52:05

Chapter 3  Radiopharmaceuticals for Cardiovascular Imaging

Table 3-1

Cmyo

Blood Flow Model Parameters For Common Myocardial Perfusion Tracers

K1

Blood A Tissue

Cmyo

Blood

K1

k2

B Tissue2

Cmyo2 k3

Tissue1 Blood

k2

C

FIGURE 3-1  Tracer kinetic models most commonly used in quantitative myocardial blood flow measurement. These are: (top, A) the net retention, (middle, B) single tissue compartment model, and (bottom, C) two tissue compartment model. Please refer to text for discussion.

from the first compartment, k3 is the uptake into the second tissue compartment, t is the timepoint the uptake is measured, and Ca(t) is the arterial concentration at timepoint. To calculate the blood blow from the compartmental model, a model for the tracer extraction is needed. The tracer extraction from the blood is often modeled using the Renkin−Crone formula,23–25 shown in Table 3-1. A plot of blood flow versus uptake is found in Figure 3-2. A very important aspect of tracer kinetics is the linearity in relation to increases in myocardial blood flow. An ideal tracer would match perfectly following the blood flow during increases to accurately reflect changes, as in microsphere data. This includes during exercise stress, which is generally two to four times resting blood flow, and pharmacologic stress, which is generally three to six times resting blood flow. If a given tracer does not follow increases in blood flow, the consequences are reduced diagnostic accuracy and lesser ability to distinguish single-vessel

ch03.indd 31

B

Rb-82

0.77

0.63

N-13 Ammonia

0.096

1.08

C-11 Acetate

0.64

1.2

F-18 Flurpiridaz

0.07

1.6

Tc-99m Sestamibi

0.87

0.44

Tc-99m Tetrofosmin

0.91

0.51

Tl-201

0.9

0.91

Note: These values are used in the Renkin−Crone model for determining the extraction fraction at particular flow rates in Columns A and B.

Cmyo1 K1

A

ischemia from multivessel ischemia. First-pass flow characteristics of common SPECT and PET tracers are illustrated in Figure 3-2. The ideal linearity is illustrated by microspheres, and the closest tracer to this is 0−15 Water, a PET tracer rarely used clinically. Of note, both common PET tracers have (rubidium and ammonia) better linearity than either SPECT tracers of sestamibi and tetrofosmin, which may partially explain their higher diagnostic sensitivity. Early studies of the 2TCM for 13N-ammonia demonstrated a high degree of accuracy compared to

Microspheres 0-15 Water

Myocardial Radiotracer Uptake

Tissue

31

F-18 Flurpiridaz

N-13 Ammonia Thallium-201 Rubidium-82 Tc-99m Sestamibi Tc-99m Tetrofosmin

Myocardial Blood Flow (mL/min/g)

FIGURE 3-2  Comparison of radiotracer uptake in relation to myocardial blood flow increases. These data are estimated curves based upon multiple literature. Note “plateau effect” in many tracers as blood flow increases.

15-02-2022 13:52:06

32

Section 1  Fundamentals of Nuclear Cardiology

microspheres.26 82Rb is a potassium analog and thus is not metabolized by the myocytes. Because of this, the flow model can be simplified using a single-tissue compartment model (1TCM).27–29 1TCM has been shown to accurately represent the estimate of myocardial blood flow with 82Rb and is highly repeatable.30 This model can be further simplified by setting k2 = 0. This model is referred to as the net retention model.31 Comparison of the net retention model with the 1TCM demonstrates a high degree of correlation between blood flow calculations in the first 2 minutes of a study. Following the first 2 minutes, the influence of the washout reduces the accuracy of this model. One of the advantages of this simple model is that it can be performed on nonlist mode, frame mode PET systems. Another advantage of this technique is that the myocardial uptake is calculated from a single late frame making it less susceptible to motion artifacts. One of the limitations of this technique is that partial volume correction cannot be determined from the data and must be supplied as a parameter. In a large study comparing myocardial blood flow calculation techniques with 82Rb, net retention, 1TCM and 2TCM models produced similar results.32,33 This result confirms the overall robustness of these methods. In addition, software packages using the same compartmental modeling algorithm produced interchangeable results. This further validates the consistency of these software packages, despite varying implementation strategies. The key limitation of this study was that no gold standard was available and therefore the overall accuracy of particular techniques could not be assessed relative to true myocardial blood flow.

SPECT RADIOPHARMACEUTICAL PERFUSION IMAGING AGENTS Information on basic properties of radionuclide SPECT MPI agents is generally obtained from cultured myocardial cells, isolated perfused hearts, or in vivo animal models.5 Precise measurements of cellular or capillary-tissue tracer kinetics are usually obtained from cell cultures and isolated perfused heart models, whereas regional tracer distribution and uptake in other organs are studied with in vivo animal models. The two most important physiologic

ch03.indd 32

factors that affect the myocardial uptake of an MPI agent are the variations in regional myocardial blood flow and the myocardial extraction of the radiotracer. This can be modeled as a function of three parameters: maximum fractional tissue extraction of the diffusible agent (Emax), capillary permeability surface area product (PScap), and the net extraction (Enet). These three major parameters can be determined using indicator−dilution techniques and radiolabeled albumin used as an intravascular reference. The difference between the intravascular albumin reference and the radiopharmaceutical that is evaluated (i.e., a diffusible perfusion agent) on a venous dilution curve is used to calculate its instantaneous cardiac extraction. The early peak of the curve, or Emax, represents the maximum fractional tissue extraction of the diffusible agent. This value is used to calculate the PScap. The Enet is the integral of the curve and is used as a measure of myocardial radiotracer retention, including both initial extraction and subsequent back-diffusion. A high value for Emax and PScap indicates a rapid blood−tissue exchange and suggests that the diffusible radiotracer will be able to assess high levels of hyperemic flow accurately. Currently, there are three commercially available myocardial perfusion SPECT tracers: (1) 201Tl thallium, (2) 99mTc tetrofosmin, and (3) 99mTc sestamibi. Initial approval for thallous chloride in 1979 provided for up to 19 mSv to be administered, as indicated for the detection of myocardial infarction.34,35 This indication was expanded to include patients for the detection of ischemia using exercise and pharmacologic stress.36–38 The approval of 99mTc-labeled sestamibi in December 1990 permitted a two-fold reduction in radiation exposure, based on the published package insert value of 9.0 mSv (two injections of 1110 MBq, MIBI insert, 8.8 mSv).39 The characteristics of SPECT MPI agents are summarized in Table 3-2.

▶▶Thallium-201 201

Tl thallium has the highest extraction fraction at high coronary blood flow rates (Fig. 3-2) amongst the three SPECT myocardial perfusion tracers commercially available today. Despite 201Tl’s high extraction fraction, it is not an ideal blood flow tracer because its long physical half-life (73.1 hours) results in high patient radiation exposure,40,41 even at low

15-02-2022 13:52:06

Chapter 3  Radiopharmaceuticals for Cardiovascular Imaging

33

Table 3-2 Comparison of Important Characteristics of SPECT Radiopharmaceuticals Tl–201

Tc-99m sestamibi

Tc-99m-Tetrofosmin

Class

Element

Isonitrile

Diphosphine

Energy level (keV)

primary 68

140

140

Emax

0.73 ± 0.10

0.38 ± 0.09

0.32 ± 0.07

Enet

0.57 ± 0.13

0.41 ± 0.15

0.23

Half-life

72 hours

6 hours

6 hours

Myocardial redistribution

Yes

No

No

Myocardial uptake (%ID)

3.0–4.0

1.4 ± 0.3

1.0–1.2

IV imaging interval

5–10 minutes

15–60 minutes

15 minutes

Target organ

Kidney

Upper large intestine

Gallbladder wall

doses (3−5 mCi). In addition to the high radiation exposure, 201Tl is also difficult imaging because of multienergy emission.42 The majority photons from thallium-201 are emitted through the process of electron capture. This process occurs when inner shell electrons are picked off by the nucleus leaving a vacancy in the lower energy shells of the atom. To compensate for this, most electrons quickly fill the inner vacancies causing a cascade of x-rays to be amended. For thallium-201, the majority of these photons are emitted near with some as low as 69 keV up to 81 keV. In addition to these characteristic x-rays, thallium-201 can also emit gamma rays at 135 keV and 167 keV (Fig. 3-3). The broad spectrum

of energy emitted by the decay of thallium-201 results in higher scatter contamination in the photopeak range. This degrades the spatial and energy resolution of these images. This combined with the lower recommended injected dose of 3 mCi makes imaging with thallium-201 challenging. Primarily because of concerns for radiation exposure (20−25 mSv), the routine use of thallium-201 as a routine perfusion agent is not recommended by the American Society of Nuclear Cardiology.43 Thallium-201 is an efficient myocardial blood flow tracer because it utilizes the potassium pump to allow for passive diffusion across the cell membrane. 201 Tl is typically delivered as thallus chloride in

18000 16000

Gamma Camera Resolution

Intensity

14000 12000

ch03.indd 33

High Energy Resolution

10000 8000 6000 4000 2000 0 –2000 0

50

150 100 Photon Energy (keV)

200

FIGURE 3-3  Photon peaks of thallium-201 at low resolution, the characteristic x-rays of 201Tl cluster into a single photo-peak at 72 keV and at higherenergy resolution, they can be split out into their component parts.

15-02-2022 13:52:06

34

Section 1  Fundamentals of Nuclear Cardiology

solution. Because thallium-201 does not have complex chemistry, it is easily produced at higher-energy cyclotron facilities (30 MeV and higher).44,45 The myocardial extraction fraction of Tl-201 in in vivo model is approximately 87% at resting flow rates.2 However, at high flow rates that may be present using pharmacologic stress (>2.0 mL/min/g), there is a diffusion limitation and a fall in tracer extraction fraction. Thallium-201 is not bound within the cell and therefore is free to diffuse back into the blood. This characteristic has advantages and disadvantages. The major drawback of diffusion of thallium-201 is that the imaging must take place within 30 minutes following tracer injection and begin within 10 minutes. In the event of camera problems or patient issues, the radiotracer can washout from the myocardium reducing the apparent severity of any perfusion defects. This characteristic can be taken advantage of for the assessment of myocardial viability. The redistribution or the filling in of a myocardial perfusion defect generally occurs between 3 and 5 hours after the injection of Tl-201 at peak stress.2,46,47 Myocardial viability can be assessed either with a small 1-mCi injection within 1 hour prior to redistribution imaging or imaging the following day 24 hours after the initial injection of thallium. Both approaches have been demonstrated to identify viable myocardium. The clinical strengths and weaknesses of thallium-201 are summarized in Table 3-3. The strengths include high linearity with myocardia blood flow and minimal uptake in the liver and gut. However, the peak energy for this tracer is low at 68 keV, resulting in suboptimal imaging quality, particularly in obese patients. Unfortunately, the half-life of 72 hours is a major deterrent to improving image quality by higher doses. In fact, the current recommended dose of 3 to 5 mCi in clinical practice results in higher radiation exposure using technetium tracers (25 mSv compared to 10−15 mSv), primarily because of concerns for radiation exposure (20−25 mSv) and overall image quality. Despite the shortcomings, there continues to be interest in developing myocardial blood flow protocols for 201Tl.48,49 High-sensitivity and high-resolution cadmium zinc telluride (CZT) SPECT imaging systems could make thallium-201 imaging for myocardial blood flow feasible in the future.49

ch03.indd 34

Table 3-3 Strengths and Weakness of 201-Thallium Strengths • Very good linearity with blood flow through pharmacologic stress • Unit dose, available clinically • Virtually no liver, gut activity • Moderate diagnostic accuracy • Validated for evaluation of hibernating myocardium Weaknesses • Very long half-life necessitating a low tracer dose clinically • Suboptimal image quality, especially in obese • Higher patient radiation exposure compared with technetium • Not recommended for routine clinical use

In summary, Tl-201 was the first cardiac nuclear tracer developed with several admirable characteristics, such as relatively high plateau effect and extraction fraction. It gave nuclear cardiology its start and was the mainstay for over 20 years. It has a long halflife of 72 hours, excellent linearity, and minimal gut and liver activity during imaging. However, the low energy window for acquisition yielding suboptimal images and the long half-life leading to unacceptably high radiation exposure to the patient have led to a steady decline in its use in comparison to the technetium-based agents. Viability assessment is still considered an important use for thallium.

▶▶Tc-99m Perfusion Agents More than 30 years ago, radiopharmaceutical perfusion agents were developed labeled with Tc-99m to circumvent the physical limitations of Tl-201. The two most widely used agents for myocardial perfusion SPECT imaging are 99mTc sestamibi and 99mTc tetrofosmin. These agents have generally favorable imaging characteristics, relatively low radiation exposure, and can be easily supplied to most centers around the world. Another advantage of these technetium imaging agents is that they are nearly completely retained in the cell, thereby minimizing the need to model tracer washout, in contrast to 201Tl.50–52 The greatest

15-02-2022 13:52:07

Chapter 3  Radiopharmaceuticals for Cardiovascular Imaging

drawback to using these agents for the absolute quantitation of myocardial blood flow is their relatively low extraction fraction at higher myocardial blood flow rates (Fig. 3-2). Another challenge with imaging these agents is a high hepatic uptake early in the acquisition. This can make quantitation and visual interpretation of the radiotracer more difficult due to photons scatter and ramp filter artifacts. These tracers have minimal redistribution and therefore two injections are necessary.53 The potential advantages of a Tc-99m−labeled agent over Tl-201 are significant and include the following:

• The 140-keV photon energy of Tc-99m, which is optimal for NaI-based gamma camera imaging.

35

Table 3-4 Strengths and Weaknesses Tc-99m Imaging agents Strengths • Better image quality than thallium-201 • Unit dose, clinical availability • Moderate diagnostic accuracy • Lower radiation exposure than Tl-201 Weaknesses • Half-life beyond image necessity • Suboptimal image quality with considerable gut, liver activity • Long rest−stress protocol requiring 2–4 hours • Considerable attenuation artifact resulting in frequent false-positive results

• The much shorter physical half-life of Tc-99m

(6 hours vs. 73 hours for Tl-201) and the better radiation dosimetry permit the administration of a higher dose of a Tc-99m−labeled compound than Tl-201 with lower radiation exposure. • The higher counts in Tc-99m images allows for the perfusion images to be ECG gated in either 8- or 16-gated frame to allow for high temporal resolution functional assessment. • Because Tc-99m is constantly available from a molybdenum generator, it can be produced either on site or delivered in unit dose form to nuclear laboratories. There are remaining issues with Tc-99m agents (Table 3-4). While the half-life of technetium is substantially shorter than Thallium-201, it is longer than necessary for the procedure of rest/stress perfusion imaging. Ideally a 2-hour half-life would allow a higher dose to improved image quality and reduce radiation exposure further. The strengths and weaknesses of technetium tracers are summarized in Table 3-4. The protocol used in most laboratories is rest/stress perfusion, which requires 2 to 4 hours for completion, which is limiting for efficiency and optimal patient care. For both technetium agents, there is considerable gut and liver activity, making interpretation difficult and false-positive conclusions resulting in unnecessary catheterizations. Finally, overall diagnostic accuracy is modest (85% sensitivity and 75% specificity, see Chapter 17). As shown in Figure 3-1, the linearity with myocardial blood flow with either sestamibi or tetrofosmin is nonlinear

ch03.indd 35

beginning in flows consistent with exercise stress and level off during pharmacologic stress. This characteristic limits the diagnostic accuracy of these agents and likely the ability to identify multivessel coronary artery disease (CAD). Tc-99m imaging agents are compounded by using byproducts of a molybdenum-99 (Mo-99) generator, called a “cow” (Fig. 3-4).54 Mo-99 has a

Vacuum Vial Saline Vial 99

99m

Tc ( )

Mo ( )

Alumina Column

Shielding

FIGURE 3-4  Saline drawn through the alumina column (elution, milking) and the daughter product, Tc (99mTc, 99Tc), is washed away, while the parent, 99Mo, remains bound.

15-02-2022 13:52:07

36

Section 1  Fundamentals of Nuclear Cardiology

half-life of 66 hours so a generator can be delivered from a distance, while at the local site is useful for approximately 1 week (Chapter 1). The generator can be eluted as often as every 6 hours and produce Tc-99m for 1 to 2 weeks. During elution, the maximal breakthrough is 0.15 mCi for each mCI of technetium-99m. Chemical purity is tested daily and should not exceed 10 mcg/ml.

▶▶Tc-99m Sestamibi The first member of the Tc-99m-isonitrile family to be evaluated in humans was the hexakis (t-butylisonitrile)-technetium (I), also known as Tc-99mTBI.6–12 Currently in use is Tc-99m-hexamibi, or Tc-99m-MIBI, by its generic name Tc-99m sestamibior Cardiolite . Tc-99m sestamibi was approved by the U.S. Food and Drug Administration (FDA) for clinical application in December 1990. Tc-99m sestamibi is a cationic complex that is taken up by myocytes in proportion to regional myocardial blood flow. Tc-99m sestamibi, which is retained within cells because of the negative charge generated on the mitochondria, has a high affinity for cytoplasm and shows very little extracellular exchange. Thus, metabolic derangements affecting myocytes’ viability would also result in decreased Tc-99m sestamibi uptake, independent of myocardial blood flow. Glover et al.12 investigated the myocardial kinetics of Tc-99m sestamibi in dogs undergoing a partial occlusion of the left circumflex coronary artery. They showed that Tc-99m sestamibi was rapidly taken up by nonischemic and ischemic myocardium at rest in proportion to regional myocardial blood flow. There was a good correlation between the initial myocardial flow at normal resting flow rates and the Tc-99m sestamibi myocardial distribution. The myocardial kinetics of Tc-99m sestamibi after pharmacologic vasodilation with dipyridamole showed that the initial myocardial uptake was closely related in a linear fashion to the regional myocardial blood flow at rates up to approximately 2.0 mL/min/g. As flow rates increase, the extraction fraction decreases leading to a nonlinear relationship between myocardial uptake and myocardial blood flow.50 However, at higher flow rates, there is a plateau in the myocardial distribution versus flow curve, resulting in an underestimation



ch03.indd 36

of coronary blood flow. Similar findings have been reported by other investigators, which parallel the kinetics of thallium-201.51,52 There is overestimation of myocardial blood flow at low flows, which is probably related to an increased extraction seen with diffusible indicators. Thus myocardial uptake of Tc-99m sestamibi is proportional to regional myocardial blood flow over the physiological flow range with decreased extraction at hyperemic flows and increased extraction at low flows. The myocardial retention of Tc-99m sestamibi is stable over a long period of time following injection (4−6 hours).53 In contrast to Tl-201, Tc-99m sestamibi shows a very slow myocardial clearance after its initial myocardial uptake. A fractional Tc-99m sestamibi clearance of 10% to 15% over a period of 4 hours was measured by Okada et al. in a canine model of partial coronary occlusion.50 This allows for patients to be stressed and injected in separate areas from imaging. This approach of staging patients has proved beneficial in many private practice laboratories, contributing to the rapid increase in utilization of sestamibi in the 1990s. Sinusas et al.55 studied the myocardial uptake of Tc-99m sestamibi and Tl-201 showed that as long as myocardial cells were still viable, the myocardial uptake of the tracer was not affected by ischemia. These data suggest that the agent can also assess myocardial viability. Tc-99m sestamibi uptake is maintained in viable myocardium but reduced in necrotic tissue. Using a dog model with coronary occlusion and reperfusion, Verani et al.56 demonstrated that the size of the perfusion defect during occlusion as detected by scintigraphic images correlated with the amount of myocardium supplied by the occluded vessel, the area at risk. Noncardiac uptake of sestamibi is high early after injection.57 The study of the upper-body organ distribution showed that the highest initial Tc-99m sestamibi concentration (counts/pixel) is found in the gallbladder and liver followed by heart, spleen, and lungs. The upper large intestine wall receives the highest dose of radioactivity, both at rest and at stress.57,58 The myocardial activity remains relatively stable over time (27 ± 4% of initial activity has cleared from the heart at 3 hours), whereas activity in the spleen and lung decreases gradually. The maximal accumulation in the gallbladder

15-02-2022 13:52:07

Chapter 3  Radiopharmaceuticals for Cardiovascular Imaging

FIGURE 3-5  The 99mTc tracers typically have higher noncardiac uptake early and this resolves over time. The top two rows are the stress and rest images of a 99mTc sestamibi study acquired at approximately 10 minutes postinjection. The bottom two rows are the stress and rest studies acquired 60 minutes postinjection in the same patient. Note the decrease in noncardiac uptake over time.

occurs approximately at 60  minutes after the injection. Similar biodistribution and radiation dosimetry are noted after exercise. This increase uptake in the liver and spleen gallbladder led most nuclear laboratories to adopt 30-minute delay between sestamibi injection and imaging.56–58 Also, there have been investigations of the effects of drinking water and of the consumption of a light fatty meal on the sub diaphragmatic uptake with varying ­success.59,60 An example of delayed clearance is shown in Figure 3-5.

▶▶Tc-99m-Tetrofosmin Tetrofosmin is a ligand that forms a lipophilic, cationic diphosphine complex with Tc-99m. Tc-99mtetrofosmin is the generic name for 1,2,-bis [bis(2-ethoxyethyl) phosphino] ethane. Tc-99mtetrofosmin shows similar myocardial uptake, retention, and blood clearance kinetics to Tc-99m sestamibi.61,62 However, its clearance from both the liver and the lung is faster than that of Tc-99m sestamibi.63 These characteristics can have an impact on the injection and imaging protocols. Sinusas et al.64 tested the hypothesis that Tc-99mtetrofosmin was a reliable coronary blood flow tracer over a physiologic range of flows seen in ischemia or

ch03.indd 37

37

infarction conditions. Uptake of myocardial Tc-99mtetrofosmin appeared to diverge from linear when coronary blood flow exceeds 1.5 to 2.0 mL/min/g of sestamibi. This roll-off in linearity appears to be lower than that of sestamibi Noncardiac uptake of tetrofosmin appears to be less than that of sestamibi. A large multicenter randomized study that included 1620 patients received sestamibi- or tetrofosmin-evaluated image quality, attenuation artifacts, and low count density. Patients receiving tetrofosmin are on average imaged 10 minutes earlier than those receiving sestamibi. Image quality is not different between either agent, and there was no significant difference between the incidence of attenuation artifacts and count density.63 In a separate study of 78 patients imaged early and late (15 minutes vs. 45 minutes) post-stress and rest injections, raw interior images were used to measure uptake in the heart, lungs, liver, and subdiaphragmatic areas. In the stress studies, there is no difference between cardiac and noncardiac uptake between the early and late images and only an 8% difference between heart and subdiaphragmatic areas.65 These improved early biokinetics of tetrofosmin may be useful for developing efficient stress-only imaging protocols and studies that measure the absolute myocardial blood flow.66 Analysis of whole-body images showed that goodquality images of the heart can be obtained as early as 5 minutes after the injection of Tc-99m-tetrofosmin, and this uptake persisted for several hours. Myocardial background clearance resulting from activity in the blood, liver, and lung was rapid. After exercise, there was less Tc-99m-tetrofosmin activity in certain organs, mainly liver, urinary bladder, and salivary glands, in comparison to the rest study. After a stress injection, the myocardial uptake of Tc-99m-tetrofosmin, although relatively stable over time, slightly decreases from 1.3% of the injected dose at 5 minutes to 1.0% at 2 hours after the injection (Fig. 3-5). Sridhara et al.12 compared Tc-99m-tetrofosmin and Tl-201 myocardial imaging in patients with documented CAD and showed that there was no significant Tc-99m-tetrofosmin−myocardial redistribution with a slow myocardial washout of approximately 4% to 5% per hour after exercise and 0.4% to 0.6% per hour after a rest injection. The estimated absorbed radiation doses at rest and at stress are given in Table 3-2.

15-02-2022 13:52:07

38

Section 1  Fundamentals of Nuclear Cardiology

The results show that both at rest and at stress, the gallbladder wall is the target organ, followed by the other excretory organs, such as upper large intestine, lower large intestine, bladder wall, and small intestine. Overall, the radiation dose to most organs is significantly reduced during exercise in comparison to rest study.41,67 In summary, these three SPECT tracers constitute the present choices in evaluating myocardial perfusion. Thallium-201 has been virtually replaced with technetium-imaging agents, and while there are small differences in characteristics between the two, clinically they appear to be equal.

NON-PERFUSION SPECT TRACERS

▶▶99mTc-Pyrophosphate Currently, the primary nonperfusion imaging procedure is to identify imaging patterns consistent with transthyretin cardiac amyloidosis (ATTR). The primary agent in the United States for this is 99m Tc-pyrophosphate ATTR cardiac amyloidosis, a significantly underdiagnosed cause of heart failure especially diastolic, which previously had limited diagnostic choices and was generally detected late when the ventricular function was already significantly affected, generally by biopsy. With the introduction of highly successful although expensive therapies for cardiac amyloidosis, there has been renewed interest in developing imaging protocols to identify those patients that would benefit from these therapies. Please see Chapter 25 for additional information about cardiac amyloidosis. During the last two decades, noninvasive detection of cardiac amyloidosis using nuclear medicine techniques has gained popularity.68–71 99mTc-labeled phosphate derivatives, initially developed as boneseeking radiotracers for bone scintigraphy, were noted to localize to amyloid deposits. Different agents were used to detect myocyte necrosis and calcifications in amyloid deposits (with localized increased tissue calcium deposits), such as 99m Tc-diphosphonate, 99mTc-pyrophosphate, 99mTcMDP (methylene-diphosphonate), and 99mTc-DPD (diphosphino-propanodicarboxylic acid). The latter radiotracer is extensively used in European countries

ch03.indd 38

but is not yet approved by the FDA. Therefore, 99mTcpyrophosphate (PYP), approved since more than 30 years for bone scintigraphy, blood pool imaging, and detection of myocardial infarction, is currently used in clinical practice in both nuclear cardiology and general nuclear medicine; the precise mechanism by which 99mTc-PYP remains unclear but is probably related to high calcium levels in amyloidosis. Different studies have shown that 99mTc-PYP cardiac imaging can distinguish ATTR from AL amyloidosis possibly because 99mTc-PYP may bind TTR amyloid fibrils more intensely than AL fibrils as a result of higher calcium containing substances in ATTR hearts. Using quantitative 99mTc-PYP cardiac imaging, Bokhari et al.72 were able to differentiate light-chain cardiac amyloidosis from the TTR-related familial and senile cardiac amyloidosis. Using a heart-to-contralateral uptake ratio of more than 1.5, they showed a sensitivity of 97% and a specificity of 100% for identifying ATTR cardiac amyloidosis (Fig. 3-6). More recently, SPECT-PYP techniques have been developed that use visual assessment of cardiac uptake versus bone uptake as a separate metric for determining the presence of ATTR amyloidosis.73 The latest multisocietal imaging guidelines for amyloid imaging recommend a 2-or 3-hour uptake phase prior to imaging, with the use of SPECT PYP71,74 typically obviating the need for late imaging.

▶▶Additional SPECT Radiopharmaceuticals One of the most encouraging agents for SPECT myocardial blood flow was 99mTc teboroxime,13 with very high linearity and a potential for a high diagnostic accuracy. However, although teboroxime was approved in 1993, it never achieved widespread use due to the high hepatic uptake and rapid washout. These two features precluded clinical use with existing cameras at the time, but the thought of using fast acquisition instrumentation with CZT cameras is an area of current investigation. Another tracer that has been investigated is I-123 rotenone.75 This agent is an inhibitor of the human mitochondrial complex I, similar to 18F flurpiridaz (discussed later). Although it does not have the same flow characteristics as 18 F flurpiridaz has, it appears to be superior to either of the 99mTc agents and is similar to 82Rb and 201Tl. Superior flow characteristics and reduced radiation

15-02-2022 13:52:07

Chapter 3  Radiopharmaceuticals for Cardiovascular Imaging

39

Rest

Resting Ischemic Defects

Scan Start Time: 08:32:11 Planar SPECT

FIGURE 3-7  TC-ECDG is a novel 99mTc label glucose analog. This agent has the potential for imaging stress-induced metabolic ischemic changes and ischemic memory.

Adj ROI Set Heart Window

Heart Contralateral

Heart

11.901

24.101

27

44

Std Dev

3.656

5.225

Total Counts

19113

38707

Area (mm sq)

4372

4372

Mean Max Pixel

H/CL

2.03

FIGURE 3-6  The heart to contralateral uptake of 99mTc pyrophosphate is elevated (>1.5) in patients with ATTR+ cardiac amyloidosis.

when compared to 201Tl make 123I rotenone a promising candidate for further development. An additional radiopharmaceutical that has undergone extensive clinical trials and is currently in use in small number of centers for neurohumoral imaging is I-123 MIBG, which is discussed in detail in Chapter 24. Another interesting SPECT agent is Technetium99m−labeled ethylenedicysteine-deoxyglucose (99mTcEC-DG), that is a glucose analog, similar to FDG.15 This agent should be sensitivity to metabolic changes caused by ischemia and hypoxia. When free fatty acid metabolism is not possible, myocyte metabolism

ch03.indd 39

switches to glucose metabolism. 99mTc-EC-DG can be taken up using the highly specific Glut-4 transporter pathway. Early Phase I images demonstrated an affinity of 99mTc-EC-DG to regions of myocardium that had severely reduced myocardial perfusion (Fig. 3-7). Preliminary imaging data demonstrate the ability of 99mTc-EC-DG to distinguish ischemia.76

CARDIAC PET RADIOPHARMACEUTICALS Positron emitting radionucleotides have the advantage of not requiring physical collimation to produce an image.77 This advantage results in higher signalto-noise images per unit injected dose. This advantage can be used to improve image resolution. The high-emission enerty level also provides the ability to produce higher-quality image, while the short half-lives of existing tracers provide lower radiation exposure. The results are that myocardial perfusion PET imaging is typically higher quality than SPECT imaging, with a higher diagnostic accuracy, lower radiation exposure, and more efficient protocols. Attenuation correction (often with computed tomography) is performed in all cardiac PET imaging studies reducing the impact of attenuation artifact that plagues SPECT imaging, such as gut/liver, as well as

15-02-2022 13:52:09

40

Section 1  Fundamentals of Nuclear Cardiology

Table 3-5 Ideal Characteristics for PET Tracers • High myocardial uptake with no or minimal myocardial redistribution. • A high target-to-background ratio with low uptake in the adjacent organs (lungs, liver, and stomach). • Linear relationship between radiotracer myocardial uptake and coronary blood flow: = 50% EF 40–49% EF < 40%

p < 0.01

9 8 7

p < 0.001

6 5 4 3 2 1 0 SSS 0–3

SSS 4–8

SSS > 8

FIGURE 10-11  Relationship between left ventricular ejection fraction, perfusion score, and all-cause mortality. (Adapted with permission from Lertsburapa K, Ahlberg AW, Bateman TM, et al. Independent and incremental prognostic value of left ventricular ejection fraction determined by stress gated rubidium 82 PET imaging in patients with known or suspected coronary artery disease. J Nucl Cardiol. 2008;15(6):745–753.)

The highest uptake does not equate to normal coro­ nary blood flow as this is a semi-quantitative mea­ sure. Hence, when reading relative perfusion images, there is a potential to miss balanced ischemia or underestimate relatively milder degree of stenosis in patients with multivessel disease. Also, with phar­ macologic vasodilator stress, one does not know whether the vasodilator agent is successful at increas­ ing blood flow. This can be overcome by measuring MBF in absolute terms, that is, in ml/g/minute. This topic is covered in detail in Chapter 11, and will be described in this chapter briefly. Currently, PET is unique as a noninvasive tool in its ability to measure regional and global MBF. MBF is a composite measure of blood flow of the epicardial coronary vessels and the microvascula­ ture. MBF is commonly provided as a relationship between rest and stress conditions as MBF Reserve (MBFR), although proper use of blood flow should

CH10.indd 167

include examination of both the rest and stress data. Gupta et al.56 demonstrated that integrating both maximal MBF and MBFR can help identify unique prognostic phenotypes in patients with CAD. There are several commercially available software tools for measuring MBF with both Rb-82 and N-13 ammo­ nia, which have been extensively validated.57 Data collection generally occurs within the first 2 minutes after injection of tracer, thus not extending the acqui­ sition time. Briefly, the clinical value of measuring MBF falls into these categories:

• Normal perfusion, normal MBF indicates low risk for epicardial or microvascular CAD and future coronary events. • Normal perfusion, abnormal MBF does not exclude either epicardial or microvascular CAD and is in a higher risk category.

15-02-2022 13:58:39

Section 2  Radionuclide Myocardial Perfusion Imaging

168

A

B

FIGURE 10-12  Example of severe diffuse CAD with normal relative uptake PET (A) images but severely reduced peak stress MBF and stress/rest MBFR, (B) globally and for all three coronary distributions. (Reproduced with permission from Bateman TM, Lance Gould K, Di Carli MF. Proceedings of the Cardiac PET Summit, 12 May 2014, Baltimore, MD: 3: Quantitation of myocardial blood flow. J Nucl Cardiol. 2015;22(3):571–578.)

CH10.indd 168

15-02-2022 13:58:41

Chapter 10  Cardiovascular Positron Emission Tomographic Imaging

• Determine success of vasodilation especially impor­

The presence of both normal perfusion and normal MBF further reduces the likelihood of CAD (Fig. 10-12). Some authors state the negative predic­ tive value to be 97% to 99%.58 The presence of abnor­ mal blood flow with normal perfusion may indicate either small vessel disease or CAD and needs further evaluation. As highlighted in a recent consensus report by Dewey et al.,59 PET is the recommended imaging modality in the investigation of patients with balanced ischemia and multivessel disease, on the basis it pro­ vides a quantitative assessment of flow. A final scenario of normal perfusion is that of blood flow data indicating no change between rest and stress. This indicates failure of augmentation of the pharmacologic stress and invali­ dates the study. Recently, there have been a number of studies investigating non-response to vasodilator stress using surrogate markers, such as splenic uptake60 and hemodynamic parameters,61 demonstrating that incor­ porating this response with the PET study findings can aid further risk stratification in this patient group. Abnormal MBF can reflect other cardiac condi­ tions such as endothelial dysfunction, diffuse micro­ vascular disease, and assess the efficacy of the stress agent.62 Prognostic data from several sources suggest that worse cardiac outcomes occur in patients with perfusion abnormalities and a severe reduction of MBF and myocardial flow reserve. Thus, the addi­ tion of MBF to PET perfusion imaging adds another dimension to the study that contributes considerably to the clinical information and subsequent manage­ ment. Any laboratory now performing or entering the PET perfusion arena should strongly consider the value of MBF. Multiple publications have demonstrated the prognostic value of MBF. Normal MBF values are associated with low coronary risk, compared to abnormal values that predict higher cardiac risk. The largest study to date was reported by Patel et al.63 who included over 12,500 consecu­ tive patients who underwent Rb-82 PET and were

CH10.indd 169

100 MBFR > 2

90 Survival (%)

tant for normal perfusion and fixed defects. • When is single vessel perfusion single vessel CAD? • Confirm that the perfusion defect is ischemia not artifact.

169

80 MBFR 1.8–2 70 MBFR < 1.8

60 50 0

1

2

3

4

5

6

7

Years after test date

FIGURE 10-13  Kaplan–Meier unadjusted survival estimates as a function of myocardial blood flow reserve at baseline. (Reproduced with permission from Patel KK, Spertus JA, Chan PS, et al. Myocardial blood flow reserve assessed by positron emission tomography myocardial perfusion imaging identifies patients with a survival benefit from early revascularization. Eur Heart J. 2020;41(6):759-768.)

followed up for all-cause mortality. An MBFR threshold of 400 μm)

Focal stenosis

Small Arteries (< 400 μm)

Diffuse atherosclerosis

Microvascular Disease

Coronary Blood Flow Pd/Pa FFR

MBFR =

Stress MBF Rest MBF

FIGURE 11-1  Schematic of coronary circulation, showing the difference between physiologic measurements of fractional flow reserve (FFR) and myocardial blood flow reserve (MBFR).

ch11.indd 176

15-02-2022 16:50:15

Chapter 11  Myocardial Blood Flow Quantitation in Clinical Practice

any nuclear cardiology tracer, including both SPECT and PET. However, practically the SPECT tracers are less than ideal due to low extraction fraction, plateau effects at higher flows, as well as camera technology. Thus at present, MBF is routinely performed with cardiac PET tracers only. Because the majority of cardiac PET laboratories (95%) employ rubidium-82 as its PET tracer, this chapter will focus upon its use and methodologies. However, the same concepts apply to the use of N-13 ammonia as radiotracer.

▶▶Collection of Myocardial Blood Flow Data Data collection for MBF begins at start of the infusion of Rb-82 or at the injection of N-13 ammonia at both rest and stress and continues until equilibrium in the myocardium is achieved. As the measurement of blood flow is dynamic, it must capture the entry of the tracer into the blood stream and its transit through the blood stream before its uptake by the myocardium; data collection, and is not feasible with

Boundary Stress Rest

177

exercise due to delay in placing the patient under the camers. For pharmacologic stress, the data collection is begun prior to tracer injection to assure capture of all data. The procedure is illustrated in Figure 11-2. The data collection for blood flow assessment occurs in conjunction with perfusion data collection and does not add to the overall acquisition time or radiation. See Chapter 10 for details on PET perfusion data collection. Processing of blood flow requires specialized software, and several different products are available.

▶▶Resting Myocardial Blood Flow Data The MBF at rest correlates closely with resting heart rate and resting systolic blood pressure. The resting rate−pressure product can be used to estimate the validity of the resting MBF numbers. In cases of tachycardia or hypertension at rest, resting blood flows can be high due to elevated resting rate−pressure product. In addition, if the patient’s

QMP Quality Review

Blood Pool Rol Stress Rest Frame

Slice Reposition ROIs

Reposition ROIs

Apply 1st frame subtraction

Arterial Input Function 10 8 6 4

ch11.indd 177

BP BP

125

100

Time(sec) Stress Myo Myo Rest

75

50

25

0

0

2

FIGURE 11-2  Collection of myocardial blood flow information and time-activity curves. Data collection is started prior to tracer injection to ensure all information is captured. Arterial tracer activity is measured at rest (green line) and stress (red line) at blood pool region of interest (left atrium for this software). Myocardial tracer activity is measured at rest (yellow line) and stress (blue line). Different software may use different modeling assumptions (net retention model, 1 and 2 tissue compartment models) to fit the data and generate flow values from the data collected and it is important to understand your software.

15-02-2022 16:50:17

178

Section 2  Radionuclide Myocardial Perfusion Imaging

blood pressure is initially high due to anxiety, etc., the resting blood flow acquired may not indicate the true resting conditions and negatively affect the overall MBFR. As MBFR is a ratio of stress-to-rest MBF, this can lead to falsely reduced MBFR in the absence of any flow-limiting epicardial or microvascular disease. It is important to make sure that the patient is comfortable before the resting blood flow is obtained. In such cases, the resting blood flow data could be corrected for resting rate−pressure product, and/or more attention paid to the absolute stress data. Alternately, if the technologist notes high HR/ BP values, the patient should be comforted and await resting blood flow acquisition until a more relaxed situation occurs. Resting blood flow is expressed as ml/g/min. At times, the resting blood flow can be higher than the calculated rate−pressure product for technical reasons again artificially reducing the MBFR results. In these cases, some software products provide a “correction factor” and/or more attention should be made for the absolute stress flow data.

▶▶Stress Myocardial Blood Flow Data The stress MBF data are collected during peak hyperemia. Data collection is begun at the time of injection of the tracer at stress, usually 60 to 120 seconds after regadenoson injection. For this reason, currently blood flow data cannot be collected with exercise stress, which would necessitate a delay in acquisition and key data lost. Generally, normal stress blood flow should be at least twice that of the resting blood flow, in both global and regional areas. As the ratio is between rest and stress, it is very important that the resting blood flow measurements reflect true resting conditions. The absolute numbers vary slightly between the different software applications and the mathematical model used to describe tracer kinetics over time. The data derived are pixel by pixel in the entire myocardium, and thus data can also be queried further using the American Society of Nuclear Cardiology (ASNC) 17-segment system of segments in each of the three vascular territories. As with rest, stress blood flow is expressed as ml/g/min. It should be noted that as the queried areas get smaller such as segment-by-segment analysis, the potential for erroneous data rises. Thus blood flow data should be interpreted in a clinical context.

ch11.indd 178

▶▶Myocardial Blood Flow Reserve Data The third set of blood flow data is the myocardial blood flow reserve, sometimes termed MBFR, or CFR. The MBFR is the ratio of the stress absolute blood flow to rest blood flow data, reported in the three coronary territories, as well as a global number. This is done on pixel-by-pixel bases, and as with rest and stress data, individual segments (17-segment display) can also be interrogated to evaluate this ratio within the perfusion abnormality. The regional data reported are the sum of pixel-wise data from each of the segments of the left anterior descending artery (LAD), right coronary artery ( RCA), or left circumflex artery (LCx) territories. In the case in which a perfusion abnormality is small, the regional data may not reflect the area of the defect, while the segmental data might. The MBFR values constitute the bulk of the published literature, especially outcomes data, usually the global values rather from individual vascular territories.

WHAT IS CONSIDERED NORMAL AND ABNORMAL MYOCARDIAL BLOOD FLOW RESERVE? Multiple studies have been published examining the range of normal MBFR values. These studies are usually based upon global myocardial flow reserve and cardiac outcomes data. A recent position statement from the ASNC and SNMMI on MBF interpretation5 recommended an approach, as described in Table 11-1. Multiple studies have supported low long-term cardiovascular event risk in patients with MBFR values of 2.00 or greater. A higher risk of adverse cardiovascular events is associated with MBFR values less than 2; cardiac outcomes worsen as the values get lower.6–8 Based on these data, the authors of the position statement agreed that 2.00 or greater should be considered normal blood flow response. The data7 also indicate that normal is independent of software program, although this should be confirmed with the individual program being used (Fig. 11-3). Patients with normal blood flow reserve with normal or small milder perfusion defects have a very high negative predictive value for high-risk CAD.9

15-02-2022 16:50:17

Chapter 11  Myocardial Blood Flow Quantitation in Clinical Practice

Cardiac death (%/yr)

30%

179

Hybrid

Schelbert (2004)

20%

Lortie (2007) Prior (2012) Yoshida (1996) Mullani (1983)

10%

1:1

0% 0

1

2 Myocardial flow reserve

3

4

FIGURE 11-3  Risk of cardiac death versus MBFR. Annual rate of death from cardiac causes as function of MFR computed using hybrid methods for estimation of input function and five different clinically used extraction models for 82Rb and 1 counterfactual model assuming 100% extraction (1:1). (Reprinted with permission from Murthy VL et al. Comparison and prognostic validation of multiple methods of quantitation of myocardial blood flow with 82Rb PET. J Nucl Med. 2014;55:1952–1958.)

Table 11-1 General Guide to Interpretation of Myocardial Blood Flow Reserve* MBFR

Interpretation

Risk

>2

Normal

Low

1.7−2

Mildly abnormal

Intermediate

1.2-1.69

Abnormal

High

1.8

80 70 60

Survival (%)

Survival (%)

(c) 1-10% Ischemia

3 4 5 Years after Test Date

80

MBFR >1.8

70 60

MBFR ≤1.8 50

MBFR ≤1.8

50 0

1

2

3 4 5 Years after Test Date

6

7

0

1

2

3 4 5 Years after Test Date

6

7

FIGURE 11-5  Reduced global MBFR was associated with a significantly greater risk of death among patients with normal, mild, moderate, or severe ischemia on relative perfusion images. (Used with permission from Patel KK et al. Myocardial blood flow reserve assessed by positron emission tomography myocardial perfusion imaging identifies patients with a survival benefit from early revascularization. Eur Heart J. 2020;41(6):759−768.)

ch11.indd 181

15-02-2022 16:50:18

182

Section 2  Radionuclide Myocardial Perfusion Imaging

no ischemia. These studies confirm the value of routinely incorporating blood flow measurement into clinical practice, which will help define a new normal in which patients are at lower cardiovascular risk than risk stratification with simple spatially relative image interpretation with either SPECT or PET.

Clinical Use of Myocardial Blood Flow Reserve in Patients with No Known CAD The interpretation of MBF results differs between those patients with no known CAD and those with a previous history of CAD, which will be discussed separately. Table 11-2 lists the potential value of MBFR in the setting of no known CAD. First, determination of adequate vasodilation in those patients undergoing pharmacologic stress PET is a powerful tool in assuring that a patient with normal perfusion is truly normal. It is well known that certain products such as caffeine interfere with the vasodilation process, and heretofore it has not been possible to determine such an effect. As illustrated in Table 11-1, the possibility of inadequate vasodilation and nondiagnostic test in the presence of normal perfusion occurs if the global MBFR is close to 1 (less than 1.2). Such finding should be considered inconclusive for the identification of either epicardial or microvascular disease. If it can be determined that the patient consumed a product interfering with vasodilation, the study could be repeated, or if not dobutamine PET MPI could be an

Table 11-2 Clinical Value of Myocardial Blood Flow Reserve in Patients with No Known CAD • Confirm successful vasodilation. • Confirm the perfusion defect is ischemia not artifact. • Identify a low-risk patient with normal perfusion, normal LVEF, and normal MBFR. • Identify coronary microvascular dysfunction or severe multivessel ischemia from epicardial disease in patients with normal perfusion and global abnormal MBFR. • Identify when single-vessel perfusion defect represents single-vessel CAD. • Identify the multivessel involvement in patients with single territory perfusion defects. • Guide revascularization therapy.

ch11.indd 182

alternative. If neither is possible, another imaging modality should be performed. A second value of MBF in no CAD is that it can assist the reader in an equivocal perfusion study (#2, Table 11-2). In this case, the presence of abnormal blood flow in the area of the questioned defect is consistent with CAD, while the finding of normal MBFR can be reassuring that the study is normal and the defect is artifact, such as patient motion during acquisition. A major value of MBF in patients with no known CAD is the presence of both normal perfusion and normal MBFR (#3, Table 11-2). This finding further reduces the likelihood of CAD beyond perfusion data alone. The negative predictive value for obstructive CAD has been reported to be as high as 97%,9 and generally is considered to exclude both epicardial (balanced ischemia) and microvascular CAD as an etiology of the patient’s symptoms, especially useful in diabetic and hypertensive patients. This scenario is an important advancement for nuclear imaging in that the previous concern for severe epicardial CAD with normal perfusion results is now minimized, reducing the likelihood of inappropriate cardiac catheterization. In normal young healthy adults without any CAD, stress flows should augment at least two to three times resting flows. In a study of 290 patients with suspected CAD undergoing Rb-82 PET, 19% of whom had left main or multivessel CAD, an MBFR more than 1.93 had a very high negative predictive value of 97% in helping rule out high-risk CAD, especially in patients with perfusion defect affecting less than 10% of LV mass.9 The necessity for downstream invasive testing for a patient with normal perfusion and blood flow should be minimal. The majority of patients with no known CAD and normal perfusion are in this category with MBF also being normal. However, as it is understood that patients with normal perfusion can have CAD, the converse also occurs, the presence of abnormal blood flow with normal perfusion (#4, Table 11-2). This pattern suggests that CAD has not been excluded and indicates either microvascular or epicardial CAD, or both. Outcome data demonstrate that the future cardiac event rate is significantly higher in this group especially compared to those with normal perfusion and blood flow. Thus, patients in this category

15-02-2022 16:50:18

Chapter 11  Myocardial Blood Flow Quantitation in Clinical Practice

of results generally require further evaluation, such as coronary computed tomographic angiography (CCTA) or cardiac catheterization, to exclude epicardial CAD, especially those patients with MBFR of 1.3 to 1.6. It should be noted that a “normal” catheterization does not exclude microvascular CAD as an etiology. As highlighted in a recent consensus report by Dewey et al.,13 PET with MBFR is the recommended imaging modality in the investigation of patients with balanced ischemia and multivessel disease, demonstrating the growing acceptance of blood flow assessment as an integral part of cardiac PET. Given that perfusion images are spatially relative, they are normalized to the hottest pixel, and can underestimate the extent of disease in case of multivessel involvement or balanced flow reduction as seen in the left main disease.4 Absolute flow quantitation with PET helps identify patients with high-risk CAD, where there is global reduction of peak hyperemic flow, as well as myocardial flow reserve in all coronary artery territories. In 120 patients undergoing Rb-82 PET, MBFR provided significant incremental value in diagnosis of three-vessel disease over relative perfusion assessment.8 In the situation of normal perfusion, as MBFR reflects the hemodynamic significance of disease throughout the entire coronary vasculature, it can also be reduced in patients with diffuse atherosclerosis or microvascular dysfunction, which is prevalent in patients with diabetes, hypertension, chronic kidney disease, and cardio-metabolic disease. While it can be difficult to differentiate whether the etiology of flow reduction is from focal epicardial stenosis, diffuse atherosclerosis, or microvascular dysfunction, some flow patterns have been suggested. A gradual decrease in peak hyperemic flows with a gradient from base to apex can suggest the presence of diffuse atherosclerotic disease in the affected coronary artery.14 Microvascular dysfunction is typically associated with global reduction in hyperemic flows and flow reserve, usually in the mild to moderately depressed range (1.6−2) without a significant basal to apical gradient of reduction in flows. That said, in cases of global reduction in flows, anatomic evaluation with coronary CT angiography or cardiac catheterization should be considered to help differentiate between multivessel epicardial disease and microvascular dysfunction.

ch11.indd 183

183

In patients with perfusion abnormalities, the number of vessels of the abnormal MBFR will assist in determining whether more extensive CAD is present (#5, Table 11-2). Clinical data indicate that in a patient with single-vessel perfusion abnormality if the MBFR is confined to the area of the defect alone, the likelihood of single vessel CAD at catheterization is high. In contrast, for patients with a single perfusion defect and abnormal blood flow beyond such as two or three vessels, there is a much higher likelihood of multivessel CAD at catheterization. MBF can help uncover significant focal epicardial disease in patients who may have a small perfusion defect, if they have regional flow reduction. In a direct same patient comparison in 208 patients, flow quantitation with PET had the highest diagnostic accuracy for predicting flow-limiting obstructive CAD with fractional flow reserve as the gold standard, compared to relative perfusion with SPECT or CCTA.15 Reduced peak hyperemic MBF and MBFR in a territory has been shown to be associated with flow-limiting stenosis in that coronary artery, though exact cutoffs may vary between laboratories and radiotracers.16,17 Typically, regional flow reduction with or without a perfusion defect, with preserved global myocardial flow reserve, confirms single-vessel involvement. In presence of focal epicardial stenosis, a sharp steep decrease in hyperemic flow values is noted between the proximal and distal segments supplied by the affected coronary artery, usually with a stressinduced perfusion defect (Fig. 11-6).14

Clinical Use of Myocardial Blood Flow Reserve in Patients with Known CAD The clinical value of MBFR measurement in patients with known CAD is listed in Table 11-3. As with any vasodilator study, a universal value of MBFR is in determining successful vasodilation, and this is as true in patients with CAD, especially those with normal perfusion or fixed defects. Once adequate vasodilation is confirmed, the clinical use in those with known CAD is different from patients with no known CAD. Patients with prior CABG may have reduction in myocardial blood flow from severe native vessel disease and/or microvascular disease despite patent grafts. As such, reduction of myocardial flow in these patients may not always indicate progression

15-02-2022 16:50:18

184

Section 2  Radionuclide Myocardial Perfusion Imaging

A Gradual Longitudinal Base to Apex Perfusion Gradient of LAD

B Abrupt Sharp Borders of Severe Focal Perfusion Defect Due to LAD Stenosis

FIGURE 11-6  (A) Flow pattern associated with diffuse atherosclerotic narrowing showing a gradual base to apex longitudinal gradient in hyperemic flows in the affected coronary artery. (B) Flow pattern associated with severe focal stenosis showing a sudden drop in peak hyperemic flow between the proximal and distal segments beyond the stenosis in the affected coronary artery with a regional perfusion defect at stress. (Used with permission from Gould KL et al. Integrating coronary physiology, longitudinal pressure, and perfusion gradients in CAD: measurements, meaning, and mortality. J Am Coll Cardiol. 2019;74(14):1785−1788.)

of disease in these patients. However, it is well recognized that patients with known CAD even after CABG/PCI an intervention can also have normal or regionally abnormal MBFR and thus, it is worthwhile evaluating blood flow in the setting of CAD, including determining success of vasodilation. As shown in Table 11-3, there is also important clinical value in identifying “culprit” vessels in-stent re-stenosis and helping guide revascularization strategies in CAD patients with chronic total occlusions. Global MBFR can help with further treatment selection beyond ischemia on relative perfusion assessment. In 12,594 patients undergoing PET, patients with MBFR 1.8 or lower had a survival benefit with early revascularization compared to medical therapy only (Fig. 11-7).18 This survival benefit was noted most prominently among patients with more than 10% ischemia on their PET, even though a trend for the same was present at lower levels of ischemia. Conversely, patients with significant perfusion defects were not observed to have a survival benefit with revascularization, in the presence of normal global MBFR. A prior study of 329 patients noted a survival benefit with low MBFR (1.8

2.02 (0.83, 4.92)

>5-1.8

1.17 (0.59, 2.32)

≥10%

MBFR ≤1.8

0.70 (0.53, 0.91)

(N= 1413) MBFR >1.8

1.51 (0.88, 2.59)

0.5

0.25

1 2 4 Hazards Ratio >

0.25

(A)

3-way Interaction P-value = 0.90

Ischemia

185

0.5

(Favors Early Revascularization)

0

1 2 3 4 Myocardial Blood Flow Reserve

5

(Favors Medical Therapy)

(B)

FIGURE 11-7  Hazard ratios for death with early revascularization versus medical therapy across levels of global MBFR on PET (A) overall and (B) subdivided within categories based on % ischemia. Patients with MBFR ≤1.8 had a survival benefit with early revascularization (PCI or CABG) within 90 days of test compared to medical therapy for both patients with and without known CAD. This was also present after accounting for % ischemia (reversible perfusion defect size and severity). (Reproduced with permission from Patel KK et al. Myocardial blood flow reserve assessed by positron emission tomography myocardial perfusion imaging identifies patients with a survival benefit from early revascularization. Eur Heart J. 2020;41(6):759−768.)

disease (CKD), and cardiomyopathy.6,8,18 Among patients with diabetes and no known CAD, the presence of reduced global MBFR was associated with a higher CV risk, similar to that of patients with known CAD/ prior MI.20 Impaired global MBFR has been shown to be associated with diastolic dysfunction and future risk of diastolic heart failure.21 Impaired MBFR is also associated with maladaptive remodeling in hypertensive patients22 and markers of myocardial injury and wall stress in those with aortic stenosis.23 As such, reduced global MBFR may be an early marker of myocardial decompensation in these patients.

Clinical Use of Myocardial Blood Flow Reserve in Heart Transplant Patients Coronary allograft vasculopathy (CAV) is a late morbid complication post heart transplant, and typically requires yearly invasive angiography for surveillance. PET with flow quantitation can help noninvasively diagnose CAV, and potentially mitigate the risk of multiple invasive surveillance angiograms.24,25 A multiparametric PET score utilizing peak MBF ( 55 provided a more than 93% sensitivity for one abnormal parameter and more than 96% specificity for two abnormal parameters for the diagnosis of CAV.

▶▶Situations When Myocardial Flow Values Are Less Useful Clinically There are certain patient populations where myocardial flow information should be interpreted with caution, as it is less useful in clinical decision making (Table 11-4). In patients with large transmural infarcts, both the absolute rest and stress blood flows are severely reduced, and MBFR may be falsely normal. In patients post-CABG who have severe native vessel disease, hyperemic flows and MBFR may be reduced regionally or globally due to that, despite the presence of patent grafts. As such, flow quantitation is less likely to help diagnose significant graft

15-02-2022 16:50:20

186

Section 2  Radionuclide Myocardial Perfusion Imaging

Table 11-4 Situations In Which Myocardial Blood Flow Assessment May Be Less Useful • Myocardial infarction • Advanced chronic kidney disease • Nonischemic cardiomyopathy • Post coronary artery bypass grafting (CABG) • Studies that fail quality control assessment

and/or native vessel disease. A global MBFR value of more than 1.2 in these patients does indicate good vasodilator response. Similarly, patients on dialysis or advanced chronic kidney disease may often have abnormal MBFR with normal perfusion due to the higher incidence of microvascular disease in these conditions. In patients with severe LV dysfunction, flows can be low due to elevated LV end diastolic pressures or interstitial fibrosis and may not accurately reflect the presence of physiologically significant CAD. Again, microvascular CAD is common in patients with nonischemic cardiomyopathies, and an abnormal MBFR in the presence of normal perfusion may not be related to epicardial CAD. CABG patients are listed here and, as mentioned previously, not infrequently have abnormal blood flow in the presence of normal perfusion. Finally, every blood

flow study should be assessed for quality to assure that data collection and processing are appropriate. If there is a technical error that cannot be corrected, blood flow data should be interpreted cautiously or stated to be unreliable. The quality control aspect of MBF assessment has been described in detail in a recent ASNC/SNMMI Information Statement by Bateman et al.5

CLINICAL APPLICATIONS OF MYOCARDIAL BLOOD FLOW RESERVE: A CASE-BASED APPROACH The unique value of MBF assessment comes in relation to other data from the PET/CT study including perfusion, function and, when available, calcium scoring. In this section are several case vignettes to assist the reader in understanding how to put the data together, and maximize the value of blood flow assessment. Case Vignette 1: A 65-year-old woman with a history of type 2 diabetes, hypertension, and coronary artery calcium score of 123 was referred for a vasodilator (regadenoson) PET MPI for symptoms of atypical chest pain. Cardiac PET (Fig. 11-8) showed normal myocardial perfusion and wall motion and an increase in LVEF from 70% at rest to 77% at peak

FIGURE 11-8  Rb-82 PET MPI images and flow data for Case Vignette 1. A 67-year-old woman with coronary artery calcium score of 123 and atypical chest pain, showing normal perfusion and flow. For details, refer to text.

ch11.indd 186

15-02-2022 16:50:21

Chapter 11  Myocardial Blood Flow Quantitation in Clinical Practice

stress, as well as a normal response. Quantitative flow information showed normal resting MBF regionally and globally and normal augmentation of blood flow with vasodilator stress and a regional and global MBFR of 2.42 (normal :more than 2.0). In this patient, despite an elevated coronary artery calcium score, normal regional and global flow reserve gave confidence that patient did respond adequately to regadenoson stress, and also pointed toward a low likelihood of high-risk flow limiting CAD. Quantitative flow information on a PET MPI can help assess the adequacy of a vasodilator stress. The blood flow results successfully confirmed successful vasodilation, as well as placed the patient in a lowrisk category for CAD events. However, the abnormal calcium score indicates non-obstructive CAD. Teaching Points:

• MBFR more than 1.2 confirms adequate vasodilator response.

• Normal perfusion and normal MBFR confirm low

risk for obstructive CAD and excludes microvascular CAD. • Normal MBFR indicates the elevated calcium score likely due to nonobstructive CAD, warranting more aggressive statin therapy.

187

Case Vignette 2: A 50-year-old man with a history of hypertension and gastroesophageal reflux and family history of premature CAD was referred for a vasodilator PET/CT for an episode of atypical chest pain. The resting ECG demonstrated inferior Q waves, although there was no history of CAD. Cardiac PET perfusion images (Fig. 11-9) showed a medium-sized area of moderate reduction in intensity which is reversible, in the distribution of the right coronary artery affecting approximately 7% of the LV myocardium. Gated images showed normal global left ventricular systolic function with an LVEF of 69% increasing to 70% at peak stress along with a reversible wall motion abnormality in the inferior wall. These findings were consistent with single-vessel ischemia. MBFR demonstrated normal resting blood flow and normal global MBFR (2.23). Absolute hyperemic flows and regional flow reserve were reduced in the RCA territory (1.75), but hyperemic flows in the LAD and LCx territories, as well as global MBFR, were normal. This pattern confirms likely single-vessel CAD at catheterization and excludes microvascular disease. As symptoms improved, an initial course of medical therapy optimization was chosen. Flow quantitation

FIGURE 11-9  Rb-82 PET MPI images and flow data for Case Vignette 2. A 50-year-old man presenting with atypical chest pain suggestive of single-vessel obstructive disease. For details, refer to text.

ch11.indd 187

15-02-2022 16:50:23

188

Section 2  Radionuclide Myocardial Perfusion Imaging

is complementary in studies with perfusion abnormalities and can help determine the severity of CAD in a given patient, in this case confirming a high likelihood of single-vessel CAD. Teaching Points:

• A decrease in regional peak hyperemic MBF and/

or regional MBFR in a territory with a reversible perfusion defect, with normal flow response in other coronary territories, is suggestive singlevessel obstructive CAD. • Clinical decisions can be made with confidence in studies suggestive of single-vessel CAD, such as altered medical therapy without the necessity of cardiac catheterization.

Case Vignette 3: A 70-year-old woman with a history of hypertension, hyperlipidemia, smoking, and diabetes was referred for a vasodilator cardiac PET for symptoms of typical anginal chest pain for 10 days. Perfusion images demonstrated a mediumsized area of moderate-intensity ischemia in the inferior and inferoseptal regions affecting approximately 8% of LV myocardium (Fig. 11-10). The gated images showed not only an increase in LVEF from 62% at rest to 66% at peak stress, but also a reversible wall motion abnormality in the inferior region. MBF at rest was normal, but the absolute stress flows along with myocardial flow reserve were decreased in all three coronary arteries with a global MBFR of 1.55. These findings suggest disease in more than

FIGURE 11-10  Rb-82 PET MPI images and flow data for Case Vignette 3. Perfusion data consistent with single territory ischemia, and flow data suggest multivessel involvement. Cardiac catheterization confirmed multivessel CAD.

ch11.indd 188

15-02-2022 16:50:28

Chapter 11  Myocardial Blood Flow Quantitation in Clinical Practice

the vascular territory, which could be in the epicardial circulation or microvasculature, in contrast to Case 2 just presented. The patient is also in a higherrisk category for cardiac events, thus cardiac catheterization was recommended. Subsequent coronary angiography showed 80% stenosis of the proximal LAD, 70% stenosis of second ramus branch, 100% stenosis of LCX with collaterals, and 95% stenosis of RCA 95%. The patient subsequently underwent three-vessel CABG. While the perfusion images point toward RCA territory involvement, the global flow reduction in all three territories suggest significant disease affecting all three coronary artery territories, which was confirmed on follow-up angiography. This case illustrates the complementary role of perfusion and blood flow and in this case identified more severe CAD likely at catheterization.

189

Teaching Points:

• Among patients with a perfusion defect in a single coronary artery territory, reduction in peak flows and flow reserve in other coronary arteries can suggest the presence of multivessel involvement. • Referral to coronary catheterization should be considered for patients with reduced flow in all three coronary artery territories with single-vessel perfusion defects, as multivessel CAD is often identified. Case Vignette 4: A 72-year-old man with a history of hypertension and dyslipidemia was referred for a vasodilator PET for worsening dyspnea and effort intolerance over 2 weeks. Figure 11-11 illustrates the results. Perfusion images were normal. Gated images showed an increase in LVEF from 66%

FIGURE 11-11  Rb-82 PET MPI images and flow data for Case Vignette 4. Perfusion images were normal; however, reduced flows in all three territories suggest possible balanced ischemia. Catheterization showed significant distal left main stenosis.

ch11.indd 189

15-02-2022 16:50:32

Section 2  Radionuclide Myocardial Perfusion Imaging

190

epicardial CAD. In addition, an MBFR above 1.7 is generally more likely microvascular and below that epicardial CAD. Teaching Points:

at rest to 69% at peak stress. Quantitative flow data showed diffusely reduced peak hyperemic flows and reduced global MBFR at 1.34. Despite normal perfusion, CAD either epicardial or microvascular disease was not excluded due to severely reduced MBFR. A subsequent coronary angiogram showed a large eccentric plaque in distal left main leading to 90% stenosis in distal left main, extending into ostial LAD and ostial LCx. The risk of multivessel disease increases with lower MBFR, and patients with global MBFR values less than 1.6 have a higher likelihood of significant multivessel disease, although cutoffs may vary based on patient characteristics, software, and radiotracer use (Fig. 11-12). In the patient in Vignette 3, perfusion images point toward RCA territory involvement, but the global flow reduction in all three territories suggests significant disease affecting all three coronary artery territories, which was confirmed on follow-up angiography. The presence of abnormal MBFR in the setting of normal perfusion requires careful consideration that either epicardial or microvascular CAD has not been excluded. Calcium scoring in this setting may assist a low or zero calcium score is more likely to be microvascular CAD, while high calcium score more likely

• In the presence of normal perfusion and abnor-

mal MBFR, it may be challenging to differentiate between multivessel epicardial disease, diffuse atherosclerosis, or microvascular dysfunction. Coronary anatomy evaluation with coronary CT angiography or cardiac catheterization can help rule out significant multivessel epicardial disease in these cases. • Moderate-to-severe global reduction of peak flows and flow reserve in all coronary artery territories without significant flow gradient from base to apex can point toward the presence of coronary microvascular dysfunction Case Vignette 5: A 55-year-old patient with a history of prior PCI to mid-LAD dyslipidemia underwent a vasodilator cardiac PET for atypical chest pain. Myocardial perfusion images showed a large area of severe reversible perfusion defect in in the distribution of LAD territory (Fig. 11-13). There was transient ischemic dilation with stress (1.23). The left ventricle was dilated at rest, with mildly

1

1.00

0.8

0.80

0.7 Sensitivity

Probability of 3-vessel CAD

0.9

0.6 0.5 0.4 0.3

0.1

(A)

0.40

CFR

SSS

0.20

0.2

0

0.60

0

0.5

1

1.5

2

2.5 MFR

3

3.5

4

4.5

5

0.00 0.0

(B)

0.2

0.4 0.6 1-Specificity

0.8

1.0

FIGURE 11-12  (A) Predicted probability (red line, 95% CI in blue lines) of severe three-vessel disease across levels of Rb82 PET MBFR showing increasing likelihood of multivessel disease with reducing MBFR. (B) ROC curves showing incremental diagnostic value of MBFR for multivessel disease compared to SSS alone. (Used with permission from Ziadi MC et al. Does quantitation of myocardial flow reserve using rubidium-82 positron emission tomography facilitate detection of multivessel coronary artery disease? J Nucl Cardiol. 2012;19(4):670−680.)

ch11.indd 190

15-02-2022 16:50:32

Chapter 11  Myocardial Blood Flow Quantitation in Clinical Practice

191

FIGURE 11-13  Rb-82 PET MPI images and flow data for Case Vignette 5 of a 55-year-old man with a history of LAD PCI and atypical chest pain showing large LAD territory perfusion defect with global reduction in flows in all three coronary territories.

reduced LVEF at rest at 46% with stress-induced LAD wall motion abnormalities. Flow quantitation showed reduced augmentation of MBFs with stress in all three territories, especially in LAD distribution with a global MBFR of 1.26. A subsequent coronary angiogram demonstrated 95% proximal LAD stenosis just proximal to the prior LAD stent, patent diagonal which is jailed, 50% ostial LCx, and 30% mid-RCA stenosis. Patient underwent successful stenting of LAD. Flow quantitation is especially helpful in diagnosis and guiding management in patients with known CAD where anatomy is already known. Among patients with known obstructive CAD who are being medically managed and present with worsening symptoms, hyperemic flow reduction in a coronary territory helps identify the cause of symptoms, and can guide revascularization decisions. Among patients with prior PCI presenting with worsening ischemic symptoms, reduction of flow in the territory supplied by the stented vessel suggests in-stent restenosis or occlusion. Among patients with chronic total occlusion where revascularization is high risk, flow reduction in the territory supplied by the vessel with CTO can help guide the decision for revascularization versus medical management.

ch11.indd 191

Teaching Points:

• A reduced global MBFR incrementally predicts

those at a higher risk of future adverse cardiovascular events, independent of perfusion defect and LVEF assessment. • MBFR helps guide post-test revascularization decision.

REPORTING OF MYOCARDIAL BLOOD FLOW RESERVE A critical issue of MBF assessment is reporting and transmitting the results to the referring healthcare provider. Murthy et al.26 and Bateman et al.5 are useful guides for this. Validity of the flow measurements should be confirmed first by checking the quality control, as described in the beginning of the chapter. Reporting sections should include the “interpretation” and “conclusion” portions. It should address the question of the referring physician and take into account the clinical context of the patient, history of CAD, and other PET findings in addition to flow information, as discussed in the case vignettes described above. Flow reserve values are more

15-02-2022 16:50:35

192

Section 2  Radionuclide Myocardial Perfusion Imaging

Table 11-5 Examples of Useful Sentences For Reporting PET MBFR In the Description • There was a rise in MBF between rest and stress, indicating successful vasodilation. • There was no rise in MBF between rest and stress, indicating failed vasodilation. • Global MBFR was (provide number) In the Conclusion • Normal MBFR confirms study normalcy, which indicates lower risk of CAD beyond normal perfusion and predicts a low risk for major coronary-related events. • Despite normal myocardial perfusion, MBFR is abnormal, placing the patient in a higher-risk category for CAD and cardiac-related events in patients with no known CAD. • There is a perfusion defect in a single coronary territory along with corresponding regional reduction in MBFR, suggesting single-vessel obstructive CAD. Normal MBFR within the remainder of the myocardium makes more extensive CAD unlikely. • While the perfusion indicates single-vessel disease, MBFR is globally reduced, raising concern for more extensive CAD. • The absence of a rise in MBF with normal perfusion does not exclude CAD. • MBFR is not reported in this patient due to technical or patient-specific concerns that can affect accuracy and inappropriate clinical decisions Adapted from Bateman TM et al. Practical guide for interpreting and reporting cardiac PET measurements of myocardial blood flow: an Information Statement from the American Society of Nuclear Cardiology, and the Society of Nuclear Medicine and Molecular Imaging. J Nucl Cardiol. 2021;28(2):768−787.

robust and comparable across different centers and should be reported over absolute values of stress and rest flows, as absolute stress flow values may differ between different software, a combined interpretation of perfusion, gated, and flow information with a summary statement should be provided, as described in Table 11-5, as referring physicians may not be familiar with the interpretation of flow information.

▶▶Future Directions There is strong evidence supporting the use of quantitative myocardial flow measurements and MBFR in

ch11.indd 192

diagnosis, prognosis, and management of patients with suspected or known CAD. As such, quantitative flow measurements should routinely be a part of interpretation and reporting of PET MPI studies. Flow measurements may vary slightly between different radiotracers, equipment, and software, and it is important to know the nuances of interpreting flow and normal values at one’s local institution. Newergeneration SPECT cameras make dynamic imaging that can be used for flow quantitation possible, which will further expand the availability of flow quantitation with SPECT. More work is needed to assess how flow measurements can be used to better differentiate phenotypes of epicardial obstructive CAD from diffuse atherosclerotic disease and microvascular dysfunction or a combination of those. How change in quantitative flow measurements with time relates to symptoms and cardiovascular events needs to be better understood to further refine the role of quantitative flow in guiding patient management.

REFERENCES 1. Shaw LJ, Iskandrian AE. Prognostic value of gated myocardial perfusion SPECT. J Nucl Cardiol. 2004;11(2):171–185. 2. Dorbala S, Hachamovitch R, Curillova Z, et al. Incremental prognostic value of gated Rb-82 positron emission tomography myocardial perfusion imaging over clinical variables and rest LVEF. JACC Cardiovasc Imaging. 2009;2(7):846–854. 3. Mc Ardle BA, Dowsley TF, deKemp RA, Wells GA, Beanlands RS. Does rubidium-82 PET have superior accuracy to SPECT perfusion imaging for the diagnosis of obstructive coronary disease? A systematic review and meta-analysis. J Am Coll Cardiol. 2012;60(18):1828–1837. 4. Berman DS, Kang X, Slomka PJ, et al. Underestimation of extent of ischemia by gated SPECT myocardial perfusion imaging in patients with left main coronary artery disease. J Nucl Cardiol. 2007;14(4):521–528. 5. Bateman TM, Heller GV, Beanlands R, et al. Practical guide for interpreting and reporting cardiac PET measurements of myocardial blood flow: an Information Statement from the American Society of Nuclear Cardiology, and the Society of Nuclear Medicine and Molecular Imaging. J Nucl Med. 2021. 6. Murthy VL, Naya M, Foster CR, et al. Improved cardiac risk assessment with noninvasive measures of coronary flow reserve. Circulation. 2011;124(20):2215–2224. 7. Murthy VL, Lee BC, Sitek A, et al. Comparison and prognostic validation of multiple methods of quantification of myocardial blood flow with 82Rb PET. J Nucl Med. 2014;55(12):1952– 1958. 8. Ziadi MC, Dekemp RA, Williams KA, et al. Impaired myocardial flow reserve on rubidium-82 positron emission tomography imaging predicts adverse outcomes in patients assessed

15-02-2022 16:50:35

Chapter 11  Myocardial Blood Flow Quantitation in Clinical Practice for myocardial ischemia. J Am Coll Cardiol. 2011;58(7): 740–748. 9. Naya M, Murthy VL, Taqueti VR, et al. Preserved coronary flow reserve effectively excludes high-risk coronary artery disease on angiography. J Nucl Med. 2014;55(2):248–255. 10. Kitkungvan D, Bui L, Johnson NP, et al. Quantitative myocardial perfusion positron emission tomography and caffeine revisited with new insights on major adverse cardiovascular events and coronary flow capacity. Eur Heart J Cardiovasc Imaging. 2019;20(7):751–762. 11. Patel KK, Spertus JA, Chan PS, et al. Myocardial blood flow reserve assessed by positron emission tomography myocardial perfusion imaging identifies patients with a survival benefit from early revascularization. Eur Heart J. 2020;41(6):759–768. 12. Herzog BA, Husmann L, Valenta I, et al. Long-term prognostic value of 13N-ammonia myocardial perfusion positron emission tomography added value of coronary flow reserve. J Am Coll Cardiol. 2009;54(2):150–156. 13. Dewey M, Siebes M, Kachelriess M, et al. Clinical quantitative cardiac imaging for the assessment of myocardial ischaemia. Nat Rev Cardiol. 2020;17(7):427–450. 14. Gould KL, Nguyen T, Johnson NP. Integrating Coronary Physiology, Longitudinal Pressure, and Perfusion Gradients in CAD: Measurements, Meaning, and Mortality. J Am Coll Cardiol. 2019;74(14):1785–1788. 15. Danad I, Raijmakers PG, Driessen RS, et al. Comparison of coronary CT angiography, SPECT, PET, and Hybrid imaging for diagnosis of Ischemic heart disease determined by fractional flow reserve. JAMA Cardiol. 2017;2(10):1100–1107. 16. Hajjiri MM, Leavitt MB, Zheng H, Spooner AE, Fischman AJ, Gewirtz H. Comparison of positron emission tomography measurement of adenosine-stimulated absolute myocardial blood flow versus relative myocardial tracer content for physiological assessment of coronary artery stenosis severity and location. JACC Cardiovasc Imaging. 2009;2(6):751–758. 17. Danad I, Uusitalo V, Kero T, et al. Quantitative assessment of myocardial perfusion in the detection of significant coronary

ch11.indd 193

193

artery disease: cutoff values and diagnostic accuracy of quantitative [(15)O]H2O PET imaging. J Am Coll Cardiol. 2014;64(14):1464–1475. 18. Patel KK, Spertus JA, Chan PS, et al. Myocardial blood flow reserve assessed by positron emission tomography myocardial perfusion imaging identifies patients with a survival benefit from early revascularization. Eur Heart J. 2020;41(6):759–768. 19. Taqueti VR, Hachamovitch R, Murthy VL, et al. Global coronary flow reserve is associated with adverse cardiovascular events independently of luminal angiographic severity and modifies the effect of early revascularization. Circulation. 2015;131(1):19–27. 20. Murthy VL, Naya M, Foster CR, et al. Association between coronary vascular dysfunction and cardiac mortality in patients with and without diabetes mellitus. Circulation. 2012;126(15):1858–1868. 21. Taqueti VR, Solomon SD, Shah AM, et al. Coronary microvascular dysfunction and future risk of heart failure with preserved ejection fraction. Eur Heart J. 2018;39(10):840–849. 22. Zhou W, Brown JM, Bajaj NS, et al. Hypertensive coronary microvascular dysfunction: a subclinical marker of end organ damage and heart failure. Eur Heart J. 2020;41(25):2366–2375. 23. Zhou W, Sun YP, Divakaran S, et al. Association of Myocardial Blood Flow Reserve With Adverse Left Ventricular Remodeling in Patients With Aortic Stenosis: The Microvascular Disease in Aortic Stenosis (MIDAS) Study. JAMA Cardiol. 2021. 24. Bravo PE, Bergmark BA, Vita T, et al. Diagnostic and prognostic value of myocardial blood flow quantification as noninvasive indicator of cardiac allograft vasculopathy. Eur Heart J. 2018;39(4):316–323. 25. Chih S, Chong AY, Erthal F, et al. PET assessment of epicardial intimal disease and microvascular dysfunction in cardiac allograft vasculopathy. J Am Coll Cardiol. 2018;71(13):1444–1456. 26. Murthy VL, Bateman TM, Beanlands RS, et al. Clinical quantification of myocardial blood flow using PET: Joint Position Paper of the SNMMI Cardiovascular Council and the ASNC. J Nucl Cardiol. 2018;25(1):269–297.

15-02-2022 16:50:35

This page intentionally left blank

READ_THINK_ITALIAN.indd 2

11/19/21 1:12 PM

Ventricular Function

Prem Soman and Saurabh Malhotra

KEY POINTS ■■

■■

■■

■■

■■

Gated single-photon emission computed tomo­g­raphy (SPECT) is a useful application to distinguish between an artifact due to soft tissue attenuation versus myocardial scar (infarction), and it also improves reader confidence. Determination of left ventricular (LV) function by gated SPECT or radionuclide angiography (RNA) has valuable prognostic value, especially for prediction of death, and this is incremental to perfusion data alone. Gated SPECT may frequently distinguish between ischemia cardiomyopathy and LV dysfunction due to a nonischemic etiology. RNA has excellent reproducibility and permits assessment of both systolic and diastolic functions and is particularly useful in guiding chemotherapy administration. Global dyssychrony may be assessed with gated SPECT and determination of histogram bandwidth and phase standard deviation. These factors may be useful in the prediction of response to resynchronization therapy or development of arrhythmias.

INTRODUCTION The assessment of ventricular function is a critical component of the evaluation of the cardiac patient

CH12.indd 195

CHAPTER

12

with important diagnostic and prognostic applications. Radionuclide imaging offers highly accurate and precise methods for the measurement of left ventricular (LV) systolic and diastolic function, either in conjunction with myocardial perfusion imaging (single-photon emission computed tomography [SPECT] or positron emission tomography [PET]) or separately in the form of radionuclide ventriculography (RNV). More recently, the ability to assess dyssneregy has added another depth to the role of nuclear imaging in the accurate assessment of patients. This chapter will discuss the role of all three of these approaches.

GATED SINGLE-PHOTON EMISSION COMPUTED TOMOGRAPHY The introduction of electrocardiographic (ECG)-gated myocardial SPECT in the 1990s expanded the application of myocardial perfusion imaging to routinely include the assessment of LV systolic function.1 The development and use of this technique occurred when technetium-based imaging agents were placed into clinical use, as these tracers provided much higher counts, and therefore image quality for measuring function. This was a critical development in the evolution of myocardial perfusion imaging. Currently, the majority of myocardial perfusion studies performed in the United States use gated SPECT technology and Tc-99m tracers. A more recent development has been the evaluation of ventricular dyssynchrony by SPECT, providing further data beyond

15-02-2022 10:45:19

196

Section 2  Radionuclide Myocardial Perfusion Imaging

myocardial perfusion. This chapter will discuss both aspects of ventricular assessment.

PRINCIPLES

▶▶Technical Considerations Hardware Requirements Gated SPECT images can be acquired using singleor multiple-detector cameras. More recently, dualheaded cameras in the 90-degree configuration have been preferred, as images can be acquired in half the time required using a single-headed system without sacrificing image quality. The majority of gated SPECT imaging is performed with high-resolution parallel hole collimators for Tc-99m studies, while all-purpose collimators are used for thallium-201 (Tl-201) studies. A 180-degree imaging arc (45-degree right anterior oblique [RAO] to 45-degree left posterior oblique projections) with a circular orbit is most commonly used, although noncircular (body contour, elliptical) orbits can also be used. The most common detector rotation mode is the “continuous step and shoot” acquisition method, in which the detector records events when stationary at each projection, and then rotates (moves) to the next projection. A “continuous” acquisition mode is also available. The standard image matrix size for gated and nongated SPECT imaging is 64 × 64 pixels, with pixel sizes of 5 to 7 mm. This size offers adequate image resolution for interpretation and quantitation of both Tl-201 and Tc-99m tomograms. Contemporary computers possess adequate processing speed and internal hard disk space to process and store large amounts of scintigraphic data. Acquisition computers are usually separate from processing computers to allow for efficient laboratory operations. In addition, unsophisticated, relatively inexpensive, three-lead gating devices are provided by manufacturers to supply the trigger to the acquisition computer.2

Gated SPECT Acquisition and Processing In an ECG-gated acquisition, a three-lead ECG provides the R-wave trigger to the acquisition computer, with two successive R-wave peaks on the ECG defining a cardiac cycle. Counts from each phase of the

CH12.indd 196

cardiac cycle are binned to a corresponding temporal “frame” within the computer. Perfusion projection images are obtained from summation of the individual frames (Fig. 12-1).1 There is a trade-off between the temporal resolution of gated Tc-99m-sestamibi/tetrofosmin images and the count density of the individual frames. Gating of myocardial perfusion is usually performed at 8 or 16 frames per R–R interval per projection to maintain the count density using a single-headed camera, although 32 frames per cycle are also possible. With dual-headed cameras, 16 frames per cycle (rather than 8 frames) are preferred as the ejection fraction (EF) results are more in line with other imaging modalities. With a multiheaded SPECT system, more frames can be acquired with no increase in acquisition time, as these systems can obtain higher count density images. Most manufacturers provide one of the two modes of gated SPECT acquisition—“fixed” or “variable”—to define the R–R interval. In fixed acquisition mode, the R–R interval is estimated by the acquisition computer prior to the study, based on previously observed 10 to 20 heartbeats, and remains fixed throughout the study. In the variable acquisition mode, the heart rate

R-R R-R patient = R-R computer

Diastole (1) Systole (3–4)

Diastole (1)

FIGURE 12-1  Principle of ECG-gated SPECT acquisition. Separate temporal frames corresponding to different phases of the cardiac cycle are acquired for each angular projection. Perfusion images are obtained from summation of the individual frames. (Reproduced with permission from Cullom SJ, Case JA, Bateman TM. Electrocardiographically gated myocardial perfusion SPECT: Technical principles and quality control considerations. J Nucl Cardiol. 1998;5(4):418–425.)

15-02-2022 10:45:19

Chapter 12  Ventricular Function

is continuously monitored throughout the study and the acquisition computer alters the duration of temporal frames as needed to bin counts equally into the prespecified intervals (8 or 16) per each previously detected R–R interval. In both of these modes (fixed and variable), the data cannot be reformatted after acquisition is complete. An alternative to this is the list mode, a technique increasingly used in contemporary radionuclide studies.3 This technique allows counts to be reformatted into temporal frames after acquisition is complete. The computer records the spatial coordinates of each detected count, as well as the timing marker that identifies the time the count was detected. After acquisition, gated SPECT frames are generated by selecting the mean R–R interval and the beat rejection criteria followed by appropriate binning of the data. The advantage of the list mode is its flexibility, and thus it potentially avoids the gating artifacts seen in the fixed- and the variablemode acquisitions. A disadvantage, however, is that it requires storage of massive amounts of data, and therefore was rarely used in clinical practice until recently, when high-capacity computers became ubiquitous. Variations in heart rate due to a variety of factors (sinus arrhythmia, other arrhythmias, patient anxiety or motion, poor ECG lead contact, etc.) can result in temporal “blurring,” that is, mixing of counts from adjacent frames. To limit acquired data to those heartbeats that are representative of the patient’s average heartbeat and to minimize temporal blurring, a beat rejection window is set by specifying the acceptable deviation of R–R interval from the expected value. A 20% (±10) window has historically been applied, although in patients with highly variable heart rates, up to a 100% (±50) acceptance window can be set. Some camera manufacturers provide an extra frame in which counts from all rejected beats are accumulated. The counts within the extra frame can be added to the nongated data after the acquisition is complete in order to generate a summed SPECT dataset for the interpretation of the static perfusion images. On most commercial systems, a premature ventricular contraction mode may be set that programs the computer to skip one or more cardiac beats before the R–R gating is reestablished. This is done to avoid mixing counts from the two successive cardiac cycles.3

CH12.indd 197

197

As can be inferred from the above discussion, the detection of an adequate R-wave signal is essential to the successful collection of image data synchronized to heart rate. In patients with severe arrhythmias, the triggering mechanism is incapable of properly identifying the R-wave and EF fluctuations, artifactual perfusion abnormalities, and wall thickening discordance may occur.3 Thus, in this situation, a nongated SPECT study may be preferred. A variety of single- and 2-day protocols may be used in conjunction with gated SPECT. As long as counts are adequate, either Tl-201 or Tc-99m perfusion tracers may be used. Either or both the acquisitions composing the stress/rest or rest/stress protocol can be gated, although the most commonly utilized is the high-dose technetium stress study because of its superior count density. Although the common practice is to gate only the post-stress image, a study by Johnson et al. reported that in 36% of patients with reversible perfusion defects, the post-stress left ventricular ejection fraction (LVEF) was >5% lower than at rest.4 This implies that global and regional LV functions obtained from post-stress–gated acquisitions are not representative of basal LV function in patients with stress-induced ischemia, and that perhaps both rest and stress images should be gated routinely, as long as the count density is adequate. It is currently recommended that both rest and stress ECG gating be performed.5 In general, resting function is of lesser quality due to lower counts and information is not reported unless changes occur between the post stress and rest images. The ECG-gated SPECT procedure results in the derivation of a time–volume curve, based on the volume of the left ventricle derived by endocardial definition at each of the 8 or 16 bins (frames) within one cardiac cycle. This is in contrast to the time–activity curve derived from RNV, a technique that calculates EF from the difference in LV cavity counts between end-systole and end-diastole, rather than from LV volumes determined by endocardial definition (see below).

▶▶Procedure for Interpretation of ECG-Gated SPECT Imaging To maximize the value of functional ECG-gated SPECT data, a systematic approach to interpretation

15-02-2022 10:45:19

198

Section 2  Radionuclide Myocardial Perfusion Imaging

Table 12-1 Sequence of ECG-Gated SPECT Myocardial Perfusion Imaging Interpretation • Observation of unprocessed (raw) data • Evaluation of individual perfusion slices • Application of quantitative perfusion software • Evaluation of individual slices for ventricular function • Quantitation of left ventricular ejection fraction • Evaluation of three-dimensional gated images • Evaluation of right ventricular function

Gated SPECT Artifact Irregular R-R Interval: Representative slices from raw unprocessed data

Processed horizontal long axis slices

is essential. Ventricular function should be interpreted only in the context of the perfusion data, as the latter has potential to influence the results and, the two sets of information are complementary. The sequence of interpretation is listed in Table 12-1 and should begin with the evaluation of rotation images followed by perfusion data and finally ventricular function. The final step is the integration of clinical information.

Evaluation of Unprocessed (Raw) Data The interpretation of gated SPECT begins with evaluation of the rotating unprocessed data for overall image quality and any information that might impact the function. This includes potential soft tissue attenuation from breast or diaphragm, and interference from extra cardiac liver or gut activity. Inspection of raw data provides assessment of the technical quality of gated SPECT acquisition. In general, images with poor counts should be interpreted with caution as they could be associated with artifacts. Periodic flashing of the display results from gating errors, occurring as a result of wide variation in the cardiac cycle during the acquisition leading to variation in counts between images (Fig. 12-2). Wide variability of R–R interval can also cause a radial blurring artifact in the gated tomographic images. This may limit the definition of end-systolic frame, affecting LVEF and end-systolic volume (ESV) measurements. The presence of gating errors can be further confirmed by graphs displaying accepted counts as a function of the projection number. If there are no gating errors, all projection curves should superimpose nearly perfectly.

CH12.indd 198

FIGURE 12-2  Example of a gated SPECT artifact, with individual slices revealing “spots” indicative of a gating error, which may then be manifest as “flashing” on the gated SPECT. (Courtesy of GV Heller, MD, PhD.)

Evaluation of Myocardial Perfusion Data This topic is discussed in detail in Chapter 14. Briefly, tomographic slices of the myocardium should be displayed according to the standardized model recommended by the American Society of Nuclear Cardiology,6 and interpreted prior to assessment of ventricular function. Using this model, the myocardium is divided into 17 segments based on three short-axis slices (the apical, mid-ventricular, and basal) and a mid-ventricular vertical long-axis slice. The presence of a defect and the degree of reversibility (differences between stress and rest) should also be noted. Quantitative confirmation of the visual observations of the perfusion data should generally be made. Fixed and reversible perfusion deficits should be carefully evaluated for regional function as described in the next section.

Evaluation of Ventricular Function Assessment of wall motion should be performed for both the left ventricle and the right ventricle. Assessment should include both global LV and RV functions, and regional function for the left ventricle. The latter is critically important in the presence of a perfusion abnormality. It is generally recommended that ventricular function be interpreted using at least three short-axis slices and one horizontal and vertical

15-02-2022 10:45:20

Chapter 12  Ventricular Function

long-axis slice at a magnification sufficient to easily assess regional changes. The short-axis slices selected should be similar to those used for perfusion interpretation to enable meaningful comparison.

Assessment of Global Ventricular Function The interpreter should first estimate the global LVEF by visual interpretation, and then confirm with the quantitative software. Confirmation of quantitated LVEF should include examining the contours, as gut and liver activity might be mistakenly included. It is important for the reader to know the lower limit of normal for a given software quantitation package, since the absolute number varies by imaging modality, as well as among software and hardware vendors. Global function can be categorized as normal (EF >50%) or mild (EF 40–50%), moderate (EF 30–40%), or severe (EF 49.8 degrees in women identifies dyssynchrony.49 Phase analysis-derived PSD and HBW have been known to have shown excellent reproducibility50 in single-center studies, due to the automated generation of these parameters with minimal operator input. However, repeatability has not been tested in a multi-center setting. In general, PSD and HBW are considered to represent a

CH12.indd 209

209

Phase Histogram: Percentage of myocardium at the OMC throughout the cardiac cycle

Phase Polarmap: Sequence of mechanical activation

FIGURE 12-12  The phase histogram shows the percentage of myocardium contracting (y-axis) at each point in the cardiac cycle (x-axis). The phase polar map is a bull’s-eye representation of the LV, showing the sequence of mechanical activation, using a color-coded scheme. This is an example of synchronous LV contraction with a narrow and highly peaked histogram and a uniform color on the phase polar map.

measure of global LV dyssynchrony. The assessment of regional LV dyssynchrony can be derived by partitioning of the global phase data in accordance with the standard 17-segment LV model. The segmental mean phase value provides an assessment of regional differences in LV contraction, from which the segmental location of the site of latest activation (SOLA) can be determined.51 It is important to note that assessment of dyssynchrony from gated SPECT is software dependent, in a fashion similar to the known variations in measurement of LV volume and EF. It is known that, Quantitative-Gated SPECT® (QGS) excludes myocardial regions with the lowest 5% of phase amplitude in the derivation of the HBW, and therefore is expected to produce values of HBW that are systematically different from those obtained from ECTb.49,52 Given these computational differences between software, it is important to use software specific cut-offs when evaluating dyssynchrony, and to utilize the same software to process-gated SPECT data when serial changes in LV dyssynchrony are to be determined. Despite the low temporal resolution, when compared to ERNA, the wider availability and the ability to simultaneously obtain comprehensive information on perfusion, function, and dyssynchrony (both global and regional) from a standard acquisition make gated SPECT a very attractive tool for dyssynchrony assessment. Clinical Application of Dyssynchrony The logical application of LV dyssynchrony assessment would be in guiding cardiac resynchronization

15-02-2022 10:45:37

210

Section 2  Radionuclide Myocardial Perfusion Imaging

in patients with medically refractory heart failure and severely reduced EF, where only two-thirds of patients selected based on clinical criteria (NYHA class II-IV, EF 120 ms) will show improvement in symptomatic and or LV function. Thus, there is interest in using imaging approaches to improve patient selection and response rates. However, more recent iterations of selection criteria afford a class I indication only to patients with QRS >150 ms with left bundle branch block (LBBB) pattern, where the response rate is >80%.53 Gated SPECT and ERNA studies have shown that the presence of LV dyssynchrony and its severity prior to cardiac resynchronization therapy (CRT) determine LV reverse remodeling and symptomatic benefit following CRT.54–56 While threshold values of abnormal dyssynchrony have been associated with CRT response in small, single-center studies, specific cutoff values of HBW or PSD, hitherto considered in isolation, have not been useful in improving patient selection for CRT. Studies have suggested an important role of myocardial scar burden in predicting CRT response,57–59 and also a relationship between SOLA with the position of the LV lead of a biventricular pacemaker.60–62 Gated SPECT is an established technique to imaging MI or scar, and its ability to identify SOLA by regional phase analysis to guide LV lead position has been reported.63 Identification of regional phase on gated SPECT in a population with HF, low EF has demonstrated the SOLA to be

always located in the lateral wall among those with LBBB, but only in 50% of patients with non-LBBB wide QRS.64 This finding may partly explain the high response rate to CRT in patients with LBBB, and suggests a role of image guidance in the placement of the LV lead in patients with non-LBBB QRS patterns. Indeed, single-center studies do suggest improved outcome when the LV lead is targeted to a viable myocardial segment with delayed activation.61,65,66 The combined value of dyssynchrony, myocardial scar and LV lead concordance with SOLA, in predicting the CRT response was evaluated in a prospective study of 44 HF patients undergoing CRT implantation.64 The presence of baseline dyssynchrony and its change immediately following resynchronization was studied on gated SPECT with a “single-injection protocol.” A prespecified algorithm comprising (a) the presence of baseline dyssynchrony, (b) the scar burden of 2 pixels), repeating the image acquisition is recommended. Motion correction algorithms, which involves a manual approach, may also be successfully applied,9,10 but usually only when the patient motion is superior–inferior. Correction of lateral or rotational motion is challenging, but it has been successfully incorporated into many software packages Multidetector systems may demonstrate abrupt motion on review of the rotating images. This is usually caused by temporal factors associated with a two-detector system and the fact that the last frame of acquisition for Detector 1 is substantially later than the first frame obtained from Detector 2. Thus, gradual motion throughout the acquisition is therefore accentuated when reviewing the rotating images. The presence of patient motion may produce artifacts and therefore reduce diagnostic accuracy.11 Not only do these artifacts resemble ischemic heart disease, but also patient motion may create the appearance of multivessel disease. “Upward creep” may also be detected by reviewing the rotating images. This

CH14.indd 238

The rotating planar images should be reviewed for the presence of abnormal activity beyond the boundaries of the myocardial structures. The presence of intense subdiaphragmatic activity, emanating either from the liver or from the gastrointestinal tract, may confound image interpretation. Once such activity is present, it may cause a negative lobe artifact, also known as a ramp filter artifact. Intense adjacent activity may cause this reconstruction artifact, for which there is no reliable correction, although iterative reconstruction (as opposed to filtered backprojection) may help.4 This type of abnormality may create an artifactual perfusion abnormality or may mask the presence of a true abnormality (Fig. 14-5). Ideally, when substantial activity is noted especially in the liver or adjacent bowel loop, image acquisition should be repeated to eliminate this type of artifact. The interpretation of SPECT images should not be restricted only to the myocardium. A variety of neoplastic lesions may also be detected with commonly used radiopharmaceuticals.3,13 These may reflect either primary or metastatic tumors and include the following types of neoplastic growths: lung, breast, sarcoma, lymphoma, thymoma, parathyroid tumor, thyroid abnormality, and kidney and hepatic tumors. Incidentally discovered clinical thyrotoxicosis can be detected by careful evaluation of the rotating planar images and may aid in the early detection of thyroid disease.14 Finally, the rotating images may reveal contamination by the radiopharmaceutical, occurring on either the skin or clothing, which once again may confound the SPECT image interpretation. Extravasation of tracer at the injection site often can be detected, especially in larger field of view cameras.

15-02-2022 17:04:06

Chapter 14  Interpretation and Reporting of SPECT and PET Myocardial Perfusion Imaging TH–6

092411

22Apr 1999

239

NWMH

Short Axis-STR

Short Axis-RST

Vertical-STR

STREES RAW

Vertical-RST

Horizontal-STR

A TH–6

092411

22Apr 1999

NMH

Short Axis-STR

Short Axis-RST

Short Axis-corr

Vertical-STR

Horizontal-STR

Vertical-RST

Horizontal-RST

B

FIGURE 14-5  Dual-isotope myocardial perfusion images of a patient felt to be at low risk for coronary artery disease. (A) Prominent activity is noted in a bowel loop immediately adjacent to the inferolateral wall on the resting images. This is clearly visible on the resting planar image. These images also demonstrate an apparent reversible inferior wall perfusion defect. (Used with permission from Thomas Holly, MD.) (B) The stress and rest images are again shown on the first two rows in the short-axis, vertical long-axis, and horizontal long-axis images. The third row represents repeat image acquisition of the poststress images following food consumption, defecation, and waiting approximately two additional hours. With the subdiaphragmatic/bowel loop activity now removed, there is no perfusion abnormality noted in the inferior wall. (Used with permission from Thomas Holly, MD.)

CH14.indd 239

15-02-2022 17:04:08

240

Section 2  Radionuclide Myocardial Perfusion Imaging

This may result in degradation of image quality due to low count density, and the study may need to be repeated. In addition, it is usually possible to distinguish between a neoplastic growth and contamination by reviewing the rotating images.

Attenuation Artifacts Soft tissue, overlying cardiac structures, may confound image interpretation. Breasts/chest soft tissue, or that related to subdiaphragmatic structures, may reduce the specificity for coronary artery disease (CAD) detection and is present in up to 40% of all studies. Photopenic areas may be noted from overlapping breast tissue even when the size of the breast is relatively small. It is often possible to appreciate where the reduction of photons may occur on the SPECT slices by reviewing the cine images (Fig. 14-6). In addition, soft tissue attenuation from the diaphragm may obscure the inferior wall, causing a false impression of an inferior wall abnormality. This occurs most commonly in men, but in obese women as well. Recognition of the superior-placed diaphragm is helpful in the interpretation of images and the enhanced recognition of a potential artifact. When such an abnormality

FIGURE 14-6  An individual frame of the rotating planar images demonstrating prominent soft tissue attenuation from the breast as depicted by the photopenic area (arrowheads). (Reproduced with permission from Hendel RC, Gibbons RJ, Bateman TM. Use of rotating (cine) planar projection images and the interpretation of a tomographic myocardial perfusion study. J Nucl Cardiol. 1999;6(2):234–240.)

CH14.indd 240

is present, prone imaging may be helpful. In patients with large body mass index (BMI), whose defect in the apical-inferior region may be diagnostically challenged, the addition of prone acquisition has been shown to improve diagnostic confidence, and when applied to stress-only MPI, it can reduce the need for unnecessary rest scans.15,16 Obviously, gated SPECT17,18 and attenuation correction19 methodologies may also be of substantial use in the correct interpretation of soft tissue abnormalities. Differential soft tissue attenuation may occur when the overlying soft tissue is present in different positions on the rest and stress images, thereby leading to the appearance of an apparently reversible perfusion defect. This challenging scenario is not helped by gated SPECT, as true reversible defects (ischemia) may demonstrate normal left ventricular (LV) thickening and motion.

▶▶Analysis of Planar Images Although tomographic MPI is considered the preferred modality for assessment of myocardial perfusion, planar imaging may be an alternative option in certain circumstances, such as in claustrophobic or critically ill patients where rapid acquisition is required, or in morbidly obese patients that do not qualify for the SPECT camera table. Electrocardiogram (ECG)-gated planar images can also be acquired. Regardless of whether the tomographic MPI is performed, planar images should always be inspected first, preferably on a linear gray scale. Soft tissue attenuation by breast tissue, diaphragm, or other sources should be noted. Breast marker has been shown to be useful in identifying true perfusion defects from breast attenuation on planar images. Similarly to the analysis of tomographic slices, segmental analysis of myocardial perfusion can be performed. The standard views for imaging positions and standardized nomenclature for myocardial segmental perfusion evaluation on planar images have been described in the American Society of Nuclear Cardiology (ASNC) myocardial perfusion planar imaging guideline.20 For qualitative assessment, the severity of perfusion defect can be classified as mild, moderate, or severe and the extent of defect as small, medium, or large. A five-point segmental scoring system can be applied for semiquantitative evaluation, which is described later in this chapter. If

15-02-2022 17:04:08

Chapter 14  Interpretation and Reporting of SPECT and PET Myocardial Perfusion Imaging

a quantitative analysis is to be performed on planar imaging, background subtraction must be applied to the images. Reversibility may also be reported from planar images.

▶▶Analysis of Tomographic Slices Analysis of Image Quality The first task is to determine whether or not adequate count statistics are present for accurate interpretation. A number of quality assurance tools are available from most manufacturers that assist in this process. The study should be graded based on overall image quality (uninterpretable, poor, fair, good, and excellent). Factors related to body habitus should be considered.

Cardiac and Lung Activity The projection data also provide an assessment of cardiac size. Left ventricular hypertrophy (LVH) may be suspected when a reduced LV cavity:wall thickness

241

ratio is noted. Prominent right ventricular uptake may be noted on the raw images or tomographic slices and may indicate right ventricular hypertrophy, such as seen in pulmonary hypertension. However, no criterion other than subjective visual impression is available for this diagnosis. Prominent lung activity may be present, which frequently is in the setting of severe LV dysfunction or extensive ischemia. However, while abnormal lung activity is an important finding associated with thallium-201 scintigraphy, there is no consensus as to its meaning with technetium-99m imaging.4 LV cavity size may be assessed first by reviewing the rotating planar images. However, the overall cavity-to-wall-thickness ratio may be more qualitatively determined by observing the SPECT slices. In addition, it should be noted if the post-stress images visually reveal a larger LV cavity than noted on the resting study (Fig. 14-7). This would be consistent with transient cavity dilation (TCD) also known as transient ischemic dilation (TID) of the LV cavity. The presence of TCD is a marker of proximal left anterior descending (LAD) and/or multivessel

FIGURE 14-7  Perfusion images from a 31-year-old man with new onset of chest pain, who had limited exercise capacity and developed marked STsegment changes during the stress test. The stress images reveal an extensive, severe defect in the anterior, septal, and apical regions, with substantial reversibility noted on the resting images. In addition, there is transient enlargement of the left ventricular cavity on the post-stress images, relative to the resting study. The transient cavity dilation is most notable on the vertical and horizontal long-axis images; the TID ratio was 1.4.

CH14.indd 241

15-02-2022 17:04:09

242

Section 2  Radionuclide Myocardial Perfusion Imaging

Table 14-2

Table 14-3

Factors Impacting on TID Calculations

Abnormal TID Cutoffs for Different Protocols and Types of Isotope Used

• Radiopharmaceutical • Mode of stress (vasodilator vs. exercise vs. combined) • Sequence of imaging (stress/rest vs. rest/stress) • Camera system (NaI vs. CZT) • Quantitative software • Noncoronary factors (micovascular disease, hypertension)

disease and a worsened prognosis.21 The TID values vary by protocols and types of isotopes used in the study, along with other factors (Table 14-2). Usually, about a 20% increase is required when using the dual-isotope protocol.21 When using a single-isotope study, a lesser amount of cavity enlargement (approximately 5–10%) is felt to be abnormal.22,23 The upper limits for TID vary based on the radiotracer and type of stress,24 as well as whether a new-generation detection, i.e., CZT, is used.25 Pharmacological MPI typically results in higher upper normal TID ratios when compared to exercise MPI,26 as also illustrated in Table 14-3. Sex difference for TID thresholds has also been noted, and should be taken into consideration when interpreting TID ratios.27 In addition to the visual assessment, quantitative analysis of the TID ratio is available on most software packages; of note, quantitative TID values differ substantially among the different quantitative methods.28 Finally, nonepicardial coronary disease may also account for TID, as in patients with microvascular disease or hypertensive heart disease. Overall, many experts believe that TID ratios for single-isotope studies in excess of 1.05 for exercise and 1.15 for vasodilator stress should raise concern of severe disease.

Perfusion Defect Defect severity is often described in a qualitative fashion (mild, moderate, and severe). A mild abnormality is one in which the clinical significance of the defect is unknown. Such an abnormality may reflect an equivocal finding. This often represents only a 10% reduction of peak tracer activity for a particular study, and generally changes image intensity with maintenance of wall thickness. Moderate and severe

CH14.indd 242

Protocol

TID Threshold

Rest Tl-201/exercise stress Tc-99m sestamibi21

1.22

Rest Tl-201/exercise stress Tc-99m sestamibi29

1.23

Rest Tl-201/pharmacologic stress Tc-99m sestamibi  Dipyridamole30

1.27

30

1.35

 Adenodine  Adenosine

31

1.36

 Regadenoson32

1.39

 Dobutamine30

1.40

Exercise stress/rest Tc-99m sestamibi29 22

1.14

Rest/exercise stress Tc-99m sestamibi

1.19

Gated rest/exercise stress Tc-99m sestamibi (end-diastolic volume)22

1.23

Regadenoson stress/rest Tc-99m sestamibi33

1.33

Rest/regadenoson stress Tc-99m tetrofosmin23

1.31

Dipyridamole stress/rest Tc-99m sestamibi (2-day)34

1.19

Gated stress/rest Tc-99m tetrofosmin (2-day; end-diastolic volume)35

1.25

Exercise Tc-99m sestamibi with CZT camera25

1.20−1.22

Regadenoson Tc-99m sestamibi with CZT camera25

1.52

Regadenoson-walk Tc-99m sestamibi with CZT camera25

1.30

Dipyridamole Tc-99m sestamibi with CZT camera25

1.28

Exercise Tc-99m tetrofosmin with CZT camera25

1.18

defects carry more important diagnostic and prognostic values. In addition, the extent of the perfusion abnormality may also be qualitatively described as small, medium, or large (Figs. 14-8 and 14-9). Although these descriptions are relative, they may

15-02-2022 17:04:09

Chapter 14  Interpretation and Reporting of SPECT and PET Myocardial Perfusion Imaging

243

FIGURE 14-8  Exercise/rest dualisotope myocardial perfusion images from a 69-year-old man with a history of hypertension, hyperlipidemia, and diabetes who presents with exertional chest pain. There is a small-sized defect of moderate severity involving the basal portion of the inferior wall. This perfusion abnormality appears completely reversible and is consistent with the significant stenosis in the right coronary artery. Subsequent coronary angiography confirms the presence of a 95% right coronary artery stenosis.

FIGURE 14-9  Adenosine/rest dualisotope myocardial perfusion imaging in a 75-year-old man with a history of known coronary artery disease and status post myocardial infarction. He is currently asymptomatic. The perfusion images demonstrate a large area of severely reduced activity in the inferior and inferoseptal walls, with a moderately severe abnormality noted in the septum and apical regions. No reversibility (ischemia) is noted.

CH14.indd 243

15-02-2022 17:04:10

244

Section 2  Radionuclide Myocardial Perfusion Imaging

be based on objective information from quantitative programs. In an attempt to describe the severity and extent as a combined value, a variety of scoring systems have been designed, the most popular being the summed stress and summed rest scores. These scores are derived by adding the point value using the range of “0” for normal perfusion to “4” for absent activity for each segment of the 17-segment model.1 A mild, moderate, or severe reduction in count should be scored as 1, 2, or 3, respectively. The difference between the summed stress score (SSS) and the SRS is called the summed difference score (SDS) and is a measure of reversibility. Usually, individual segments with a ≥2-grade improvement on the resting study are felt to represent substantial ischemia. The size of the defect should be noted, as small, medium, or large (Table 14-4). Alternatively, the defect may be described as percent myocardium, which may be estimated by taking the SSS (or SDS) and dividing by 68 in order to generate percent myocardium. The type of perfusion abnormality should also be described. A fixed perfusion defect (i.e., one that is the same on both the post-stress and rest images) is often equated to a myocardial scar, especially when the abnormality is of severe intensity. However, a fixed perfusion abnormality may also reflect severe myocardial ischemia and the presence of myocardial viability. A reversible abnormality is a perfusion abnormality noted on the post-stress images, but largely normalizes on the resting images. In many

Table 14-4 Semiquantitative Defect Analysis Percent of Left Ventricle

Number of Segments*

Small

5–10

1–2

Medium

>10–20

3–4

Large

>20

≥5

*

Data from Hendel RC, Budoff MJ, Cardella JF, et al. ACC/ AHA/ACR/ASE/ASNC/HRS/ NASCI/RSNA/SAIP/SCAI/ SCCT/SCMR/SIR. Key data elements and definitions for cardiac imaging. J Am Coll Cardiol. 2009;53:91–124.

CH14.indd 244

cases, some interpreters may use the term partially reversible. It is critical to determine whether it is a predominantly reversible defect or only a minimally reversible abnormality. Quantitatively, reversibility has a variety of definitions, but is often associated with a 20% to 30% improvement in regional activity. “Reverse redistribution” describes a pattern where a defect noted on the rest images is either not present or less severe on the stress images. This finding may be noted in the setting of a subendocardial (nontransmural) scar and provides evidence of viability.36 Although well described with thallium-201 scintigraphy, when this pattern is present with Tc-99m sestamibi or tetrofosmin, an artifact should be suspected. This finding likely relates to low count density of the resting study especially with a same day rest/stress protocol and does not correlate with significant coronary artery lesions.37 The perfusion abnormalities should also be identified by their location. Standard terminology has now been accepted.1 nomenclature of such abnormalities (Fig. 14-10). The 17-segment model should be used for reference with regard toThe perfusion abnormalities should be described as being present in the apical, anterior, inferior, or lateral walls/ regions. Terms like “posterior” or “distal” should not be used. The perfusion defect may also be described as occurring within a vascular distribution, although, the distribution of an individual coronary artery is highly variable. By convention, the 17 segments have been assigned specific vascular distributions so as to standardize interpretation and reporting. As a general rule, the lateral wall is assigned to the circumflex distribution, the anterior and antero- septal regions to the LAD coronary artery, the infero- septal and inferior walls to the right coronary artery, and the apex is usually assigned to the LAD distribution.

Quantitative Analysis A variety of software tools are presently available, including the following products that are commercially available. These quantitative programs usually refer an individual patient’s data to a normal reference profile. The comparison of individual studies to such a reference is often displayed as a polar map. The “blackedout” segments usually reflect an area of activity that is

15-02-2022 17:04:10

Chapter 14  Interpretation and Reporting of SPECT and PET Myocardial Perfusion Imaging

245

Left ventricular segmentation 1 7 2

6 8

13

14

17

9

12

16 11

15

3

5 10

4 1. Basal anterior 2. Basal anteroseptal 3. Basal inferoseptal 4. Basal inferior 5. Basal inferolateral 6. Basal anterolateral

A

7. Mid anterior 8. Mid anteroseptal 9. Mid inferoseptal 10. Mid inferior 11. Mid inferolateral 12. Mid anterolateral

13. Apical anterior 14. Apical septal 15. Apical inferior 16. Apical lateral 17. Apex

Coronary artery territories Vertical long-axis

Short-axis Apical

Mid 7

13 14

8

12

2

Mid

6 17

16 9 15

B

Basal 1

LAD

11 10

RCA

3

5 4

LCX

FIGURE 14-10  (A) A polar-plot depiction of left ventricular segmentation according to the 17-segment model. The recommended nomenclature is noted for each segment below. (B) The 17-segment model, obtained by three individual short-axis slices, as well as one mid-cavity vertical long-axis slice. A depiction of the coronary artery distribution is also noted. (Reproduced with permission from Cerqueira MD, Weissman NJ, Dilsizian V, et al. Standardized myocardial segmentation and nomenclature for tomographic imaging of the heart: a statement for healthcare professionals from the Cardiac Imaging Committee of the Council on Clinical Cardiology of the American Heart Association. Circulation. 2002;105(4):539–542.)

CH14.indd 245

15-02-2022 17:04:10

246

Section 2  Radionuclide Myocardial Perfusion Imaging

FIGURE 14-11  Stress/rest perfusion imaging demonstrates a large inferior, interoseptal, and interolateral defect, which has a small area of reversibility (ischemic) in the apex. The polar plots demonstrate the large extent of the defect, with the hatched region indicating reversibility.

below the threshold deemed as normal (Fig. 14-11). In many cases, it may represent a value such as 2.5 standard deviations below the mean value for a normal patient population; individual programs have specific thresholds. These thresholds and normal reference files are often different depending on the radiopharmaceutical. Furthermore, additional techniques, such as attenuation correction, may also alter the profiles. Most important, however, is that the normal reference files are sex-specific unless attenuation correction methodology is employed. In addition to the polar map or bull’s-eye projection, circumferential profiles may also be created again demonstrating where the count density falls below a specific threshold and is, therefore, deemed abnormal. Quantitative analysis for most of the software programs has been validated in multiple studies and usually published in peer-reviewed journals,38–41 which is further discussed in Chapter 15. However, it is advised that the quantitative analysis be used as a tool and guide, serving as a “second observer.” These quantitative computer-assisted tools should not be used for primary analysis. Following the visual

CH14.indd 246

inspection of the tomographic slices, quantitative interpretation may be examined. Any discrepancies or previously unrecognized abnormalities may then be reviewed. However, the individual interpreter must “overread” the computer-assisted interpretations, as many technical problems may develop and lead to false results. Therefore, quantitative analysis is not a substitute for an expert interpretation but should be used as an adjunct to assist the interpreter.

▶▶Gated SPECT It is recommended that gated SPECT be employed for essentially all myocardial perfusion SPECT imaging studies42 with a standardized approach for the interpretation of gated SPECT data. This critical component of contemporary perfusion imaging is discussed in detail in Chapter 12. The gated SPECT data are often displayed in different fashions, depending on the software. Irrespective of the display, the most critical information is often demonstrated in the mid-ventricular slices from each of the orthogonal axes. Gated SPECT should be displayed

15-02-2022 17:04:11

Chapter 14  Interpretation and Reporting of SPECT and PET Myocardial Perfusion Imaging

247

FIGURE 14-12  Computer-assisted detection of the endocardial surfaces on gated SPECT with the “contours” demonstrating the boundaries of the endocardial and epicardial surfaces (4 D-M SPECT™).

as a cine loop, and the interpreter should observe the images for overall global function, examining the endocardial surfaces and their excursion. In addition to the motion of the endocardial walls, myocardial thickening may be determined by the increase in brightness noted on gated SPECT display resulting from the partial volume effect. It is possible that the thickening (brightening) may be normal, although the excursion is abnormal. This is often seen in settings such as following previous cardiac surgery, where the septum appears dyskinetic or akinetic, but thickens (brightens) normally. If there is difficulty localizing the endocardial surfaces, most software will provide “contours” where the computer provides a line for what it believes to be the endocardial surface (Fig. 14-12). This may be used to assist the interpreter in evaluation of endocardial motion. Regional abnormalities may also be determined especially by examining multiple axes. Similar geographic schema to that noted for perfusion imaging should be employed when describing regional wall motion abnormalities. A great variety exists with regard to the use of displays for gated SPECT information. While black

CH14.indd 247

and white or “hot body/thermal” may demonstrate brightening very effectively, a number of different color tables have also been used to assist in evaluating myocardial brightening or increases in count intensity. Overall, however, the monochromatic color tables are usually recommended. It is recommended that ECG gating be performed for both stress and rest acquisitions, in case poor data quality is present on one set of images. Gated SPECT data reflect global and regional LV function at the time of image acquisition, and hence stress gated SPECT images may not reflect the impact of stress (ischemia) on ventricular function. However, the performance of gated SPECT for both stress and resting states permits an evaluation of ventricular function during potentially differing physiologic states. A reduction of global LV function or regional wall thickness on the stress images has been shown to be a useful prognostic marker43,44

Gated SPECT Quality It is critical to have sufficient count density to examine for the accurate interpretation of gated SPECT data. A number of software programs analyze each of the

15-02-2022 17:04:11

Section 2  Radionuclide Myocardial Perfusion Imaging

248

frames to determine if the count density is adequate. The overall image quality should help the interpreter determine whether or not the study is interpretable. Another concern is that of poor ECG gating, such as with an arrhythmia, as it is manifested as a flashing on the rotating planar images45 (Fig. 14-13).

A

This is due to the variation in the beat–beat interval. For the most part, however, the overall global function data, including the ejection fraction, are still well preserved. If anything, cardiac arrhythmias more often affect the myocardial perfusion data than they do the gated SPECT information and its impact on

1

2

3

4

5

6

7

8

9

10

11

12

13

14

15

16

17

18

19

20

21

22

23

24

25

26

27

28

29

30

31

32

Spin 18

Spin

Spin

18

18

GS GATED SPECT PROJECTION

GS GATED SPECT PROJECTION

B GS GATED SPECT PROJECTION

1

100

0

100

1

100

FIGURE 14-13  (A) Thirty-two summed projections of gated SPECT study demonstrating multiple bright dots scattered in the field of view, especially the 16th−19th frames. (B) Last three frames of the 18th projection. These “dots” are due to an arrhythmia, with the shorter cardiac cycles having lower counts accumulation in the later frames. Normalization may then be applied for reduce the apparent “flashing, resulting in in the production of hot pixels, giving a sparkling appearance, known as a “flickering” artifact. (Reproduced with permission from Qutbi M. Flickering or hot-pixel artifact on gated myocardial perfusion SPECT imaging: an illustrated review for technologists. J Nucl Cardiol. 2017;25:671−7.)

CH14.indd 248

15-02-2022 17:04:14

Chapter 14  Interpretation and Reporting of SPECT and PET Myocardial Perfusion Imaging

249

A

B

FIGURE 14-14  Histogram of patients with (A) a regular rhythm and narrow range of beat rate and (B) with an irregular rhythm with a substantial portion of beats lying outside of the R–R interval window.

functional information. The use of a heart rate histogram may be useful in this setting to explain apparent count “drop out” (Fig. 14-14).

Semiquantitative Description Global and regional wall motion abnormalities should be defined as normal, hypokinetic, akinetic, or dyskinetic. It is possible to further subdivide the hypokinesis, although it may be difficult to differentiate between mild and moderate hypokinesis. A five-point scoring system for thickening and wall motion has been described ranging from normal function to mild, moderate or severe hypokinesis, to akinesis and dyskinesis. Usually,

CH14.indd 249

wall thickening correlates well with wall motion. However, following cardiac surgery, there is usually a reduced excursion of the septum, with normal thickening, a finding that is a normal variant.

Quantitative Analysis Global LV function may be accurately quantified and described specifically using an ejection fraction determination. Each software program has been well validated and reveals good correlation with other methodologies. When the ejection fraction is more than 70%, such as occurring in patients with small LV cavities, it is suggested to describe this as either “normal” or “≥70%,”

15-02-2022 17:04:14

250

Section 2  Radionuclide Myocardial Perfusion Imaging

as it is somewhat nonsensical to describe an ejection fraction of 92%. More qualitative descriptions may also be used such as “normal function” or those studies possessing mild, moderate, or severely reduced LV systolic function. However, given the overall validation of cardiac software packages, it is recommended to quantitatively describe the ejection fraction. LV volumes, both end-systolic and end-diastolic, may also be noted. Regional wall motion abnormalities and regional myocardial thickening have also been accurately quantified using several cardiac software packages. This information is often graphically depicted as a three-dimensional plot. This may be used to assist the interpreter in determinations of abnormal function. However, most of these methods have been less well validated than the global ejection fraction determination. Therefore, the tools for regional determinations should be used as an adjunctive technique to assist the interpreter.

▶▶Attenuation Correction A number of manufacturers possess well-validated methods for correcting soft tissue attenuation.19 Although different techniques have been employed by most vendors, the literature now supports the conclusion that diagnostic specificity is improved. In addition, attenuation correction may assist in the improved detection of multivessel disease and left main stenosis. It is also likely that attenuation correction will assist in prognostic applications. It is, however, critical to understand the workings of each system, as they are widely different. The interpreter of myocardial perfusion SPECT imaging should note each system’s benefits and potential limitations. It is critical that the quality of the transmission map used for correcting the emission data be of high quality (Fig. 14-15). The counts should be adequate, as determined by the manufacturer, and truncation should

Stress

Short axis

Vertical long axis

Stress AC

Stress Stress AC

Horizontal long axis

Stress Stress AC

FIGURE 14-15  Stress images obtained in a 49-year-old woman with a low likelihood for coronary artery disease. The stress images demonstrate an apparent anterior and anteroapical wall perfusion abnormality. Following attenuation correction (Vantage Pro™, Philips), a more uniform appearance is present (stress AC) and the images are clearly normal. (Reproduced with permission from Gary Heller, MD, PhD.)

CH14.indd 250

15-02-2022 17:04:16

Chapter 14  Interpretation and Reporting of SPECT and PET Myocardial Perfusion Imaging

be absent or minimal. If the quality of the transmission scan is suboptimal, the attenuation-corrected images should not be used. All attenuation correction methods may cause artifacts, especially when used incorrectly. It is critical to ensure that the attenuation correction

251

map, obtained by either CT or radioactive line source, is registered correctly to overlay the emission data, otherwise inaccurate correction may occur with the resultant creation of artifacts (Fig. 14-16). Therefore, the interpreter should know the specifics of such attenuation correction-derived

A

B

C

NM Transaxiails STRESS

NM Transaxials STRESS

Fused Transaxials STRESS

Fused Transaxials STRESS

FIGURE 14-16  Alignment of SPECT and CT-based attenuation correction. (A) An uncorrected Tc-99m tetrofosmin SPECT perfusion image. (B) Attenuation-corrected images but with poor registration of the SPECT emission data overlaid on the CT data, with a resultant lateral wall defect. (C) With proper registration, CT attenuation correction reveals normal myocardial perfusion. (Reproduced with permission from Abbott BG, Case JA, Dorbala S, et al. Contemporary cardiac SPECT imaging-innovations and best practices: An information statement from the American Society of Nuclear Cardiology. Circ Cardiovasc Imaging. 2018;11(9):e000020.)

CH14.indd 251

15-02-2022 17:04:17

252

Section 2  Radionuclide Myocardial Perfusion Imaging

64

63

65

64

66

66

62

Uncorrected

63

Attenuation-Corrected

65

64

artifacts, as well as its effectiveness in correcting soft tissue attenuation. It is recommended that the uncorrected SPECT images be examined in addition to the attenuationcorrected data.4,5 It is critical to understand the occurrence of the correction and its impact on the images. With the understandings of the benefits and limitations of each system, as well as comparing the uncorrected and corrected information, the interpreter can then gain the true value of the attenuation-corrected perfusion imaging. Caution is often advised when there is prominent activity from overlying structures such as the bowel or liver. This can directly impact on the interpretation of an inferior wall abnormality. Specifically, if substantial hepatic or subdiaphragmatic activity is present, a true inferior wall perfusion abnormality may be masked by attenuation correction. Importantly, apical thinning is also far more prominent on attenuation-corrected SPECT images (Fig. 14-17). It is, therefore, critical to understand the “normal” appearance of an attenuation-corrected

CH14.indd 252

FIGURE 14-17  Horizontal long-axis images of a patient with normal myocardial perfusion. The top two rows depict standard, noncorrected stress and rest SPECT images, with the pink arrow pointed at the apex. The bottom two rows are following attenuation correction, demonstrating an apical defect (yellow arrows), which is due to apical thinning and commonly seen following attenuation correction.

image. Likewise, the right ventricle is far more prominent on attenuation-corrected SPECT imaging. This does not reflect the right ventricular enlargement or hypertrophy, instead improves visualization of the structure. Obviously, a “learning curve” is required to understand the impact of attenuation correction on the LV apex and right ventricle.

▶▶Alternate Patient Positioning In addition to commercially available attenuation correction software, combined supine-prone myocardial perfusion SPECT without attenuation correction has been shown to enhance the recognition of soft tissue attenuation and increase the specificity and diagnostic accuracy in the detection of CAD, especially in women and obese patients.15,16 By shifting the heart anteriorly and the diaphragm and subdiaphragmatic organs inferiorly, prone position improves inferior wall attenuation artifact. However, prone-only MPI is not recommended due to possible artifactual anterior and anteroseptal

15-02-2022 17:04:18

Chapter 14  Interpretation and Reporting of SPECT and PET Myocardial Perfusion Imaging

253

S U

S

U

S

U

S U

defects from the close proximity of the myocardium to anterior bony structures, for example, ribs and sternum, resulting in false-positive results. The use of combined supine-prone MPI has similar prognostic values compared to attenuationcorrected MPI. The presence of perfusion defects on supine acquisitions but normal on prone images indicates a good prognosis and a low risk for subsequent cardiovascular events, similar to that of patients with normal supine-only studies.15,16 With the development of upright or semiupright camera systems, prone imaging is not possible, but the use of upright and supine position offers the opportunity to recognize artifacts in a fashion similar to supine/prone imaging (Fig. 14-18). This is especially valuable for obese patients, as it allows for improved artifact recognition and a reduced requirement for rest images.46

▶▶Solid-State Detector Cadmium Zinc Telluride Cameras The interpretation of images obtained with CZT detectors is similar to conventional SPECT, although the markedly improved camera sensitivity is likely to

CH14.indd 253

FIGURE 14-18  Supine (S; first rows) and upright (U; second rows) SPECT images obtained from D_SPECT camera illustrating a prominent inferior wall abnormality that almost completely resolves with upright positioning. (Reproduced with permission from Ben-Haim S, Almukhailed O, Neill J, et al. Clinical value of supine and upright myocardial perfusion imaging in obese patients using the D-SPECT camera. J Nucl Cardio. 2014;21:478-85.)

produce images of enhanced quality, with myocardial walls appearing thinner and an enlarged LV cavity. In cameras containing multiple detectors, review of the traditional cine of planar images is not possible but a “panogram” provides the opportunity to gauge patient motion (Fig. 14-3). Upright imaging may not only allow for less interference with extracardiac, splanchnic activity, but may also introduce unique artifacts such as those seen in the inferoapical region; this may be recognized by performing both upright and supine imaging (Fig. 14-18).47

▶▶Incorporation of Clinical Data Beyond information regarding the patient’s body habitus, clinical data, including risk factors, may influence image interpretation. As such, these data should be considered only after careful image interpretation. How this information should be weighed with regard to final impression is controversial, although most experts recommend not including this in the final conclusion. Others have suggested that the clinical information may slightly alter the conclusion, such as from an equivocal diagnosis to one with slightly increased clarity. Data regarding

15-02-2022 17:04:19

254

Section 2  Radionuclide Myocardial Perfusion Imaging

prior cardiac events, including revascularization and myocardial infarction (MI), must be considered so as to add to the clinical relevance of the study.

▶▶Artificial Intelligence Recently, artificial intelligence (AI) tools have been developed to aid in image acquisition, quantification, interpretation, and reporting. These instruments incorporate computer-based algorithms, which mimic human intelligence but incorporate huge databases and collective knowledge so as to recognize image patterns. AI includes machine learning, which uses existing observations and outcomes to more accurately predict future observations, and deep learning, which utilizes neural networks for image interpretation. A more detailed discussion of AI and its applications to nuclear cardiology may be found in Chapter 15.

▶▶Summary—Interpretation A systematic process of interpretation of myocardial perfusion SPECT images is critical for the highest level of accuracy and clinical utility. Attention must be paid to the quality of MPI data. In recent years, a variety of guidelines and position papers have focused on optimal techniques for image interpretation.1–6 A complete listing of all items to be considered, including those that are standard, recommended, or optional, may be found in the most recent version of the ASNC guidelines for image interpretation.6 A common vocabulary and format also ensure that high-quality interpretation and reporting are achieved. Advanced imaging technology, including quantitative analysis, gated SPECT, attenuation correction, and AI, has improved the value of SPECT imaging, but the “reader” of such images must be cognizant of the advantages and pitfalls for these methods and understand the potential impact of these techniques on the final interpretation.

REPORTING OF SPECT MYOCARDIAL PERFUSION IMAGING The final product of a nuclear cardiology procedure is the report. This directly reflects not only on the performance of the imaging study performed, but also on its interpretation. It is ultimately the critical piece

CH14.indd 254

of information that guides patient management. As such, it is essential that the report be inclusive yet clear in its meaning. The quality of the report reflects not only on the interpreting physician, but also on the nuclear cardiology laboratory and on the field of nuclear cardiology itself. Many current nuclear cardiology reports fall far short of the mark of clarity. Words such as “suggestive of,” “possible,” and other qualifiers should be avoided. In addition, the unique descriptions of perfusion abnormalities such as describing the severity as “>2” or the use of words such as paradoxical should be removed. While the intent of the reader may be well known within one institution, significant problems may arise should the report migrate out of the individual laboratory’s usual realm. In describing the location of various perfusion abnormalities, consistency must be present and words such as posterior are now not recommended. Finally, ubiquitous and somewhat insulting comments, such as “clinical correlation is suggested,” should be avoided. The latter phrase need not be included, as the correlation of all imaging results should be performed with regard to the clinical history. Finally, descriptions such as “of unknown significance” or “equivocal” should be restricted to the most extreme of the circumstances. A number of publications have delineated the importance of a nuclear cardiology report and guidelines have been proposed to define the components of the content of this report.2,6,42,48–50 These guidelines are composed of tables listing variables, their descriptions, priority of inclusion within the report, and the permissible responses.6 Obviously, individuality is critical, and reports will need to be adjusted to suit the laboratory, as well as the individual nuclear cardiologist. However, some standardization is critical so as to optimize the information and to provide the most clinically relevant data to the referring physician. ASNC strongly supports the use of SR to improve communication and reporting of nuclear cardiology procedures.6 Currently, reports are often variable in content and in form. While this is acceptable, ambiguity and a lack of an impression are not helpful to the patient, the referring physician, or nuclear cardiology. The most important goal of a nuclear cardiology report should be the communication of critical findings to the referring physician and their clinical implications. In addition, the report should serve

15-02-2022 17:04:19

Chapter 14  Interpretation and Reporting of SPECT and PET Myocardial Perfusion Imaging

to document information that is pertinent to reimbursement and accreditation licensure. The latter includes drug management and radiation safety. Finally, certification and accreditation are based often on nuclear cardiology reports, and organizations such as IAC-Nuclear/PET (http://www.intersocietal. org/nuclear/) will review those reports to ensure that they are consistent with the standards of nuclear cardiology.51 The key elements of the report, as mandated by Intersocietal Accreditation Commission (IAC), are shown in Table 14-5. A majority of the nuclear

Table 14-5 Reporting Standards-Mandatory Data Elements (Based on IAC Standards) • Name, Address, and Phone Number of Imaging facility • Name of examination • Patient information including full name, sex, DOB, height/weight, or BMI • Requesting provider • Interpreting physician • Date of examination • Clinical indications and pertinent history • Procedural description, including radionuclide amount and route of administration, time from injection to imaging • Administered nonradioactive pharmaceuticals, include name, dose, and route, and timing • Anatomic area imaged • Views obtained, i.e., SPECT or SPECT/CT • CT procedure, if applicable, including technique, dose, and contrast • Stress test results, including protocol, duration, peak hemodynamic parameters, stress symptoms, ECG findings at rest and stress, and percent maximum predicted heart rate • Image quality and explanation of suboptimal studies • Image results including defect description as to size/ extent, severity, location (17-segment model) and type (fixed, reversible, mixed), as well as functional results (quantitative LVEF, global and regional wall motion). • Comparison to other imaging or nonimaging studies, or inclusion of statement that no prior studies were available. • Conclusion/Impression (summary of perfusion and functional findings) Data from Intersocietal Accreditation Commission (IAC). The IAC standards and guidelines for nuclear/PET accreditation. http://www. intersocietal.org/nuclear/standards/IACNuclearPETStandards2016.pdf. Published September 15, 2016.

CH14.indd 255

255

laboratories across the nation were reported to be noncompliant, with the reporting standards in at least one element. However, compliance improved in laboratories that applied for accreditation in sequential cycles.52 This topic is further discussed in Chapter 7.

▶▶Components of the Report Patient Data Patient name and date of birth, as well as name, contact information, and affiliation of the referring physician, must be present. Information that is important not only to the interpretation of the study, but also to provide clinical relevance, must be included in the report (Table 14-6). This includes the age, sex, and body habitus of the patient. Height, weight, body surface area, and chest circumference are items that may be included. Relevant clinical information including past cardiac history such as MI or the performance of revascularization should be included. In addition, major cardiac risk factors should be noted. The patient’s current medications should be included, as they may impact the interpretation of the results. They also provide documentation of the current clinical status of the subject.

Indications The indication for the procedure should be fairly delineated to allow full understanding of why the study was performed, as well as to demonstrate the appropriateness of the study based on the previously

Table 14-6 Clinical Information • Demographics (age, sex, and race) • Body habitus (height and weight) • Symptoms • Medications • Cardiac risk factors • Prior cardiac events • Prior diagnostic tests • Therapeutic cardiac procedures • Primary (and secondary) indications • Appropriate use documentation

15-02-2022 17:04:19

256

Section 2  Radionuclide Myocardial Perfusion Imaging

described ACCF/ASNC appropriateness criteria for SPECT MPI and ACCF/AHA/ASE/ASNC/HFSA/ HRS/SCAI/SCCT/SCMR/STS multimodality appropriate-use criteria for the detection and risk assessment of stable ischemic heart disease.53–55 Ideally, this should be obtained from the report, as well as from the patient history. The key indications for the procedure include the following:

• Diagnosis of CAD • Evaluation of the extent and severity of known • • • • •

CAD Risk assessment, including perioperative risk Determination of myocardial viability Assessment of acute chest pain syndromes Evaluation of structural heart disease Assessment of heart failure

The actual indication may also be refer to a specific diagnostic code to assist in matters related to reimbursement, such as providing documentation of medical necessity. Many laboratories include the ICD10 codes directly in the report. A comprehensive list of indications is found in the 2017 ASNC Imaging Guidelines.6 If pharmacologic testing is performed, the reason why exercise testing was not performed should be included. Given increased emphasis on resource utilization, the report should also contain appropriate use criteria documentation, including the specific appropriate use criteria selected and the category of appropriateness.

Procedure Procedures should be well delineated in the report so as to provide a frame of reference for subsequent comparisons, as well as to explain the testing results directly to the referring physician (Table 14-7). The mode of stress should be noted. If it is exercise, the specific protocol such as Naughton or Bruce should be noted. The duration of exercise should be stated, as well as the protocol stage achieved. Finally, the number of metabolic equivalents (METS) should be specified. The adequacy of the test results should be stated, such as the target heart rate achieved in terms of the maximum predicted heart rate and the reasons for test termination, such as general fatigue or hypotension. If pharmacologic stress testing is performed,

CH14.indd 256

Table 14-7 Procedure • Type and protocol of stress procedure • Adequacy of results • Symptoms during protocol • Hemodynamic response (heart rate and blood pressure) • ECG changes • Reason for test termination • Radiopharmaceuticals utilized (with dose) • Imaging protocol • Attenuation/scatter correction • Functional data

the agent and dose should be specified. In addition, whether adjunctive exercise was performed and its type should be noted. The presence of symptoms, hemodynamic responses, as well as arrhythmias, should be well delineated. The heart rate, as well as blood pressure changes, should be noted. Electrocardiographic changes, including those that deviate from the baseline ECG, should be mentioned. The resting ECG should be stated, especially if there are abnormalities noted such as left bundle branch block (LBBB), LVH, or nonspecific ST/T-wave abnormalities. The content of the report may vary if separate reports are generated for the stress test and the perfusion imaging data. However, even when a separate stress test report is created, it is advised that the perfusion data be reported along with a minimum of stress data including: exercise duration, maximum heart rate and percent maximum predicted heart rate, blood pressure response, symptoms, and ECG findings. The MPI agent and dose administered at peak stress and at rest should be noted. If the stress is continued after the radiopharmaceutical injection, the duration of the continuation of stress should be mentioned. The actual imaging protocol should also be stated. For example, planar or SPECT, as well as whether the imaging was performed in a supine, prone, or upright position, should be stated. The reason must be provided if the routine protocol was replaced with an alternative protocol (e.g., attenuation correction, patient motion, or body habitus). A 1- or 2-day protocol should be specified and the

15-02-2022 17:04:19

Chapter 14  Interpretation and Reporting of SPECT and PET Myocardial Perfusion Imaging

radiopharmaceutical should be documented. If gated SPECT or first-pass imaging was performed, again this should be well described and noted whether it was performed at stress or rest. If stress-only imaging was obtained, then it should be clearly stated. Finally, advanced imaging techniques such as attenuation correction should be noted.

Image Findings The results or findings portion of the report should provide an in-depth discussion of all findings noted on review of the nuclear cardiology study. It should be comprehensive and attempt to describe all pertinent findings (Table 14-8). The first portion of the results section should deal with image quality. This may be stated as excellent, good, fair, or poor quality, but at least inadequate quality should be noted. Extracardiac activity should also be described and correlated with any potential clinical information. The perfusion defect characteristics should be carefully described. The well-defined 17-segment model should be used to describe the size, severity, and location of perfusion abnormalities (Fig. 14-9).1 The location of the abnormality in terms of segmentation and vascular territory should be described to the best of the interpreter’s abilities and using standard nomenclature.4,5 Clear delineation between single- and multivessel disease should be performed. Defect size should be noted as small (1–2 segments), medium (3–4 segments), or large (≥5 segments) (Table 14-4).56 Further defect description should include the type (ischemic, reversible, persistent, and mixed) and severity (mild, moderate, and severe) of the finding. The European Association of Nuclear Medicine and the European Association Table 14-8 Image Findings • Study quality • Defect description (size, reversibility, severity, and location) • Extensiveness (TID, lung activity, and right ventricular activity) • Left ventricular function (global and regional) • Extracardiac activity

CH14.indd 257

257

of Cardiovascular Imaging (EACVI) strongly recommend that reports should be written in simple language with limited use of technical terms and abbreviations, and qualitative descriptions (e.g., small-, medium-, or large-sized, or slightly, moderately, or severely reduced) should be replaced with quantitative assessment.57 Quantitative image processing is recommended, as is the reporting of such information. This may be performed by documenting findings, such as the SSS or perfusion defect score, which provides the percentage of myocardial involvement (see Chapter 15 for additional details). Markers of extensive disease, such as multivessel distribution or the presence of abnormal tracer distribution in the lungs, should be carefully noted. In addition, the presence of cavity enlargement, either immediately following stress or on both stress and rest images, should be described. If TID is noted, the TID ratio should be contained in the report. LV function assessment is a mainstay of MPI. Both global and regional function should be described qualitatively, as well as quantitatively. The left ventricular ejection fraction (LVEF) should be stated if gated SPECT was performed. If the LVEF is between 50% and 70%, the interpreter may describe this as normal, as well as report the quantitative value. Hyperdynamic function is defined as >70%, but it is well recognized that this finding is common with gated SPECT and is often related to small LV cavities. Defining function as “hyperdynamic” or more than 70% may be superior to nonsensical values, such as an LVEF of 91%. Mild, moderate, and severe LV dysfunctions are defined as 40% to 49%, 30% to 39%, and less than 30%, respectively. Both the rest and post-stress functions should be noted, if available. Regional defects should be carefully described as hypokinetic, akinetic, or dyskinetic, and the location given. Optionally, images may be included in the final report if they clearly represent the finding and impression portions of the report without causing confusion to the clinician. A standardized scale should be used and only a limited number of images should be selected.57 Findings related to perfusion, size, and function of the right ventricle may be reported, especially if specific indications for their assessment are present.

15-02-2022 17:04:19

258

Section 2  Radionuclide Myocardial Perfusion Imaging

Conclusion/Impression The impression is frequently the only portion of the report read by the referring physician (Table 14-9). It is therefore critical that it be crisp in its meaning, and the final diagnosis must be clear. Most reports should begin the impression section with a statement that perfusion imaging is either normal or abnormal. The terms “equivocal,” “possible,” or “probable” should be used infrequently.2,6,50 A study may still be interpreted as normal even if there are perfusion abnormalities noted in the results section (Tables 14-8 and 14-9). This discrepancy should be explained in the impression section briefly by commenting on whether or not this was believed to be due to an artifact, such as soft tissue, LBBB, or patient motion. Conclusions regarding perfusion defects must contain a statement of whether these findings indicate ischemia, infarction, or both. Additionally, the number of coronary territories and possibly specific vascular distributions should be noted, if possible. The functional information should be incorporated in a brief manner describing whether LV function is reduced and whether regional wall motion abnormalities are present. The amount of LV dysfunction should be semiquantitated, such as stating that there is moderately reduced LV systolic function. Finally, the perfusion and function findings must be integrated in the final impression, as wall motion may help to distinguish between an attenuation artifact and a true fixed defect, consistent with scar. In addition to conclusions about the perfusion and ventricular function data, the “Impression” section must also summarize the results of stress testing,

Table 14-9 Impression • Normal or abnormal (minimum use of “equivocal”) • Recognition of artifact, if relevant • Global and regional LV function • Summary and correlation of ECG findings • Integration with clinical information • Address the clinical indication for testing

CH14.indd 258

including the significance of electrocardiographic findings. The “Conclusion” section is also the place for correlation of the perfusion imaging data with clinical information, data from the results of the stress test, and any correlation with angiographic data, as it is known. A comparison to prior studies should be undertaken, and a direct statement regarding any significant changes from previous studies should be made. As noted in the recent ASNC reporting guidelines, ECG and imaging findings should be described as concordant or discordant, although the latter may be challenging especially when the ECG findings are abnormal but the imaging is negative6 As the diagnostic accuracy of perfusion imaging is superior to that of ECG stress testing, an abnormal ECG response to exercise with normal perfusion images should be considered to be a false-positive, especially in the setting of resting ST-segment abnormalities. Of note, however, is that ECG changes occurring during a vasodilator infusion may portend a worse prognosis, even in the setting of normal SPECT images.58 Finally, the impression must directly address the question that was asked. Therefore, specific comments should be made regarding the indications and reasons for performing the procedure. For example, many laboratories performing a diagnostic study will comment directly on the likelihood of CAD. Some practitioners, however, are concerned about the legal ramifications of such a statement and opt not to include this type of language. Following an MI, the findings of a second vascular distribution and/or peri-infarction ischemia should be noted and equated with an increased risk of subsequent cardiac events. Likewise, perioperative assessments should comment specifically in the report on whether an increased risk for perioperative cardiac complications is present based on the study. For acute imaging procedures, the interpreter should note whether there is any evidence of ongoing ischemia or MI. It is now encouraged that reports provide information about the clinical risk for a specific patient, based on the scintigraphic findings. This has been outlined in multiple publications and is often used for defining the appropriate use of coronary revascularization.59 Specific

15-02-2022 17:04:19

Chapter 14  Interpretation and Reporting of SPECT and PET Myocardial Perfusion Imaging

Table 14-10 Clinical Risk Statements • Low (70%)

Noninvasive stress testing ± Invasive angiography

FIGURE 17-12  Coronary artery disease (CAD) severity identified by coronary CT angiography and recommended management. Patients with a normal coronary CT angiography can be safely reassured. Follow-up for preventive therapy is recommended for nonobstructive (65 years of age. Am J Cardiol. 2012;110:1092–1099. 29. Froelicher VF, Lehmann KG, Thomas R, et al. The electrocardiographic exercise test in a population with reduced workup bias: diagnostic performance, computerized interpretation, and multivariable prediction. Ann Intern Med. 1998;128:965. 30. Berman DS, Hachmovitch R. Risk assessment in patients with stable coronary artery disease: Incremental value of nuclear imaging. J Nucl Cardiol. 1996;3:S41–S49. 31. Hachamovitch R, Berman DS, Kiat H, et al. Exercise myocardial perfusion spect in patients without known coronary artery disease: incremental prognostic value and use in risk stratification. Circulation. 1996;93:905–914. 32. Gianrossi R, Detrano R, Mulvihill D, et al. Exercise-induced ST depression in the diagnosis of coronary artery disease. A meta-analysis. Circulation. 2018;80:87–98. 33. Fleischmann KE, Hunink MGM, Kuntz KM, Douglas PS. Exercise echocardiography or exercise spect imaging? A metaanalysis of diagnostic test performance. JAMA. 1998;280:913. 34. Parker MW, Iskandar A, Limone B, et al. Diagnostic accuracy of cardiac positron emission tomography versus single photon emission computed tomography for coronary artery disease: a bivariate meta-analysis. Circulation Cardiovasc Imaging. 2012;5: 700–707. 35. Iskandar A, Limone B, Parker MW, et al. Gender differences in the diagnostic accuracy of SPECT myocardial perfusion imaging: a bivariate meta-analysis. J Nucl Cardiol Official Publ Am Soc Nucl Cardiol. 2012;20:53–63. 36. Leppo J. Comparison of pharmacologic stress agents. J Nucl Cardiol. 1996;3:S22–S26. 37. Borges-Neto S, Mahmarian JJ, Jain A, Roberts R, Verani MS. Quantitative thallium-201 single photon emission computed tomography after oral dipyridamole for assessing the presence, anatomic location and severity of coronary artery disease. J Am Coll Cardiol. 1988;11:962–969. 38. Thomas G, Tammelin BR, Schiffman GL, et al. Safety of regadenoson, a selective adenosine A2A agonist, in patients with chronic obstructive pulmonary disease: A randomized, double-blind, placebo-controlled trial (RegCOPD trial). J Nucl Cardiol. 2008;15:319–328. 39. Hage FG, Ghimire G, Lester D, et al. The prognostic value of regadenoson myocardial perfusion imaging. J Nucl Cardiol Official Publ Am Soc Nucl Cardiol. 2015;22:1214–1221. 40. Cerqueira MD, Nguyen P, Staehr P, et al. Effects of age, gender, obesity, and diabetes on the efficacy and safety of the selective A2A agonist regadenoson versus adenosine in myocardial perfusion imaging integrated ADVANCE-MPI trial results. JACC Cardiovasc Imaging. 2008;1:307–316. 41. Iskandrian AE, Bateman TM, Belardinelli L, et al. Adenosine versus regadenoson comparative evaluation in myocardial perfusion imaging: Results of the ADVANCE phase 3 multicenter international trial. J Nucl Cardiol. 2007;14:645–658.

15-02-2022 14:15:09

Chapter 17  Evaluation of Patients with Suspected Coronary Artery Disease 42. Mahmarian JJ, Peterson LE, Xu J, et al. Regadenoson provides perfusion results comparable to adenosine in heterogeneous patient populations: A quantitative analysis from the ADVANCE MPI trials. J Nucl Cardiol. 2014;22:248–261. 43. Hays JT, Mahmarian JJ, Cochran AJ, Verani MS. Dobutamine thallium-201 tomography for evaluating patients with suspected coronary artery disease unable to undergo exercise or vasodilator pharmacologic stress testing. J Am Coll Cardiol. 1993;21:1583–1590. 44. Shehata AR, Gillam LD, Mascitelli VA, et al. Impact of acute propranolol administration on dobutamine-induced myocardial ischemia as evaluated by myocardial perfusion imaging and echocardiography. Am J Cardiol. 1997;80:268–272. 45. Marwick TH, Cho I, Hartaigh BÓ, Min JK. Finding the gatekeeper to the cardiac catheterization laboratory: coronary CT angiography or stress testing? J Am Coll Cardiol. 2015;65:2747–2756. 46. Choi JY, Lee KH, Kim SJ, et al. Gating provides improved accuracy of differentiating artifacts from true lesions in equivocal fixed defects on technetium 99m tetrofosmin perfusion SPECT*1, *2. J Nucl Cardiol. 1998;5:395–401. 47. Smanio PEP, Watson DD, Segalla DL, et al. Value of gating of technetium-99m sestamibi single-photon emission computed tomographic imaging. J Am Coll Cardiol. 30:1687–1692. 48. Taillefer R, DePuey EG, Udelson JE, et al. Comparative diagnostic accuracy of Tl-201 and Tc-99m sestamibi SPECT imaging (perfusion and ECG-gated SPECT) in detecting coronary artery disease in women. J Am Coll Cardiol. 1997;29:69–77. 49. Taasan V, Wokhlu A, Taasan MV, et al. Comparative accuracy of supine-only and combined supine-prone myocardial perfusion imaging in men. J Nucl Cardiol. 2015;23:1470–1476. 50. Hayes SW, Lorenzo A De, Hachamovitch R, et al. Prognostic implications of combined prone and supine acquisitions in patients with equivocal or abnormal supine myocardial perfusion SPECT. J Nucl Med Official Publ Soc Nucl Med. 2003;44:1633–1640. 51. Gutstein A, Bental T, Solodky A, Mats I, Zafrir N. Prognosis of stress-only SPECT myocardial perfusion imaging with prone imaging. J Nucl Cardiol Official Publ Am Soc Nucl Cardiol. 2016;25:809–816. 52. Hendel RC, Berman DS, Cullom SJ. Multicenter clinical trial to evaluate the efficacy of correction for photon attenuation and scatter in SPECT myocardial perfusion imaging. Circulation. 1999;99(21):2742–2749. 53. Heller GV, Bateman TM, Johnson LL, et al. Clinical value of attenuation correction in stress-only Tc-99m sestamibi SPECT imaging. J Nucl Cardiol. 2004;11:273–281. 54. Huang J.-Y, Huang C-K, Yen R-F, et al. Diagnostic performance of attenuation-corrected myocardial perfusion imaging for coronary artery disease: a systematic review and meta-analysis. J Nucl Med. 2016;57:1893–1898. 55. Ardestani A, Ahlberg AW, Katten DM, et al. Risk stratification using line source attenuation correction with rest/stress Tc99m sestamibi SPECT myocardial perfusion imaging. J Nucl Cardiol. 2014;21:118–126. 56. Hendel RC, Heller GV, Links J, et al. American society of nuclear cardiology and society of nuclear medicine joint position statement. J Nucl Cardiol. 2002;9:135–143. 57. Bateman TM, Dilsizian V, Beanlands RS, et al. American Society of Nuclear Cardiology and Society of Nuclear Medicine and Molecular Imaging Joint Position Statement on the Clini-

CH17.indd 321

321

cal Indications for Myocardial Perfusion PET. J Nucl Med. 2016;57:1654–1656. 58. McArdle BAM, Dowsley TF, deKemp RA, Wells GA, Beanlands RS. Does rubidium-82 PET have superior accuracy to SPECT perfusion imaging for the diagnosis of obstructive coronary disease? A systematic review and meta-analysis. J Am Coll Cardiol. 2012;60:1828–1837. 59. Gowd BMP, Heller GV, Parker MW. Stress-only SPECT myocardial perfusion imaging: a review. J Nucl Cardiol Official Publ Am Soc Nucl Cardiol. 2014;21:1200–1212. 60. Vanzetto G, Ormezzano O, Fagret D, et al. Long-Term additive prognostic value of thallium-201 myocardial perfusion imaging over clinical and exercise stress test in low to intermediate risk patients: Study in 1137 patients with 6-year followup. Circulation. 1999;100:1521–1527. 61. Winther S, Schmidt SE, Mayrhofer T, et al. Incorporating coronary calcification into pre-test assessment of the likelihood of coronary artery disease. J Am Coll Cardiol. 2020;76:2421– 2432. 62. Anand DV, Lim E, Hopkins D, et al. Risk stratification in uncomplicated type 2 diabetes: prospective evaluation of the combined use of coronary artery calcium imaging and selective myocardial perfusion scintigraphy. Eur Heart J. 2006;27:713–721. 63. Budoff MJ, Dowe D, Jollis JG, et al. Diagnostic performance of 64-multidetector row coronary computed tomographic angiography for evaluation of coronary artery stenosis in individuals without known coronary artery disease: Results from the prospective multicenter ACCURACY (assessment by coronary computed tomographic angiography of individuals undergoing invasive coronary angiography) trial. J Am Coll Cardiol. 2008;52:1724–1732. 64. Arbab-Zadeh A, Miller JM, Rochitte CE, et al. Diagnostic accuracy of ct coronary angiography according to pretest probability of coronary artery disease and severity of coronary arterial calcification: The CorE-64 International, Multicenter Study. J Am Coll Cardiol. 2012;59:379–87. 65. Min JK, Dunning A, Lin FY, et al. Age- and sex-related differences in all-cause mortality risk based on coronary computed tomography angiography findings results from the International Multicenter CONFIRM (coronary ct angiography evaluation for clinical outcomes: An International Multicenter Registry) of 23,854 patients without known coronary artery disease. J Am Coll Cardiol. 2011;58:849–860. 66. Douglas PS, Hoffmann U, Patel MR, et al. Outcomes of anatomical versus functional testing for coronary artery disease. N Engl J Medicine. 2015;372:1291–1300. 67. SCOT HEART Investigators. CT coronary angiography in patients with suspected angina due to coronary heart disease (SCOT-HEART): an open-label, parallel-group, multicentre trial. Lancet. 2015;385:2383–2391. 68. SCOT HEART Investigators, Adamson PD, Berry C, et al. Coronary CT angiography and 5-year risk of myocardial infarction. N Engl J Med. 2018;379:924–933. 69. Divakaran S, Cheezum MK, Hulten EA, et al. Use of cardiac CT and calcium scoring for detecting coronary plaque: implications on prognosis and patient management. Br J Radiology. 2015;88:20140594. 70. Saraste A, Barbato E, Capodanno D, et al. Imaging in ESC clinical guidelines: chronic coronary syndromes. European Hear J - Cardiovasc Imaging. 2019;20:1187–1197.

15-02-2022 14:15:09

322

Section 3  Indications and Applications

71. Williams MC, Hunter A, Shah ASV, et al. Use of coronary computed tomographic angiography to guide management of patients with coronary disease. J Am Coll Cardiol. 2016;67:1759–1768. 72. Lee JM, Choi KH, Koo B-K, et al. Prognostic implications of plaque characteristics and stenosis severity in patients with coronary artery disease. J Am Coll Cardiol. 2019;73:2413–2424. 73. Oikonomou EK, Marwan M, Desai MY, et al. Non-invasive detection of coronary inflammation using computed tomography and prediction of residual cardiovascular risk (the CRISP CT study): a post-hoc analysis of prospective outcome data. Lancet. 2018;392:929–939. 74. Chen MY, Rochitte CE, Arbab-Zadeh A, et al. Prognostic value of combined CT angiography and myocardial perfusion imaging versus invasive coronary angiography and nuclear stress perfusion imaging in the prediction of major adverse cardiovascular events: The CORE320 Multicenter Study. Radiology. 2017;284:55–65. 75. Driessen RS, Danad I, Stuijfzand WJ, et al. Comparison of coronary computed tomography angiography, fractional flow reserve, and perfusion imaging for ischemia diagnosis. J Am Coll Cardiol. 2019;73:161–173. 76. Gligorova S, Agrusta M. Pacing stress echocardiography. Cardiovasc Ultrasoun. 2005;3:36. 77. Picano E, Lattanzi F. Dipyridamole echocardiography. A new diagnostic window on coronary artery disease. Circulation. 1991;83:III19–III26. 78. Wei K, Ragosta M, Thorpe J, et al. Noninvasive quantification of coronary blood flow reserve in humans using myocardial contrast echocardiography. Circulation. 2001;103:2560–2565. 79. Abdelmoneim SS, Dhoble A, Bernier M, et al. Quantitative myocardial contrast echocardiography during pharmacological stress for diagnosis of coronary artery disease: a systematic review and meta-­analysis of diagnostic accuracy studies. European J ­Echocardiogr J Work Group Echocardiogr Eur Soc Cardiol. 2009;10:813–825. 80. Verani MS. Stress myocardial perfusion imaging versus echocardiography for the diagnosis and risk stratification of patients with known or suspected coronary artery disease. Semin Nucl Med. 1999;29:319–329. 81. Quiñones MA, Verani MS, Haichin RM, et al. Exercise echocardiography versus 201Tl single-photon emission computed

CH17.indd 322

tomography in evaluation of coronary artery disease. Analysis of 292 patients. Circulation. 1992;85:1026–1031. 82. Sicari R, Nihoyannopoulos P, Evangelista A, et al. Stress Echocardiography Expert Consensus Statement—Executive Summary: European Association of Echocardiography (EAE) (a registered branch of the ESC). Eur Heart J. 2008;30:278–289. 83. Picano E, Bedetti G, Varga A, Cseh E. The comparable diagnostic accuracies of dobutamine-stress and dipyridamolestress echocardiographies: a meta-analysis. Coronary Artery Dis. 2000;11:151–159. 84. Picano E, Molinaro S, Pasanisi E. The diagnostic accuracy of pharmacological stress echocardiography for the assessment of coronary artery disease: a meta-analysis. Cardiovasc Ultrasoun. 2008;6:30. 85. Rainbird AJ, Mulvagh SL, OH JK, et al. Contrast dobutamine stress echocardiography: Clinical practice assessment in 300 consecutive patients. J Am Soc Echocardiog. 14, 378–385. 86. Porter TR, Xie F, Kricsfeld A, Chiou A, Dabestani A. Improved endocardial border resolution during dobutamine stress echocardiography with intravenous sonicated dextrose albumin. J Am Coll Cardiol. 1994;23:1440–1443. 87. Nandalur KR, Dwamena BA, Choudhri AF, Nandalur MR, Carlos RC. Diagnostic performance of stress cardiac magnetic resonance imaging in the detection of coronary artery disease. J Am Coll Cardiol. 2007;50:1343–1353. 88. Schwitter J, Wacker CM, van Rossum AC, et al. MR-IMPACT: comparison of perfusion-cardiac magnetic resonance with single-photon emission computed tomography for the detection of coronary artery disease in a multicentre, multivendor, randomized trial. Eur Heart J. 2008;29:480–489. 89. Taylor A, Keegan J, Jhooti P, Firmin D, Pennell D. Calculation of a subject-specific adaptive motion-correction factor for improved real-time navigator echo-gated magnetic resonance coronary angiography. J Cardiov Magn Reson. 1999;1:131–138. 90. Sakuma H, Ichikawa Y, Chino S, et al. Detection of coronary artery stenosis with whole-heart coronary magnetic resonance angiography. J Am Coll Cardiol. 2006;48:1946–1950. 91. Arai AE, Schulz-Menger J, Berman D, et al. Gadobutrol enhanced cardiac magnetic resonance imaging for detection of coronary artery disease. J Am Coll Cardiol. 2020;76:1536–1547.

15-02-2022 14:15:09

Evaluation of Patients with Known Coronary Artery Disease Javier Gomez and Rami Doukky

KEY POINTS ■■

■■

■■

■■

■■

CH18.indd 323

In patients with known coronary artery disease (CAD), abnormal single-photon emission computed tomography (SPECT) or positron emission tomography (PET) myocardial perfusion imaging (MPI) is associated with an increase in risk of adverse cardiac events, proportional to the extent and severity of the perfusion abnormalities. SPECT and PET MPI can determine extent, severity, and localization of perfusion abnormalities in patients with multivessel CAD or prior revascularization procedures, guiding management strategies. Routine MPI testing after coronary revascularization in patients with stable or no symptoms is not supported by current guidelines. SPECT and PET MPI provide safe and effective risk stratification in patients presenting with acute coronary syndromes or myocardial infarction who have not received or have a contraindication to invasive coronary angiography. SPECT and PET MPI can be useful to assess response to medical therapy in patients with established CAD.

CHAPTER

18

In recent decades, there has been a dramatic decline in the rate of death attributable to cardiovascular disease.1 This trend is credited primarily to the development and implementation of effective treatment strategies, including medical therapy, interventions such as coronary artery bypass graft (CABG) surgery and percutaneous coronary interventions (PCI), and successful treatment for acute myocardial infarction. With these advances in mind, the decision to undergo myocardial perfusion imaging (MPI) evaluation in patients with known ischemic heart disease is an important one, particularly in asymptomatic patients. This chapter will evaluate the role of stress MPI in patients with known coronary artery disease (CAD) in a variety of settings, including medical therapy, postinterventions (CABG, PCI), and following myocardial infarction.

MYOCARDIAL PERFUSION IMAGING AND CHRONIC ISCHEMIC HEART DISEASE

▶▶Indications for Stress Myocardial Perfusion Imaging Stress testing is an important tool in the longitudinal assessment of patients with known CAD, especially when there is a change in the frequency or pattern of symptoms. Once the decision has been made to

15-02-2022 11:21:47

324

Section 3  Indications and Applications

further assess the patient, the choice comes to stress with or without imaging. The use of exercise tolerance testing (ETT) without imaging to make management decisions in this patient population requires several important considerations. The American College of Cardiology/American Heart Association (ACC/AHA) guidelines for ETT strongly recommend an imaging study as part of the evaluation in patients with baseline EGG abnormalities (pre-excitation, paced ventricular rhythm, ≥1 mm resting ST-segment depression, and complete left bundle branch block [LBBB]).2 In addition, in the CAD population, over 60% of candidates for stress are unable to complete an exercise protocol, therefore requiring pharmacologic stress with imaging. The use of digoxin, presence of left ventricular hypertrophy, or any resting ST-segment depression decreases the specificity of exercise testing, while sensitivity may remain unaffected.2 Importantly, several other subsets of patients benefit incrementally with the use of radionuclide imaging, including patients with previous myocardial infarction (MI) and/or coronary revascularization procedures (CABG or PCI), patients with prior angiography demonstrating significant disease (where identification of the lesion causing myocardial ischemia is important), individuals with high risk for future events (e.g., diabetics), and patients with a previous positive MPI.2–6 In addition, the question in known CAD patients is often specific for location of disease and the ETT alone is unable to provide such data due to the low sensitivity. For example, a negative ETT (without imaging) would not rule out myocardial ischemia in this population. Among patients with known CAD, MPI also adds prognostic information even for those with high exercise capacity.7 In one study of 926 patients with known CAD who achieved ≥10 metabolics equivalent of task (METs) on treadmill exercise and were followed for a mean of 32 months, Peclat et al. demonstrated that patients who were able to achieve ≥10 METs had a lower annualized event rate than those who achieved = 10 METs Normal (1) >= 10 METs Abnormal (2) < 10 METs (3)

0.95 0.90 0.85 0.80 0

12

24

36

Time (months) Number at risk

Pairwisw comparisons

p value

0

12

24

36

1 vs 2

0.023

310

302

257

137

1 vs 3

5 years post-CABG (n = 1544). There were statistically significant increases in the rates of cardiac death as a function of SSS category (P = 0.049 and 0.005 for ≤5 and >5 years, respectively). CABG, coronary artery bypass graft surgery; SSS, summed stress score. (Reproduced with permission from Zellweger MJ, Lewin HC, Lai S, et al. When to stress patients after coronary artery bypass surgery? Risk stratification in patients early and late post-CABG using stress myocardial perfusion SPECT: implications of appropriate clinical strategies. J Am Coll Cardiol. 2001;37:144−152.)

CH18.indd 329

15-02-2022 11:21:50

330

Section 3  Indications and Applications

▶▶S/P Percutaneous Coronary Intervention The explosion of PCI use in general, as well as increasingly complex lesions and higher risk patients with single- or multivessel disease, has created a necessity for the detection of restenosis and disease progression, as well as risk assessment. A number of clinical studies have documented the usefulness of stress SPECT MPI for identifying restenosis in patients after PCI.32,33 The optimal time of performing SPECT imaging after PCI is somewhat controversial.21 Currently, MPI is considered reasonable in patients with atypical symptoms within a few weeks post-PCI, or those in whom additional ischemia detection is warranted due to incomplete revascularization. Patients with typical symptoms early following PCI are best evaluated in the catheterization laboratory. Studies by Giedd et al.34 and Zellweger et al.35 suggested that when performed ≥6 months following PCI, MPI can reliably identify patients most at risk for poor longterm outcome. However, there are no randomized clinical data to support the notion that routine use of MPI post-PCI can improve patients’ outcomes. Peterson et al. evaluated a cohort of 1848 patients who had PCI, of whom 241 were asymptomatic when they had follow-up SPECT MPI. Among those who had the study within the first 2 years (n=138), no patient required revascularization,

whereas in those who had the study after 2 years (n = 103), two patients underwent revascularization. These results suggest that routine stress testing after PCI in asymptomatic patients has low yield, especially within the first 2 years.36 When performed shortly after PCI, MPI can identify problems related to the target vessel. Zellweger et al. investigated a cohort of 476 patients who underwent SPECT MPI 6 months after PCI and followed them for a mean of 1 year. As shown in Figure 18-6, those who had target vessel ischemia had significantly higher rate of major adverse cardiac events, mostly driven by an increase in target vessel revascularization.23 In this cohort, ischemia was silent in 68% of patients. However, the impact of coronary revascularization on the outcomes of patients with silent target vessel ischemia is not established. In contrast, when performed late after PCI, SPECT MPI often identifies CAD in remote arteries, rather than in the target vessel, most likely due to disease progression. In a cohort of patients who underwent routine SPECT MPI 5 years after PCI, Zellweger et al. demonstrated that abnormal perfusion imaging was frequent irrespective of symptoms, and its utility lies in the detection of persistent or progressive CAD in remote vessel areas rather than in the diagnosis of late intervention-related problems in the treated vessels.31 These findings are consistent with our understanding of the timeline of in-stent

P < 0.0001 30 25

NO TVI TVI

32.4

P < 0.0001 26.5

%

20 15 10 5

P = 0.3 6.1

5.9 2.9

3.2

0 CD or MI

CH18.indd 330

TVR

MACE

FIGURE 18-6  Event rates in patients with targetvessel ischemia versus patients without. TVI, target vessel ischemia; CD, cardiac death; MI, myocardial infarction; TVR, target revascularization; and MACE, major adverse cardiac events (CD, MI, TVR). (Reprinted with permission from Zellweger MJ, Kaiser C, Brunner-La Rocca HP, et al. Value and limitations of target-vessel ischemia in predicting late clinical events after drug-eluting stent implantation. J Nucl Med. 2008;49:550−556.)

15-02-2022 11:21:50

Chapter 18  Evaluation of Patients with Known Coronary Artery Disease

restenosis and PCI-related complications, which tend to occur within the first year after intervention. It should be noted, regardless of these findings , that AUC criteria do not support imaging asymptomatic patients post-PCI.

MYOCARDIAL PERFUSION IMAGING IN ACUTE CORONARY SYNDROME PATIENTS

▶▶Thrombolytic Therapy for Acute ST-Segment Elevation Myocardial Infarction (STEMI) The most effective therapy for patients experiencing STEMI is timely reperfusion therapy with primary PCI. Unfortunately, this is not possible in every setting, and in these cases, thrombolytic therapy has been shown to improve myocardial salvage and reduce mortality rate compared to patients not receiving reperfusion therapy. However, even after successful thrombolytic therapy, patients may remain at risk for future events. Post-thrombolysis MPI provides valuable prognostic information by quantifying the infarcted myocardium and the myocardium at risk. Basu et al. demonstrated that reversible perfusion abnormality (ischemia), postlytic therapy predicts adverse events (death, re-infarction, CHF, and unstable angina) during follow up (range, 8−32 months) with a hazard ratio of 8.1 (95% confidence interval [CI] 2.7–23.8, P < 0.001).37 Similarly, Dakik et al. showed that the estimated cardiac event rate after thrombolytic therapy doubled with every 10% decrease in LVEF and 10% increase in MPI defect size.38 Based on these data, select uncomplicated STEMI patients who receive successful thrombolytic therapy are reasonable candidates for stress MPI.

▶▶ST-Elevation Myocardial Infarction without Reperfusion Therapy Patients who survive STEMI without receiving any reperfusion therapy are also at risk for re-infarction, heart failure, and cardiac death. Thus, they are candidates for risk stratification. Clinically unstable

CH18.indd 331

331

post-STEMI patients (heart failure, arrhythmias, and hemodynamic instability) and those with clinical or electrocardiographic evidence of recurrent ischemia need to undergo coronary angiography to determine further management. However, those with uncomplicated STEMI may be candidates for noninvasive risk assessment. Pharmacologic stress MPI has been shown to be superior to conventional submaximal ETT with modified Bruce protocol as a risk stratification tool in these patients.39–43 Stress testing with dipyridamole, adenosine, or regadenoson may be performed safely as early as 2 to 4 days following an uncomplicated MI. This approach can shorten hospital stay, and more importantly, it has been shown to have an excellent prognostic value, as patients with low-risk myocardial perfusion scan (i.e., without evidence of reversible perfusion defect) have low risk for cardiac events.39–41 Mahmarian et al. showed that a large area of myocardial ischemia (>10% of the left ventricle) on adenosine stress MPI following noncomplicated MI is superior to coronary angiography in identifying patients at increased risk for cardiac events.41 These data paved the way for the INSPIRE trial.44 In this multicenter prospective study, Mahamarian et al. enrolled 728 clinically stable survivors of acute MI (ST-elevation and non-ST-elevation acute coronary syndrome [ACS]) who did not receive coronary revascularization and underwent adenosine SPECT MPI within 10 days of admission. Three risk groups were prospectively defined based on perfusion defect size (PDS), ischemia perfusion defect size (IPDS), and left ventricular ejection fraction: (1) low risk (PDS < 20%); (2) intermediate risk (PDS > 20% and IPDS 20% and IPDS > 10%). The study also prospectively defined discharge and treatment strategies based on MPI risk group and ejection fraction. The cardiac events/death and reinfarction rates were, respectively, 5.4% and 1.8% in the low risk, 14% and 9.2% in the intermediate risk, and 18.6% and 11.6% in the high-risk group (P < 0.0001), demonstrating a significant increase in event rates with increasing burden of perfusion abnormalities. Furthermore, the study established the value of commonly measured MPI variables to improve the precision of assessing risk beyond TIMI risk score and ejection fraction, as shown in

15-02-2022 11:21:50

Section 3  Indications and Applications

332 60

Global X2 Value

50 40

*p < 0.001 vs. TIMI risk †p < 0.001 vs. TIMI risk + LVEF ‡p < 0.01 vs. TIMI risk + LVEF + PDS Total cardiac events Cardiac death and reinfarction

‡ †

*

30 20



*

10 0 TIMI Risk

TIMI Risk + LVEF

TIMI Risk + LVEF + PDS

Figure 18-7.44 More importantly, the study prospectively demonstrated that low-risk patients are candidates for safe early discharge, whereas patients in the intermediate- and high-risk groups should be considered for early invasive management. Moreover, in the high-risk group, coronary revascularization was associated with significant reduction in the cardiac event rate (10% vs. 32%, P = 0.049). Thus, the INSPIRE trial demonstrated that MPI not only defines risk, but can also guide patient management. These findings may also be relevant in patients who present late after an ST-elevation myocardial infarction.44

▶▶Non-ST-Elevation Acute Coronary Syndrome It is generally accepted that intermediate- and highrisk patients with non-ST-elevation ACS benefit from early invasive strategy aimed at early coronary revascularization.45 However, patients with low-risk non-ST-elevation ACS, such as those with low TIMI risk score and negative biomarkers, are candidates for a noninvasive risk assessment, especially if they respond well to initial medical management.46–48 Additionally, noninvasive risk stratification prior to invasive testing should be considered in ACS patients with a relative contraindication for coronary angiography, such as those with bleeding diatheses or chronic kidney disease.49 Exercise or pharmacologic stress MPI can be used safely to assess risk for cardiac events (death or MI)

CH18.indd 332

TIMI Risk + LVEF + PDS + Ischemia

FIGURE 18-7  Incremental prognostic value of radionuclide myocardial perfusion imaging variables to TIMI risk score − data from the INSPIRE trial. LVEF, left ventricular ejection fraction; PDS, perfusion defect size. (Reproduced, with permission from Mahmarian JJ, Shaw LJ, Filipchuk NG, et al. A multinational study to establish the value of early adenosine technetium99m sestamibi myocardial perfusion imaging in identifying a low-risk group for early hospital discharge after acute myocardial infarction. J Am Coll Cardiol. 2006;48:2448−2457.)

in patients with unstable angina following the initial response to medical management.50–52 The safety of adenosine and dipyridamole vasodilator stress 2 to 7 days after ACS is well established.39–42 More recently, regadenoson vasodilator stress has been shown to be associated with similarly low adverse event rates among patients undergoing in-hospital MPI for the assessment of ACS or elevated cardiac biomarkers.43 SPECT MPI has been shown to be safe and effective for risk stratification in patients presenting with ACS. Brown et al. and Stratmann et al. demonstrated that stress MPI can effectively risk stratify patients with unstable angina for long-term adverse cardiac events, and thus identifying patients with inducible ischemia as candidates for coronary angiography and revascularization.50,52 However, with demonstrated effectiveness of early invasive management of ACS, the role of stress MPI in the risk stratification of ACS patients in the modern era has significantly diminished. Nonetheless, there may be a role for MPI in the risk assessment of low-risk non-ST-elevation ACS and those with relative contraindication for coronary angiography.45

MYOCARDIAL PERFUSION IMAGING TO ASSESS EFFICACY OF MEDICAL THERAPY Intensive medical therapy with risk factor modification is essential in the management of patients with

15-02-2022 11:21:51

Chapter 18  Evaluation of Patients with Known Coronary Artery Disease

CAD, as demonstrated by both the COURAGE and ISCHEMIA trials.13,16 While high-risk patients gain a survival benefit from CABG, low- and moderate-risk patients have equivalent outcomes with respect to mortality with either approach (medical management or revascularization). Appropriate medical therapy would include aspirin, beta-blockers, lipid-lowering agents, and angiotensin-converting enzyme (ACE) inhibitors in diabetics and patients with impaired LV function. Since the degree and extent of ischemia predict future events, MPI has been used to assess the impact of medical management on the burden of myocardial ischemia in patients with known CAD. Importantly, the use of antianginal medications is usually avoided before performing diagnostic SPECT MPI studies for the detection of CAD; however, it is important to perform the studies while the patient is on antianginal regimen for the purpose of assessing prognosis and response to therapy. Data from Mahmarian et al. demonstrated that quantitative exercise Tl-201 MPI is highly reproducible and can be used to accurately interpret temporal changes in myocardial perfusion in individual patients.53 Moreover, in the ADVANCE MPI trials, Mahmarian et al. demonstrated, in same patient controls, that adenosine and regadenoson produce similar perfusion defects; thus temporal changes with repeat testing can be followed when either of these vasodilator stress agent is used.54 The beneficial impact of various pharmacologic interventions on the clinical outcomes of patients with CAD has been well established. Medical therapy has been associated with improvement in myocardial perfusion defects as a result of either decreased oxygen demand (beta blockers)55–57 or improved coronary blood flow (nitrates, calcium channel blockers [CCBs], and statins).58–60 In asymptomatic patients with CAD and known myocardial perfusion abnormalities, the 2013 ACCF/AHA/ASNC multimodality AUC for radionuclide MPI deem stress testing “rarely appropriate” when last stress imaging study was done less than 2 years prior, but “may be appropriate” if previous stress imaging was performed ≥2 years beforehand.46

▶▶Beta Blockers and Imaging Results Among the antianginal medications, beta-blockers have been shown in multiple studies to markedly decrease the burden of exercise-induced ischemia or

CH18.indd 333

333

even normalize the test.55,61 The anti-ischemic effect of beta-blockers is mediated primarily by decreasing the heart rate, prolonging diastole, increasing coronary perfusion time and myocardial oxygen extraction, and decreasing myocardial oxygen consumption in ischemic tissue.56 At the cellular level, beta-blockers alter myocyte metabolism providing additional myocardial protection against ischemia. One week of oral propranolol treatment has been shown to improve MPI abnormalities in men with established CAD.55 Similarly, acute propranolol administration in patients with dobutamine-induced reversible perfusion abnormalities has been shown to dampen heart rate response (peak heart rate 83 ± 18 vs. 125 ± 17, P < 0.001), rate−pressure product (14,169 ± 4248 vs. 19,894 ± 3985; P < .001), and myocardial ischemia score (6.9 ± 5.8 vs. 10.1 ± 7.1, P = 0.047) despite a higher infusion dose.56 The extent of perfusion abnormalities has been shown to be significantly reduced with beta blockers even with the use of vasodilator stress.61 Similarly, PET MPI is affected by the use of beta blockers. In a study of 36 patients who underwent N-13 ammonia PET MPI before and after 12 weeks of either carvedilol or metoprolol, Koepfli et al. showed that the use of beta blockers was associated with a decrease in resting MBF proportional to cardiac work, as well as a significant increase in MBF during hyperemia resulting in increased CFR in the stenotic segments (+15%, P < 0.05) with no significant change in remote segments.62

▶▶Nitrates and Imaging Results The anti-ischemic effect of nitroglycerin has been attributed to the redistribution of blood flow from normal to ischemic myocardium through dilation of collateral vessels. Additionally, nitroglycerine reduces myocardial oxygen consumption by producing a systemic vasodilation leading to a reduction in systemic venous return, left ventricular dimensions, and myocardial wall stress. Either in conjunction with beta blockers57 or alone,58,60 both of these agents decrease the size of reversible defects (particularly in patients with large ischemic perfusion defects). Mahmarian et al. prospectively evaluated whether short-term (6.1 ± 1.8 days) transdermal nitroglycerin patches could limit the extent of exercise-induced LV ischemia, as assessed by quantitative Tl-201 tomography. Patients

15-02-2022 11:21:51

334

Section 3  Indications and Applications

randomized to receive active patch therapy had a significant reduction in their total PDS (−8.9 ± 11.1%) compared with placebo-treated patients (−1.8 ± 6.1%, P = .04). The reduction in PDS was most apparent in those with the largest (≥ 20%) baseline perfusion defects (−11.4 ± 13.4% vs. 1.0 ± 3.6%, respectively, P = 0.02). Nitrate therapy did not significantly reduce heart rate, blood pressure, or double product, in consistence with its known mechanisms of action.60

▶▶Calcium Channel Blockers and Imaging Results The primary benefit of CCBs seems to be a reduction in myocardial oxygen demand, which is achieved by decreased arterial tone, peripheral vascular resistance, intraventricular pressure, and wall stress. CCBs also enhance myocardial perfusion through their effects on myocardial microcirculation and metabolism. They improve coronary flow in CAD by selectively dilating larger arterioles and may prevent coronary spasm.58 Limited data suggest that acute administration of nifedipine before exercise planar MPI resulted in significant improvement in perfusion (defined as ≥20% increase) in approximately one-half of patients and in one-fourth of segments compared with no CCB administration. Chronic administration of nifedipine and nicorandil in two separate studies before exercise SPECT reduced the defect extent and severity.58

▶▶Lipid-Lowering Agents The impact of lipid-lowering agents in the secondary prevention of coronary disease has been demonstrated in multiple large studies, such as CARE,59 CTT,63 4S,64 TNT,65 PROVE-IT,66 and JUPITER.67 Statins improve endothelial function and preserve coronary perfusion independent of cholesterol reduction. They increase smooth muscle relaxation, decrease oxidative stress, and prevent vascular inflammation. Although statins may halt the progression or even cause a regression of atherosclerosis as assessed by coronary angiography and intravascular ultrasound, the degree of regression is slight compared with the substantial improvement in clinical outcomes which is likely related to plaque

CH18.indd 334

stabilization. Using rest-dipyridamole PET, Gould et al. demonstrated that there were statistically significant improvements in size and severity of perfusion abnormalities after intensive 90-day cholesterol lowering compared to baseline control.68 Thus, short-term intensive cholesterol lowering improves myocardial perfusion before anatomic regression of stenosis occurs. Schwartz et al. used SPECT imaging in patients with CAD and hypercholesterolemia to assess serial changes in myocardial perfusion associated with cholesterol lowering therapy.69 Following improvement in total cholesterol (pretreatment: 223 ± 51, post-treatment: 147 ± 33, P < 0.001), the stress defect score (% ischemic myocardium) was significantly improved (pretreatment: 19 ± 16, post-treatment: 9 ± 13, P = 0.022). Furthermore, the same investigators studied the effect of shortterm (6 weeks) or long-term (6 months) pravastatin in dyslipidemic patients with baseline MPI ischemic defects.70,71 Despite a significant reduction of lowdensity lipoprotein (LDL) at 6 weeks (33%, P < .001), myocardial perfusion scores were reduced only at 6 months (12.6 ± 5.7 at baseline, 9.4 ± 6.2 at 6 months, P = 0.01). The time course of reduced perfusion abnormalities, rather than LDL reduction, paralleled documented clinical benefit.59,64,68,72

▶▶Angiotensin-Converting Enzyme Inhibitors There is notable evidence that ACE inhibitors exert a beneficial effect in patients with known coronary disease. The mechanism for ACE inhibitors benefit is complex (improved endothelial function, vasodilation and reduced afterload, antiplatelet effect, and inhibition of neurohormonal activation). There is no large study to examine their direct anti-ischemic mechanism using MPI. In two studies, ACE inhibition was associated with improved epicardial73 and microvascular blood flow;74 the mechanism is predominantly endothelium mediated. After 12 weeks of treatment, enalapril delayed the onset of ischemic ST-segment depression during ETT (5.6 ± 1.9 minute in the enalapril group vs. 4.4 ± 1.3 minute in the placebo group, P < 0.05) without affecting the double product.75 Further studies are needed to elucidate a direct anti-ischemic mechanism of ACE inhibitors and explore the role of MPI in monitoring such an effect.

15-02-2022 11:21:51

Chapter 18  Evaluation of Patients with Known Coronary Artery Disease

▶▶Lifestyle Modifications The widespread interest in the noninvasive management of coronary atherosclerosis has brought new attention to the impact of various lifestyle changes on the prognosis of coronary disease. Diet, exercise, and behavioral interventions are generally advised for patients with documented coronary disease. The impact of these changes on the extent of atherosclerosis, as determined by angiography, is modest. However, after 5 years of intensive risk factor modification, the size and severity of perfusion abnormalities on rest-dipyridamole PET imaging improved in the intervention group compared to controls.76 The ISCHEMIA trial, which compared the effects of OMT versus the combination of OMT and coronary revascularization in patients with moderate to severe ischemia, failed to demonstrate a statistically significant difference between the two groups in reducing adverse cardiac events. One of the explanations for these results is the aggressive incorporation of lifestyle modifications and medication adherence, which lead to lower than expected event rates in both study groups. The study goals to reduce risk factors included smoking cessation, moderate intensity physical activity for ≥30 minutes ≥5 times per week, reduced saturated fat consumption to 1.6 (N=339)

CAD-/DMNI MPI/EF (N=682)

FIGURE 19-11  Annualized cardiac mortality among patients with diabetes mellitus or coronary artery disease. Note that CFR reclassifies the risk of diabetics without overt CAD into a high-risk category similar to the risk of those with overt CAD, when CFR is abnormal, or low-risk category similar to those without diabetes or CAD. CAD, coronary artery disease; DM, diabetes mellitus; CFR, coronary flow reserve; Nl, normal; MPI, myocardial perfusion imaging; EF, left ventricular ejection fraction. (Data from Murthy VL, Naya M, Foster CR, et al. Association between coronary vascular dysfunction and cardiac mortality in patients with and without diabetes mellitus. Circulation. 2012;126(15):1858−1868.)

The prognostic value of coronary vascular health, as measured by CFR, extends not only to clinical risk assessment, but is also incremental to coronary atherosclerotic burden. Naya et al. demonstrated that normal CFR (≥ 2.0) further classifies risk among patients with wide spectrum of CACS. Even low-risk patients with CACS of 0 were reclassified into a lower-risk stratum when CFR is normal.194 The prognostic value of CFR extends beyond not only CACS, but also coronary angiography. Taqueti et al. demonstrated that despite the strong correlation between CFR and MACE, the correlation between CFR and angiographic CAD burden is rather weak (correlation coefficient r = −0.26). The authors identified patients with low CAD burden but impaired CFR to be at higher risk for MACE, similar to those with high CAD burden.195 The same group demonstrated that women referred for coronary angiography had a significantly lower burden

CH19.indd 355

of obstructive CAD in comparison with men but were not protected from cardiac events. Impaired CFR, rather than CAD burden, was associated with excess cardiovascular risk in women relative to men. As shown in Figure 19-12, CFR seems to explain excess cardiovascular risk, particularly among women with low CAD burden.196 There is now a large body of evidence establishing CFR as a powerful tool that provides incremental risk stratification beyond routine assessment of clinical characteristics, LV function, extent and severity of perfusion abnormalities, and even anatomic CAD burden. It was suggested that a normal CFR in patients with normal MPI portend a 3-year “warranty” of low MACE risk.197 In clinical practice, a CFR cutoff of 1.8 to 2.0 is generally used; greater CFR values are associated with excellent outcomes, while lower values correlate with an increasing risk of MACE.

15-02-2022 12:26:23

356

Section 3  Indications and Applications 2.0

Female Male

Log adjusted hazard

1.5

1.0

0.5

p or = 10 metabolic equivalents predicts a very low risk of inducible ischemia: does myocardial perfusion imaging have a role? J Am Coll Cardiol. 2009;54(6):538–545. 100. Bourque JM, Charlton GT, Holland BH, Belyea CM, Watson DD, Beller GA. Prognosis in patients achieving >/=10 METS on exercise stress testing: was SPECT imaging useful? J Nucl Cardiol. 2011;18(2):230–237. 101. Smith L, Myc L, Beller GA, Bourque JM. A high exercise workload of ≥ 10 METs predicts a very low risk of significant ischemia in patients of advanced age. J Nucl Cardiol. 2015;22(4):776 [Abstract]. 102. Joye JD, Schulman DS, Lasorda D, Farah T, Donohue BC, Reichek N. Intracoronary Doppler guide wire versus stress single-photon emission computed tomographic thallium-201 imaging in assessment of intermediate coronary stenoses. J Am Coll Cardiol. 1994;24(4):940–947. 103. Miller DD, Donohue TJ, Younis LT, et al. Correlation of pharmacological 99mTc-sestamibi myocardial perfusion imaging with poststenotic coronary flow reserve in patients with angiographically intermediate coronary artery stenoses. Circulation. 1994;89(5):2150–2160. 104. Deychak YA, Segal J, Reiner JS, et al. Doppler guide wire flow-velocity indexes measured distal to coronary stenoses associated with reversible thallium perfusion defects. Am Heart J. 1995;129(2):219–227. 105. Voudris V, Avramides D, Koutelou M, et al. Relative coronary flow velocity reserve improves correlation with stress myocardial perfusion imaging in assessment of coronary artery stenoses. Chest. 2003;124(4):1266–1274. 106. Pijls NH, De Bruyne B, Peels K, et al. Measurement of fractional flow reserve to assess the functional severity of coronary-artery stenoses. N Engl J Med. 1996;334(26):1703–1708. 107. De Bruyne B, Bartunek J, Sys SU, Heyndrickx GR. Relation between myocardial fractional flow reserve calculated from coronary pressure measurements and exercise-induced myocardial ischemia. Circulation. 1995;92(1):39–46. 108. Alqaisi F, Albadarin F, Jaffery Z, et al. Prognostic predictors and outcomes in patients with abnormal myocardial perfusion imaging and angiographically insignificant coronary artery disease. J Nucl Cardiol. 2008;15(6):754–761.

15-02-2022 12:26:27

366

Section 3  Indications and Applications

109. Yokota S, Ottervanger JP, Mouden M, Timmer JR, Knollema S, Jager PL. Prevalence, location, and extent of significant coronary artery disease in patients with normal myocardial perfusion imaging. J Nucl Cardiol. 2014;21(2):284–290. 110. Hachamovitch R, Di Carli MF. Nuclear cardiology will remain the "gatekeeper" over CT angiography. J Nucl Cardiol. 2007;14(5):634–644. 111. Min JK, Shaw LJ, Berman DS. The present state of coronary computed tomography angiography a process in evolution. J Am Coll Cardiol. 2010;55(10):957–965. 112. Shaw LJ, Hachamovitch R, Berman DS, et al. The economic consequences of available diagnostic and prognostic strategies for the evaluation of stable angina patients: an observational assessment of the value of precatheterization ischemia. Economics of Noninvasive Diagnosis (END) Multicenter Study Group. J Am Coll Cardiol. 1999;33(3):661–669. 113. Hachamovitch R, Hayes SW, Friedman JD, Cohen I, Berman DS. A prognostic score for prediction of cardiac mortality risk after adenosine stress myocardial perfusion scintigraphy. J Am Coll Cardiol. 2005;45(5):722–729. 114. Germano G, Kavanagh PB, Kavanagh JT, Wishner SH, Berman DS, Kavanagh GJ. Repeatability of automatic left ventricular cavity volume measurements from myocardial perfusion SPECT. J Nucl Cardiol. 1998;5(5):477–483. 115. DePuey EG, Rozanski A. Using gated technetium-99m-sestamibi SPECT to characterize fixed myocardial defects as infarct or artifact. J Nucl Med. 1995;36(6):952–955. 116. Taillefer R, DePuey EG, Udelson JE, Beller GA, Latour Y, Reeves F. Comparative diagnostic accuracy of Tl-201 and Tc99m sestamibi SPECT imaging (perfusion and ECG-gated SPECT) in detecting coronary artery disease in women. J Am Coll Cardiol. 1997;29(1):69–77. 117. Smanio PE, Watson DD, Segalla DL, Vinson EL, Smith WH, Beller GA. Value of gating of technetium-99m sestamibi single-photon emission computed tomographic imaging. J Am Coll Cardiol. 1997;30(7):1687–1692. 118. Sharir T, Germano G, Kang X, et al. Prediction of myocardial infarction versus cardiac death by gated myocardial perfusion SPECT: risk stratification by the amount of stressinduced ischemia and the poststress ejection fraction. J Nucl Med. 2001;42(6):831–837. 119. Kapetanopoulos A, Ahlberg AW, Taub CC, Katten DM, Heller GV. Regional wall-motion abnormalities on poststress electrocardiographic-gated technetium-99m sestamibi single-photon emission computed tomography imaging predict cardiac events. J Nucl Cardiol. 2007;14(6):810–817. 120. Hida S, Chikamori T, Tanaka H, et al. Diagnostic value of left ventricular function after stress and at rest in the detection of multivessel coronary artery disease as assessed by electrocardiogram-gated SPECT. J Nucl Cardiol. 2007;14(1):68–74. 121. Druz RS, Akinboboye OA, Grimson R, Nichols KJ, Reichek N. Postischemic stunning after adenosine vasodilator stress. J Nucl Cardiol. 2004;11(5):534–541. 122. Johnson LL, Verdesca SA, Aude WY, et al. Postischemic stunning can affect left ventricular ejection fraction and regional wall motion on post-stress gated sestamibi tomograms. J Am Coll Cardiol. 1997;30(7):1641–1648. 123. Paul AK, Hasegawa S, Yoshioka J, et al. Characteristics of regional myocardial stunning after exercise in gated myocardial SPECT. J Nucl Cardiol. 2002;9(4):388–394.

CH19.indd 366

124. Toba M, Kumita S, Cho K, Ibuki C, Kumazaki T, Takano T. Usefulness of gated myocardial perfusion SPECT imaging soon after exercise to identify postexercise stunning in patients with single-vessel coronary artery disease. J Nucl Cardiol. 2004;11(6):697–703. 125. Doukky R, Frogge N, Bayissa YA, et al. The prognostic value of transient ischemic dilatation with otherwise normal SPECT myocardial perfusion imaging: A cautionary note in patients with diabetes and coronary artery disease. J Nucl Cardiol. 2013;20(5):774–784. 126. Golzar Y, Olusanya A, Pe N, et al. The significance of automatically measured transient ischemic dilation in identifying severe and extensive coronary artery disease in regadenoson, single-isotope technetium-99m myocardial perfusion SPECT. J Nucl Cardiol. 2015;22(3):526–534. 127. Hashimoto J, Kubo A, Iwasaki R, et al. Gated singlephoton emission tomography imaging protocol to evaluate myocardial stunning after exercise. Eur J Nucl Med. 1999;26(12):1541–1546. 128. del Val Gomez M, Gallardo FG, San Martin MA, Garcia A, Terol I. Ischaemic related transitory left ventricular dysfunction in 201Tl gated SPECT. Nucl Med Commun. 2005;26(7):601–605. 129. Gomez J, Golzar Y, Fughhi I, Olusanya A, Doukky R. The significance of post-stress decrease in left ventricular ejection fraction in patients undergoing regadenoson stress gated SPECT myocardial perfusion imaging. J Nucl Cardiol. 2018;25(4):1313–1323. 130. Otaki Y, Fish MB, Miller RJH, Lemley M, Slomka PJ. Prognostic value of early left ventricular ejection fraction reserve during regadenoson stress solid-state SPECT-MPI. J Nucl Cardiol. 2021; DOI:10.1007/s12350-020-02420-w.. 131. Dorbala S, Hachamovitch R, Curillova Z, et al. Incremental prognostic value of gated Rb-82 positron emission tomography myocardial perfusion imaging over clinical variables and rest LVEF. JACC Cardiovasc Imaging. 2009;2(7):846–854. 132. Gomez J, Golzar Y, Fughhi I, Olusanya A, Doukky R. The significance of post-stress decrease in left ventricular ejection fraction in patients undergoing regadenoson stress gated SPECT myocardial perfusion imaging. J Nucl Cardiol. 2018;25(4):1313–1323. 133. Smith P, Farag A, Bhambhvani P, Iskandrian A, Hage FG. Prognostic value of absent left ventricular ejection fraction reserve with regadenoson SPECT MPI. J Nucl Cardiol. 2020. 134. Chen J, Garcia EV, Folks RD, et al. Onset of left ventricular mechanical contraction as determined by phase analysis of ECG-gated myocardial perfusion SPECT imaging: development of a diagnostic tool for assessment of cardiac mechanical dyssynchrony. J Nucl Cardiol. 2005;12(6):687–695. 135. Aggarwal H, AlJaroudi WA, Mehta S, et al. The prognostic value of left ventricular mechanical dyssynchrony using gated myocardial perfusion imaging in patients with endstage renal disease. J Nucl Cardiol. 2014;21(4):739–746. 136. Pazhenkottil AP, Buechel RR, Husmann L, et al. Long-term prognostic value of left ventricular dyssynchrony assessment by phase analysis from myocardial perfusion imaging. Heart. 2011;97(1):33–37. 137. McLaughlin MG, Danias PG. Transient ischemic dilation: a powerful diagnostic and prognostic finding of stress myocardial perfusion imaging. J Nucl Cardiol. 2002;9(6):663–667.

15-02-2022 12:26:27

Chapter 19  Risk Stratification with Myocardial Perfusion Imaging 138. Henzlova MJ, Duvall WL, Einstein AJ, Travin MI, Verberne HJ. ASNC imaging guidelines for SPECT nuclear cardiology procedures: Stress, protocols, and tracers. J Nucl Cardiol. 2016;23(3):606–639. 139. Elhabyan AK, Reyes BJ, Hallak O, et al. Subendocardial ischemia without coronary artery disease: is elevated left ventricular end diastolic pressure the culprit? Curr Med Res Opin. 2004;20(5):773–777. 140. Xu Y, Arsanjani R, Clond M, et al. Transient ischemic dilation for coronary artery disease in quantitative analysis of sameday sestamibi myocardial perfusion SPECT. J Nucl Cardiol. 2012;19(3):465–473. 141. Slomka PJ, Berman DS, Germano G. Normal limits for transient ischemic dilation with 99mTc myocardial perfusion SPECT protocols. J Nucl Cardiol. 2017;24(5):1709–1711. 142. Hu LH, Sharir T, Miller RJH, et al. Upper reference limits of transient ischemic dilation ratio for different protocols on newgeneration cadmium zinc telluride cameras: A report from REFINE SPECT registry. J Nucl Cardiol. 2020;27(4):1180–1189. 143. Weiss AT, Berman DS, Lew AS, et al. Transient ischemic dilation of the left ventricle on stress thallium-201 scintigraphy: a marker of severe and extensive coronary artery disease. J Am Coll Cardiol. 1987;9(4):752–759. 144. McClellan JR, Travin MI, Herman SD, et al. Prognostic importance of scintigraphic left ventricular cavity dilation during intravenous dipyridamole technetium-99m sestamibi myocardial tomographic imaging in predicting coronary events. Am J Cardiol. 1997;79(5):600–605. 145. Mazzanti M, Germano G, Kiat H, et al. Identification of severe and extensive coronary artery disease by automatic measurement of transient ischemic dilation of the left ventricle in dual-isotope myocardial perfusion SPECT. J Am Coll Cardiol. 1996;27(7):1612–1620. 146. Kinoshita N, Sugihara H, Adachi Y, et al. Assessment of transient left ventricular dilatation on rest and exercise on Tc-99m tetrofosmin myocardial SPECT. Clin Nucl Med. 2002;27(1):34–39. 147. Kritzman JN, Ficaro EP, Corbett JR. Post-stress LV dilation: the effect of imaging protocol, gender and attenuation correction [abstract]. J Nucl Med. 2001;42(supple):50P. 148. Williams K, Schnieder C. Transient ischemic dilation (TID) with pharmacological stress dual isotope SPECT [abstract]. Circulation. 2000;102:II–546. 149. Abidov A, Bax JJ, Hayes SW, et al. Integration of automatically measured transient ischemic dilation ratio into interpretation of adenosine stress myocardial perfusion SPECT for detection of severe and extensive CAD. J Nucl Med. 2004;45(12):1999–2007. 150. Katz JS, Ruisi M, Giedd KN, Rachko M. Assessment of transient ischemic dilation (TID) ratio in gated SPECT myocardial perfusion imaging (MPI) using regadenoson, a new agent for pharmacologic stress testing. J Nucl Cardiol. 2012;19(4):727–734. 151. Lester D, El-Hajj S, Farag AA, et al. Prognostic value of transient ischemic dilation with regadenoson myocardial perfusion imaging. J Nucl Cardiol. 2016;23(5):1147–1155. 152. Kakhki VR, Sadeghi R, Zakavi SR. Assessment of transient left ventricular dilation ratio via 2-day dipyridamole Tc-99m sestamibi nongated myocardial perfusion imaging. J Nucl Cardiol. 2007;14(4):529–536.

CH19.indd 367

367

153. Bestetti A, Di Leo C, Alessi A, Triulzi A, Tagliabue L, Tarolo GL. Post-stress end-systolic left ventricular dilation: a marker of endocardial post-ischemic stunning. Nucl Med Commun. 2001;22(6):685–693. 154. Rischpler C, Higuchi T, Fukushima K, et al. Transient ischemic dilation ratio in 82Rb PET myocardial perfusion imaging: normal values and significance as a diagnostic and prognostic marker. J Nucl Med. 2012;53(5):723–730. 155. Abidov A, Germano G, Berman DS. Transient ischemic dilation ratio: a universal high-risk diagnostic marker in myocardial perfusion imaging. J Nucl Cardiol. 2007;14(4):497–500. 156. Valdiviezo C, Motivala AA, Hachamovitch R, et al. The significance of transient ischemic dilation in the setting of otherwise normal SPECT radionuclide myocardial perfusion images. J Nucl Cardiol. 2011;18(2):220–229. 157. Abidov A, Bax JJ, Hayes SW, et al. Transient ischemic dilation ratio of the left ventricle is a significant predictor of future cardiac events in patients with otherwise normal myocardial perfusion SPECT. J Am Coll Cardiol. 2003;42(10):1818–1825. 158. Bourque JM. Contemporary relevance of TID: Based on the company it keeps. J Nucl Cardiol. 2015;33(3):535–538. 159. Gibson RS, Watson DD, Carabello BA, Holt ND, Beller GA. Clinical implications of increased lung uptake of thallium-201 during exercise scintigraphy 2 weeks after myocardial infarction. Am J Cardiol. 1982;49(7):1586–1593. 160. Boucher CA, Zir LM, Beller GA, et al. Increased lung uptake of thallium-201 during exercise myocardial imaging: clinical, hemodynamic and angiographic implications in patients with coronary artery disease. Am J Cardiol. 1980;46(2):189–196. 161. Kaminek M, Myslivecek M, Skvarilova M, et al. Increased prognostic value of combined myocardial perfusion SPECT imaging and the quantification of lung Tl-201 uptake. Clin Nucl Med. 2002;27(4):255–260. 162. Cox JL, Wright LM, Burns RJ. Prognostic significance of increased thallium-201 lung uptake during dipyridamole myocardial scintigraphy: comparison with exercise scintigraphy. Can J Cardiol. 1995;11(8):689–694. 163. Daou D, Delahaye N, Lebtahi R, et al. Diagnosis of extensive coronary artery disease: intrinisic value of increased lung 201 T1 uptake with exercise SPECT. J Nucl Med. 2000;41(4):567–574. 164. Bacher-Stier C, Sharir T, Kavanagh PB, et al. Postexercise lung uptake of 99mTc-sestamibi determined by a new automatic technique: validation and application in detection of severe and extensive coronary artery disease and reduced left ventricular function. J Nucl Med. 2000;41(7):1190–1197. 165. Mahmarian JJ, Cerqueira MD, Iskandrian AE, et al. Regadenoson induces comparable left ventricular perfusion defects as adenosine: a quantitative analysis from the ADVANCE MPI 2 trial. JACC Cardiovasc Imaging. 2009;2(8):959–968. 166. Berman DS, Kang X, Gransar H, et al. Quantitative assessment of myocardial perfusion abnormality on SPECT myocardial perfusion imaging is more reproducible than expert visual analysis. J Nucl Cardiol. 2009;16(1):45–53. 167. Iskandrian AE, Garcia EV, Faber T, Mahmarian JJ. Automated assessment of serial SPECT myocardial perfusion images. J Nucl Cardiol. 2009;16(1):6–9. 168. Mahmarian JJ, Peterson LE, Xu J, et al. Regadenoson provides perfusion results comparable to adenosine in heterogeneous patient populations: A quantitative analysis from the ADVANCE MPI trials. J Nucl Cardiol. 2015;22(2):248–261.

15-02-2022 12:26:27

368

Section 3  Indications and Applications

169. Giri S, Shaw LJ, Murthy DR, et al. Impact of diabetes on the risk stratification using stress single-photon emission computed tomography myocardial perfusion imaging in patients with symptoms suggestive of coronary artery disease. Circulation. 2002;105(1):32–40. 170. Iskandrian AE, Hage FG, Shaw LJ, Mahmarian JJ, Berman DS. Serial myocardial perfusion imaging: defining a significant change and targeting management decisions. JACC Cardiovasc Imaging. 2014;7(1):79–96. 171. Losordo DW, Vale PR, Hendel RC, et al. Phase 1/2 placebocontrolled, double-blind, dose-escalating trial of myocardial vascular endothelial growth factor 2 gene transfer by catheter delivery in patients with chronic myocardial ischemia. Circulation. 2002;105(17):2012–2018. 172. Farzaneh-Far A, Phillips HR, Shaw LK, et al. Ischemia change in stable coronary artery disease is an independent predictor of death and myocardial infarction. JACC Cardiovasc Imaging. 2012;5(7):715–724. 173. Marwick TH, Shan K, Patel S, Go RT, Lauer MS. Incremental value of rubidium-82 positron emission tomography for prognostic assessment of known or suspected coronary artery disease. Am J Cardiol. 1997;80(7):865–870. 174. Dorbala S, Di Carli MF, Beanlands RS, et al. Prognostic value of stress myocardial perfusion positron emission tomography: results from a multicenter observational registry. J Am Coll Cardiol. 2013;61(2):176–184. 175. Dorbala S, Hachamovitch R, Curillova Z, et al. Incremental prognostic value of gated Rb-82 positron emission tomography myocardial perfusion imaging over clinical variables and rest LVEF. JACC Cardiovasc Imaging. 2009;2(7):846–854. 176. Patel KK, Spertus JA, Chan PS, et al. Extent of myocardial ischemia on positron emission tomography and survival benefit with early revascularization. J Am Coll Cardiol. 2019;74(13):1645–1654. 177. Kanayama S, Matsunari I, Kajinami K. Comparison of gated N-13 ammonia PET and gated Tc-99m sestamibi SPECT for quantitative analysis of global and regional left ventricular function. J Nucl Cardiol. 2007;14(5):680–687. 178. Bravo PE, Chien D, Javadi M, Merrill J, Bengel FM. Reference ranges for LVEF and LV volumes from electrocardiographically gated 82Rb cardiac PET/CT using commercially available software. J Nucl Med. 2010;51(6):898–905. 179. Dorbala S, Vangala D, Sampson U, Limaye A, Kwong R, Di Carli MF. Value of vasodilator left ventricular ejection fraction reserve in evaluating the magnitude of myocardium at risk and the extent of angiographic coronary artery disease: a 82Rb PET/CT study. J Nucl Med. 2007;48(3):349–358. 180. White HD, Norris RM, Brown MA, Brandt PW, Whitlock RM, Wild CJ. Left ventricular end-systolic volume as the major determinant of survival after recovery from myocardial infarction. Circulation. 1987;76(1):44–51. 181. Sharir T, Kang X, Germano G, et al. Prognostic value of poststress left ventricular volume and ejection fraction by gated myocardial perfusion SPECT in women and men: genderrelated differences in normal limits and outcomes. J Nucl Cardiol. 2006;13(4):495–506. 182. Kay J, Dorbala S, Goyal A, et al. Influence of sex on risk stratification with stress myocardial perfusion Rb-82 positron emission tomography: Results from the PET (Positron Emission Tomography) Prognosis Multicenter Registry. J Am Coll Cardiol. 2013;62(20):1866–1876.

CH19.indd 368

183. Lertsburapa K, Ahlberg AW, Bateman TM, et al. Independent and incremental prognostic value of left ventricular ejection fraction determined by stress gated rubidium 82 PET imaging in patients with known or suspected coronary artery disease. J Nucl Cardiol. 2008;15(6):745–753. 184. Murthy VL, Bateman TM, Beanlands RS, et al. Clinical quantification of myocardial blood flow using PET: Joint Position Paper of the SNMMI Cardiovascular Council and the ASNC. J Nucl Cardiol. 2018;25(1):269–297. 185. Murthy VL, Naya M, Foster CR, et al. Improved cardiac risk assessment with noninvasive measures of coronary flow reserve. Circulation. 2011;124(20):2215–2224. 186. Ziadi MC, Dekemp RA, Williams KA, et al. Impaired myocardial flow reserve on rubidium-82 positron emission tomography imaging predicts adverse outcomes in patients assessed for myocardial ischemia. J Am Coll Cardiol. 2011;58(7):740–748. 187. Naya M, Murthy VL, Taqueti VR, et al. Preserved coronary flow reserve effectively excludes high-risk coronary artery disease on angiography. J Nucl Med. 2014;55(2):248–255. 188. Patel KK, Spertus JA, Chan PS, et al. Myocardial blood flow reserve assessed by positron emission tomography myocardial perfusion imaging identifies patients with a survival benefit from early revascularization. Eur Heart J. 2020;41(6):759–768. 189. Gould KL, Johnson NP, Roby AE, et al. Regional, artery-specific thresholds of quantitative myocardial perfusion by PET associated with reduced myocardial infarction and death after revascularization in stable coronary artery disease. J Nucl Med. 2019;60(3):410–417. 190. Gupta A, Taqueti VR, van de Hoef TP, et al. Integrated noninvasive physiological assessment of coronary circulatory function and impact on cardiovascular mortality in patients with stable coronary artery disease. Circulation. 2017;136(24):2325–2336. 191. Murthy VL, Naya M, Foster CR, et al. Association between coronary vascular dysfunction and cardiac mortality in patients with and without diabetes mellitus. Circulation. 2012;126(15):1858–1868. 192. Charytan DM, Skali H, Shah NR, et al. Coronary flow reserve is predictive of the risk of cardiovascular death regardless of chronic kidney disease stage. Kidney Int. 2018;93(2):501–509. 193. Murthy VL, Naya M, Foster CR, et al. Coronary vascular dysfunction and prognosis in patients with chronic kidney disease. JACC Cardiovasc Imaging. 2012;5(10):1025–1034. 194. Naya M, Murthy VL, Foster CR, et al. Prognostic interplay of coronary artery calcification and underlying vascular dysfunction in patients with suspected coronary artery disease. J Am Coll Cardiol. 2013;61(20):2098–2106. 195. Taqueti VR, Hachamovitch R, Murthy VL, et al. Global coronary flow reserve is associated with adverse cardiovascular events independently of luminal angiographic severity and modifies the effect of early revascularization. Circulation. 2015;131(1):19–27. 196. Taqueti VR, Shaw LJ, Cook NR, et al. Excess cardiovascular risk in women relative to men referred for coronary angiography is associated with severely impaired coronary flow reserve, not obstructive disease. Circulation. 2017;135(6):566–577. 197. Herzog BA, Husmann L, Valenta I, et al. Long-term prognostic value of 13N-ammonia myocardial perfusion positron

15-02-2022 12:26:27

Chapter 19  Risk Stratification with Myocardial Perfusion Imaging emission tomography added value of coronary flow reserve. J Am Coll Cardiol. 2009;54(2):150–156. 198. Murthy VL, Naya M, Taqueti VR, et al. Effects of sex on coronary microvascular dysfunction and cardiac outcomes. Circulation. 2014;129(24):2518–2527. 199. Maron DJ, Hochman JS, Reynolds HR, et al. Initial invasive or conservative strategy for stable coronary disease. N Engl J Med..2020;382(15):1395–1407. 200. Greenland P, Blaha MJ, Budoff MJ, Erbel R, Watson KE. Coronary calcium score and cardiovascular risk. J Am Coll Cardiol. 2018;72(4):434–447. 201. Bellasi A, Lacey C, Taylor AJ, et al. Comparison of prognostic usefulness of coronary artery calcium in men versus women (results from a meta- and pooled analysis estimating all-cause mortality and coronary heart disease death or myocardial infarction). Am J Cardiol. 2007;100(3):409–414. 202. Mittal TK, Pottle A, Nicol E, et al. Prevalence of obstructive coronary artery disease and prognosis in patients with stable symptoms and a zero-coronary calcium score. Eur Heart J Cardiovasc Imaging. 2017;18(8):922–929. 203. Berman DS, Wong ND, Gransar H, et al. Relationship between stress-induced myocardial ischemia and atherosclerosis measured by coronary calcium tomography. J Am Coll Cardiol. 2004;44(4):923–930. 204. Pyslar N, Doukky R. Myocardial perfusion imaging and coronary calcium score: A marriage made in heaven. J Nucl Cardiol. 2019; DOI:10.1007/s12350-019-01966-8. 205. Schenker MP, Dorbala S, Hong ECT, et al. Interrelation of coronary calcification, myocardial ischemia, and outcomes in patients with intermediate likelihood of coronary artery disease. Circulation. 2008;117(13):1693–1700. 206. Betancur J, Commandeur F, Motlagh M, et al. Deep learning for prediction of obstructive disease from fast myocardial perfusion SPECT: a multicenter study. JACC Cardiovasc Imaging. 2018;11(11):1654–1663. 207. Motwani M, Dey D, Berman DS, et al. Machine learning for prediction of all-cause mortality in patients with suspected coronary artery disease: a 5-year multicentre prospective registry analysis. Eur Heart J. 2017;38(7):500–507. 208. Haro Alonso D, Wernick MN, Yang Y, Germano G, Berman DS, Slomka P. Prediction of cardiac death after adenosine myocardial perfusion SPECT based on machine learning. J Nucl Cardiol. 2019;26(5):1746-1754. 209. Gomez J, Doukky R, Germano G, Slomka P. New trends in quantitative nuclear cardiology methods. Current Cardiovascular Imaging Reports. 2018;11(1):1. 210. Betancur J, Otaki Y, Motwani M, et al. Prognostic value of combined clinical and myocardial perfusion imaging data using machine learning. Curr Cardiovasc Imaging Rep. 2018;11(7):1000–1009. 211. Juarez-Orozco LE, Knol RJJ, Sanchez-Catasus CA, MartinezManzanera O, van der Zant FM, Knuuti J. Machine learning in the integration of simple variables for identifying patients with myocardial ischemia. J Nucl Cardiol. 2020;27(1):147–155. 212. Gomez J, Doukky R. Artificial intelligence in nuclear cardiology. J Nucl Med. 2019;60(8):1042–1043. 213. Brindis RG, Douglas PS, Hendel RC, et al. ACCF/ASNC appropriateness criteria for single-photon emission computed tomography myocardial perfusion imaging (SPECT MPI): a report of the American College of Cardiology Foundation Quality Strategic Directions Committee Appropriate-

CH19.indd 369

369

ness Criteria Working Group and the American Society of Nuclear Cardiology endorsed by the American Heart Association. J Am Coll Cardiol. 2005;46(8):1587–1605. 214. Hendel RC, Berman DS, Di Carli MF, et al. ACCF/ASNC/ ACR/AHA/ASE/SCCT/SCMR/SNM 2009 Appropriate Use Criteria for Cardiac Radionuclide Imaging: A Report of the American College of Cardiology Foundation Appropriate Use Criteria Task Force, the American Society of Nuclear Cardiology, the American College of Radiology, the American Heart Association, the American Society of Echocardiography, the Society of Cardiovascular Computed Tomography, the Society for Cardiovascular Magnetic Resonance, and the Society of Nuclear Medicine. Endorsed by the American College of Emergency Physicians. J Am Coll Cardiol. 2009;53(23):2201–2229. 215. Alexander S, Doukky R. Effective risk stratification of patients on the basis of myocardial perfusion SPECT is dependent on appropriate patient selection. Curr Cardiol Rep. 2015;17(1):549. 216. Doukky R, Hayes K, Frogge N. Appropriate use criteria for SPECT myocardial perfusion imaging: Are they appropriate for women? J Nucl Cardiol. 2016;23(4):695–705. 217. Elgendy IY, Mahmoud A, Shuster JJ, Doukky R, Winchester DE. Outcomes after inappropriate nuclear myocardial perfusion imaging: A meta-analysis. J Nucl Cardiol. 2016;23(4):680-9.. 218. Dos Santos MA, Santos MS, Tura BR, Felix R, Brito AS, De Lorenzo A. Budget impact of applying appropriateness criteria for myocardial perfusion scintigraphy: The perspective of a developing country. J Nucl Cardiol. 2016;23(5):1160–1165. 219. Doukky R, Frogge N, Appis A, et al. Impact of appropriate use on the estimated radiation risk to men and women undergoing radionuclide myocardial perfusion imaging. J Nucl Med. 2016;57(8):1251–1257. 220. Lin GA, Dudley RA, Lucas FL, Malenka DJ, Vittinghoff E, Redberg RF. Frequency of stress testing to document ischemia prior to elective percutaneous coronary intervention. JAMA. 2008;300(15):1765–1773. 221. Levine GN, Bates ER, Blankenship JC, et al. 2011 ACCF/AHA/ SCAI Guideline for Percutaneous Coronary Intervention. A report of the American College of Cardiology Foundation/ American Heart Association Task Force on Practice Guidelines and the Society for Cardiovascular Angiography and Interventions. J Am Coll Cardiol. 2011;58(24):e44–e122. 222. Hillis LD, Smith PK, Anderson JL, et al. 2011 ACCF/AHA Guideline for Coronary Artery Bypass Graft Surgery. A report of the American College of Cardiology Foundation/ American Heart Association Task Force on Practice Guidelines. Developed in collaboration with the American Association for Thoracic Surgery, Society of Cardiovascular Anesthesiologists, and Society of Thoracic Surgeons. J Am Coll Cardiol. 2011;58(24):e123–e210. 223. Khawaja FJ, Jouni H, Miller TD, Hodge DO, Gibbons RJ. Downstream clinical implications of abnormal myocardial perfusion single-photon emission computed tomography based on appropriate use criteria. J Nucl Cardiol. 2013;20(6):1041–1048. 224. Brown KA. Cardiac risk defined by stress myocardial perfusion imaging: Impact on physician decision making and cost savings. J Nucl Cardiol. 2002;9(1):124–126.

15-02-2022 12:26:28

370

Section 3  Indications and Applications

225. Underwood SR, Godman B, Salyani S, Ogle JR, Ell PJ. Economics of myocardial perfusion imaging in Europe--the EMPIRE Study. Eur Heart J. 1999;20(2):157–166. 226. Sabharwal NK, Stoykova B, Taneja AK, Lahiri A. A randomized trial of exercise treadmill ECG versus stress SPECT myocardial perfusion imaging as an initial diagnostic strategy in stable patients with chest pain and suspected CAD: cost analysis. J Nucl Cardiol. 2007;14(2):174–186. 227. Shaw LJ, Mieres JH, Hendel RH, et al. Comparative effectiveness of exercise electrocardiography with or without myocardial perfusion single photon emission computed tomography in women with suspected coronary artery disease: results from the What Is the Optimal Method for Ischemia Evaluation in Women (WOMEN) trial. Circulation. 2011;124(11):1239–1249.

CH19.indd 370

228. Shaw LJ, Marwick TH, Berman DS, et al. Incremental cost-effectiveness of exercise echocardiography vs. SPECT imaging for the evaluation of stable chest pain. Eur Heart J. 2006;27(20):2448–2458. 229. Hlatky MA, Shilane D, Hachamovitch R, Dicarli MF. Economic outcomes in the Study of Myocardial Perfusion and Coronary Anatomy Imaging Roles in Coronary Artery Disease registry: the SPARC Study. J Am Coll Cardiol. 2014;63(10):1002–1008. 230. Mark DB, Federspiel JJ, Cowper PA, et al. Economic outcomes with anatomical versus functional diagnostic testing for coronary artery disease. Ann Intern Med. 2016;165(2):94–102.

15-02-2022 12:26:28

Nuclear Cardiovascular Imaging in Special Populations Robert C. Hendel, Michael C. Desiderio and Gary V. Heller

KEY POINTS ■■

■■

■■

■■

■■

■■

CH20.indd 371

There is abundant literature support for nuclear cardiac imaging in specific populations, with value for both diagnosis and risk stratification. Nuclear cardiac imaging provides important diagnostic and risk stratification data with both single-photon emission computed tomography (SPECT) and positron emission tomography (PET) myocardial perfusion imaging (MPI), while abnormal myocardial blood flow (MBF) assessment identifies microvascular disease with independent risk of coronary events. Data on MBF are increasingly important in specific populations, such as diabetics and women. In diabetic patients, patients with normal exercise SPECT carry a low risk of coronary events, while those with abnormal MBF are at the highest risk of events. Overall, MPI in female patients provides similar diagnostic and risk stratification data in male patients, while MBF data provide further risk stratification especially in nonobstructive disease states. Renal failure patients are at high risk for cardiac events, and myocardial perfusion data provide important risk stratification.

■■

■■

CHAPTER

20

In the elderly population, exercise testing alone does not provide adequate discrimination between normal and abnormal results, while SPECT and PET MPI provide incremental prognostic value. In obese patients, the use of attenuation correction, alternate patient positioning, and altered radiopharmaceutical dosing/ protocols is recommended, which will then provide similar clinical information to that obtained in a nonobese population.

INTRODUCTION Previous chapters have demonstrated the important role of myocardial perfusion imaging (MPI) for the diagnosis and risk stratification of epicardial coronary artery disease (CAD) in the general population of patients with known or suspected CAD. Of importance are patient groups that are at higher risk of CAD and adverse outcomes, in which MPI can identify both low- and high-risk groups. There is a robust literature validating the use of MPI in these more specific patient cohorts. This chapter will describe the value of MPI for the assessment of CAD and risk stratification in special populations consisting of diabetic patients, women, patients with chronic renal disease, obese patients, and the elderly. These five patient populations present their own unique challenges from a diagnostic, as well as prognostic, perspective. In the past, the role of nuclear imaging has

15-02-2022 17:44:05

372

Section 3  Indications and Applications

primarily focused upon identifying patients at risk for epicardial CAD and its consequences. There is a growing body of literature demonstrating that many patients in these groups have disease processes that are more extensive than epicardial disease alone. At the same time, an emerging role for nuclear perfusion imaging is in the evaluation of atherosclerotic disease of the coronary microvasculature (coronary microvascular disease, CMD) through the assessment of myocardial blood flow (MBF) and myocardial blood flow reserve (MBFR, also known as coronary flow reserve [CFR]). While the primary purpose of this chapter is to present data on the role of nuclear cardiology for epicardial CAD, its new role expanding beyond the microvascular disease for each patient group will also be discussed.

▶▶Cardiac Microvascular Disease: Clinical Consequences and its Importance in Special Populations Chest pain without obstructive coronary disease is a clinical entity, which has received renewed interest largely due to advances in cardiac imaging. Endothelial dysfunction of the CMD has been implicated in chest pain syndromes, exertional dyspnea, and possibly heart failure for many years.1–3 CMD is associated with subclinical epicardial disease but is still impacted by traditional cardiovascular risk factors, such as hypertension, hyperlipidemia, smoking, and diabetes.1,2 It is also a highly prevalent disorder among women and diabetes.1,3 Our current understanding of the coronary vascular system is that it is a continuum of vessels from the larger epicardial arteries traversing to smaller vessels into the coronary capillary bed . These smaller vessels (pre-arterioles and arterioles) are involved in the regulation of blood flow, likely to a greater degree than the epicardial arteries. Figure 20-1 illustrates the disease processes affecting the microvascular tree, including atherosclerotic disease and coronary spasm, and the clinical impact of CMD even in the absence of epicardial disease is well described.1–3 Until recently, there were few diagnostic modalities available to evaluate CMD, especially noninvasively. Cardiac positron emission tomography (PET) has emerged to the forefront of the noninvasive diagnostic arena due to the ability to evaluate CMD

CH20.indd 372

through the measurement of MBF,4–6 although there is promise for novel SPECT techniques, which are currently in development. Multiple software programs for assessing MBF using cardiac PET are available, giving this procedure considerable visibility. MBF is described in detail in Chapter 11. Briefly, MBF data are obtained at both rest and stress conditions, with the comparison between the two conditions termed MBFR; the ratio between the two conditions MBFR can be determined globally, as well as regionally, with correlations with perfusion data. It is thought that dysfunction of these smaller vascular beds may precede overt epicardial disease and contribute to cardiovascular risk independently, especially in patients with diabetes, hypertension, and renal failure.2 Furthermore, the role of MBFR assessment of multivessel disease (MVD) is indeed a powerful diagnostic tool. Data have demonstrated that CFR is an independent risk factor for hard events and major adverse cardiovascular events in patients evaluated for ischemia after adjusting for patient characteristics and traditional cardiac risk factors. CFR provides incremental risk stratification even beyond the estimates of left ventricular (LV) systolic function and the extent and severity of ischemia.3 This effect was confirmed in patients regardless of perfusion data (Fig. 20-2).5 The literature regarding the presence and consequences of microvascular disease in each of the five special patient populations will be addressed in this chapter.

DIABETES MELLITUS In 2014, the global prevalence of diabetes was estimated to be 9% among adults aged 18+ years as per the World Health Organisation7 and is only rising. In the United States, the Centers for Disease Control and Prevention (CDC) alone estimates that 29 million people (9.3% prevalence) suffer from diabetes, with one in four people unaware of their diabetic status.8 This increase in prevalence of diabetes mirrors the obesity epidemic in the United States with 33% of the population being obese.9 The overall prevalence of CAD has been estimated to be as high as 55% in diabetic patients versus 4% in the general population, and CAD is the leading cause of death in diabetics. In a landmark Finnish observational trial, the authors

15-02-2022 17:44:05

Chapter 20  Nuclear Cardiovascular Imaging in Special Populations

373

Normal Structure and Function of Coronary Macro- and Microcirculation Epicardial Arteries >400 µm

Pre-Arterioles 100-400 µm

Arterioles 40-100 µm

Capillaries 5 years, and two cardiac risk factors of smoking, hypertension, hypercholesterolemia, and positive family history of CAD) in addition to diabetes.34 The rate of silent myocardial infarction (SMI) in this study was 22% and “overt or silent CAD progression” (cardiac events + new ischemia/scar) was seven-fold higher in patients with a normal than an abnormal MPS (35.6% vs. 4.6%), documenting the screening efficacy of MPS in these patients. Patients randomized to revascularization had similar rates of symptomatic CAD progression, but lower rates of asymptomatic CAD (more ischemia or new scar) progression (54.3% vs. 15.8%). Thus, performance of MPI in high-risk diabetics may help identify those at higher risk of MACE.34 The treatment of this group of patients with invasive versus medical therapies needs to be tested in large-scale randomized trials.

▶▶Myocardial Blood Flow and Microvascular Disease in Diabetic Patients Diabetes is considered a CAD risk equivalent in patients, and SPECT data corroborate this concept.23 This higher risk for cardiovascular events, even among patients without obstructive disease, could implicate another aspect, namely, microvascular disease CMD.3 A summary of key studies using MPI in patients with diabetes is presented in Table 20-1. Murthy et al. examined patients with and without diabetes using 82Rb-PET MPI and found that addition of MBF data to clinical and imaging risk models improved risk discrimination for both diabetics and nondiabetics.35 Diabetic patients without known CAD but with impaired MBF reserve (MBFR) experienced a rate of cardiac death comparable to that for nondiabetic patients with known CAD (2.8% per year vs. 2.0% per year). Conversely, diabetics without known CAD and preserved MBFR had very low annualized cardiac mortality, which was similar to patients without known CAD or diabetes mellitus and normal

CH20.indd 380

stress perfusion and systolic function (0.3% per year vs. 0.5% per year)33 (Fig. 20-9). The Murthy study included those with and without perfusion abnormalities. In order to examine the impact of CMD, a recent study focused on microvascular disease.36 These investigators examined those patients with normal PET perfusion in diabetic and nondiabetic patients with MBF measured. They found that diabetic patients with abnormal MBFR and normal perfusion had significantly higher event rates than those with normal MBFR, the latter of which was similar to a normal population. Thus, evaluation of blood flow in diabetic patients could provide further insights into characterizing risk of cardiac events in this patient group.

▶▶Conclusions: Diabetic Patients Diabetes has been established as a coronary disease equivalent, and diagnosis and risk stratification for CAD are of considerable importance to identify high- and low-risk patients for coronary events to gear therapy accordingly. The nuclear imaging data presented offer both opportunities, with clear clinical benefits (Table 20-2). A strategy to be considered is to perform exercise stress imaging in exercise capable symptomatic patients, as this will stratify high- and low-risk diabetic patients26 For those unable to exercise, pharmacologic stress MPI is of considerable value. The measurement of MBF may further stratify higher-risk patients, as abnormal blood flow portends a higher risk of coronary events in diabetic patients than those diabetic patients with normal blood flow.4,35 Table 20-2 Value of Radionuclide Imaging in Diabetic Patients • MPI is an effective tool in the diagnosis and risk stratification of diabetic patients. • Diabetic patients with longstanding and more severe disease are at greater risk for cardiac events. • While routine stress MPI is not recommended for asymptomatic diabetic patients, the value of stress MPI in symptomatic patients is well demonstrated. • The ability to perform exercise in diabetic patients conveys a lower risk of cardiac events, particularly those with normal imaging results. • Myocardial blood flow assessment identifies “at-risk” diabetic patients with evidence of microvascular disease.

15-02-2022 17:44:18

Chapter 20  Nuclear Cardiovascular Imaging in Special Populations 14%

N = 2423 CD = 122

12%

P = 0.89

CAD+/DM+ (N = 606) CAD+/DM– (N = 669)

Cardiac mortality

10% P = 0.37

8%

CAD–/DM+ CFR ≤1.6 (N = 227)

6% P = 0.01 4%

P = 0.01

P = 0.005

CAD–/DM– NI MPI/EF (N = 682)

2% 0%

P = 0.65 0

1

2 Years

WOMEN While cardiovascular disease (CVD), including CAD and stroke, is the leading cause of death in both men and women in the United States, more women die than men each year.9 In 2013, CVD caused about one death every 80 seconds in women and was responsible for more number of deaths than were cancer, chronic lower respiratory disease, and diabetes combined. Younger women (aged 35−45 years) for the first time in the last four decades had higher CAD death rates likely representing the influence of the obesity epidemic in younger women.9,37 American College of Cardiology/ American Heart Association (ACC/AHA) guidelines recommend classification of women into three CVD risk categories: 1. High risk (individuals with CAD, stroke, peripheral arterial disease, chronic kidney disease [CKD], and diabetes) 2. At risk: individuals with risk factors of hypertension, dyslipidemia, obesity, smoking, physical inactivity, family history of premature CAD, poor exercise tolerance, lupus, rheumatoid arthritis, preeclampsia, pregnancy-induced hypertension, and gestational diabetes 3. At optimal risk: absence of any risk factors. High-risk equivalent states, including peripheral arterial disease and longstanding or poorly controlled

CH20.indd 381

CAD–/DM+ CFR >1.6 (N = 339)

3

381

FIGURE 20-9  Annualized cardiac mortality in diabetic and nondiabetic patients with or without coronary artery disease (CAD) and with preserved or reduced coronary flow reserve (CFR). MPI, myocardial perfusion imaging; EF, ejection fraction; Nl MPI, normal myocardial perfusion imaging; CD, cardiac death. (Reproduced with permission from Murthy VL, Naya M, Foster CR, et al. Association between coronary vascular dysfunction and cardiac mortality in patients with and without diabetes mellitus. Circulation. 2012;126(15):1858−1868.)

diabetes mellitus for women aged more than 40 years, also categorize a woman at high ischemic heart disease (IHD) risk. In the presence of certain high-risk markers (peripheral vascular disease and longstanding/poorly controlled diabetes), the IHD risk estimations can be elevated by one category.38 This section will review the data on women and heart disease and the value of radionuclide MPI in assessment and prognosis.

▶▶Diagnostic Testing In Women The diagnosis of CAD in women is challenging due to the atypical symptoms experienced by women, which leads to less referral for stress testing and cardiac catheterization in women with suspected CAD.39,40 A meta-analysis revealed that women with an acute MI had lower odds of presenting with chest pain than men (odds ratio 0.63; 95% confidence interval [CI] 0.59−0.68) and were more likely than men to present with atypical symptoms.41 Similar findings were also observed in a prospective cohort study, which showed that despite chest pain being the most common presentation of ACS in both sexes, women presented more frequently without chest pain than men (19.0% vs. 13.7%).42 The misdiagnosis of chest pain in women remains a contemporary issue. Adding to the challenge of diagnosing heart disease in women is the high prevalence of nonobstructive disease (40–60%) at the time

15-02-2022 17:44:18

382

Section 3  Indications and Applications

of cardiac catheterization in women with chest pain. This finding was confirmed by Jespersen et al., which showed higher incidence of nonobstructive CAD in women than in men (65% vs. 32%).43 In this study, normal coronary arteries and nonobstructive CAD were associated with 52% and 85% increased risk of MACE (cardiovascular mortality, hospitalization for MI, heart failure, or stroke) and with 29% and 52% increased risk of all-cause mortality, respectively.43 Similar findings were demonstrated in the 10-year follow up of the WISE (Women’s Ischemia Syndrome Evaluation) study, which showed that two-thirds (62%) of women with angina had nonobstructive disease. The 10-year adverse outcome rates (cardiovascular death or MI rate) in the women with nonobstructive CAD were almost double (12.8% vs. 6.7%) than that observed in women with angiographically normal coronary arteries.44 These differences in the presentation and prognosis of CAD in women could be related to the increased incidence of CMD and different characteristics of plaque morphology in women presenting with ACS.45 However, studies using optical coherence tomography did not reveal

a sex difference in the prevalence of plaque rupture or erosion.46,47 Exercise stress testing or exercise tolerance testing (ETT) is the most common screening test used to detect CAD. The diagnostic accuracy of ETT is only modest, with a sensitivity and specificity for the detection of a 50% luminal stenosis or greater of 47% to 80% and 63% to 73%, respectively.48,49 While to overall sensitivity of this test is moderate, in certain populations it may be useful, particularly with regards to outcomes. To examine ETT in comparison to cardiac SPECT imaging, the optimal initial method of ischemia evaluation in symptomatic women with low-to-intermediate likelihood of CAD was assessed in the WOMEN trial.50 In this trial, 824 symptomatic women with low-to-intermediate pretest probability of CAD were randomized to one of two diagnostic strategies: ETT alone or exercise SPECT MPI. While this study did not address diagnostic accuracy, at 2 years, there was no difference in major adverse cardiac events (98.0% for ETT and 97.7% for MPI; P = 0.59; Fig. 20-10). The clinical implications of this

Cumulative event-free survival (%)

100%

p = 0.59

90%

80%

70%

60%

ETT Excercise MPI

0%

0 0.1 0.2 0.3 0.4 0.5 0.6 0.7 0.8 0.9 1 1.1 1.2 1.3 1.4 1.5 1.6 1.7 1.8 1.9 Time to follow-up (in years)

Number at risk 388 384

382 373

382 373

380 372

379 368

2 376 367

FIGURE 20-10  Comparison of outcomes in a randomized trial between exercise tolerance testing (ETT) versus exercise myocardial perfusion imaging (MPI) strategy in women with chest pain. (Reproduced with permission from Shaw LJ, Mieres JH, Hendel RH, et al. Comparative effectiveness of exercise electrocardiography with or without myocardial perfusion single photon emission computed tomography in women with suspected coronary artery disease: results from the What Is the Optimal Method for Ischemia Evaluation in Women (WOMEN) trial. Circulation. 2011;124(11):1239−1249.)

CH20.indd 382

15-02-2022 17:44:20

Chapter 20  Nuclear Cardiovascular Imaging in Special Populations

trial’s results are noteworthy. For low−intermediaterisk women capable of performing exercise, routine ETT without imaging appears a reasonable first-line test resulting in 2-year outcomes equivalent to those achieved with exercise MPI.50 The low−intermediate risk woman is a candidate for an exercise ECG if she is functionally capable and has a normal or interpretable rest ECG. Women with intermediate− high IHD risk with an abnormal 12-lead rest ECG (i.e., with resting ST-segment abnormalities) may be referred for stress imaging, as well as women at any risk unable to complete and exercise bout of diagnostic level. Women at high IHD risk with stable symptoms may be referred for a stress imaging modality for functional assessment of their ischemic burden and for guiding post-test and anti-ischemic therapeutic decision making.38

▶▶Diagnostic Value of SPECT Imaging in Women As noted above, the diagnostic accuracy of the ETT alone is moderate, which becomes an important issue in intermediate to high likelihood female patients in which treatment strategies depend upon testing results. The importance of MPI in improving the diagnostic accuracy of stress testing in women was demonstrated in a contemporary meta-analysis of 14 SPECT studies, which demonstrated a sensitivity of 81% and specificity of 78% in women with no known CAD,51 substantially higher than that of ETT alone. SPECT MPI therefore has a well-established role for the diagnosis of ischemic heart disease.52,53 The higher sensitivity of SPECT MPI is particularly useful in identifying the presence of ischemia in women with potentially false-positive exercise tolerance tests.54 In a report of more than 4000 patients of which more than one-third were women, exercise MPI showed a net reclassification improvement of 36% over exercise treadmill stress tests (ETT).55 Thus, additional ischemia heart disease risk information was available by performance of MPI in one out of every three patients.55 While radionuclide imaging has a higher sensitivity than ETT in the detection of single-vessel disease, the highest accuracy has been found in women with multiple-vessel disease compared to those with single-vessel disease.56,57

CH20.indd 383

383

It has also been questioned whether stress SPECT MPI is less accurate in women than in men, similar to ETT, likely due to attenuation artifact. To address this issue, Iskandar et al.58 performed a bivariate metaanalysis on 26 studies that met criteria. In contrast to ETT, SPECT imaging provided similar and high sensitivity and specificity for both genders. However, some differences in SPECT diagnostic accuracy between men and women have been noted, and may be related to increased recognition of cardiovascular microvascular disease, which is more prevalent in women and leads to reduced specificity when compared with coronary angiography due to the absence of obstructive disease. Other factors, such as the imaging of patients with a lower pretest likelihood of disease, reduced exercise capacity, and a lower maximum heart rate may also explain the reduced diagnostic sensitivity in women.45,59 Specificity of SPECT in women may also be reduced due to the commonly seen artifacts related to breast tissue attenuation; attenuation correction or alternate position protocols (i.e., supine/prone) should be routinely performed (see Chapter 9). The accuracy of radionuclide imaging is affected by several factors in women, particularly breast attenuation, body habitus, and small heart size.60–62 To appreciate potential breast attenuation artifact, one of the best approaches is to begin with a review of the unprocessed rotating images. On standard display if a defect correlates with an area of soft tissue on the raw images and the wall motion in the same region is normal with gated SPECT, the finding is consistent with an artifact and not CAD. The use of technetium-99 imaging agents (higher energy) and ECG gating using this concept has been shown to improve specificity in women.63,64 Other techniques to increase specificity of MPI include use of supine/prone MPI or the use of attenuation correction.59,65 Unfortunately, despite the improvement in diagnostic accuracy noted with these methods, they are largely underutilized.65 The overall correlation between sum stress scores for two readers improved with supine/prone imaging in female patients undergoing supine/prone MPI (0.86 vs. 0.75) as compared to supine only MPI.66 The use of alternate positioning with systems is also highly recommended to adjudicate attenuation artifact. Solid-state detector technology (i.e., cadmium zinc telluride [CZT]) may provide

15-02-2022 17:44:20

Section 3  Indications and Applications

high diagnostic accuracy in women, comparable to that with men,67 and provide up to 77% reclassification of equivocal scans in women, as well as reducing the need for rest scanning by more than 50%.68 Due to the smaller heart size in some women, as noted above, normal limits for LVEF and for transient ischemic dilation ratios may be higher in women and sex-based normal limits and software should be used to improve diagnostic accuracy.59 Pharmacologic stress is an important alternative in women with limited exercise capacity. Dipyridamole, adenosine, and regadenoson stress protocols have been found to be comparable to exercise imaging in primarily male populations69 In a prospective study of 201 women, SPECT MPI had a 95% sensitivity, 66% specificity, and 85% accuracy in the detection of coronary stenosis greater than 70% regardless of presenting symptoms, prior history of MI, or pretest probability of coronary disease.70 A more recent trial with regadenoson stress MPI revealed similar perfusion results to that obtained with adenosine, irrespective of patient-specific factors, including gender.71 Pharmacologic stress testing with adenosine or regadenoson has a higher rate of adverse effects in women, with increased incidence of chest pain, GI discomfort, dizziness, and headache, perhaps due to the smaller volume of distribution of these agents in women.72,73 Additionally, women with normal MPI studies more frequently had ischemic ECG changes following an adenosine infusion than men.74

▶▶Risk Stratification of Women with SPECT MPI In addition to a higher diagnostic accuracy, SPECT MPI has demonstrated value in predicting cardiacrelated outcomes (cardiac death, nonfatal MI), based upon imaging results of the study. In a large singlecenter study by Hachamovitch et al., female patients were followed for up to 3 years post-testing for cardiac events (cardiac death and MI) following adenosine SPECT imaging.75 They reported patients with normal or mildly abnormal imaging results had a low annualized event rate ( 50 old Diabetes mellitus < 10 years Left ventricular hypertropy Smoking On hemodialysis > 1 year

ECG CXR Rest echocardiogram

Intermediate

Known CAD At least 3 risk factors: Hypertension Dyslipidenmia Age > 50 old Diabetes mellitus < 10 years Left ventricular hypertropy Smoking On hemodialysis > 1 year

ECG CXR Rest echocardiogram Stress/Rest SPECT MPI

High

Active cardiac condition Ischemia on stress test LVEF < 35% Diabetes mellitus ≥ 10 years Patients ≥ 50 years old

Left and right cardiac cathetrerizaiton with coronary angiography

CAD, Coronary artery disease; CXR, chest x-ray; ECG, electrocardiogram; LVEF, left ventricular ejection fraction; MPI, myocardial perfusion imaging; SPECT, single-photon emission computed tomography.

CH21.indd 414

15-02-2022 10:00:52

Chapter 21  Preoperative Risk Assessment for Noncardiac Surgery

who are not able to achieve the target heart rate due to limited functional capacity. Radionuclide MPI has emerged as a useful tool for detection of CAD in these patients. In a study of ESLD patients, Zoghbi et al. showed that radionuclide MPI had 99% negative predictive value for adverse perioperative cardiovascular events.59 In a study of over 2500 patients referred for liver transplantation who had undergone SPECT MPI, Duvall et al. found that approximately 8% of patients had abnormal perfusion results, with obstructive disease found in 1% of patients on angiography.60 However, this may be because the patients in their study tended to be younger and less likely to have risk factors for CAD compared to the general population. Despite these favorable results, there is a theoretical concern that ESLD patients have increased vasodilation at baseline in these patients due to underlying liver disease, thereby reducing the potential to achieve maximal hyperemia with vasodilators (i.e., regadenoson or adenosine) and thereby may limit sensitivity for the detection of CAD.61 Cardiac CT (CAC and CCTA) is an emerging risk assessment tool for the pre-liver transplant population. In a study of 1045 patients with ESLD, CCTA showed a low prevalence of obstructive CAD (7.9%),62. Routine preoperative CCTA is believed to have low yield for the detection of significant CAD in preliver transplant patients.63 However, CCTA has a good negative predictive value to exclude obstructive CAD63 and is considered safe for use in ESLD patients.64 Elevated CAC of more than 400 has been shown to be associated with an increased risk of MACE after liver transplantation65 and may be a useful and inexpensive test to determine the need for subsequent evaluation. Therefore, based on the current research, a strategy combining CCTA and CAC may be useful for the detection of CAD in these patients. Although these studies are encouraging, further research is needed to better define the role of cardiac CT for liver transplant patients. Invasive coronary angiography remains the gold standard for the diagnosis of significant CAD in the preliver transplant population. Revascularization of significant CAD prior to liver transplant may reduce postoperative MACE and mortality.66 However, coronary angiography in ESLD patients may carry significant risk of vascular complications and bleeding due

CH21.indd 415

415

to the underlying coagulopathy caused by ESLD,67 although transradial catheterization appears to be safe in ESLD patients.68

PATIENTS WITH CHRONIC HEART FAILURE UNDERGOING NONCARDIAC SURGERY Over 6 million people in the United States have HF , with its prevalence expected to increase significantly over the next 10 years,69 partly due to an aging population and improved survival of HF patient.70 As a result, a significant portion of these patients may undergo noncardiac surgery during their lifetime. The presence of chronic HF has been shown to portend worse prognosis in patients undergoing noncardiac surgery,70 and thus these patients need special attention during the preoperative process. Unfortunately, there is no clear consensus on the optimal preoperative management of HF patients prior to surgery.71 However, the overall goal of the preoperative risk assessment in patients with HF is to identify patients who are at increased risk of developing decompensated HF in the perioperative setting, as well as those at risk of superimposed ischemia and to take steps to optimize these patients prior to surgery. As with patients without HF, preoperative functional status can have a major impact on the perioperative course among HF patients. Preoperative testing in these patients generally follows the same course as for patients without HF, and includes an electrocardiogram, a chest radiograph, an echocardiogram, and stress testing if appropriate. Left ventricular ejection fraction (EF) is an important prognostic marker—reduced EF is associated with increased perioperative MACE.72 Measurement of natriuretic peptide levels such as brain natriuretic peptide (BNP) is also utilized by some centers, as elevated BNP levels have been shown to be associated with increased perioperative MACE.73 Stress testing and coronary angiography in HF patients are indicated as per preoperative risk assessment guidelines. Regarding risk assessment tools, both the RCRI and NSQIP risk calculators are recommended tools for preoperative risk assessment. Additionally, Andersson et al. have developed a risk index from Danish registry data that utilizes variables included in the RCRI index

15-02-2022 10:00:52

416

Section 3  Indications and Applications

plus several additional variables to predict 30-day mortality in HF patients undergoing noncardiac surgery.74 However, this index is not externally validated. The mainstay of preoperative management of patients with HF is initiation or continuation of goaldirected medical therapy and to optimize patient’s clinical status prior to surgery, including the use of beta-blocker therapy.71 The role of revascularization in ischemic cardiomyopathy prior to surgery is controversial at this time and not routinely recommended.71

CONCLUSION A comprehensive preoperative cardiac risk assessment prior to noncardiac surgery involves an assessment of surgical- and patient-specific risk factors, with further testing if indicated to identify optimal risk reduction strategies. Clinical markers, physical examination, and functional capacity are among the most important elements of estimating perioperative risk. Noninvasive testing is helpful in the group of patients estimated to be at intermediate risk, who have limited functional capacity and are being evaluated for high-risk surgery. Radionuclide MPI has a long history of clinical utility for such patients, demonstrating excellent risk stratification for patients at intermediate clinical risk and offers a physiology-based assessment for nearly all patients groups, even those unable to perform physical exercise. A thorough perioperative evaluation offers an opportunity to initiate or modify cardiac care so as to provide benefit in both the short- and long-term outcome for patients with known or suspected ischemic heart disease. Additionally, preoperative assessment serves to inform the surgeon and anesthesiologist, as well as the patient and their family, about the anticipated risk of the planned procedure.

REFERENCES 1. Devereaux PJ, Sessler DI. Cardiac complications in patients undergoing major noncardiac surgery. N Engl J Med. 2015;373: 2258–2269. 2. Bartels K, Karhausen J, Clambey ET, Grenz A, Eltzschig HK. Perioperative organ injury. Anesthesiology. 2013;119:1474–1489. 3. Botto F, Alonso-Coello P, Chan MT, et al. Myocardial injury after noncardiac surgery: a large, international, prospective cohort

CH21.indd 416

study establishing diagnostic criteria, characteristics, predictors, and 30-day outcomes. Anesthesiology. 2014;120:564–578. 4. Puelacher C, Lurati Buse G, Seeberger D, et al. Perioperative myocardial injury after noncardiac surgery: incidence, mortality, and characterization. Circulation. 2018;137:1221–1232. 5. Fleisher LA, Fleischmann KE, Auerbach AD, et al. ACC/ AHA guideline on perioperative cardiovascular evaluation and management of patients undergoing noncardiac surgery: a report of the American College of Cardiology/American Heart Association Task Force on practice guidelines. J Am Coll Cardiol. 2014;64:e77–e137. 6. Jacobs AK, Anderson JL, Halperin JL, et al. The evolution and future of ACC/AHA clinical practice guidelines: a 30-year journey: a report of the American College of Cardiology/ American Heart Association Task Force on practice guidelines. Circulation. 2014;130:1208–1217. 7. Lee TH, Marcantonio ER, Mangione CM, et al. Derivation and prospective validation of a simple index for prediction of cardiac risk of major noncardiac surgery. Circulation. 1999;100:1043–1049. 8. Gupta PK, Gupta H, Sundaram A, et al. Development and validation of a risk calculator for prediction of cardiac risk after surgery. Circulation. 2011;124:381–387. 9. Cohen ME, Ko CY, Bilimoria KY, et al. Optimizing ACS NSQIP modeling for evaluation of surgical quality and risk: patient risk adjustment, procedure mix adjustment, shrinkage adjustment, and surgical focus. J Am Coll Surg. 2013;217:336–46 e1. 10. Morris CK, Ueshima K, Kawaguchi T, Hideg A, Froelicher VF. The prognostic value of exercise capacity: a review of the literature. Am Heart J. 1991;122:1423–1431. 11. Cutler BS, Wheeler HB, Paraskos JA, Cardullo PA. Applicability and interpretation of electrocardiographic stress testing in patients with peripheral vascular disease. Am J Surg. 1981;141:501–506. 12. Eagle KA, Coley CM, Newell JB, et al. Combining clinical and thallium data optimizes preoperative assessment of cardiac risk before major vascular surgery. Ann Intern Med. 1989;110:859–866. 13. Eagle KA, Singer DE, Brewster DC, Darling RC, Mulley AG, Boucher CA. Dipyridamole-thallium scanning in patients undergoing vascular surgery. Optimizing preoperative evaluation of cardiac risk. JAMA. 1987;257:2185–2189. 14. van Diepen S, Bakal JA, McAlister FA, Ezekowitz JA. Mortality and readmission of patients with heart failure, atrial fibrillation, or coronary artery disease undergoing noncardiac surgery: an analysis of 38 047 patients. Circulation. 2011;124:289–296. 15. Healy KO, Waksmonski CA, Altman RK, Stetson PD, Reyentovich A, Maurer MS. Perioperative outcome and long-term mortality for heart failure patients undergoing intermediateand high-risk noncardiac surgery: impact of left ventricular ejection fraction. Congest Heart Fail. 2010;16:45–49. 16. Hendel RC, Whitfield SS, Villegas BJ, Cutler BS, Leppo JA. Prediction of late cardiac events by dipyridamole thallium imaging in patients undergoing elective vascular surgery. Am J Cardiol. 1992;70:1243–1249. 17. Eagle KA, Berger PB, Calkins H, et al. ACC/AHA guideline update for perioperative cardiovascular evaluation for noncardiac surgery---executive summary a report of the American College of Cardiology/American Heart Association Task Force on Practice Guidelines (Committee to Update the 1996

15-02-2022 10:00:52

Chapter 21  Preoperative Risk Assessment for Noncardiac Surgery Guidelines on Perioperative Cardiovascular Evaluation for Noncardiac Surgery). Circulation. 2002;105:1257–1267. 18. Shaw LJ, Eagle KA, Gersh BJ, Miller DD. Meta-analysis of intravenous dipyridamole-thallium-201 imaging (1985 to 1994) and dobutamine echocardiography (1991 to 1994) for risk stratification before vascular surgery. J Am Coll Cardiol. 1996;27:787–798. 19. Thai JN, Abidov A, Jie T, Krupinski EA, Kuo PH. Nuclear myocardial perfusion imaging versus stress echocardiography in the preoperative evaluation of patients for kidney transplantation. J Nucl Med Technol. 2015;43:201–205. 20. L’Italien GJ, Paul SD, Hendel RC, et al. Development and validation of a Bayesian model for perioperative cardiac risk assessment in a cohort of 1,081 vascular surgical candidates. J Am Coll Cardiol. 1996;27:779–786. 21. Stratmann HG, Younis LT, Wittry MD, Amato M, Mark AL, Miller DD. Dipyridamole technetium 99m sestamibi myocardial tomography for preoperative cardiac risk stratification before major or minor nonvascular surgery. Am Heart J. 1996;132:536–541. 22. Stratmann HG, Younis LT, Wittry MD, Amato M, Miller DD. Dipyridamole technetium-99m sestamibi myocardial tomography in patients evaluated for elective vascular surgery: prognostic value for perioperative and late cardiac events. Am Heart J. 1996;131:923–929. 23. Koutelou MG, Asimacopoulos PJ, Mahmarian JJ, Kimball KT, Verani MS. Preoperative risk stratification by adenosine thallium 201 single-photon emission computed tomography in patients undergoing vascular surgery. J Nucl Cardiol. 1995;2:389–394. 24. Wolk MJ, Bailey SR, Doherty JU, et al. ACCF/AHA/ASE/ ASNC/HFSA/HRS/SCAI/SCCT/SCMR/STS 2013 multimodality appropriate use criteria for the detection and risk assessment of stable ischemic heart disease: a report of the American College of Cardiology Foundation Appropriate Use Criteria Task Force, American Heart Association, American Society of Echocardiography, American Society of Nuclear Cardiology, Heart Failure Society of America, Heart Rhythm Society, Society for Cardiovascular Angiography and Interventions, Society of Cardiovascular Computed Tomography, Society for Cardiovascular Magnetic Resonance, and Society of Thoracic Surgeons. J Am Coll Cardiol. 2014;63:380–406. 25. Beller GA, Zaret BL. Contributions of nuclear cardiology to diagnosis and prognosis of patients with coronary artery disease. Circulation. 2000;101:1465–1478. 26. Ghadri JR, Fiechter M, Verauth K, et al. Coronary calcium score as an adjunct to nuclear myocardial perfusion imaging for risk stratification before non-cardiac surgery. J Nucl Med. 2012;53:1081–1086. 27. Fathala A, Aljefri A, Alsugair A, Abouzied M. Coronary artery calcification detected by PET/CT scan as a marker of yocardial ischemia/coronary artery disease. Nucl Med Commun. 2011;32:273–278. 28. Hwang JW, Kim EK, Yang JH, et al. Assessment of perioperative cardiac risk of patients undergoing noncardiac surgery using coronary computed tomographic angiography. Circ Cardiovasc Imaging. 2015;8. 29. Sheth T, Butler C, Chow B, et al. The coronary CT angiography vision protocol: a prospective observational imaging cohort study in patients undergoing non-cardiac surgery. BMJ Open. 2012;2.

CH21.indd 417

417

30. Koshy AN, Ha FJ, Gow PJ, et al. Computed tomographic coronary angiography in risk stratification prior to non-cardiac surgery: a systematic review and meta-analysis. Heart. 2019;105:1335–1342. 31. Smilowitz NR, Berger JS. Perioperative cardiovascular risk assessment and management for noncardiac surgery: a review. JAMA. 2020;324:279–290. 32 Santry HP, Gillen DL, Lauderdale DS. Trends in bariatric surgical procedures. JAMA. 2005;294:1909–1917. 33. Gemignani AS, Muhlebach SG, Abbott BG, Roye GD, Harrington DT, Arrighi JA. Stress-only or stress/rest myocardial perfusion imaging in patients undergoing evaluation for bariatric surgery. J Nucl Cardiol. 2011;18:886–892. 34. Bateman TM, Heller GV, McGhie AI, et al. Diagnostic accuracy of rest/stress ECG-gated Rb-82 myocardial perfusion PET: comparison with ECG-gated Tc-99m sestamibi SPECT. J Nucl Cardiol. 2006;13:24–33. 35. Raggi P. Pre-renal transplant risk stratification: a perpetual quandary. JACC Cardiovasc Imaging. 2018;11:855–858. 36. Kasiske BL, Chakkera HA, Roel J. Explained and unexplained ischemic heart disease risk after renal transplantation. J Am Soc Nephrol. 2000;11:1735–1743. 37. Kasiske BL, Guijarro C, Massy ZA, Wiederkehr MR, Ma JZ. Cardiovascular disease after renal transplantation. J Am Soc Nephrol. 1996;7:158–165. 38. Weir MR. Is there an optimal strategy for pretransplant cardiovascular screening? Transplantation. 2015;99:656–657. 39. Wang LW, Fahim MA, Hayen A, et al. Cardiac testing for coronary artery disease in potential kidney transplant recipients. Cochrane Database Syst Rev. 2011:CD008691. 40. West JC, Napoliello DA, Costello JM, et al. Preoperative dobutamine stress echocardiography versus cardiac arteriography for risk assessment prior to renal transplantation. Transpl Int. 2000;13(Suppl 1):S27–S30. 41. Tita C, Karthikeyan V, Stroe A, Jacobsen G, Ananthasubramaniam K. Stress echocardiography for risk stratification in patients with end-stage renal disease undergoing renal transplantation. J Am Soc Echocardiogr. 2008;21:321–326. 42. Helve S, Laine M, Sinisalo J, et al. Even mild reversible myocardial perfusion defects predict mortality in patients evaluated for kidney transplantation. Eur Heart J Cardiovasc Imaging. 2018;19:1019–1025. 43. Abuzeid W, Iwanochko RM, Wang X, Kim SJ, Husain M, Lee DS. Prognostic impact of SPECT-MPI after renal transplantation. J Nucl Cardiol. 2017;24:295–303. 44. Helve S, Nieminen T, Helantera I, et al. The value of myocardial perfusion imaging in screening coronary artery disease before kidney transplantation. Clin Transplant. 2020;34:e13894. 45 Wilson RS, Lin T, Chambers CE, Kadry Z, Jain AB. Assessing cardiovascular risk in the prerenal transplant population: Comparison of myocardial perfusion imaging and coronary angiography with risk factor stratification. Clin Transplant. 2019;33:e13735. 46. Rabbat CG, Treleaven DJ, Russell JD, Ludwin D, Cook DJ. Prognostic value of myocardial perfusion studies in patients with end-stage renal disease assessed for kidney or kidneypancreas transplantation: a meta-analysis. J Am Soc Nephrol. 2003;14:431–439. 47. AlJaroudi W, Anokwute C, Fughhi I, et al. The prognostic value of heart rate response during vasodilator stress myocardial perfu-

15-02-2022 10:00:52

418

Section 3  Indications and Applications

sion imaging in patients with end-stage renal disease undergoing renal transplantation. J Nucl Cardiol. 2019;26:814–822. 48. Gordi T, Blackburn B, Lieu H. Regadenoson pharmacokinetics and tolerability in subjects with impaired renal function. J Clin Pharmacol. 2007;47:825–833. 49. Aljaroudi W, Hermann D, Hage F, Heo J, Iskandrian AE. Safety of regadenoson in patients with end-stage renal disease. Am J Cardiol. 2010;105:133–135. 50. Doukky R, Rangel MO, Wassouf M, Dick R, Alqaid A, Morales Demori R. The safety and tolerability of regadenoson in patients with end-stage renal disease: the first prospective evaluation. J Nucl Cardiol. 2013;20:205–213. 51. Doukky R, Morales Demori R, et al. Attenuation of the side effect profile of regadenoson: a randomized double-blinded placebo-controlled study with aminophylline in patients undergoing myocardial perfusion imaging. “The ASSUAGE trial”. J Nucl Cardiol. 2012;19:448–457. 52. Winther S, Svensson M, Jorgensen HS, et al. Prognostic value of risk factors, calcium score, coronary cta, myocardial perfusion imaging, and invasive coronary angiography in kidney transplantation candidates. JACC Cardiovasc Imaging. 2018;11:842–854. 53. Chen J, Budoff MJ, Reilly MP, et al. Coronary artery calcification and risk of cardiovascular disease and death among patients with chronic kidney disease. JAMA Cardiol. 2017;2:635–643. 54. Carey WD, Dumot JA, Pimentel RR, et al. The prevalence of coronary artery disease in liver transplant candidates over age 50. Transplantation. 1995;59:859–864. 55. Fili D, Vizzini G, Biondo D, et al. Clinical burden of screening asymptomatic patients for coronary artery disease prior to liver transplantation. Am J Transplant. 2009;9:1151–1157. 56. Targher G, Day CP, Bonora E. Risk of cardiovascular disease in patients with nonalcoholic fatty liver disease. N Engl J Med. 2010;363:1341–1350. 57. Pruthi J, Medkiff KA, Esrason KT, et al. Analysis of causes of death in liver transplant recipients who survived more than 3 years. Liver Transpl. 2001;7:811–815. 58 Johnston SD, Morris JK, Cramb R, Gunson BK, Neuberger J. Cardiovascular morbidity and mortality after orthotopic liver transplantation. Transplantation. 2002;73:901–906. 59. Zoghbi GJ, Patel AD, Ershadi RE, Heo J, Bynon JS, Iskandrian AE. Usefulness of preoperative stress perfusion imaging in predicting prognosis after liver transplantation. Am J Cardiol. 2003;92:1066–1071. 60. Duvall WL, Singhvi A, Tripathi N, Henzlova MJ. SPECT myocardial perfusion imaging in liver transplantation candidates. J Nucl Cardiol. 2020;27:254–265.

CH21.indd 418

61. Davidson CJ, Gheorghiade M, Flaherty JD, et al. Predictive value of stress myocardial perfusion imaging in liver transplant candidates. Am J Cardiol. 2002;89:359–360 62. An J, Shim JH, Kim SO, et al. Prevalence and prediction of coronary artery disease in patients with liver cirrhosis: a registry-based matched case-control study. Circulation. 2014; 130:1353–1362. 63. Di Carli MF, Blankstein R. Low yield of routine preoperative coronary computed tomography angiography in patients evaluated for liver transplantation. Circulation. 2014;130:1337–1339. 64. Poulin MF, Chan EY, Doukky R. Coronary computed tomographic angiography in the evaluation of liver transplant candidates. Angiology. 2015;66:803–810. 65. Kong YG, Ha TY, Kang JW, Hwang S, Lee SG, Kim YK. Incidence and predictors of increased coronary calcium scores in liver transplant recipients. Transplant Proc. 2015;47:1933–1938. 66. Maddur H, Bourdillon PD, Liangpunsakul S, et al. Role of cardiac catheterization and percutaneous coronary intervention in the preoperative assessment and management of patients before orthotopic liver transplantation. Liver Transpl. 2014;20:664–672. 67. Keeffe BG, Valantine H, Keeffe EB. Detection and treatment of coronary artery disease in liver transplant candidates. Liver Transpl. 2001;7:755–761. 68. Jacobs E, Singh V, Damluji A, et al. Safety of transradial cardiac catheterization in patients with end-stage liver disease. Catheter Cardiovasc Interv. 2014;83:360–366. 69. Virani SS, Alonso A, Benjamin EJ, et al. Heart Disease and Stroke Statistics-2020 Update: A Report From the American Heart Association. Circulation. 2020;141:e139–e596. 70. Upshaw J, Kiernan MS. Preoperative cardiac risk assessment for noncardiac surgery in patients with heart failure. Curr Heart Fail Rep. 2013;10:147–156. 71. Hernandez AF, Newby LK, O’Connor CM. Preoperative evaluation for major noncardiac surgery: focusing on heart failure. Arch Intern Med. 2004;164:1729–1736. 72. Xu-Cai YO, Brotman DJ, Phillips CO, et al. Outcomes of patients with stable heart failure undergoing elective noncardiac surgery. Mayo Clin Proc. 2008;83:280–288. 73. Dernellis J, Panaretou M. Assessment of cardiac risk before non-cardiac surgery: brain natriuretic peptide in 1590 patients. Heart. 2006;92:1645–1650. 74. Andersson C, Gislason GH, Hlatky MA, et al. A risk score for predicting 30-day mortality in heart failure patients undergoing non-cardiac surgery. Eur J Heart Fail. 2014;16:1310–1316.

15-02-2022 10:00:52

Radionuclide Imaging in Heart Failure Gautam V. Ramani and Prem Soman

KEY POINTS ■■ ■■

■■

■■

■■

■■

CH22.indd 419

Heart failure (HF) is a highly prevalent condition with [a] high mortality rate. Accurate quantitation of left ventricular function is possible with gated single-photon emission computed tomography (SPECT), positron emission tomography (PET), and radionuclide ventriculography. Nuclear cardiology is helpful in distinguishing ischemic from nonischemic cardiomyopathy in many patients, particularly in those without symptoms or risk factors for coronary artery disease. Nuclear cardiology techniques can accurately quantify myocardial ischemia and viability. Risk stratification of HF patients may be enhanced with neurohumoral imaging, assessment of myocardial blood flow, and ventricular remodeling. PET has particular utility in patients with suspected cardiac sarcoidosis and for the surveillance of posttransplant patients for allograft vasculopathy.

CHAPTER

22

INTRODUCTION Despite many advances in recognition and treatment, chronic heart failure (HF) is an increasingly prevalent condition with a high mortality rate.1,2 The successful treatment of HF patients requires establishing an accurate diagnosis; identifying potentially reversible etiologies; determining the optimal therapy, which may be medical, percutaneous, or surgical; and assessing risk of patients at high risk for worsening HF or sudden cardiac death. Several of these aspects of HF care can be gainfully evaluated by radionuclide imaging. This chapter will provide a broad overview of established applications of radionuclide imaging in HF. Additional chapters will provide a more detailed overview of specific techniques and their applications. The clinician has several goals when evaluating an HF patient. Once a clinical diagnosis of the syndrome of HF is made, the initial step is usually the assessment of left ventricular function, often with the accurate determination of left ventricular ejection fraction (LVEF). Approximately one-half of patients will have HF with preserved ejection fraction (HFpEF, EF ≥ %50%), while the remainder will have either HF with reduced ejection fraction (HFrEF, EF < 40%) or HF with midrange EF (41−50%), a recently proposed category.3

15-02-2022 10:08:54

420

Section 3  Indications and Applications

Radionuclide imaging methods including singlephoton emission computed tomography (SPECT), radionuclide ventriculo­graphy (RVG), and positron emission tomography (PET) can all provide highly accurate and repeatable quantitative measurements of LVEF. Despite a similar prevalence to HFrEF, absent specific infiltrative etiologies, such as cardiac amyloidosis or sarcoidosis, no therapeutics have been shown to improve survival in patients with HFpEF. Treatment of HFpEF focuses upon the identification of treatable causes including coronary artery disease (CAD) and management of comorbidities including obesity, hypertension, and atrial fibrillation. Patients with HFpEF require an evaluation for CAD, and absent presentation with acute coronary syndrome or chest pain, this is often done with nuclear imaging.4 A specific case in point regarding radionuclide imaging in HFpEF patients is cardiac amyloidosis. The emergence of Tc-99 pyrophosphate imaging as a noninvasive diagnostic standard for ATTR cardiomyopathy (Chapter 25) has been transformative for the field by unmasking a hitherto

unrecognized prevalence of the condition in patients with HFpEF.5 Together with the introduction of first specific therapies for cardiac amyloidosis, Tc-99 pyrophosphate has invigorated clinical and academic interest in the field. Similarly, PET imaging with F-18 fluorodeoxyglucose (FDG) is the only available approach to distinguish between inactive and inflammatory infiltrates in cardiac sarcoidosis (Chapter 26); a distinction that guides the decision to institute high-dose corticosteroids.6 For patients with HFrEF, a critical early step is the determination of etiology. Etiology evaluation can include identifying specific and potentially remediable causes such as valvular disease, CAD, toxin-induced and metabolic cardiomyopathies, infiltrative diseases, and arrhythmias. Radionuclide imaging has critical roles in the determination of HF etiology and the identification of patients for coronary revascularization (Chapter 23). When extensive CAD is found, decision making is required regarding the risks/benefits of coronary revascularization. Myocardial viability testing (Chapter 23) may be of value in decision making. Figure 22-1

Clinical Diagnosis of Heart Failure

LV Function Assessment

HFrEF Is significant coronary artery disease present?

HFpEF No

Yes

Identify and treat special Specific cardiomyopathies e.g., sarcoidosis, amyloidosis

NICM

ICM

Are there targets for revascularization? Yes Is patient high risk for surgical intervention? Yes Viability testing for further risk assessment

No

No

GDMT Device Therapy VAD/Transplant

Consider surgical revascularization

FIGURE 22-1  Scheme for the evaluation of patients with heart failure. Arrows indicate steps where radionuclide imaging has application. HF, heart failure; LV, left ventricle; HFrEF, heart failure with reduced ejection fraction; HFpEF, heart failure with preserved ejection fraction; CAD, coronary artery disease; ICM, ischemic cardiomyopathy; NICM, nonischemic cardiomyopathy; GDMT, guidelinedirected medical therapy; ICD, implantable cardioverter defibrillator; CRT, cardiac resynchronization therapy; VAD, ventricular assist device.

CH22.indd 420

15-02-2022 10:08:54

Chapter 22  Radionuclide Imaging in Heart Failure

illustrates an algorithm for evaluation of patients with newly diagnosed cardiomyopathy. For patients with nonischemic cardiomyopathy (NICM) and persistent LV systolic dysfunction after coronary revascularization, a combination of guidelinedirected medical therapy and device therapy in selected patients (implantable cardioverter defibrillator [ICD], and cardiac resynchronization therapy [CRT]) form the cornerstone of current recommendations. Evolving techniques such as myocardial sympathetic neuronal function imaging (Chapter 24) and dyssynchrony imaging (Chapter 12) may have relevance to further refine the selection of patients for ICD and CRT. Furthermore, PET/CT imaging with F-18 FDG has established utility in the challenging area of diagnosing device infections (Chapter 26). A minority of patients with progressive, refractory HF will receive advanced HF therapies, including left ventricular assist devices and heart transplantation. In posttransplant patients, radionuclide imaging has important prognostic value, which may influence therapeutic options in patients with suspected allograft vasculopathy.

▶▶Determination of Heart Failure Etiology The etiology of HF varies considerably depending on the population studied.7 CAD, including myocardial infarction, is the strongest risk factor for development of HF.8 Based on clinical trial data on patients with established HF, CAD is the attributed etiology for 60 to 70% of HF in the United States.9 However, the mere presence of CAD in the setting of a cardiomyopathy does not imply an ischemic etiology to the LV dysfunction. What is traditionally referred to as significant CAD in the literature, that is, ≥50% luminal stenosis, may be encountered in 15% to 30% of patients with a dilated (nonischemic) cardiomyopathy, and thus may not be sufficiently sensitive for accurate risk stratification of the HF population. In other words, a distinction must be made of CAD that is etiologically related to the HF from that which is simply coexistent with nonischemic LV systolic dysfunction. Felker et al. addressed this question and tested a more stringent definition of ischemic cardiomyopathy for the characterization of HF patients.10 They defined ischemic cardiomyo­ pathy as LV dysfunction with one or more of the

CH22.indd 421

421

following angiographic criteria: significant left main or proximal left anterior descending coronary artery stenosis, at least two-vessel disease with ≥70% stenosis, or single-vessel disease with prior myocardial infarction, or prior coronary revascularization. For example, a patient with LV dysfunction and a 70% stenosis of one major epicardial vessel without antecedent myocardial infarction or revascularization would be adjudicated to the non-ischemic cardiomyopathy (NICM) group (with coexisting, but not causally related CAD). Using these more restrictive criteria, patients with LV dysfunction and singlevessel CAD have a prognosis comparable to those with NICM.5 Patients with true CAD-related HF have a worse prognosis than those with NICM, but the former may improve cardiac function dramatically with revascularization, highlighting the critical importance of an accurate diagnosis. The literature regarding the use of SPECT for the diagnosis of underlying CAD in LV dysfunction has primarily focused upon patients with chronic HF, with scant data addressing the diagnosis of CAD in new-onset HF. In the setting of newly diagnosed HFrEF, the identification of underlying CAD and potential “at risk” dysfunctional myocardium that might recover with coronary revascularization is critical. Although current practice guidelines specifically mandate coronary angiography as an initial evaluation strategy only in HF patients with angina, chest pain is often absent in patients with ischemic cardiomyopathy, even those with significant amounts of viable myocardium.4,11 The Investigation of Myocardial Gated SPECT Imaging (IMAGING) in Heart Failure trial specifically addressed the utility of gated SPECT as an initial diagnostic modality in the de novo acute HF setting.12 A total of 201 patients hospitalized with new-onset HF were prospectively enrolled and underwent exercise or pharmacologic SPECT during the index hospitalization. At the physician’s discretion, approximately one-third of the patients underwent coronary angiography. Using a summed stress score (SSS) more than 3 to define an abnormal study, SPECT had a sensitivity of 96% and a negative predictive value of 96% for the diagnosis of ischemic cardiomyopathy using the criteria proposed by Felker but was less accurate in detecting limited-extent CAD

15-02-2022 10:08:54

422

Section 3  Indications and Applications

Table 22-1 Performance Characteristics of Gated SPECT Tc-99m Sestamibi for CAD Diagnosis in Patients with NewOnset Heart Failure from the IMAGING in Heart Failure Study

CAD Definition

Any CAD: ≥70% Stenosis in Any Coronary Artery

Extensive CAD: Stenosis ≥70% in the LM or Proximal LAD, ≥70% in ≥2 Major Epicardial Coronary Arteries or Any Stenosis ≥70% with a Prior MI or Coronary Revascularization

CAD prevalence by angiography

51%

36%

Sensitivity (95% CI)

82% (66–92)

96% (81–99)

Specificity (95% CI)

57% (40–72)

56% (41–71)

PPV

67%

55%

NPV

75%

96%

CAD, coronary artery disease; LM, left main coronary artery; LAD, left anterior descending coronary artery; NPV, negative predictive value; PPV, positive predictive value. Criteria for positive SPECT was summed stress score >3. Reproduced with permission from Soman P, Lahiri A, Mieres JH, et al. Etiology and pathophysiology of new-onset heart failure: evaluation by myocardial perfusion imaging. J Nucl Cardiol. 2009;16(1):82–91.

(Table 22-1). Thus, this study provides proof of concept of the utility of myocardial SPECT for the initial characterization of patients presenting with severe new-onset HF. Such patients who have normal stress myocardial SPECT are very unlikely to have underlying CAD that is responsible for their HF (Fig. 22-2). Several previous studies have established the utility of myocardial perfusion imaging (MPI) for the diagnosis of CAD in chronic HF.13 Although many of these studies predated contemporary MPI, they uniformly demonstrated a very high negative predictive value for excluding CAD. Using gated SPECT imaging, Danias et al. demonstrated that the summed stress score (SSS) for an ischemic cardiomyopathy was far greater than that noted with an NICM (32.9, 95% confidence interval [CI] 28.6−37.1 vs. 6.9, 95% CI 5.4−8.4). In fact all nonischemic CM patients had an SSS of ≤14 as opposed to those with an ischemic CM, where the SSS was ≥21. Additionally, regional wall motion variability was significantly lower in NICM patients. The combined use of wall motion and perfusion data appears to provide excellent delineation of the type of CM, although this was a small study.14 Thus, in the setting of both new-onset and established HF and global systolic dysfunction, a normal stress myocardial perfusion scan virtually excludes a diagnosis of ischemic LV dysfunction.

CH22.indd 422

A major concern is that of balanced ischemia due to extensive CAD, which might be missed or underestimated due to the fact that the MPI assessment of regional myocardial perfusion is relative. While it is unlikely that a patient with severe and extensive CAD will have no angina and a normal and rest/stress ECG and MPI, given the critical importance of excluding CAD in this population and the lack of substantive clinical trial data with MPI, patients with new-onset HF, especially those who have risk factors for CAD or have hemodynamic instability, often undergo diagnostic coronary angiography for this purpose. MPI with PET is more sensitive than SPECT for the detection of CAD but has not been specifically tested in the context of new-onset HF. The ability to quantify absolute myocardial blood flow with PET may offer additional advantages by facilitating the identification of microvascular disease and for prognostication.15,16 From a practical perspective, most new-onset HF patients who have angina or CAD risk factors should undergo diagnostic coronary angiography. HF patients with a low probability of CAD, with clinical circumstances suggestive of nonischemic LV dysfunction, can have a rest/stress MPI as the initial diagnostic test with reliable accuracy. In patients with known CAD being evaluated for newonset or established HF, a rest/stress MPI may provide invaluable information on ischemia, viability,

15-02-2022 10:08:54

Chapter 22  Radionuclide Imaging in Heart Failure

423

FIGURE 22-2  Categorization of heart failure etiology using technetium-99m sestamibi MPI. (A) Left ventricular (LV) dilation (abnormal LV systolic function by gated SPECT not shown) with large, fixed perfusion defects in the septum, anterior wall, apex, and inferior wall suggestive of CAD-related (“ischemic”) cardiomyopathy. (B) Normal stress–rest perfusion and LV size (normal LV EF on gated SPECT not shown) indicative of heart failure likely related to diastolic mechanisms. (C) LV dilation (with abnormal LV systolic function on gated SPECT, not shown) and normal perfusion suggestive of non-CAD related (“nonischemic”). (Reproduced with permission from Soman P, Lahiri A, Mieres JH, et al. Etiology and pathophysiology of new-onset heart failure: evaluation by myocardial perfusion imaging. J Nucl Cardiol. 2009;16(1):82–91.)

and quantitative LV function, which can be used to drive important management decisions such as the choice between targeted percutaneous or surgical revascularization. It is important to recognize that mild fixed perfusion defects are common in NICM and may reflect true physiological phenomena, such as myocardial fibrosis or abnormal coronary vasodilator reserve, and have prognostic significance.17–19 Inferior defects may also be caused by diaphragmatic attenuation and further attenuation from LV dilatation and may be accentuated in patients with cardiomyopathies. Attenuation correction is helpful in identifying soft tissue artifacts in SPECT imaging, but its effect on diagnostic accuracy for CAD has not been specifically tested in the HF population.

CH22.indd 423

▶▶Selecting Patients for Coronary Revascularization In selected HF patients with LV systolic dysfunction, coronary revascularization may offer a unique opportunity for a “cure.” The selection of patients for coronary revascularization requires a consideration of its potential benefits against perioperative mortality and morbidity. Patients with severe LV systolic dysfunction are at the highest risk but may also derive the most benefit. While the concept of preserved myocardial viability (Chapter 23)and its impact on prognosis appears physiologically sound, the evidence from clinical trials has not been conclusive. The Surgical Treatment for Ischemic Heart Failure (STITCH) trial randomized 1212 patients with LVEF

15-02-2022 10:08:57

424

Section 3  Indications and Applications

less than 35% to medical therapy or coronary artery bypass grafting (CABG) and medical therapy.20 A subset of enrolled patients underwent viability testing at the discretion of the treating physician.21 While CABG afforded patients a mortality benefit in the long term (10-year follow up) but not early on, there was no interaction between viability testing and outcome (reported only with the short-term results).22 However, it remains unclear whether these results reflect a lack of power (type II error), lack of test sensitivity (viability testing was performed with SPECT or dobutamine echocardiography, neither F-18 FDG PET nor cardiac magnetic resonance was used), or a true clinical result. The multicenter Canadian Positron Emission Tomography and Recovery Following Revascularization, Phase -2 (PPAR 2) randomized 430 patients with CAD and severe LV systolic dysfunction to F-18 FDG PET or standard care-based decision for revascularization. At one year, the primary outcome of cardiac death, MI, and cardiac hospitalization was not different between the groups by the prespecified intention to treat analysis. However, a posthoc per protocol analysis showed a reduction in the adverse outcomes in PET group, an observation that was also noted in the STITCH trial.23 The STITCH trial is the largest randomized study examining surgical versus medical therapy in patients with ischemic cardiomyopathy. Further analysis revealed better outcomes after CABG in patients with preserved functional capacity, multivessel CAD, lower EF, and higher end-systolic volume.22 One practical approach, based largely upon the STITCH trial, might be to forgo viability testing in younger patients with extensive CAD, angina, good coronary target vessels, and average surgical risk. For older patients and those at high surgical risk, particularly those with severely reduced systolic function, the demonstration specifically of hibernating or stunned myocardium may further guide risk assessment.24 While improvement in LVEF is often a goal of surgical revascularization, Samady et al. demonstrated that lack of improvement in LVEF following CABG was not associated with poorer outcomes compared to patients with improved LVEF. The authors hypothesized that improved tissue perfusion with surgical revascularization had beneficial effects on reduction of infarction size independent of LVEF

CH22.indd 424

improvement, such as with ventricular remodeling or electrical instability.25

▶▶Radionuclide Approaches to Risk Stratification in Heart Failure The role of myocardial sympathetic neuronal imaging for risk stratification in HF patients is discussed in Chapter 24. The state of the sympathetic nerve terminal function in the heart can be assessed by the kinetics of the sympathetic neurotransmitter norepinephrine at the junction of the cardiac presynaptic sympathetic nerve and the cardiac myocyte, that is, the synaptic cleft. A functional impairment of the norepinephrine uptake 1 transporter (NET-1) in chronic HF leads to a progressive decline in norepinephrine uptake. I-123 metaiodobenzylguanidine (mIBG) is a SPECT agent that was approved by the Food and Drug Administration for imaging myocardial sympathetic nerve terminal function in 2013.26 Its uptake parallel’s norepinephrine activity.27 The ADMIRE-HF (AdreView Myocardial Imaging for Risk Evaluation in Heart Failure) trial established the prognostic utility of I-123 mIBG imaging in patients with HF.26 C-11−labeled PET agents have also been used for sympathetic neuronal imaging, but the requirement for a cyclotron in close proximity makes clinical use logistically difficult.28 A newer agent F-18 flubrobenguagne (LMI1195) is currently undergoing clinical evaluation for the evaluation of myocardial sympathetic innervation by PET.29 Other radionuclide approaches have also been proven valuable for risk stratification in HF. A preserved myocardial flow reserve on Rb-82 PET MPI is indicative of a more benign prognosis in patients with ischemic and NICM compared to patients with a low myocardial flow reserve, as demonstrated elegantly in a study of 510 patients followed up for 8 months.30 The use of PET-derived myocardial flow reserve to identify low-risk patients is a very promising approach to risk stratification and is addressed again in the section on cardiac transplantation, related to allograft vasculopathy (Chapter 23). The cellular and interstitial changes that underlie the phenomenon of LV remodeling are accompanied by a transformation of the normally ellipsoid LV into a more spherical shape. LV shape indices, such as the sphericity index derived from echocardiography,

15-02-2022 10:08:57

Chapter 22  Radionuclide Imaging in Heart Failure

425

FIGURE 22-3  Illustration of two cases with comparable left ventricular ejection fraction (LVEF) but different left ventricular shape indices (LVSI). The patient with a (A) normal ellipsoid-shaped LV and preserved LVSI had no symptoms, whereas the patient with a (B) remodeled, spherical LV and abnormal LVSI (B) had severe symptoms. (Reproduced with permission from Abidov A, Slomka PJ, Nishina H, et al. Left ventricular shape index assessed by gated stress myocardial perfusion SPECT: initial description of a new variable. J Nucl Cardiol. 2006;13(5):652–659.)

have established utility in predicting prognosis and response to therapy in HF patients.31–33 Douglas et al. showed that survival in 56 dilated cardiomyopathy patients was worse in those with a more spherical left ventricle (based on a ratio of short-to-long axis end-diastolic dimensions) resulting in maldistribution of afterload.31 Others have shown a transformation of a spherical LV in dilated cardiomyopathy to a more ellipsoid shape in response to therapy.32,33 Analogous measurements derived from gated SPECT have shown similar prognostic value (Fig. 22-3).34

▶▶Role of Radionuclide Imaging in Device and Advanced Heart Failure Therapies The use of sympathetic neuronal imaging agents for prognostication in HF has been discussed earlier in this chapter. While the ADMIRE study established

CH22.indd 425

the prognostic utility of I-123 mIBG imaging specifically for arrhythmic risk stratification in HF and the agent was subsequently approved by the FDA for this purpose, it is not currently used to refine patient selection for ICD therapy. A major deterrent is that existing recommendations derived from multiple large clinical trials are based on an EF cutoff of ≤35%, and robust imaging data to advocate for modification of these criteria are lacking. An analogous situation is that of LV dyssynchrony assessment.35 While several single- and multicenter trials have established the feasibility of gated SPECT (and echocardiography) to measure LV intraventricular dyssynchrony (see Chapter 12), the approach has not been adopted clinically. Thus, while the existing criteria for patient selection for ICD and CRT result in a significant proportion of patients not benefitting from these expensive therapies, larger clinical trials are needed to refine these parameters based on imaging criteria.

15-02-2022 10:08:58

426

Section 3  Indications and Applications

Another area of great clinical challenge is the surveillance of patients after HT for the detection of coronary allograft vasculopathy (CAV). Currently, posttransplant patients undergo annual surveillance coronary angiography, and those who cannot undergo angiography typically obtain dobutamine stress echocardiography. CAV is characterized by diffuse arterial hyperplasia, often involving branch vessels, as opposed to focal stenoses and therefore, may be missed by conventional coronary “lumino­ graphy,” particularly in the early stages. Intriguing features include its development in young patients without traditional risk factors for atherosclerosis and unpredictable timeline of onset post heart transplant (HT).36 Once developed, there is no therapy proven to definitively reverse CAV, and the clinical course is usually one of the progressive symptoms and allograft dysfunction, especially when identified in the first few years post HT.37 The role of MPI for posttransplant follow up has been evaluated. Both SPECT perfusion imaging38 and PET perfusion with myocardial flow reserve estimation39–42 have been found to have good prognostic utility for this purpose. Single-center studies indicate that normal perfusion on SPECT or myocardial flow reserve on PET predicts an excellent prognosis in the intermediate term (2–5 years). More recently, abnormal coronary flow reserve identified by 82Rb PET scanning, performed early post HT, was found to be predictive of late-term mortality.40 The ability to risk stratify patients using noninvasive approaches would be an important clinical advantage for posttransplant patients who are already burdened with a substantial load of testing and have a high incidence of renal insufficiency. Also, these data attest to the fact that, analogous to atherosclerotic CAD, functional testing provides important prognostic information that may not be available from anatomy-based diagnostic approaches.43

▶▶Molecular Imaging in Heart Failure Molecular mechanisms of HF are operative at the preclinical “at risk” stage (Stage A of the ACC/AHA classification), and targeted imaging approaches have greatly enhanced our understanding of HF pathophysiology. It is hoped that the clinical translation of molecular imaging approaches will identify

CH22.indd 426

specific processes that may predominate in individual patients or patient groups, and explain the heterogeneity in response to therapy, and facilitate personalizing medical care. Such approaches include the imaging cellular mechanisms such as apoptosis (annexin-V)44,45 and the renin–angiotensin system (F-18 captopril, F-18 lisinopril),46 myocardial sympathetic innervation, and myocardial metabolism (C-11 palmitate, I-123 BMIPP, F-18 FDG).47 Molecular imaging techniques have also been applied with success to the monitoring of regenerative cell therapy.48

SUMMARY In summary, several established radionuclide imaging approaches can be used with advantage in the evaluation and management of the HF patient. Radionuclide imaging applications in this prevalent and pervasive condition continue to evolve and expand, offering unique insights into pathophysiology. The future clinical translation of molecular imaging approaches may offer opportunities to personalize therapy.

REFERENCES 1. Ponikowski P, Voors AA, Anker SD, et al. 2016 ESC Guidelines for the diagnosis and treatment of acute and chronic heart failure: The Task Force for the diagnosis and treatment of acute and chronic heart failure of the European Society of Cardiology (ESC). Developed with the special contribution. Eur J Heart Fail. 2016;18:891–975. 2. Virani SS, Alonso A, Benjamin EJ, et al. Heart disease and stroke statistics—2020 update: A report from the American Heart Association. Circulation. 2020;141(9):e139–e596. 3. Yancy CW, Jessup M, Bozkurt B, et al. 2017 ACC/AHA/HFSA Focused Update of the 2013 ACCF/AHA Guideline for the Management of Heart Failure: A Report of the American College of Cardiology/American Heart Association Task Force on Clinical Practice Guidelines and the Heart Failure Society of Amer. J Am Coll Cardiol. 2017;70:776–803. 4. Yancy CW, Jessup M, Bozkurt B, et al. 2013 ACCF/AHA Guideline for the Management of Heart Failure: A Report of the American College of Cardiology Foundation/American Heart Association Task Force on Practice Guidelines. Circulation. 2013;128:e240–e327. 5. Gillmore JD, Maurer MS, Falk RH, et al. Nonbiopsy diagnosis of cardiac transthyretin amyloidosis. Circulation. 2016;133:2404–2412. 6. Osborne MT, Hulten EA, Singh A, et al. Reduction in (1)(8)Ffluorodeoxyglucose uptake on serial cardiac positron

15-02-2022 10:08:58

Chapter 22  Radionuclide Imaging in Heart Failure emission tomography is associated with improved left ventricular ejection fraction in patients with cardiac sarcoidosis. J Nucl Cardiol. 2014;21:166–174. 7. Felker GM, Thompson RE, Hare JM, et al. Underlying causes and long-term survival in patients with initially unexplained cardiomyopathy. N Engl J Med. 2000;342:1077–1084. 8. He J, Ogden LG, Bazzano LA, Vupputuri S, Loria C, Whelton PK. Risk factors for congestive heart failure in US men and women: NHANES I epidemiologic follow-up study. Arch Intern Med. 2001;161:996–1002. 9. Gheorghiade M, Sopko G, De L, et al. Navigating the crossroads of coronary artery disease and heart failure. Circulation. 2006;114:1202–1213. 10. Felker GM, Shaw LK, O’Connor CM. A standardized definition of ischemic cardiomyopathy for use in clinical research. J Am Coll Cardiol. 2002;39:210–218. 11. Cleland JG, Pennell DJ, Ray SG, et al. Myocardial viability as a determinant of the ejection fraction response to carvedilol in patients with heart failure (CHRISTMAS trial): randomised controlled trial. Lancet. 2003;362:14–21. 12. Soman P, Lahiri A, Mieres JH, et al. Etiology and pathophysiology of new-onset heart failure: Evaluation by myocardial perfusion imaging. J Nucl Cardiol. 2009;16:82–91. 13. Udelson JE, Shafer CD, Carrio I. Radionuclide imaging in heart failure: assessing etiology and outcomes and implications for management. J Nucl Cardiol. 2002;9:40S–52S. 14. Danias PG, Papaioannou GI, Ahlberg AW, et al. Usefulness of electrocardiographic-gated stress technetium-99m sestamibi single-photon emission computed tomography to differentiate ischemic from nonischemic cardiomyopathy. Am J Cardiol. 2004;94:14–19. 15. Majmudar MD, Murthy VL, Shah RV, et al. Quantification of coronary flow reserve in patients with ischaemic and nonischaemic cardiomyopathy and its association with clinical outcomes. Eur Heart J Cardiovasc Imaging. 2015;16:900–909. 16. Bravo PE, di Carli MF, Dorbala S. Role of PET to evaluate coronary microvascular dysfunction in non-ischemic cardiomyopathies. Heart Fail Rev. 2017;22:455–464. 17. Chikamori T, Doi YL, Yonezawa Y, Yamada M, Seo H, Ozawa T. Noninvasive identification of significant narrowing of the left main coronary artery by dipyridamole thallium scintigraphy. Am J Cardiol. 1991;68:472–477. 18. Iles L, Pfluger H, Lefkovits L, et al. Myocardial fibrosis predicts appropriate device therapy in patients with implantable cardioverter-defibrillators for primary prevention of sudden cardiac death. J Am Coll Cardiol. 2011;57:821–828. 19. van den Heuvel AF, van Veldhuisen DJ, van der Wall EE, et al. Regional myocardial blood flow reserve impairment and metabolic changes suggesting myocardial ischemia in patients with idiopathic dilated cardiomyopathy. J Am Coll Cardiol. 2000;35:19–28. 20. Velazquez EJ, Lee KL, Deja MA, et al. Coronary-Artery Bypass Surgery in Patients with Left Ventricular Dysfunction. N Engl J Med. 2011;364:1607–1616. 21. Bonow RO, Maurer G, Lee KL, et al. Myocardial Viability and Survival in Ischemic Left Ventricular Dysfunction. N Engl J Med. 2011;364:1617–1625. 22. Panza JA, Velazquez EJ, She L, et al. Extent of coronary and myocardial disease and benefit from surgical revascularization

CH22.indd 427

427

in ischemic LV dysfunction [Corrected]. J Am Coll Cardiol. 2014;64:553–561. 23. Beanlands RSB, Nichol G, Huszti E, et al. F-18-Fluorode­ oxyglucose Positron Emission Tomography Imaging-Assisted Management of Patients With Severe Left Ventricular Dysfunction and Suspected Coronary Disease. A Randomized, Controlled Trial (PARR-2). J Am Coll Cardiol. 2007;50:2002– 2012. 24. Kandolin RM, Wiefels CC, Mesquita CT, et al. The Current Role of Viability Imaging to Guide Revascularization and Therapy Decisions in Patients With Heart Failure and Reduced Left Ventricular Function. Can J Cardiol. 2019;35:1015–1029. 25. Samady H, Elefteriades JA, Abbott BG, Mattera JA, McPherson CA, Wackers FJ. Failure to improve left ventricular function after coronary revascularization for ischemic cardiomyopathy is not associated with worse outcome. Circulation. 1999;100:1298–1304. 26. Jacobson AF, Senior R, Cerqueira MD, et al. Myocardial iodine-123 meta-iodobenzylguanidine imaging and cardiac events in heart failure. Results of the prospective ADMIREHF (AdreView Myocardial Imaging for Risk Evaluation in Heart Failure) study. J Am Coll Cardiol. 2010;55:2212–2221. 27. Soman P, Travin MI, Gerson M, Cullom SJ, Thompson R. I-123 MIBG Cardiac Imaging. J Nucl Cardiol. 2015;22:677–685. 28. Sasano T, Abraham MR, Chang KC, et al. Abnormal sympathetic innervation of viable myocardium and the substrate of ventricular tachycardia after myocardial infarction. J Am Coll Cardiol. 2008;51:2266–2275. 29. Zelt JGE, Britt D, Mair BA, Rotstein BH, et al. Regional Distribution of Fluorine-18-Flubrobenguane and Carbon-11Hydroxyephedrine for Cardiac PET Imaging of Sympathetic Innervation. JACC: Cardiovascular Imaging. 2021;14(7): 1425–1436. 30. Majmudar MD, Murthy VL, Shah RV, et al. Quantification of coronary flow reserve in patients with ischaemic and non-ischaemic cardiomyopathy and its association with clinical outcomes. Eur Heart J Cardiovasc Imaging. 2015;16(8):900–909. 31. Douglas PS, Morrow R, Ioli A, Reichek N. Left ventricular shape, afterload and survival in idiopathic dilated cardiomyopathy. J Am Coll Cardiol. 1989;13:311–315. 32. Hall SA, Cigarroa CG, Marcoux L, Risser RC, Grayburn PA, Eichhorn EJ. Time course of improvement in left ventricular function, mass and geometry in patients with congestive heart failure treated with beta-adrenergic blockade. J Am Coll Cardiol. 1995;25:1154–1161. 33. Lowes BD, Gill EA, Abraham WT, et al. Effects of carvedilol on left ventricular mass, chamber geometry, and mitral regurgitation in chronic heart failure. Am J Cardiol. 1999;83:1201–1205. 34. Abidov A, Slomka PJ, Nishina H, et al. Left ventricular shape index assessed by gated stress myocardial perfusion SPECT: initial description of a new variable. J Nucl Cardiol. 2006;13:652–659. 35. Chen J, Garcia EV, Bax JJ, Iskandrian AE, Borges-Neto S, Soman P. SPECT myocardial perfusion imaging for the assessment of left ventricular mechanical dyssynchrony. J Nucl Cardiol. 2011;18:685–694. 36. Libby P. The vascular biology of atherosclerosis. In: Libby P, Bonow RO, Mann DL, Zipes DP, eds. Braunwald’s Heart Disease. Philadelphia: Saunders; 2008:985–1002.

15-02-2022 10:08:58

428

Section 3  Indications and Applications

37. Costanzo MR, Naftel DC, Pritzker MR, et al. Heart transplant coronary artery disease detected by coronary angiography: A multiinstitutional study of preoperative donor and recipient risk factors. J Heart Lung Transplant. 1998;17:744–753. 38. Manrique A, Bernard M, Hitzel A, et al. Diagnostic and prognostic value of myocardial perfusion gated SPECT in orthotopic heart transplant recipients. J Nucl Cardiol. 2010;17:197–206. 39. Mc Ardle BA, Davies RA, Chen L, et al. Prognostic Value of Rubidium-82 Positron Emission Tomography in Patients After Heart Transplant. Circ Cardiovasc Imaging. 2014;7:930–937. 40. Feher A, Srivastava A, Quail MA, et al. Serial assessment of coronary flow reserve by rubidium-82 positron emission tomography predicts mortality in heart transplant recipients. JACC: Cardiovasc Imaging. 2020;13:109–120. 41. Bravo PE, Bergmark BA, Vita T, et al. Diagnostic and prognostic value of myocardial blood flow quantification as non-invasive indicator of cardiac allograft vasculopathy. Eur Heart J. 2018;39:316–323. 42. Miller RJH, Manabe O, Tamarappoo B, et al. Comparative prognostic and diagnostic value of myocardial blood flow and

CH22.indd 428

myocardial flow reserve after cardiac transplantation. J Nucl Med. 2020;61:249–255. 43. Soman P, McNamara D. Editorial: Surveillance for post transplant coronary artery vasculopathy: Shifting gears from diagnosis to prognosis. J Nucl Cardiol. 2010;17:172–174. 44. Thimister PW, Hofstra L, Liem IH, et al. In vivo detection of cell death in the area at risk in acute myocardial infarction. J Nucl Med. 2003;44:391–396. 45. Korngold EC, Jaffer FA, Weissleder R, Sosnovik DE. Noninvasive imaging of apoptosis in cardiovascular disease. Heart Fail Rev. 2008;13:163–173. 46. Dilsizian V, Eckelman WC, Loredo ML, Jagoda EM, Shirani J. Evidence for tissue angiotensin-converting enzyme in explanted hearts of ischemic cardiomyopathy using targeted radiotracer technique. J Nucl Med. 2007;48:182–187. 47. Dilsizian V. Metabolic adaptation to myocardial ischemia: the role of fatty acid imaging. J Nucl Cardiol. 2007;14: S97–S99. 48. Bengel F. Nuclear imaging in cardiac cell therapy. Heart Fail Rev. 2006;11:325–332.

15-02-2022 10:08:58

SECTION

4

BEYOND PERFUSION IMAGING

CH23.indd 429

15-02-2022 18:49:30

This page intentionally left blank

READ_THINK_ITALIAN.indd 2

11/19/21 1:12 PM

Nuclear Cardiology Procedures in the Evaluation of Myocardial Viability Christiane Wiefels, Fernanda Erthal, Benjamin Chow, Gary V. Heller and Rob S.B. Beanlands

KEY POINTS ■■

■■

■■

■■

CH23.indd 431

Following an acute or chronic ischemic injury, dysfunctional myocardium can be classified as viable (stunned or hibernating myocardium) or nonviable (necrotic and scar). Hibernating myocardium is characterized by reduced perfusion at rest after repeated episodes of ischemia and/or stunning, and despite being dysfunctional, it is viable. It has the capacity for functional recovery if adequate and timely revascularization can be achieved. During fasting conditions, free-fatty acids are the preferred energy substrate of the myocardium. However, ischemia can cause a shift toward glucose utilization making positron emission tomography (PET) with 18 F-fluorodeoxyglucose (FDG) a highly sensitive means to evaluate the presence of hibernating myocardium. While all techniques (18FDG PET, dobutamine echocardiography, 201Tl-single-photon emission computed tomography [SPECT], 99m Tc-SPECT, and cardiac magnetic resonance imaging [CMR] can be used for viability assessment and guidance in decision making, some may be better in certain circumstances. Among these 18FDG PET and CMR are considered more sensitive, while

■■

■■

CHAPTER

23

dobutamine echocardiogram and CMR are considered more specific. The use of viability studies is best targeted to patients at higher risk for cardiac death or other cardiac events whose benefit from cardiac revascularization may be considered less certain due to factors such as comorbidities and poor vascular targets. The revascularization benefit must outweigh the risk of a potential surgery or intervention and enable improved outcome and quality of life. Several prospective outcomes trials have demonstrated clinical benefit when revascularization is guided by the presence or absence of viable tissue by 18FDG PET imaging, especially with viable myocardium over 20%.

INTRODUCTION One in every eight deaths is attributable to heart failure (HF). The prevalence of HF continues to increase. In the United States, HF has increased from 5.7 million in 2012 to 6.5 million in 2014, and it is estimated that this will be more than 8 million people by 2030. This will lead to 127% increase in healthcare costs amounting to approximately $69.7 billion.1,2 Although survival of HF patients has improved, it remains a disease with poor outcome

15-02-2022 18:49:30

432

Section 4  Beyond Perfusion Imaging

with a 50% mortality at 5 years.3 Given the growing prevalence of ischemic heart failure (IHF) and its high mortality rate, the optimal management strategies for IHF have been the focus of research for the past several decades. Evidence, including randomized controlled trials, has accumulated and supports the notion that patients with IHF may benefit from viability imaging to guide therapy and revascularization decisions.4–12 Many studies have suggested that viability assessment using noninvasive testing is a predictor of LV function and outcome benefit with revascularization.7,8,10–13 However, the viability substudies of the recent Surgical Treatment for Ischemic Heart Failure (STICH) and STICH Extension Study (STICHES) trials called some of these observations into question (discussed in details below).14,15 The STICH(ES) trial demonstrated long-term outcome benefit for revascularization in patients with left ventricle (LV) dysfunction, angina, and suitable anatomy. The viability substudies (that used singlephoton emission computed tomography [SPECT] and dobutamine echocardiography to define viability), remind us that viability imaging is not always needed in patients with LV dysfunction who are candidates for revascularization. Rather, it is most useful when revascularization decisions are most difficult, in patients with primarily HF symptoms, multiple comorbidities, and suboptimal vascular targets who may not have been candidates for the STICH trial. This chapter will review viability concepts, different nuclear imaging methods used to assess viability, supportive data, and recommendations. The clinical evidence for viability testing, as well as ongoing trials and future directions, will be discussed.

VIABILITY CONCEPTS After myocardial injury, dysfunctional myocardium can have different histologic changes: viable (stunned and/or hibernating myocardium) or nonviable (necrotic and scar). These categories are critical concepts, and misunderstandings can sometimes lead to incorrect clinical decision making. Nuclear medicine techniques are an important tool and help distinguishing the different molecular and cellular changes associated with these processes (Table 23-1).16

CH23.indd 432

▶▶Viable versus Nonviable Myocardium Dysfunctional myocardium can be dichotomized into viable or nonviable myocardium. In the latter, the tissue is replaced by irreversible fibrosis and therefore cannot be reversed and thus cannot be improved with revascularization. Conversely, viable myocardium can have variable function and preserved metabolism and can be subtended with either preserved or impaired blood flow. In cases of dysfunctional but viable myocardium, from repeated episodes or persistent ischemia, restoring coronary flow may result in metabolic and LV function recovery.17,18 The main goal of viability imaging is to define viable myocardium to guide decision making for therapies with the goal of improving LV function and clinical outcomes.4–13

▶▶Myocardial Stunning Stunned myocardium refers to a postischemic state where there is mismatch between function and flow. Although resting coronary flow has returned to normal, myocardial function remains impaired. Depending on the duration and severity of the ischemic insult,19–22 the presence of stunning can last minutes, days, or weeks. In postischemic states, the distinction between stunning and hibernation is often very difficult, particularly following acute myocardial infarction. This may be due in part to the fact that there is frequently a mixture of ischemia, stunning, and hibernation. It is important to note that all of these states represent viable myocardium. In the stunned myocardium condition, metabolic alterations prevail over structural changes, and may recover spontaneously without intervention, although there are small studies indicating that no glucose uptake early after infarction indicates a nonviable condition that does not change over time. Electron microscopy of stunned myocardium shows normal or just mildly degenerated cells.18 Metabolic changes can be complex and progress over time. Observed metabolic derangement includes a decrease in calcium sensitivity of myofilaments.22 A glucose transporter (GLUT) 4 translocation to the sarcolemma and an increase in glucose uptake have been observed;23 so too has a decrease in glucose uptake on metabolic imaging post-STEMI revascularization.24 If the injury persists or in cases of

15-02-2022 18:49:30

Chapter 23  Nuclear Cardiology Procedures in the Evaluation of Myocardial Viability

433

Table 23-1 Characteristics of Viable and Nonviable Dysfunctional Myocardium and Their Clinical Relevance Glucose Metabolism (FDG)

Function

Structural Changes

Potential to Recover/ Clinical Relevance

Variable (normal, increased, or reduced)

Reduced

No

Likely to recover if ischemic injury does not persist or becomes repetitive; revascularization can prevent recurrent stunning

Reduced (downregulated)

Preserved or increased (perfusionmetabolism mismatch)

Reduced

Yes, some

May have partial or full recovery if adequate revascularization can be achieved

Ischemia

Preserved at rest, impaired at stress

Normal at rest, increased at stress

Preserved at rest, impaired at stress

No

May benefit from revascularization to prevent recurrent ischemia

Scar

Reduced

Reduced

Absent

Fibrosis

Unlikely to recover with or without revascularization

Myocardium

Flow/Perfusion

Stunned

Preserved at rest (following transient ischemic insult)

Hibernation

Reproduced with permission from Kandolin RM, Wiefels CC, Mesquita CT, et al. The current role of viability imaging to guide revascularization and therapy decisions in patients with heart failure and reduced LV function. Can J Cardiol. 2019;35(8):1015–1029.

repetitive stunning, myocardial changes can progress to a hibernating state (viable), and continued ischemic insults can lead to irreversible fibrosis/scar (nonviable).17,25,26 By preventing recurrent ischemic insults, stunned myocardium is expected to recover.As such, imaging early in the post-MI period following large MIs (STEMIs or large non-STEMIs) may be misleading and challenging. False positives or false negatives can occur in the early phase post-MI as viable myocardium may recover spontaneously without needing revascularization; variable metabolic derangements in the stunned myocardium and potential effects of no reflow-phenomenon or inflammation could also alter tracer uptake and lead to equivocal or misleading viability assessment. Thus, viability imaging is often best avoided within 2 to 4 weeks of myocardial infarction.

▶▶Hibernation Hibernating myocardium is thought to progress from repeated episodes of ischemia or stunning17,27

CH23.indd 433

and by definition, remains viable and therefore has the capacity for functional recovery after adequate revascularization.4,17,28 Myocytes in a hibernating state lose variable amounts of contractile material (sarcomeres) without significant changes in the cell volume. The absence of volume changes is an important characteristic that helps differentiate viable myocardium from atrophic degeneration.29 Cellular volume, previously occupied by myofilaments, is replaced by glycogen29 (Fig. 23-1). This adaptive downregulation response is important to prevent a “supply–demand imbalance” during periods of ischemia.27 Mitochondria apparatus is preserved, and functional and oxidative metabolism is only mildly reduced and can be measured by positron emission tomography (PET) with 11C-acetate (acetate is transported into the mitochondria directly via acetyl-CoA and then enters into the tricarboxylic acid cycle to produce energy).30 Similarly, myocardial glucose metabolism is preserved and can be measured by PET with

15-02-2022 18:49:30

434

Section 4  Beyond Perfusion Imaging

A

B

C

FIGURE 23-1  (A) “Light micrograph of myocardium showing normal cardiomyocytes with virtually no glycogen (PAS staining in red). (B) Transmission electron micrograph of normal cardiac myocyte. (C) Representative light micrograph of biopsy sample of human hibernating myocardium. Cardiac myocytes are depleted of their contractile material and filled with glycogen (PAS-positive staining). (D) Representative transmission electron micrograph of a hibernating cardiomyocyte. Myolytic cytoplasm is devoid of sarcomeres and filled with glycogen. Magnification: A and C, ×320; B, ×7100; and D, ×7500. (Reproduced with permission from Vanoverschelde J-LJ, Wijns W, Borgers M, et al. Chronic myocardial hibernation in humans from bedside to bench. Circulation. 1997;95(7):1961–1971.)

D

Table 23-2 Imaging Modalities and Mechanisms for Viability Detection Imaging Modality

Method Target

Indicator of Viability

18

Glucose metabolism

Normal perfusion/FDG = viable (not ischemic at rest) Perfusion-Metabolism a) mismatch = hibernation b) match = scar (nonviable)

SPECT (Tl-201)

Myocardial perfusion/Na/K ATPase activity (membrane integrity)

Uptake = viable Redistribution on delayed imaging after stress, rest or reinjection = ischemia or hibernation

SPECT (Tc-99m-based)

Myocardial perfusion/arterioles vasodilatation Mitochondrial integrity

Uptake = viable Mismatch between rest and post nitro images = hibernating

Dobutamine echocardiography/CMR

Contractile reserve

Improvement in wall motion with low dose dobutamine

Delayed enhancement CMR

Fibrosis tissue

Absence = viable or small amount of scar

Late enhancement CT

Fibrosis tissue

Absence or small amount of scar

Microvascular integrity

Homogeneous contrast intensity

FDG PET

Myocardial contrast echocardiography 18

18

FDG PET, positron emission tomography with fluorodeoxyglucose; CT, computed tomography; CMR, cardiac magnetic resonance; SPECT, singlephoton emission computed tomography; Tl-201, thallium-201; Tc-99m, technetium-99m.

CH23.indd 434

15-02-2022 18:49:31

Chapter 23  Nuclear Cardiology Procedures in the Evaluation of Myocardial Viability 18

fluorodeoxyglucose (18FDG), a glucose analogue that is transported into the myocyte and converted to FDG-6-phosphate. This preservation of metabolism is key and enables cardiac metabolic imaging to distinguish between viable and nonviable myocardium. Several modalities are available for viability assessment: cardiac PET, SPECT, low-dose dobutamine echocardiography (LDD), dobutamine cardiac magnetic resonance (D-CMR), delayed-enhancement CMR (DE-CMR), delayed-enhancement computed tomography (CT),4 and myocardial contrast echo.31,32 Metabolic and cellular targets and findings suggestive of viability for each modality are outlined in Table 23-2. The clinically available imaging methods are summarized below.

435

viability, and one does not have an advantage over the other.

Imaging Protocols for Thallium-201 SPECT Thallium is a potassium analog and its uptake is both a passive and active process requiring the normal function of the sodium–potassium ATPase pump and cellular membrane integrity.43 Since membrane integrity is a requisite for cell viability, thallium-201 uptake visualized by SPECT images is indicative of myocardium viability. Different protocols are described to assess viability with 201Tl-SPECT. The American Society of Nuclear Cardiology (ASNC) guidelines44 provide procedural guidance (Figs. 23-2 and 23-3). Although the rest-redistribution protocol can be performed,

NUCLEAR IMAGING METHODS FOR VIABILITY

▶▶Single-Photon Emission Computed

Inject Tl-201 Rest

Day 1

Day 2

Tomography (SPECT) Myocardium viability imaging can be performed with thallium-201 (201Tl) or 99m-technetium (99mTc)–based tracers and relies on the integrity of the sarcolemma and mitochondria, respectively.33–35 Comparisons between 201Tl-SPECT, 99mTc-SPECT, and 18FDG PET show that 99mTc-based imaging may underestimate the amount of viable myocardium,36–40 while one direct comparison study observed that 201 Tl provided comparable information with 18FDG PET.38 Assessing value of viability methods is difficult yielding studies with low patient number and single modality imaging. That said, a meta-analysis of 40 studies (1119 patients) using 201Tl-SPECT showed a mean sensitivity, specificity, predictive positive value (PPV), and negative predictive value (NPV) of 87%, 54%, 67%, and 79%, respectively.4 A meta-analysis of 25 studies (721 patients) that used 99mTc-SPECT to assess viability showed sensitivity, specificity, PPV, and NPV of 83%, 65%, 74%, and 76%, respectively. Other studies showed similar values between 201Tland 99mTc-based imaging;41,42 however, neither is as sensitive as 18FDG PET. Overall, both approaches are considered reasonable means to assess myocardial

CH23.indd 435

15 min.

Rest Imaging

Review

Time

24 hour Redistribution Imaging

Review

Optional, Depending on Physician’s Interpretation of the Images

Inject Tl-201 Rest

15 min.

Time

Rest Imaging

Review

3–4 hour Delay

3–4 hour Redistribution Imaging

Review

Optional, Depending on Physician’s Interpretation of the Images

FIGURE 23-2  201Tl rest-redistribution protocol for viability assessment. (Reproduced with permission from Henzlova MJ, Duvall WL, Einstein AJ, et al. ASNC imaging guidelines for SPECT nuclear cardiology procedures: Stress, protocols, and tracers. J Nucl Cardiol. 2016;23(3):606–639.)

15-02-2022 18:49:32

Section 4  Beyond Perfusion Imaging

436

Day 1

Inject Tl-201 Stress

Stress

15 min.

Stress Imaging

Day 2

Review 2.5–4 hour Delay

Time

15 min.

Stress Imaging

Review 2.5–4 hour Delay

15 min.

Reinjection Imaging

Day 1

Review

15 min.

Day 2 Reinject Tl-201

Stress Imaging

Review 2.5–4 hour Delay

Time

Rest Review Imaging

24 hour Imaging

Review

Optional, Depending on Physician’s Interpretation of the Images Day 1

Inject Tl-201 Stress

Time

Rest Review Imaging

Optional, Depending on Physician’s Interpretation of the Images

Inject Tl-201 Stress

Stress

Review

Reinject Tl-201

Time

Stress

24 hour Redistribution Imaging

Optional, Depending on Physician’s Interpretation of the Images

Inject Tl-201 Stress

Stress

Rest Review Imaging

15 min.

Day 2 Reinject Tl-201

Stress Imaging

Review 2.5–4 hour Delay

Rest Review Imaging

15 min.

Reinjection Imaging

24 hour Imaging

Review

Optional, Depending on Physician’s Interpretation of the Images

FIGURE 23-3  201Tl stress-rest imaging protocols. (Reproduced with permission from Henzlova MJ, Duvall WL, Einstein AJ, et al. ASNC imaging guidelines for SPECT nuclear cardiology procedures: Stress, protocols, and tracers. J Nucl Cardiol. 2016;23(3):606–639.)

CH23.indd 436

15-02-2022 18:49:32

Chapter 23  Nuclear Cardiology Procedures in the Evaluation of Myocardial Viability

the most common protocol starts with a stress phase and an injection of 2.5 to 3.5 mCi of 201Tl at peak stress. After 10 to 15 minutes, the stress images are acquired with redistribution (rest) imaging acquired 2.5 to 4 hours later and can be reported as a regular stress/rest study. When a persistent (fixed) defect is present and viability assessment is required, a lateredistribution imaging can be performed at 18 to 24 hours. 201Tl redistribution is a continual process that requires blood supply to the viable tissue and thus its uptake is also related to perfusion and the severity of coronary artery stenosis.45 Studies have shown that in late images (8–72 hours), the viable myocardium segments show thallium redistribution (reversible defects), while truly nonviable myocardium appears as a persistent (fixed) defect on the late perfusion images.45–48 To optimize the protocol, or shorten the study, 1 to 2 mCi of 201Tl can be reinjected after the rest study, with the reinjection image acquired after.44 A subanalysis of a pooled meta-analysis compared 201Tl-SPECT rest-redistribution with the 201 Tl-SPECT reinjection protocol and showed comparable sensitivities (87% for both protocols) but with higher specificity and PPV for 201Tl-SPECT rest-redistribution (56% vs. 50% and 71% vs. 58%, respectively) (P < 0.05 for both).4 On the other hand, head-to-head comparison between 24-hour redistribution and the reinjection protocol suggested that reinjection may provide superior image quality and better diagnostic information with a significantly greater ability to identify hibernating myocardium.49 When comparing the reinjection protocol to 18FDG PET, there was very good correlation, although 201Tl may have inferior sensitivity compared to glucose metabolism assessment with PET.4,38,50,51 The difference in sensitivity may be because viability study with PET considers information from both perfusion and metabolism. Figure 23-4 shows an example of 201 Tl-SPECT rest-redistribution imaging, demonstrating viability in the infero/lateral region (generally accepted as 50% counts or greater).

Imaging Protocols for 99m-Tc-Based SPECT 99m

Technetium-sestamibi and 99mTc-tetrosfomin have lipophilic properties and enter cells passively. However, their retention by the mitochondria is

CH23.indd 437

SAO

HLA

437

VLA

201

TI Rest

201 TI Redistribution

FIGURE 23-4  201Tl rest-redistribution SPECT in short axis (SAO), horizontal long axis (HLA), and vertical long axis (VLA) showing a mismatch area (viable myocardium) in the mid-todistal inferior and inferolateral walls (yellow arrow) and distal anterior and apex (red arrow).

an active process and depends on mitochondrial membrane integrity.52 99mTc-based radiotracers (sestamibi and tetrofosmin) have almost no redistribution when compared to 201Tl53 and different approaches are suggested to increase sensitivity for viability detection. While resting 99mTc-based perfusion imaging can be used alone to define viability, several studies have shown that the use of nitrates improves viability detection when compared to rest 99mTc-based imaging and correlates with improvement after revascularization.42,54–58 Nitroglycerin has the capacity to increase the blood flow of epicardial and collateral vessels and increase the sensitivity to defect viable myocardium.57 The study is usually performed with two images: a rest perfusion study and a rest perfusion study following oral administration of 10 mg of isosorbide nitrate, given 10 to 15 minutes before the image acquisition. Although the exact mechanism is not well understood, it is proposed that it may be related to improvement in blood flow secondary to vasodilation improving blood supply to the hibernating myocardium and therefor tracer uptake.57 Studies have also demonstrated that abnormal contractility can lead to perfusion defects with SPECT perfusion imaging.59,60 Incremental value can be achieved by adding electrocardiogram (ECG) gating and dobutamine to 99mTc-based imaging, enabling the assessment of both perfusion and contractile reserve in a single study.61 Dual-isotope imaging with 99mTc at stress and 201 Tl at rest is also possible. The 201Tl rest portion

15-02-2022 18:49:32

438

Section 4  Beyond Perfusion Imaging

is used for viability interpretation with the addition of late redistribution and reinjection to increase the test sensitivity. However, this protocol is associated with a significantly higher radiation dose and may not be the ideal primary test when other diagnostic options are available44 and is now generally discouraged.

Imaging Interpretation of SPECT Viability Images Imaging interpretation should be performed carefully and according to the image acquisition protocol chosen, depending on the patient characteristics and institutional availability. When rest201Tl-stress SPECT imaging is performed, the interpretation is related to the protocol used. In a rest/stress/redistribution protocol,

the first set of images (rest/stress) will define the presence of ischemia (reversible defect) or persistent defect (scar). The third image of late redistribution is acquired to evaluate if the persistent defect is reversible with more time, therefore representing viable hibernating myocardium.50 A persistent (fixed) severe defect (40%)

Lower LVEF (1) c.  ≥ Grade 2 diastolic dysfunction†

ATTR/AL

2.

CMR a.  LV wall thickness >ULN for sex on SSFP cine CMR b.  Global ECV >0.40 c.  Diffuse LGE† d. Abnormal gadolinium kinetics typical for amyloidosis, myocardial nulling prior to blood pool nulling

ATTR/AL

3. PET: 18F-florbetapir† or 18F-florbetaben PET† ‡ a.  Target to background (LV myocardium to blood pool) ratio >1.5 b.  Retention index >0.030 min-1

ATTR/AL

AL, amyloidogenic light chain; ATTR, amyloidogenic transthyretin; ECV, extracelullar volume; LGE, late gadolinium enhancement; LS, longitudinal strain; LV, left ventricular; SSFP, steady-state free precession; ULN, upper limit of normal, at midcavity-level ULN for women/men were 7 mm/9 mm (long axis) and 7 mm/8 mm (short axis), respectively (Reproduced with permission from Kawel N, Turkbey EB, Carr JJ, et al. Normal left ventricular myocardial thickness for middle-aged and older subjects with steady-state free precession cardiac magnetic resonance: the multi-ethnic study of atherosclerosis. Circulation Cardiovasc Imaging. 2012;5:500–508.). These consensus recommendations were based on moderate-quality evidence from one or more well-designed, well-executed nonrandomized studies, observational studies, registries, or meta-analyses of such studies. The PET recommendations were based on more limited data. *Endomyocardial biopsy should be considered in cases of equivocal 99mTc-PYP, DPD, HMDP scan. When 99mTc-PYP, DPD, and HMDP are positive in the context of any abnormal evaluation for serum/urine immunofixation or serum-free light chain assay, or MGUS, this should not be seen as diagnostic for ATTR cardiac amyloidosis. In these instances, referral to a specialist amyloid center for further evaluation and consideration of biopsy is recommended. † Off-label use of FDA-approved commercial products. ‡18 F-flutemetamol not studied systematically in the heart. 11C-Pittsurgh B compound is not FDA approved and not available to sites without a cyclotron in proximity. Reproduced with permission from Dorbala S, Ando Y, Bokhari S et al. ASNC/AHA/ASE/EANM/HFSA/ISA/SCMR/SNMMI expert consensus recommendations for multimodality imaging in cardiac amyloidosis: Part 2 of 2-Diagnostic criteria and appropriate utilization. J Nucl Cardiol. 2019;26:2065–2123.

ch25.indd 484

15-02-2022 10:34:02

Chapter 25  Imaging Cardiac Amyloidosis

the diagnosis of AL. Emerging studies of cardiac amyloidosis such as with 18F-florbetapir,38 11C-pittsburgh B compound,39 and 18F-florbetaben40 show its value for the detection of AL cardiac amyloidosis (see section PET Amyloid Binding Tracer Imaging)

Prognosis The role of bone avid scintigraphy for evaluating prognosis has been variable. Patients with a positive bone avid scintigraphy study have worse prognosis compared to those with negative scans, but the visual grade was not prognostic (Fig. 25-3).41 For patients with ATTRv, it has been shown that the heart-to-whole body ratio (>7.5) combined with LV wall thickness (>12 mm) was associated with the highest rate of major adverse cardiovascular events (defined as cardiovascular death, hospitalization, or stroke).42 A heart-to-­contralateral lung ratio of ≥1.6 on planar 99mTc-PYP scan is associated with worse survival.43 Among individuals with

0.00

0.25

0.50

0.75

1.00

left ventricular wall thickening, those with HF and peripheral sensory neuropathy, patients with low flow low gradient aortic stenosis, patients with known or suspected hereditary ATTR cardiac amyloidosis, 99m TcPYP/DPD/HMDP scan was rated as appropriate to assess for ATTR cardiac amyloidosis.9 Patients with familial ATTR and suspected cardiac amyloidosis can also be evaluated with 99mTcPYP/DPD scan, but recent literature suggests a lower sensitivity of bone avid scintigraphy for certain patients with ATTRv.36,37 A negative bone avid cardiac scintigraphy scan, however, does not exclude AL amyloidosis, and further evaluation is necessary.1 Also, 25% of patients with AL amyloidosis manifest Grade 2/3 uptake on bone avid scintigraphy. For these reasons, it is critically important to exclude AL amyloidosis in all patients undergoing bone avid scintigraphy with a cardiac phenotype suspicious for an infiltrative process. In these patients, involved organ biopsy (bone marrow, fat pad, kidney, or endomyocardial biopsy) can be performed to confirm

485

0

20

40 Follow-up (months)

60

80

PeruginiGrade = 0

28

22

12

3

0

PeruginiGrade = 1

28

20

13

7

0

PeruginiGrade = 2

436

272

109

14

0

PeruginiGrade = 3

110

74

40

7

0

Number at risk

PeruginiGrade = 0

PeruginiGrade = 1

PeruginiGrade = 2

PeruginiGrade = 3

FIGURE 25-3  Prognostic value of 99mTc-DPD imaging stratified by visual score. Patients with Grade 0 uptake of 99mTcDPD survived longer than patients with Grade 1, 2, or 3 uptake (P < 0.02). But survival did not differ between 99mTcDPD visual Grade 1, Grade 2, and Grade 3 patients. (Reproduced with permission from Hutt DF, Fontana M, Burniston M, et al. Prognostic utility of the Perugini grading of 99mTc-DPD scintigraphy in transthyretin (ATTR) amyloidosis and its relationship with skeletal muscle and soft tissue amyloid. Eur Heart J Cardiovasc Imaging. 2017;18:1344-1350.)

ch25.indd 485

15-02-2022 10:34:03

486

Section 4  Beyond Perfusion Imaging

V30M mutation ATTRv, a heart-to-mediastinal (H/M) ratio of less than 1.6 on late 123I-mIBG imaging was associated with a significantly worse 5-year mortality (42% vs. 7% for H/M ratio of rib uptake). The top row illustrates PYP planar images, the middle row shows SPECT/CT fusion images, and the bottom row shows heart to contralateral ratio quantification. HCL refers to the ratio of regional myocardial count to that of the contralateral lung. (Reproduced with permission from Hanna M, Ruberg FL, Maurer MS, et al. Cardiac scintigraphy with technetium99m-labeled bone-seeking tracers for suspected amyloidosis: JACC Review Topic of the Week. J Am Coll Cardiol. 2020;75:2851-2862.) Grade 2

Grade 3

0 5

Normal

SUVmax = 1.7 SUVmean = 0.98 CAA = 378 %ID = 0.59

SUVmax = 2.86 SUVmean = 1.54 CAA = 535.4 %ID = 0.82

SUV 5

SUVmax = 5.29 SUVmean = 2.24 CAA = 1054 %ID = 1.17

SUVmax = 4.47 SUVmean = 2.95 CAA = 1258 %ID = 1.44

0 SUV

FIGURE 25-5  99mTc-PYP SPECT/CT quantitative imaging. Quantitative imaging using SPECT/CT demonstrates a gradual increase in quantitative PYP metrics (SUV, standardized uptake value; CAA, cardiac amyloid activity; %ID, % injected dose) from Grade 0 to Grade 4. The top row illustrates PYP SPECT images and the bottom row illustrates SPECT/CT fusion images. (Reproduced with permission from Dorbala S, Park MA, Cuddy S, et al. Absolute quantitation of cardiac (99m)Tc-pyrophosphate using cadmium zinc telluride-based SPECT/CT. J Nucl Med. 2020;62(5):716–722.)

radiotracers may be seen in AL amyloidosis and hence it is imperative to exclude AL amyloidosis (using serum and urine immunofixation electrophoresis and serumfree light chain assay) in all patients undergoing bone avid scintigraphy to make sure a deadly disease is not missed. A major pitfall is myocardial blood pool uptake in early images, which can masquerade as a positive scan on planar images (Fig. 25-6). Hence, planar-only images are not sufficient to diagnose ATTR cardiac amyloidosis. SPECT imaging is essential to distinguish myocardial activity from blood pool activity and to

ch25.indd 487

diagnose cardiac amyloidosis. If blood pool activity is noted, delayed images can be obtained to increase the contrast between the myocardium and the blood pool. A negative bone avid scintigraphy may be seen in certain patients with hereditary ATTR cardiac amyloidosis; hence, if clinical suspicion of amyloidosis remains high, endomyocardial biopsy may be considered. 99mTcmethylene diphosphonate (MDP), a commonly used tracer for bone scanning, is not recommended for the evaluation of cardiac ATTR ­amyloidosis, as it is not sensitive to diagnose cardiac ATTR amyloidosis.47

15-02-2022 10:34:10

488

Section 4  Beyond Perfusion Imaging

Positive PYP ≠ ATTR; Diagnosis = AL

Positive PYP = blood pool uptake, no amyloid

Always screen for AL

Always perform SPECT

Heart Failure with typical echo and/or CMR Negative sFLC, serum/urine IFE Positive PYP with SPECT Accurate Diagnosis = ATTR-CM Perform TTR DNA sequence

Negative PYP, Clinical suspicion persists Cardiac biopsy: Diagnosis = ATTRv Perform biopsy if strong clinical suspicion

FIGURE 25-6  Pitfalls of 99mTc-bone avid tracer scintigraphy. The major pitfalls of 99mTc-bone avid tracer scintigraphy include: (1) making sure that AL amyloidosis is not missed; (2) evaluating SPECT images to eliminate blood pool activity as a cause of a false-positive planar image; and (3) considering endomyocardial biopsy despite a negative 99mTc- PYP/DPD/HMDP scan if clinical suspicion remains high. (Reproduced with permission from Hanna M, Ruberg FL, Maurer MS, et al. Cardiac scintigraphy with technetium-99m-labeled bone-seeking tracers for suspected amyloidosis: JACC Review Topic of the Week. J Am Coll Cardiol. 2020;75:2851-2862.)

▶▶Reporting Report should contain the presence and degree of myocardial uptake, as well as other incidental findings on whole-body imaging and SPECT/CT, if performed. If qualitative analysis is requested, the heart-to-contralateral-lung ratio on planar i­maging can be reported as well. Readers are referred to American Society of Nuclear Cardio­logy (ASNC) recommendation for more details (Table 25-2).9,49

▶▶PET Amyloid Binding Tracer Imaging PET is emerging as a useful tool in the evaluation of cardiac amyloidosis. PET tracers,

ch25.indd 488

11C

-Pittsburgh, 18F-florbetapir, 18F-florbetaben, and F-flutemetamol, were originally developed for the imaging of beta-amyloid in Alzheimer’s disease; however, recently these tracers have been used in the evaluation of cardiac amyloidosis. An autoradiography study demonstrated specific binding 18 F-florbetapir to AL and ATTR amyloid deposits on myocardial sections.50 These PET tracers allow for not only a qualitative visual assessment of amyloid burden, but also a quantitative evaluation, which may prove useful in the evaluation of disease progression and response to therapy. In the initial studies, using 11 C-Pittsburgh B compound, uptake of the radiotracer was seen in all AL and ATTR cardiac amyloid patients (n = 10) and in none of the healthy controls 18

15-02-2022 10:34:13

Chapter 25  Imaging Cardiac Amyloidosis

489

Table 25-2 Recommendations for Standardized Reporting of (99mTc-PYP/DPD/HMDP Imaging for Cardiac Amyloidosis Parameters

Elements

Demographics

Patient name, age, sex, reason for the test, date of study, prior imaging procedures, and biopsy results if available (Required)

Methods

Imaging technique, radiotracer dose and mode of administration, interval between injection and scan, and scan technique (planar and SPECT) (required)

Findings

Image quality Visual scan interpretation (required) Semiquantitative interpretation in relation to rib uptake (required) Quantitative findings H/CL lung ratio (Optional; recommended for positive scans)

Ancillary findings

Whole-body imaging if planar whole-body images are acquired (optional) Interpret CT for attenuation correction if SPECT/CT scanners are used (recommended)

Conclusions

1. An overall interpretation of the findings into categories of (1) not suggestive of ATTR cardiac amyloidosis; (2) strongly suggestive of ATTR cardiac amyloidosis; or (3) equivocal for ATTR cardiac amyloidosis after exclusion of a systemic plasma cell dyscrasia (required) a.  Not suggestive: A semiquantitative visual grade of 0. b. Equivocal: If diffuse myocardial uptake of 99mTc-PYP/DPD/HMDP is visually confirmed and the semiquantitative visual grade is 1 or there is interpretive uncertainty of Grade 1 versus Grade 2 on visual grading. c. Strongly suggestive: If diffuse myocardial uptake of 99mTc-PYP/DPD/HMDP is visually confirmed, a semiquantitative visual grade of 2 or 3. 2. Statement that evaluation for AL amyloidosis by serum FLCs, serum, and urine immunofixation is recommended in all patients undergoing 99mTc-PYP/DPD/HMDP scans for cardiac amyloidosis. (Required) 3. Statement that results should be interpreted in the context of prior evaluation and referral to a hematologist or amyloidosis expert is recommended if either: a. Recommended echo/CMR is strongly suggestive of cardiac amyloidosis and 99mTc-PYP/ DPD/HMDP is not suggestive or equivocal and/or b.  FLCs are abnormal or equivocal. (Recommended)

AL, amyloid light chain; ATTR, amyloid transthyretin; CMR, cardiovascular magnetic resonance; echo, echocardiography; FLC, free light chain; H/CL, heart-to-contralateral lung ratio. Reproduced with permission from Dorbala S, Ando Y, Bokhari S, et al. ASNC/AHA/ASE/EANM/HFSA/ISA/SCMR/SNMMI Expert Consensus Recommendations for Multimodality Imaging in Cardiac Amyloidosis: Part 1 of 2—Evidence Base and Standardized Methods of Imaging. Circulation: Cardiovasc Imaging. 2021;14(7):685–725.

(n = 5). In addition, in certain hereditary forms of ATTR amyloid, amyloid deposition was detected by 11C-Pittsburgh B compound, in the setting of a normal 99mTc-PYP/DPD scan, which points to the potential use in the detection of early cardiac amyloidosis disease.37 However, although 11C-Pittsburgh B compound, demonstrated the ability to reliably image amyloid deposits, the short half-life (20 minutes) necessitated the need for an onsite cyclotron,

ch25.indd 489

thus limiting its clinical use. 18F-florbetapir demonstrated a similar affinity for binding beta-amyloid, as 11 C-Pittsburgh B compound; however, 18F-florbetapir has a longer half-life (~110 minutes), making it a better candidate for widespread clinical use. These results were duplicated in subsequent studies using 18 F-florbetapir and 18F-florbetaben.40,51 Multiple indices were utilized for the quantitative assessment of radiotracer uptake, including myocardial retention

15-02-2022 10:34:13

490

Section 4  Beyond Perfusion Imaging

Mean Standardized Uptake Value

10 EARLY SCAN

INTERMEDIATE SCAN

LATE SCAN

8 6 4 2 0 0

5

10

15

20

25 30 35 Time (Min)

40

45

50

55

60

Immunoglobulin Light-Chain Amyloidosis Transthyretin-Related Amyloidosis Non-Cardiac Amyloidosis

Immunoglobulin Light-Chain Amyloidosis

Transthyretin-Related Amyloidosis

Non-Cardiac Amyloidosis

FIGURE 25-7  18F-florbetaben PET CT time activity curves in AL and ATTR cardiac amyloidosis and in nonamyloid controls. Standardized uptake value of 18F-florbetaben is highest in AL compared to ATTR and control patients. Notable, after 30 minutes, the myocardial uptake in the ATTR group is indistinguishable from control group. (Reproduced with permission from Genovesi D, Vergaro G, Giorgetti A, et al. [18F]-Florbetaben PET/CT for differential diagnosis among cardiac immunoglobulin light chain, transthyretin amyloidosis, and mimicking conditions. JACC Cardiovasc Imaging. 2021;14(1):246–255.)

index, background-to-target ratio, and the myocardial SUV. Although quantitative indices were able to differentiate between cardiac amyloidosis patients and controls (including patients with hypertensive heart disease), they were unable to differentiate between ATTR and AL amyloidosis patients.17,52 Quantitative measures of myocardial radiotracer

ch25.indd 490

uptake were lower in ATTR, compared to AL cardiac amyloidosis with 18F-florbetaben (Fig. 25-7) and similar findings were described with the other travers as well.17,40,51 A proposed diagnostic algorithm incorporating radionuclide imaging, echocardiography and CMR, is shown in Fig. 25-8.53

15-02-2022 10:34:20

Chapter 25  Imaging Cardiac Amyloidosis

491

Symptoms, ECG, Echo, MRI, or Biomarkers suggestive of cardiac amyloidosis

Screen for the presence of a monoclonal protein Order the following three tests: - Serum kappa/lamba free light chain ratio (abnormal if ratio is 1.65) - Serum protein immunofixation (abnormal if monoclonal protein is detected) - Urine protein immunofixation (abnormal if monoclonal protein is detected) 1 or more abnormal

all normal

Bone Scintigraphy Available?

Biopsy of clinically involved organ (cardiac or renal) or fat pad* If fat pad is negative, biopsy of involved organ is required

Non-invasive evaluation with Bone Scintigraphy - Positive Bone Scintigraphy

- Positive Congo Red and - Tissue typing by mass spectrometry or immunostaining yes

AL, ATTR, other amyloidosis and/or MGUS

Cardiac Amyloidosis Unlikely

ATTR Amyloidosis

Genetic Testing

ATTRwt

Referral for Bone Scintigraphy or invasive evaluation with Heart Biopsy - Positive Congo Red and - Tissue typing by mass spectrometry or immunostaining

Negative or indeterminate

yes

no

Cardiac Amyloidosis Unlikely

no

yes

Referral to Hematology and

ATTRm

Consider Heart Biopsy if suspicion is high

yes

ATTR Amyloidosis

no

Cardiac Amyloidosis Unlikely

Genetic Testing

ATTRwt

ATTRm

FIGURE 25-8  A proposed algorithm for the evaluation of patients with suspected cardiac amyloidosis. (Reproduced with permission from Maurer MS, Bokhari S, Damy T, et al. Expert consensus recommendations for the suspicion and diagnosis of transthyretin cardiac amyloidosis. Circ Heart Fail. 2019;12:e006075.)

▶▶Conclusion Radionuclide imaging of cardiac ATTR amyloidosis with 99mTc-PYP/DPD is easy to perform with essentially no contraindications. Image analysis is relatively straightforward and nuclear imaging can provide important adjunct diagnostic and prognostic information to echocardiography and CMR. 123I-mIBG imaging can be useful to identify myocardial denervation and stratify risk of adverse clinical outcomes in individuals with ATTRv. Amyloid-binding PET radiotracers that are approved for beta-amyloid brain imaging are currently under investigation and show promise,

ch25.indd 491

particularly for quantitation and characterization of cardiac amyloidosis, and may prove a useful tool in the future.

REFERENCES 1. Dorbala S, Cuddy S, Falk RH. How to image cardiac amyloidosis: a practical approach. JACC Cardiovasc Imaging. 2020;13:1368–1383. 2. Kyle RA, Gertz MA. Primary systemic amyloidosis: clinical and laboratory features in 474 cases. Semin Hematol. 1995;32:45–59. 3. Ruberg FL, Berk JL. Transthyretin (TTR) cardiac amyloidosis. Circulation. 2012;126:1286–1300.

15-02-2022 10:34:23

492

Section 4  Beyond Perfusion Imaging

4. Rapezzi C, Merlini G, Quarta CC, et al. Systemic cardiac amyloidoses: disease profiles and clinical courses of the 3 main types. Circulation. 2009;120:1203–1212. 5. Maurer MS, Schwartz JH, Gundapaneni B, et al. Tafamidis treatment for patients with transthyretin amyloid cardiomyopathy. N Engl J Med. 2018;379:1007–1016. 6. Benson MD, Waddington-Cruz M, Berk JL, et al. Inotersen treatment for patients with hereditary transthyretin amyloidosis. N Engl J Med. 2018;379:22–31. 7. Adams D, Gonzalez-Duarte A, O’Riordan WD, et al. Patisiran, an RNAi therapeutic, for hereditary transthyretin amyloidosis. N Engl J Med. 2018;379:11–21. 8. Mohammed SF, Mirzoyev SA, Edwards WD, et al. Left ventricular amyloid deposition in patients with heart failure and preserved ejection fraction. JACC Heart Fail. 2014;2: 113–122. 9. Dorbala S, Ando Y, Bokhari S, et al. ASNC/AHA/ASE/EANM/ HFSA/ISA/SCMR/SNMMI expert consensus recommendations for multimodality imaging in cardiac amyloidosis: Part 2 of 2-Diagnostic criteria and appropriate utilization. J Nucl Cardiol: Official Publ Am Soc Nucl Cardiol. 2019;26:2065– 2123. 10. Quarta CC, Solomon SD, Uraizee I, et al. Left ventricular structure and function in transthyretin-related versus lightchain cardiac amyloidosis. Circulation. 2014;129:1840–1849. 11. Dorbala S, Ando Y, Bokhari S, et al. ASNC/AHA/ASE/EANM/ HFSA/ISA/SCMR/SNMMI expert consensus recommendations for multimodality imaging in cardiac amyloidosis: Part 1 of 2-evidence base and standardized methods of imaging. J Nucl Cardiol: Official Publ Am Soc Nucl Cardiol. 2019. 12. Martinez-Naharro A, Treibel TA, Abdel-Gadir A, et al. Magnetic resonance in transthyretin cardiac amyloidosis. J Am Coll Cardiol. 2017;70:466–477. 13. Zhao L, Tian Z, Fang Q. Diagnostic accuracy of cardiovascular magnetic resonance for patients with suspected cardiac amyloidosis: a systematic review and meta-analysis. BMC Cardiovasc Disord. 2016;16:129. 14. Fontana M, Pica S, Reant P, et al. Prognostic value of late gadolinium enhancement cardiovascular magnetic resonance in cardiac amyloidosis. Circulation. 2015;132: 1570–1579. 15. Karamitsos TD, Piechnik SK, Banypersad SM, et al. Noncontrast T1 mapping for the diagnosis of cardiac amyloidosis. JACC Cardiovasc Imaging. 2013;6:488–497. 16. Fontana M, Banypersad SM, Treibel TA, et al. Native T1 mapping in transthyretin amyloidosis. JACC Cardiovasc Imaging. 2014;7:157–165. 17. Dorbala S, Vangala D, Semer J, et al. Imaging cardiac amyloidosis: a pilot study using (18)F-florbetapir positron emission tomography. Eur J Nucl Med Mol Imaging. 2014;41: 1652–1662. 18. Osborne DR, Acuff SN, Stuckey A, Wall J. A routine PET/CT protocol with simple calculations for assessing cardiac amyloid using 18F-Florbetapir. Front Cardiovasc Med. 2015;2. 19. Antoni G, Lubberink M, Estrada S, et al. In vivo visualization of amyloid deposits in the heart with 11C-PIB and PET. J Nucl Med: Official Publ, Soc Nucl Med. 2013;54:213–220. 20. Lee SP, Lee ES, Choi H, et al. (11)C-Pittsburgh B PET imaging in cardiac amyloidosis. JACC Cardiovasc Imaging. 2015;8: 50–59.

ch25.indd 492

21. Coutinho MC, Cortez-Dias N, Cantinho G, et al. Reduced myocardial 123-iodine metaiodobenzylguanidine uptake: a prognostic marker in familial amyloid polyneuropathy. Circulation Cardiovasc Imaging. 2013;6:627–636. 22. Delahaye N, Dinanian S, Slama MS, et al. Cardiac sympathetic denervation in familial amyloid polyneuropathy assessed by iodine-123 metaiodobenzylguanidine scintigraphy and heart rate variability. Eur J Nucl Med. 1999;26:416–424. 23. Noordzij W, Glaudemans AW, van Rheenen RW, et al. (123) I-Labelled metaiodobenzylguanidine for the evaluation of cardiac sympathetic denervation in early stage amyloidosis. Eur J Nucl Med Mol Imaging. 2012;39:1609–1617. 24. Tanaka M, Hongo M, Kinoshita O, et al. Iodine-123 metaiodobenzylguanidine scintigraphic assessment of myocardial sympathetic innervation in patients with familial amyloid polyneuropathy. J Am Coll Cardiol. 1997;29:168–174. 25. Gillmore JD, Maurer MS, Falk RH, et al. Nonbiopsy diagnosis of cardiac transthyretin amyloidosis. Circulation. 2016;133:2404–2412. 26. Gonzalez-Lopez E, Gallego-Delgado M, Guzzo-Merello G, et al. Wild-type transthyretin amyloidosis as a cause of heart failure with preserved ejection fraction. Eur Heart Journal. 2015;36:2585–2594. 27. Bennani Smires Y, Victor G, Ribes D, et al. Pilot study for left ventricular imaging phenotype of patients over 65 years old with heart failure and preserved ejection fraction: the high prevalence of amyloid cardiomyopathy. Int J Cardiovasc Imaging. 2016;32:1403–1413. 28. Castano A, Narotsky DL, Hamid N, et al. Unveiling transthyretin cardiac amyloidosis and its predictors among elderly patients with severe aortic stenosis undergoing ­transcatheter aortic valve replacement. Eur Heart J. 2017;38:2879–2887. 29. Scully PR, Patel KP, Treibel TA, et al. Prevalence and outcome of dual aortic stenosis and cardiac amyloid pathology in patients referred for transcatheter aortic valve implantation. Eur Heart J. 2020. 30. Sperry BW, Reyes BA, Ikram A, et al. Tenosynovial and cardiac amyloidosis in patients undergoing carpal tunnel release. J Am Coll Cardiol. 2018;72:2040–2050. 31. Dorbala S, Park MA, Cuddy S, et al. Absolute quantitation of cardiac (99m)Tc-pyrophosphate using cadmium zinc telluridebased SPECT/CT. J Nucl Med: Official Publ, Soc Nucl Med. 2020. 32. Dorbala S, Kijewski MF, Park MA. Quantitative bone-avid tracer SPECT/CT for cardiac amyloidosis: a crucial step forward. JACC Cardiovasc Imaging. 2020;13:1364–1367. 33. Ross JC, Hutt DF, Burniston M, et al. Quantitation of (99m) Tc-DPD uptake in patients with transthyretin-related cardiac amyloidosis. Amyloid : Int J Exp Clin Investigation : Official J Int Soc Amyloidosis. 2018;25:203–210. 34. Papatheofanis FJ. Quantitation of biochemical markers of bone resorption following strontium-89-chloride therapy for metastatic prostatic carcinoma. J Nucl Med: Official Publ, Soc Nucl Med. 1997;38:1175–1179. 35. Scully PR, Morris E, Patel KP, et al. DPD quantification in cardiac amyloidosis: a novel imaging biomarker. JACC Cardiovasc Imaging. 2020;13:1353–1363. 36. Musumeci MB, Cappelli F, Russo D, et al. Low sensitivity of bone scintigraphy in detecting Phe64Leu mutation-related transthyretin cardiac amyloidosis. JACC Cardiovasc Imaging. 2019.

15-02-2022 10:34:23

Chapter 25  Imaging Cardiac Amyloidosis 37. Pilebro B, Arvidsson S, Lindqvist P, et al. Positron emission tomography (PET) utilizing Pittsburgh compound B (PIB) for detection of amyloid heart deposits in hereditary transthyretin amyloidosis (ATTR). J Nucl Cardiol: Official Publ Am Soc Nucl Cardiol. 2018;25:240–248. 38. Cuddy SAM, Bravo PE, Falk RH, et al. Improved quantification of cardiac amyloid burden in systemic light chain amyloidosis: redefining early disease? JACC Cardiovasc Imaging. 2020;13:1325–1336. 39. Rosengren S, Skibsted Clemmensen T, Tolbod L, et al. Diagnostic accuracy of [11C]PIB positron emission tomography for detection of cardiac amyloidosis. JACC: Cardiovasc Imaging. 2020:3375. 40. Genovesi D, Vergaro G, Giorgetti A, et al. [18F]-­Florbetaben PET/CT for differential diagnosis among cardiac immunoglobulin light chain, transthyretin amyloidosis, and mimicking conditions. JACC Cardiovasc Imaging. 2020. 41. Hutt DF, Fontana M, Burniston M, et al. Prognostic utility of the Perugini grading of 99mTc-DPD scintigraphy in transthyretin (ATTR) amyloidosis and its relationship with skeletal muscle and soft tissue amyloid. Eur Heart J Cardiovasc Imaging. 2017;18:1344–1350. 42. Rapezzi C, Quarta CC, Guidalotti PL, et al. Role of (99m)TcDPD scintigraphy in diagnosis and prognosis of hereditary transthyretin-related cardiac amyloidosis. JACC CardiovascImaging. 2011;4:659–670. 43. Castano A, Haq M, Narotsky DL, et al. Multicenter Study of Planar Technetium 99m Pyrophosphate Cardiac Imaging: Predicting Survival for Patients With ATTR Cardiac Amyloidosis. JAMA Cardiol. 2016. 44. Martinez-Naharro A, Abdel-Gadir A, Treibel TA, et al. CMRverified regression of cardiac al amyloid after chemotherapy. JACC Cardiovasc Imaging. 2018;11:152–154. 45. Castano A, DeLuca A, Weinberg R, et al. Serial scanning with technetium pyrophosphate (Tc-PYP) in advanced ATTR car-

ch25.indd 493

493

diac amyloidosis. J Nucl Cardiol: Official Publ Am Soc Nucl Cardiol. 2015. 46. Dorbala S, Kijewski MF, Park MA. Quantitative molecular imaging of cardiac amyloidosis: The journey has begun. J Nucl Cardiol: Official Publ Am Soc Nucl Cardiol. 2015. 47. Perugini E, Guidalotti PL, Salvi F, et al. Noninvasive etiologic diagnosis of cardiac amyloidosis using 99mTc-3,3-diphosphono-1,2-propanodicarboxylic acid scintigraphy. J Am Coll Cardiol. 2005;46:1076–1084. 48. Hanna M, Ruberg FL, Maurer MS, et al. Cardiac scintigraphy with technetium-99m-labeled bone-seeking tracers for ­suspected amyloidosis: JACC Review Topic of the Week. J Am Coll Cardiol. 2020;75:2851–2862. 49. Dorbala S, Bokhari S, Miller E, Bullock-Palmer R, Soman P, Thompson R. ASNC practice points: 99mtechnetium-­ pyrophosphate imaging for transthyretin cardiac amyloidosis. Online: Am Soc Nucl Cardiol. 2016. 50. Park MA, Padera RF, Belanger A, et al. 18F-Florbetapir Binds Specifically to Myocardial Light Chain and Transthyretin Amyloid Deposits: Autoradiography Study. Circulation Cardiovasc Imaging. 2015;8. 51. Rosengren S, Skibsted Clemmensen T, Tolbod L, et al. Diagnostic accuracy of [(11)C]PIB positron emission tomography for detection of cardiac amyloidosis. JACC Cardiovasc Imaging. 2020;13:1337–1347. 52. Law WP, Wang WY, Moore PT, Mollee PN, Ng AC. Cardiac amyloid imaging with 18F-florbetaben positron emission tomography: a pilot study. J Nucl Med: Official Publ, Soc Nucl Med. 2016. 53. Maurer MS, Bokhari S, Damy T, et al. Expert consensus recommendations for the suspicion and diagnosis of transthyretin cardiac amyloidosis. Circ Heart Fail. 2019;12: e006075.

15-02-2022 10:34:24

This page intentionally left blank

READ_THINK_ITALIAN.indd 2

11/19/21 1:12 PM

18

F-FDG PET/CT for Imaging Cardiac Sarcoidosis and Inflammation Cesia Gallegos, Bryan D. Young and Edward J. Miller

KEY POINTS ■■

■■

■■

■■

■■

CH26.indd 495

Fluorine-18 fluorodeoxyglucose (18F-FDG) positron emission tomography/computer tomo-graphy (PET/CT) plays a critical role in the diagnosis and treatment assessment of indivi duals with suspected or known cardiac sarcoidosis (CS) and is included in diagnostic algorithms of multisociety guidelines Diagnosis of active inflammation and assessment of response to treatment are the two primary indications for 18F-FDG PET/ CT in CS. Cardiac PET/CT imaging of perfusion and metabolism with 18F-FDG has the potential to risk stratify patients with CS for future cardiac events. Several studies have shown the usefulness of 18F-FDG PET/CT as an additional tool in detecting infections associated with devices and grafts, now incorporated into guidelines for the management of infective endocarditis. 18 F-FDG PET/CT has high sensitivity to detect infection in challenging causes of cardiac device infections and prosthetic valves, although the specificity may be low, particularly in the early postoperative period.

CHAPTER

26

INTRODUCTION Molecular imaging with hybrid positron emission tomography/computer tomography (PET/CT) using fluorine-18 fluorodeoxyglucose (18F-FDG) is essential in oncology for the diagnosis and prognosis of malignancy. In cardiovascular medicine, 18F-FDG has historically been used for the assessment of myocardial hibernation/viability. More recently, the usefulness of 18F-FDG PET/CT has been investigated for the diagnosis of cardiac infections, such as cardiac implantable electronic device (CIED) infections, infective endocarditis (both native valve and prosthetic valve), and myocardial inflammatory conditions, such as sarcoidosis. This chapter will describe the role of 18F-FDG imaging for cardiac sarcoidosis (CS), which includes indications, patient preparation, imaging and reporting, as well as potential value in the assessment of treatment success. In addition, the role of 18F-FDG in assessing potential cardiovascular infections such as prosthetic cardiac valves, devices, and leads will also be discussed. 18 F-FDG is a cyclotron-produced glucose analog with a half-life of 110 minutes. 18F-FDG undergoes facilitated diffusion across the cardiomyocyte sarcolemma (primarily through GLUT1 and GLUT4 channels),1 where it is phosphorylated by hexokinase to 18F-FDG-6-phosphate and is not metabolized further. The trapped 18F-FDG-6-phosphate within the cell provides the imaging signal, acting as a surrogate marker for cellular glucose metabolism, representing the integrated process of uptake and phosphorylation.

15-02-2022 10:52:22

496

Section 4  Beyond Perfusion Imaging

GLUT1 and GLUT3 synthesis and cell membrane expression are upregulated in activated macrophages, which facilitate increased glucose utilization. In addition, circulating cytokines and growth factors are thought to increase the affinity of glucose transporters for 18F-FDG. This high glycolytic activity from infiltrates of active inflammatory cells is the rationale for utilizing 18F-FDG for imaging inflammation and infection. However, increased glucose uptake is not specific for inflammation. Myocardial ischemia, for example, is a potent stimulus for increased glucose utilization by increased cell surface mobilization of GLUT4 in cardiomyocytes through insulin-independent pathways. Therefore, the suppression of endogenous myocardial glucose utilization for the purpose of imaging inflammation is key to the technique and is discussed later in this chapter. 18

F-FDG PET/CT IMAGING OF CARDIAC SARCOIDOSIS

Sarcoidosis is a multisystem disorder characterized by tissue accumulation of inflammatory noncaseating granulomas and can eventually lead to organ damage due to both inflammation and subsequent scarring and fibrosis. Systemic sarcoidosis affects the lungs/thoracic lymph nodes in approximately 80% of individuals. The prevalence of cardiac involvement in sarcoidosis is of some debate, but may be evident in approximately 10% to 30% of patients without cardiac symptoms.2 The exact percentage is unknown, especially given the lack of an imaging “gold standard” for diagnosis. Clinical manifestations range from arrhythmias (conduction abnormalities and ventricular arrhythmias) and sudden cardiac death (SCD), to congestive heart failure, valvular or pericardial disorders, and myocardial ischemia, which is rare. Despite not affecting most patients with systemic sarcoidosis, CS represents the cause of death in up to 25% of sarcoidosis in the United States.2 Early diagnosis is important as these complications, particularly complete heart block, are potentially ameliorated with early treatment. Various clinical diagnostic criteria for CS have been proposed, including those from the World Association of Sarcoidosis and other Granulomatous disorder (WASOG)3 and the Japanese Ministry of Health and Welfare (JMHW) in

CH26.indd 496

1992 with revision in 2006 and again in 2017 (Table 26-1).2,4,5 The 2006 JMHW criteria included histological diagnosis by endomyocardial biopsy (EMB) or a clinical diagnosis by extracardiac biopsy-proven sarcoidosis in conjunction with findings on ECG, echocardiography, myocardial perfusion imaging with 201 thallium, 99mtechnetium, inflammation imaging with 67gallium (no longer used at most centers due to limited accuracy and high radiation exposure), and cardiac magnetic resonance (CMR).2,4,6 Unlike other infiltrative cardiomyopathies such as amyloidosis, myocardial involvement in sarcoidosis is typically patchy, accounting for the low yield of right ventricular EMB. Because of this, 18F-FDG PET/CT plays a major role in the contemporary diagnosis and management of CS.7 Acknowledging this, in 2014, the Heart Rhythm Society (HRS) expert consensus statement included 18F-FDG uptake in PET/CT in its clinical criteria (Table 26-1).6 The JMHW also revised its criteria in 2017 to include the use of 18F-FDG PET, as shown in Table 26-1.2,4,5 An important aspect of all of these criteria is the lack of consensus for a diagnostic pathway for the diagnosis of isolated CS, though the 2017 JMHW guidelines address this.5 In addition, none of these criteria have been prospectively validated, making a probabilistic diagnostic framework, incorporating 18F-FDG PET and CMR, an appealing approach.8 The first part of this chapter will focus on the technical aspects of imaging of CS using 18F-FDG PET and its role in the diagnosis, assessment of disease activity and treatment response, and prognosis.

▶▶Imaging Technique Dietary Preparation The metabolic preparation of 18F-FDG PET/CT for CS aims to suppress normal myocardial glucose utilization in order to reduce endogenous myocardial glucose uptake, therefore enhancing detection of glucose-avid inflammatory macrophages. This is accomplished by shifting myocardial metabolism to fatty acid use through one or multiple mechanisms, including prolonged fasting, high fat/low carbohydrate diet, or intravenous unfractionated heparin. There have been several small prospective studies comparing different dietary preps for 18F-FDG PET

15-02-2022 10:52:22

Chapter 26  18F-FDG PET/CT for Imaging Cardiac Sarcoidosis and Inflammation

497

Table 26-1 Revised JMHW and HRS Diagnostic Criteria for Cardiac Sarcoidosis JMHW Criteria 2017

HRS Criteria 2014

Histologic diagnosis group CS confirmed by EMB, and histologic or clinical diagnosis of extra CS

Histologic diagnosis from myocardial tissues Noncaseating granuloma on EMB with no alternative cause identified

Clinical diagnosis group Histologic or clinical diagnosis of extra CS • Two or more of the five major criteria or • One of the five major criterion and two or more of the three minor criteria Major criteria • High-grade atrioventricular block (including complete heart block) or fatal ventricular arrhythmia (sustained ventricular tachycardia or ventricular fibrillation) • Basal thinning of intraventricular septum or abnormal ventricular wall anatomy (ventricular aneurysm, thinning of the middle or upper ventricular septum, and regional ventricular wall thickening) • Abnormally high uptake with 67Ga or 18FDG PET • Late gadolinium enhancement in cardiac MRI Minor criteria • Electrocardiography: ventricular tachycardia, PVCs, RBBB, abnormal axis, abnormal Q wave • Perfusion defects on myocardial perfusion imaging • EMB: moderate or severe myocardial 1fibrosis or monocyte infiltration

Clinical diagnosis Probable diagnosis of CS exists if: – There is histologic diagnosis of extra CS* and – One or more of the following is present: • Cardiomyopathy or atrioventricular block responsive to immunosuppressive treatment* • Unexplained reduced LVEF (18 hours) was more effective than shorter duration of fasting with concomitant heparin administration. A systematic review of dietary preparations prior to 18 F-FDG PET for CS by Atterton-Evans et al.11 aimed to identify the optimal dietary prescription for suppression of physiological 18F-FDG uptake. Though their findings suggested that carbohydrate restriction prior to PET imaging may improve myocardial FDG suppression, it was also evident that there is a lack of dietary details available in the published literature, as well as variations in the measures of scan acceptability. The Joint SNMMI-ASNC Expert Consensus Document on the Role of 18F-FDG PET/CT in CS2 recommends two options for dietary preparation. The preferred option is two high-fat meals (>35 g),

15-02-2022 10:52:22

498

Section 4  Beyond Perfusion Imaging A Excellent

23

B Moderate

23

C Poor

16

63

62

63

69

63

65

FIGURE 26-1  Cardiac 18F-FDG PET imaging demonstrating variable suppression in three patients without cardiac disease. (A) Excellent myocardial suppression with blood pool activity that exceeds that of the myocardium. (B) Moderate myocardial suppression with diffuse low-level myocardial 18F-FDG uptake and nonspecific focally increased uptake in the papillary muscles and lateral wall. (C) Poor myocardial suppression with diffuse 18F-FDG uptake throughout the heart. (Reproduced with permission from Osborne MT, Hulten EA, Murthy VL, et al. Patient preparation for cardiac fluorine-18 fluorodeoxyglucose positron emission tomography imaging of inflammation. J Nucl Cardiol. 2017;24:86–99.)

low carbohydrate (18 hours), particularly for patients who cannot follow Option 1.2 Other adjunctive medications, such as the use of heparin to induce lipolysis and increase serum-free fatty acids, can be used in addition to dietary preparation. Though the role of heparin is uncertain, the most common practice is to administer a single 50 UI/kg IV bolus 15 minutes before FDG administration. The optimal preparation may be to provide detailed dietary instruction for a high-fat/low-carbohydrate diet for breakfast, lunch, and dinner the day prior to the study, followed by a prolonged fast beginning at 6 pm and extending until the test is performed the following morning (approximately 15 hours). This approach has been validated in that this preparation reliably reduces circulating insulin and increases FFA measured immediately prior to imaging.12 It is important to provide detailed patient education for the dietary preparation, as shown by Christopoulos et al.,13 where they evaluated the effect of patient education of the SNMMI/ ASNC-recommended protocol in two groups. One group received suggested meal examples, while the second group received more extensive reinforcement of instruction by nursing staff and review of dietary log was performed. In this study, the group that received more education using a structured preparation protocol was highly successful in achieving suppression of physiologic myocardial FDG uptake. Despite these recommendations and dietary instructions, about 20% of patients may still show

CH26.indd 498

nonspecific uptake despite appropriate dietary preparation.14 In a prospective analysis of the test/retest repeatability of 18F-FDG PET in 15 patients within 2 weeks and without change in treatment, it was found that despite appropriate insulin suppression and dietary preparation, there was qualitative discordance in cardiac 18F-FDG uptake between the first and second test in 4 of 15 patients12. Overall, these data suggest that 18F-FDG PET/CT protocols for imaging of inflammation may require further optimization. Figure 26-1 presents examples of complete, moderate, and poor suppression of physiological myocardial 18F-FDG uptake.15 Table 26-2 lists some sample foods for the sarcoid 18 F-FDG PET diet.2,14

Protocol and Acquisition PET/CT imaging protocols for CS include both an 18 F-FDG exam (including a cardiac-centered acquisition, potentially along with a more extended skull base to mid-thigh field of view) and a separate rest myocardial perfusion imaging study with PET using 13 N-ammonia or 82Rubidium (82Rb) or alternatively2 with SPECT using 99mTc sestamibi or 99mTc tetrofosmin. These protocols allow for the assessment of both inflammation (via 18F-FDG) and “scar” via the rest perfusion imaging, along with the potential to identify “extra-cardiac” sarcoidosis on the extended field of view. Table 26-3 outlines a summary protocol for the 18F-FDG PET/CT study for CS. PET/CT imaging is preferred for concomitant attenuation

15-02-2022 10:52:32

Chapter 26  18F-FDG PET/CT for Imaging Cardiac Sarcoidosis and Inflammation

499

Table 26-2 Dietary Preparation and Dietary Recommendations Prior to FDG PET/CT Category

Recommendations

OK to consume

• Eggs (prepared without milk or cheese) • Oil (an option for patients who are vegan or are unable to eat and have enteral access) and butter • Clear liquids (unsweetened water, tea, coffee.) • Meat fried in oil or butter without breading or broiled (chicken, turkey, bacon, meat-only sausage, hamburgers, steak, fish)

Acceptable

• Fasting for 18 hours or longer if patient cannot eat and has no enteral access or if patient has dietary restrictions preventing consumption of advised diet

Avoid

• Vegetables, beans, nuts, fruits, and juices • Bread, grain, rice, pasta, and all baked goods • Sweetened, grilled, or cured meats or meat with carbohydrate-containing additives (some sausages, ham, sweetened bacon) • Dairy products (milk, cheese, etc.) aside from butter • Candy, gum, lozenges, sugar, and sucralose (Splenda; Heartland Food Products Group) • Alcoholic beverages, soda, and sports drinks • Mayonnaise, ketchup, tartar sauce, mustard, and other condiments • Dextrose-containing intravenous medications

Reproduced with permission from Chareonthaitawee P, Beanlands RS, Chen W, et al. Joint SNMMI–ASNC Expert Consensus Document on the Role of 18F-FDG PET/CT in Cardiac Sarcoid Detection and Therapy Monitoring. J Nuc Med. 2017;58:1341-1353.

Table 26-3 Imaging Protocol for 18F-FDG PET/CT Imaging for Cardiac Sarcoidosis Patient Preparation High fat, low to zero carbohydrate diet (see Table 26-2) for at least two meals 24 hours prior to the test, followed by overnight fast Avoid strenuous activity for at least 24 hours Avoid Dextrose containing intravenous fluids/medication Resting Myocardial Perfusion Imaging Rest myocardial perfusion imaging using standard protocols of 82rubidium PET, 99mTc SPECT, or 13N-ammonia PET If 82 Rb, administer 20-30mci and acquire rest study (3D) F-FDG PET/CT Imaging

18

Radiopharmaceutical F-FDG administered intravenously Administer 8-10 mCi 18F-FDG Post rest 82Rb

18

Uptake Period 90-minute uptake phase following 18F-FDG administration (keep same uptake period for follow-up scan as at baseline) Patient should be kept in a quiet room that is shielded for radiation protection No food intake during the uptake period No exercise to avoid muscle uptake Imaging Protocol Dedicated cardiac 18F-FDG scan: 20-minute (2D) or 10-minute (3D) image acquisition in the cardiac bed position followed by limited whole-body 18F-FDG scan: Base of skull through abdomen, 3 minutes per bed position for 3D PET. Data from Chareonthaitawee P, Beanlands RS, Chen W, et al. Joint SNMMI–ASNC Expert Consensus Document on the Role of 18F-FDG PET/CT in Cardiac Sarcoid Detection and Therapy Monitoring. J Nuc Med. 2017;58:1341-1353.

CH26.indd 499

15-02-2022 10:52:32

500

Section 4  Beyond Perfusion Imaging

correction and anatomical localization, but PETonly imaging can also be performed on dedicated PET scanner, realizing that this may lead to challenges in localizing focal hotspots of 18F-FDG. After resting perfusion, 18F-FDG is administered followed by a 60- or 90-minute incubation and cardiac acquisition ranging from 10 to 30 minutes depending on scanner, acquisition mode, count rate, and tracer dose.2

▶▶Image Interpretation Interpretation of the perfusion and 18F-FDG integrated study involves four steps: (1) interpretation of perfusion images for perfusion defects and left ventricular function; (2) interpretation of cardiac 18F-FDG PET/CT, preferably including review of hybrid fused PET/CT images using a visual and if possible, a quantitative SUV scale; (3) integration of perfusion and 18F-FDG PET/CT data; and (4) interpretation of “extra-cardiac” 18F-FDG PET/CT (thoracic and/or limited whole-body) for the assessment of “extra-cardiac” sarcoidosis activity. Images should be interpreted by readers with experience in 18F-FDG imaging of the heart in conjunction with clinical data, ECG findings, and other results of imaging studies, such as echocardiography and CMR. Analysis of FDG PET/CT should be done concomitantly with rest myocardial perfusion images. First, the rest perfusion and FDG images should be coregistered to superimpose the perfusion imaging contours onto the FDG images. This can assist with anatomic localization of potentially focal left ventricular FDG uptake. Next, the resting perfusion images should be analyzed using the standard methodologies for perfusion defects, regional and global left ventricular function, and resting regional and global myocardial blood flow (if performed). Interpretation of cardiac 18F-FDG PET/CT from hybrid fused PET/CT images can then be performed using both a visual method and, if possible, an SUV-based display scale. Review of FDG uptake from cardiac formatted imaging display systems can also be employed, which can be useful to determine where anatomically focal FDG uptake is located in the myocardium. However, standard nuclear cardiology software systems that only display normalized, nonquantitative FDG images should likely be used

CH26.indd 500

with some caution due to the difficulty in assessing the total degree of cardiac and extracardiac disease activity, and due to normalization artifacts in the cardiac images. If available, semiquantitative analysis of fused FDG PET/CT using hybrid imaging fusion software to display and review axial fused FDG/CT images may be performed and has been shown to provide added value. With this approach, a 0 to 7-g/ml SUV body weight scale is employed to interpret the presence, location, and intensity of cardiac FDG uptake on the images. This is particularly important and useful in images with low-intensity FDG uptake (that would be artifactually intensified using normalized cardiac displays) and/or if there is cardiac FDG uptake outside of the left ventricle (such as the right ventricular free wall which is an important prognostic feature).16 Additional measures of cardiac metabolic volume and activity (CMV/CMA) may be obtained, that rely on identification of voxels with an SUV intensity above the defined threshold, then calculating the volume of voxels and measuring the mean SUV intensity of this volume. Figure 26-2 shows the methodology for SUV-threshold-based measurement of FDG uptake.17 There are several quantitative metrics for the assessment of SUVmax (Table 26-4). Although data support their use, there is no preferred method or SUV threshold that can distinguish active CS from normal myocardium or nonspecific uptakes. It is suggested that studies performed at the same institution with the same protocol compare SUVmax, measuring volume of inflamed tissue using an SUV threshold, and measuring target-to-background ratio comparing myocardial SUV to blood pool between scans, as outlined in Table 26-4. A comparison between visual and quantitative measurement of FDG uptake in CS is described in Table 26-5.2 Integration of FDG and myocardial perfusion interpretation usually falls into one of the following patterns: both normal perfusion and metabolism, abnormal perfusion or metabolism, or abnormal perfusion and metabolism. These findings can be then classified into four categories: (1) normal: normal perfusion and FDG; (2) early disease: no or mild perfusion defect with “matching” FDG uptake; (3) progressive disease: moderate perfusion defect with increased corresponding FDG uptake; and

15-02-2022 10:52:32

Chapter 26  18F-FDG PET/CT for Imaging Cardiac Sarcoidosis and Inflammation Step 1: Measure LVBP SUVMax (ex. SUVmax=2.3 g/ml)

A

501

Step 2: Define Cardiac ROI SUVMax (ex. SUVmax=7.9 g/ml; blue dot)

Step 3: Examine all 3 cardiac planes (axial, coronal, sagittal)

[i.e., Ensure no FDG-avid non-cardiac structures in ROI (LN, liver, etc.]

Step 4: Calculate SUV Detection Threshold

B

=

LVBP SUVmax * 1.5 Cardiac SUVmax

Ex: ((2.3 g/ml * 1.5)/7.9 g/ml)*7.9 g/ml = 3.5 g/ml

((2.3 * 1.5)/7.9)*7.9 = 3.5 g/ml

((2.3 g/ml * 2.0)/7.9 = 4.6 g/ml

Sagittal

Axial

((2.3 * 1.0)/7.9)*7.9 = 2.3 g/ml

*Cardiac SUVmax

Step 5: Record Cardiac Metabolic Volume (cm3) and Cardiac Metabolic Activity (g glucose)

FIGURE 26-2  (A and B) Method of SUV-threshold measurement of FDG uptake. Examples of detection thresholds (1.0× LVBP SUVmax [left], 1.5× LVBP SUVmax [middle, red box], and 2.0× LVBP SUVmax [right]) are depicted in B. (Reproduced with permission from Ahmadian A, Brogan, A, Berman J, et al. Quantitative interpretation of FDG PET/CT with myocardial perfusion imaging increases diagnostic information in the evaluation of cardiac sarcoidosis. J Nucl Cardiol. 2014;21: 925–939.)

CH26.indd 501

15-02-2022 10:53:01

502

Section 4  Beyond Perfusion Imaging

Table 26-4 Quantitative Metrics for Assessment 18F-FDG Uptake Metric

Definition

Advantage

Outcome

SUVmax

Measure SUVmax in the myocardium using ROI tool

Maximal intensity of inflammation independent of extent. More reproducible than SUVmean

None

SUVmean

SUVmean in 17 segments

Less sensitive to image noise

None

Coefficient of variance

Standard deviation of uptake divided by average uptake in 17 segments

Measures heterogeneity of inflammatory activity

None

Volume (and intensity) of metabolically active myocardium

VOI tool tissue with uptake above a predefined threshold of SUV18

Extent of inflammation can be estimated

Associated with combined endpoint of VT/CHF/Heart block

Heart-to-blood pool ratio

Cardiac SUVmax-to-aortic SUVmax ratio

Corrects for background pool

None

Adapted from Chareonthaitawee P, Beanlands RS, Chen W, et al. Joint SNMMI–ASNC Expert Consensus Document on the Role of 18F-FDG PET/CT in Cardiac Sarcoid Detection and Therapy Monitoring. J Nucl Med. 2017;58:1341−1353.

Table 26-5 Comparison of Visual versus Quantitative 18F-FDG PET for Cardiac Sarcoidosis Parameter

Visual Assessment

Quantitative Assessment

Method

Qualitative; Based on visual assessment of dedicated cardiac images and whole-body images

Requires dedicated workstation to calculate SUVmax and volume of inflammation

Advantages

Rapid

More reproducible

Pitfalls

Subject to normalization

No single best technique for quantification and has unknown optimal threshold for determining SUV volume. Technique-dependent and may vary at every institution.

Recommendation

Evaluation of whole-body images as these are less subject to differences in normalization than dedicated cardiac images

Assess both severity (SUVmax) and extent (volume of 18F-FDG)

Adapted from Chareonthaitawee P, Beanlands RS, Chen W, et al. Joint SNMMI–ASNC Expert Consensus Document on the Role of 18F-FDG PET/CT in Cardiac Sarcoid Detection and Therapy Monitoring. J Nucl Med. 2017;58:1341−1353.

(4) advanced disease (fibrosis): severe perfusion defect with minimal or no FDG uptake (Figure 26-3).19 Finally, it is critical to review the entire FDG PET/ CT field of view. This can be a valuable adjunct by providing evidence of active extracardiac sarcoidosis

CH26.indd 502

(or lack thereof) and potentially identifying a site for biopsy evidence of noncaseating granulomas. In addition, one should be aware of other areas of potential pathological FDG uptake, such as lung nodules, breast tissue, hepatic lesions, etc. Individuals with

15-02-2022 10:53:01

Chapter 26  18F-FDG PET/CT for Imaging Cardiac Sarcoidosis and Inflammation Perfusion PET (e.g., 82Rb, 99mTc, or 13N )

18F-FDG

PET

503

Interpretation Normal Pattern • Normal perfusion • No FDG uptake • Physiologic FDG uptake fully suppressed

Normal Pattern • Normal perfusion • Diffuse Physiologic uptake

Early CS vs. Normal Variant • Active Inflammation • Normal perfusion with focal anterolateral FDG uptake

Progressive CS • Active Inflammation • Focal inferolateral perfusion defect with matching FDG uptake

Advanced CS • Irreversible fibrosis • Inferoseptal perfusion defect with no FDG uptake

FIGURE 26-3  Cardiac sarcoid PET patterns of perfusion and 18F-FDG uptake. (Reproduced with permission from Kadkhodayan A, Chareonthaitawee P, Raman SV, et al. Imaging if inflammation in unexplained cardiomyopathy. J Am Coll Cardiol Cardiovasc Imaging. 2016;9(5):603–617.)

sarcoidosis have a two-fold higher risk of malignancy;15 hence, the whole-body images are evaluated also for coexisting undiagnosed malignancy. Knowledge of the normal biodistribution of 18F-FDG is essential to understand normal variants and identify pathological uptake on the whole-body images.

Artifacts and Pitfalls The major pitfall in the interpretation of 18F-FDG PET studies for CS is the lack of specificity of the technique. As mentioned above, the test/retest repeatability of 18F-FDG PET scans for CS is suboptimal.12 In addition, we have shown that the use of “normalized” image review can lead to oversensitivity in the diagnosis of CS and potentially unneeded treatment with immunosuppression and/or implantable cardioverter

CH26.indd 503

defibrillator (ICD) implantations.20 Therefore, it is imperative that patients are adequately prepped in order to maximize suppression of physiological glucose utilization by normal myocardium (Fig. 26-1). Diffuse myocardial 18F-FDG uptake is more likely due to insufficient suppression of physiological uptake rather than CS. Likewise, focal increase in 18 F-FDG uptake in the lateral wall, especially with normal perfusion in the lateral wall, likely represents a normal variant16 and should likely not be used as the sole criterion for the diagnosis or management of CS.

▶▶Reporting The report should include clinical presentation and absence/presence of extracardiac sarcoidosis.

15-02-2022 10:53:15

504

Section 4  Beyond Perfusion Imaging

It is also useful to include the results of prior studies such as CMR or prior 18F-FDG PET/CT with quantification if applicable. It is important to include a dedicated section for the FDG PET acquisition protocol. This section includes the dose of 18F-FDG and postinjection imaging time and dose and time of rest perfusion radiotracer injection. Information should also document the fasting time and diet information, diabetes status, and fasting glucose level. In the interpretation section, describe the quality of the study including adequacy of suppression of physiological glucose utilization and scan artifacts. Areas of cardiac uptake should be noted and should list the LV blood pool and cardiac SUVmax, cardiac metabolic volume, and activity calculated on fused hybrid FDG/CT viewing software, followed by the final sarcoid interpretation (normal, abnormal, and nondiagnostic/diffuse). Extracardiac FDG uptake should also be reported. Finally, the results of perfusion (including LV size, RV size, tracer uptake, and perfusion defects), resting myocardial blood flow if available, gated PET findings (including rest LVEF, LV volumes, wall motion, and RV function), are described. The final impression summarizes the above, concluding that the study is normal or abnormal consistent with active cardiac sarcoid, scar or active sarcoid, and scar. If possible, the probability of CS (no CS 90%) should be reported and if applicable, recommendations for further imaging (CMR) or histologic evaluation.

▶▶Indications for 18F-FDG PET/CT Diagnosis of Cardiac Sarcoidosis While the most definitive diagnosis of CS is based on a positive EMB, this is infrequently effective in everyday practice. Cardiac biopsy involves sampling the right ventricular aspect of the interventricular septum, but in CS, different parts of the myocardium may harbor granulomas in different stages of inflammation, fibrosis, or both inflammation and fibrosis. This focal nature makes the

CH26.indd 504

sensitivity of biopsy low due to sampling error and biopsy precludes assessment of disease activity in the whole heart.21,22 Hence, increased focal uptake of 18F-FDG PET/CT may help elucidate a probabilistic23 (highly probable, >90% likelihood of organ involvement, probable, 50–90% ;possible, 120 to 200 mg/dL based on various guidelines54,55 should be rescheduled if possible, as altered biodistribution from hyperglycemia could decrease the specificity of 18F-FDG PET/CT. Adequate suppression of myocardial FDG uptake is important in the evaluation of suspected cardiac valve infection, as endogenous myocardial uptake may mask pathological FDG uptake in adjacent valvular tissue.

Reporting The report should include the indication, the date of surgery/device implantation and medications such as antibiotics or corticosteroids, technique (including type and dose of radiopharmaceuticals), the quality of the study, 18F-FDG PET/CT cardiac and extracardiac findings, as well as a final impression (presence of suspicious 18F-FDG uptake, any unexpected findings).44

Alternative Nuclear Approaches for Imaging of Inflammation and Infection Scintigraphy with Labeled Autologous White Blood Cells (WBC scan) WBC scintigraphy or WBC scans are commonly used imaging methods for detecting infection and inflammation. It detects tagged (radiolabeled) WBC that are migrating to the site of infection. In many situations, it is considered the gold standard, however, it has several shortcomings. Foremost, it requires careful

CH26.indd 513

513

patient selection. For example, it cannot be used in patients with agranulocytosis as one of the requirements is to have at least 2000 leukocytes (cells/uL), making it unsuitable for chronic or opportunistic infections. Additionally, the uptake or phagocytosis of the radiolabeled WBCs in the bone marrow can make it difficult to distinguish between reactive and infected marrow. From the technical standpoint, it is a time-consuming protocol that requires direct manipulation of blood products, specifically, extraction of WBC from the patient’s whole blood sample. This is followed by incubation with a radiotracer (usually either 99mTc or 111-indium) and washing to remove excess tracer. The WBCs are then reinjected into the patient prior to imaging, exposing both the patient and medical personnel to infections throughout the process.56,57 Gallium-67 Scintigraphy Gallium-67 (67Ga) is a cyclotron produced tracer that has been used for localizing infection and inflammation for more than three decades. It is an iron analogue that binds to lactoferrin, via transferrin, which is present in high concentration in inflammatory tissue/polymorphonuclear leukocytes. It emits multiple (4) levels of energy: 70 keV, 190 keV, 200 keV, and 394 keV. Because of significant scatter effect unless using a high energy collimator, higher activity cannot be administered given the poor dosimetry of this tracer. As stated in the earlier sections of this chapter, it has been used to image sarcoidosis and it is part of the JMHW criteria for imaging of CS. Because of its low specificity (~50% at most) and variable specificity (60−90%), in addition to its poor imaging characteristics, and high radiation dose, it has largely fallen out of favor.58,59

Conclusions The main strength of 18F-FDG PET/CT is its high sensitivity to detect inflammation and infection, while its main limitation is its limited specificity. It is not useful in the early postoperative period (4–8 weeks) due to expected postoperative inflammation that could lead to false-positive findings on 18F-FDG PET/CT. Certain types of devices and grafts can also cause false-positive results and low-grade infections can be missed. Development of more specific

15-02-2022 10:54:12

514

Section 4  Beyond Perfusion Imaging

radiotracers for imaging infection60 will be useful to improve the specific diagnosis of device infection.

REFERENCES 1. Avril N. GLUT1 Expression in Tissue and (18)F-FDG Uptake. J Nucl Med. 2004;45:930. 2. Chareonthaitawee P, Beanlands RS, Chen W, et al. Joint SNMMI–ASNC Expert Consensus Document on the Role of 18F-FDG PET/CT in Cardiac Sarcoid Detection and Therapy Monitoring. J Nucl Med. 2017;58:1341–1353. 3. Judson MA, Costabel U, Drent M, et al. The WASOG sarcoidosis organ assessment instrument: An update of a previous clinical tool. Sarcoidosis Vasculitis and Diffuse Lung Diseases. 2014;31:19–27. 4. Hiraga H, Yuwai K, Hiroe M. Diagnostic standard and guidelines for sarcoidosis. Jpn J Sarcoidosis Granulomatous Disord. 2007;27:89–102. 5. Terasaki F, Yoshinaga K. New guidelines for diagnosis of cardiac sarcoidosis in Japan. Ann Nucl Cardiol. 2017;3: 42–45. 6. Birnie DH, Sauer WH, Bogun F, et al. HRS expert consensus statement on the diagnosis and management of arrhythmias associated with cardiac sarcoidosis. Heart Rhythm. 2014;11:1305–1323. 7. Hulten E, Aslam S, Osborne M, Abbasi S, Bittencourt MS, Blankstein R. Cardiac sarcoidosis-state of the art review. Cardiovasc Diagn Ther. 2016;6:50–63. 8. Vita T, Okada DR, Veillet-Chowdhury M, et al. Complementary value of cardiac magnetic resonance imaging and positron emission tomography/computed tomography in the assessment of cardiac sarcoidosis. Circ Cardiovasc Imaging. 2018;11:e007030. 9. Demeure F, Hanin FX, Bol A, et al. A randomized trial on the optimization of 18F-FDG myocardial uptake suppression: implications for vulnerable coronary plaque imaging. J Nucl Med: Official Publ, Soc Nucl Med. 2014;55:1629–1635. 10. Morooka M, Moroi M, Uno K, et al. Long fasting is effective in inhibiting physiological myocardial 18F-FDG uptake and for evaluating active lesions of cardiac sarcoidosis. EJNMMI Research. 2014;4:1. 11. Atterton-Evans V, Turner J, Vivanti A, Robertson T. Variances of dietary preparation for suppression of physiological 18 F-FDG myocardial uptake in the presence of cardiac sarcoidosis: A systematic review. J Nucl Cardiol. 2020;27:481–489. 12. Alvi RM, Young BD, Shahab Z, et al. Reproducibility and optimization of 18-fluuoro-deoxyglucose positron emission tomography for evaluation of cardiac sarcoidosis. J Am Coll Cardiol. 2019;73:1659. 13. Christopoulos G, Jouni H, Acharya GA, et al. Suppressing physiologic 18-fluorodeoxyglucose uptake in patients undergoing positron emission tomography for cardiac sarcoidosis: The effect of a structured patient preparation protocol. J Nucl Cardiol: Official Publ Am Soc Nucl Cardiol. 2021;28(2): 661–667. 14. Osborne MT, Hulten EA, Murthy VL, et al. Patient preparation for cardiac fluorine-18 fluorodeoxyglucose positron emission tomography imaging of inflammation. J Nucl Cardiol. 2017;24:86–99.

CH26.indd 514

15. Bonifazi M, Bravi F, Gasparini S, et al. Sarcoidosis and cancer risk: systematic review and meta-analysis of observational studies. Chest. 2015;147:778–791. 16. Blankstein R, Osborne M, Naya M, et al. Cardiac positron emission tomography enhances prognostic assessments of patients with suspected cardiac sarcoidosis. J Am Coll Cardiol. 2014;63:329–336. 17. Ahmadian A, Brogan A, Berman J, et al. Quantitative interpretation of FDG PET/CT with myocardial perfusion imaging increases diagnostic information in the evaluation of cardiac sarcoidosis. J Nucl Cardiol. 2014;21:925–939. 18. Abikhzer G, Martineau P, Gregoire J, Finnerty V, Harel F, Pelletier-Galarneau M. FDG-PET CT for the evaluation of native valve endocarditis. J Nucl Med. 2020;61:645. 19. Kadkhodayan A, Chareonthaitawee P, Raman SV, Cooper LT. Imaging of Inflammation in Unexplained Cardiomyopathy. JACC: Cardiovasc Imaging. 2016;9:603–617. 20. Varghese M, Smiley D, Bellumkonda L, Rosenfeld LE, Zarret B, Miiller E. Quantitative interpretation of FDG PET for cardiac sarcoidosis reclassifies visually interpreted exams and potentially impacts downstream interventions. Sarcoidosis Vasc Diffuse Lung Dis. 2018;35:342–353. 21. Bennett MK, Gilotra NA, Harrington C, et al. Evaluation of the Role of Endomyocardial Biopsy in 851 Patients With Unexplained Heart Failure From 2000–2009. Circulation: Heart Failure. 2013;6:676–684. 22. Uemura A, Morimoto S-I, Hiramitsu S, Kato Y, Ito T, Hishida H. Histologic diagnostic rate of cardiac sarcoidosis: Evaluation of endomyocardial biopsies. Am Heart J. 1999;138: 299–302. 23. Judson MA, Costabel U, Drent M, et al. The WASOG Sarcoidosis Organ Assessment Instrument: An update of a previous clinical tool. Sarcoidosis Vasc Diffuse Lung Dis. 2014;31:19–27. 24. Sharp MB, Soufer A, Abdelmessih M, Rosenfeld LE, Baldassarre LA, Miller EJ. Using FDG-PET to guide targeted cardiac magnetic resonance imaging in patients with suspected cardiac sarcoidosis. J Nucl Cardiol. 2020;27:688–690. 25. Slart RHJA, Glaudemans AWJM, Lancellotti P, et al. A joint procedural position statement on imaging in cardiac sarcoidosis: from the Cardiovascular and Inflammation & Infection Committees of the European Association of Nuclear Medicine, the European Association of Cardiovascular Imaging, and the American Society of Nuclear Cardiology. J Nucl Cardiol. 2018;25:298–319. 26. Writing group; Document reading group; EACVI Reviewers. A joint procedural position statement on imaging in cardiac sarcoidosis: from the Cardiovascular and Inflammation & Infection Committees of the European Association of Nuclear Medicine, the European Association of Cardiovascular Imaging, and the American Society of Nuclear Cardiology. Eur Heart J Cardiovasc Imaging. 2017;18:1073–1089. 27. Tahara N, Tahara A, Nitta Y, et al. Heterogeneous myocardial FDG uptake and the disease activity in cardiac sarcoidosis. JACC Cardiovasc Imaging. 2010;3:1219–1228. 28. Ahmadian A, Pawar S, Govender P, Berman J, Ruberg FL, Miller EJ. The response of FDG uptake to immunosuppressive treatment on FDG PET/CT imaging for cardiac sarcoidosis. J Nucl Cardiol: Official Publ Am Soc Nucl Cardiol. 2017;24: 413–424. 29. Osborne MT, Hulten EA, Singh A, et al. Reduction in 18F-fluorodeoxyglucose uptake on serial cardiac positron emission

15-02-2022 10:54:12

Chapter 26  18F-FDG PET/CT for Imaging Cardiac Sarcoidosis and Inflammation tomography is associated with improved left ventricular ejection fraction in patients with cardiac sarcoidosis. J Nucl Cardiol. 2014;21:166–174. 30. Yazaki Y, Isobe M, Hayasaka M, Tanaka M, Fujii T, Sekiguchi M. Cardiac sarcoidosis mimicking hypertrophic cardiomyopathy: clinical utility of radionuclide imaging for differential diagnosis. Jpn Circ J. 1998;62:465–468. 31. Bravo PE, Raghu G, Rosenthal DG, et al. Risk assessment of patients with clinical manifestations of cardiac sarcoidosis with positron emission tomography and magnetic resonance imaging. Int J Cardiol. 2017;241:457–462. 32. Greulich S, Deluigi CC, Gloekler S, et al. CMR imaging predicts death and other adverse events in suspected cardiac sarcoidosis. JACC: Cardiovasc Imaging. 2013;6:501–511. 33. Dilsizian V, Bacharach SL, Beanlands RS, et al. ASNC imaging guidelines/SNMMI procedure standard for positron emission tomography (PET) nuclear cardiology procedures. J Nucl Cardiol: Official Publ Am Soc Nucl Cardiol. 2016;23:1187–1226. 34. Habib G, Lancellotti P, Antunes MJ, et al. 2015 ESC Guidelines for the management of infective endocarditis: The Task Force for the Management of Infective Endocarditis of the European Society of Cardiology (ESC). Endorsed by: European Association for Cardio-Thoracic Surgery (EACTS), the European Association of Nuclear Medicine (EANM). Eur Heart J. 2015;36:3075–3128. 35. Ricciardi A, Sordillo P, Ceccarelli L, et al. 18-Fluoro-2-deoxyglucose positron emission tomography-computed tomography: an additional tool in the diagnosis of prosthetic valve endocarditis. Int J Infect Dis. 2014;28:219–224. 36. Saby L, Laas O, Habib G, et al. Positron emission tomography/ computed tomography for diagnosis of prosthetic valve endocarditis: increased valvular 18F-fluorodeoxyglucose uptake as a novel major criterion. J Am Coll Cardiol. 2013;61:2374–2382. 37. Sarrazin JF, Philippon F, Tessier M, et al. Usefulness of fluorine-18 positron emission tomography/computed tomography for identification of cardiovascular implantable electronic device infections. J Am Coll Cardiol. 2012;59:1616–1625. 38. Spacek M, Belohlavek O, Votrubova J, Sebesta P, Stadler P. Diagnostics of "non-acute" vascular prosthesis infection using 18F-FDG PET/CT: our experience with 96 prostheses. Eur J Nucl Med Mol Imaging. 2009;36:850–858. 39. Granados U, Fuster D, Pericas JM, et al. Diagnostic accuracy of 18F-FDG PET/CT in infective endocarditis and implantable cardiac electronic device infection: a cross-sectional study. J Nucl Med: Official Publ, Soc Nucl Med. 2016;57:1726–1732. 40. Duval X, Le Moing V, Tubiana S, et al. Impact of systematic whole-body 18F-fluorodeoxyglucose PET/CT on the management of patients suspected of infective endocarditis: The Prospective Multicenter TEPvENDO Study. Clin Infect Dis. 2020. 41. Li JS, Sexton DJ, Mick N, et al. Proposed modifications to the Duke criteria for the diagnosis of infective endocarditis. Clin Infect Dis. 2000;30:633–638. 42. Habib G, Thuny F, Avierinos JF. Prosthetic valve endocarditis: current approach and therapeutic options. Prog Cardiovasc Dis. 2008;50:274–281. 43. Pizzi MN, Roque A, Fernández-Hidalgo N, et al. Improving the diagnosis of infective endocarditis in prosthetic valves and intracardiac devices with 18F-fluordeoxyglucose positron emission tomography/computed tomography angiography: Initial Results at an Infective Endocarditis Referral Center. Circulation. 2015;132:1113–1126.

CH26.indd 515

515

44. Nuvoli S, Fiore V, Babudieri S, et al. The additional role of 18F-FDG PET/CT in prosthetic valve endocarditis. Eur Rev Med Pharmacol Sci. 2018;22:1744–1751. 45. de Camargo RA, Sommer Bitencourt M, et al. The role of 18F-fluorodeoxyglucose positron emission tomography/ computed tomography in the diagnosis of left-sided endocarditis: native vs prosthetic valves endocarditis. Clin Infect Dis. 2020;70:583–594. 46. Harrison JL, Prendergast BD, Sandoe JA. Guidelines for the diagnosis, management and prevention of implantable cardiac electronic device infection. Heart. 2015;101:250–252. 47. Ploux S, Riviere A, Amraoui S, et al. Positron emission tomography in patients with suspected pacing system infections may play a critical role in difficult cases. Heart Rhythm. 2011;8:1478–1481. 48. Chen W, Dilsizian V. Is F-flurodeoxyglucose positron emission tomography/computed tomography more reliable than clinical standard diagnosis for guiding patient management decisions in cardiac implantable electronic device infection? Circulation: Cardiovasc Imaging. 2019;12:e009453. 49. Dell’Aquila AM, Mastrobuoni S, Alles S, et al. Contributory role of fluorine 18-fluorodeoxyglucose positron emission tomography/computed tomography in the diagnosis and clinical management of infections in patients supported with a continuous-flow left ventricular assist device. Ann Thorac Surg. 2016;101:87–94; discussion 94. 50. Valentine RJ. Diagnosis and management of aortic graft infection. Semin Vasc Surg. 2001;14:292–301. 51. Keidar Z, Pirmisashvili N, Leiderman M, Nitecki S, Israel O. 18F-FDG uptake in noninfected prosthetic vascular grafts: incidence, patterns, and changes over time. J Nucl Med: Official Publ, Soc Nucl Med. 2014;55:392–395. 52. Berger P, Vaartjes I, Scholtens A, et al. Differential FDGPET uptake patterns in uninfected and infected central prosthetic vascular grafts. Eur J Vasc Endovasc Surg. 2015;50:376–383. 53. Zhuang H, Alavi A. 18-fluorodeoxyglucose positron emission tomographic imaging in the detection and monitoring of infection and inflammation. Semin Nucl Med. 2002;32:47–59. 54. Jamar F, Buscombe J, Chiti A, et al. EANM/SNMMI Guideline for 18F-FDG Use in Inflammation and Infection. J Nucl Med. 2013;54:647–658. 55. Surasi DS, Bhambhvani P, Baldwin JA, Almodovar SE, O’Malley JP. 18F-FDG PET and PET/CT patient preparation: a review of the literature. J Nucl Med Technol. 2014;42:5–13. 56. Love C, Palestro CJ. Radionuclide imaging of infection. J Nucl Med Tech. 2004;32:47–57. 57. Polvoy I, Flavell RR, Rosenberg OS, Ohliger MA, Wilson DM. Nuclear imaging of bacterial infection: The State of the Art and Future Directions. J Nucl Med. 2020;61:1708–1716. 58. Keijsers RGM, van den Heuvel DAF, Grutters JC. Imag ing the inflammatory activity of sarcoidosis. Eur Respir J. 2013;41:743–751. 59. Malinowska E, Doboszyńska A, Śliwińska A, et al. The use of 67Ga scintigraphy in patients with sarcoidosis. Nucl Med Rev. 2018;21:59–65. 60. Tsopelas C. Radiotracers used for the scintigraphic detection of infection and inflammation. ScientificWorldJ. 2015;2015:676719–676719.

15-02-2022 10:54:12

This page intentionally left blank

READ_THINK_ITALIAN.indd 2

11/19/21 1:12 PM

Hybrid Imaging: SPECT–CT and PET–CT Cory Henderson, Patrycja Galazka and Sharmila Dorbala

KEY POINTS ■■

■■

■■

■■

Hybrid single-photon emission computed tomography and computed tomography (SPECT–CT) and positron emission tomography computed tomography (PET–CT) imaging have grown tremendously over the last two decades. Depending upon the equipment, the CT component of SPECT/CT and PET/CT images is useful for attenuation correction, identification of coronary artery calcification, CT coronary angiography, or localiza­tion of  radiotracer uptake in radiopharmaceutically avid images. Coronary artery calcium score when combined with a myocardial perfusion imaging is valuable for the management of patients with suspected ischemic heart disease. Cardiac PET/MR provides simultaneous PET and MR acquisitions and is currently used primarily for research applications.

BACKGROUND There has been significant growth in hybrid singlephoton emission computed tomography and computed tomography (SPECT–CT) and positron emission tomography computed tomography (PET–CT)

ch27.indd 517

CHAPTER

27

sys­tems over the past decade, driven in large part by oncologic imaging. A fortuitous byproduct of this has been the development, application, and validation of myocardial perfusion imaging (MPI) using these hybrid systems. Cardiac hybrid SPECT–CT and PET–CT systems offer distinct advantages compared with traditional SPECT or PET MPI. The CT provides for excellent attenuation correction and improves the specificity of MPI, CT-derived coronary artery calcium (CAC) score adds substantial incremental diagnostic and prognostic information to MPI, and hybrid scanners offer the ability to combine a physiologic assessment of perfusion, function, or metabolism with an anatomic assessment of atherosclerosis and structural heart disease. Hybrid SPECT–CT and PET–CT imaging also offers unprecedented opportunities for molecular cardiology research. The primary focus of this chapter is to discuss the clinical applications of hybrid radionuclide MPI with calcium scoring and coronary CTA.

SPECT–CT AND PET–CT The hardware of SPECT–CT and PET–CT scanners comprises a conventional SPECT scanner or a PET scanner coupled with a CT scanner of various configurations. While all SPECT–CT and PET–CT scanners offer CT-based attenuation correction, calcium scoring (≥4 slice MDCT) and coronary CTA (≥64 slice MDCT) may be performed only on certain hybrid SPECT–CT and PET–CT scanners. Sample hybrid PET–CT and SPECT–CT protocols

15-02-2022 17:17:50

518

Section 4  Beyond Perfusion Imaging Rubidium-82 or N13-ammonia

A

Positioning scan and CTAC

CAC*

Rest images

Rest images

Rubidium-82 or N13-ammonia

Pharmacological stress infusion

Stress test

Technetium-99m

B

Stress images

lodinated CT contrast

CTAC*

CT coronary angiogram

Stress images Technetium-99m

Positioning scan and CTAC

CAC

Rest images

Rest images

Exercise/ pharmacological stress

Stress test

Positioning scan and CTAC

lodinated CT contrast

Stress images

CT coronary angiogram

Stress images

FIGURE 27-1  Sample protocols for (A) PET/CT and (B) SPECT/CT myocardial perfusion imaging. CTAC, CT for AC (10 mA, 120 keV, nongated free breathing); CAC, calcium score CT scan (300 mA, 140 keV, ECG-gated CT scan with breath hold); *, optional.

are shown in Figure 27-1A and B, respectively. The CAC score and/or coronary CTA study can be performed sequentially right before or after the SPECT or PET scan or at a separate setting.

ATTENUATION CORRECTON WITH CT FOR BOTH SPECT AND PET Attenuation correction using transmission scanning employing external radioactive sources or cardiac CT improves the count uniformity of the image and helps distinguish attenuation artifacts from real defects. It also offers the possibility of stress-only imaging, with potential savings of time, cost, and radiation dose. Also, accurate attenuation correction enables precise measurements of absolute radiotracer concentration in the myocardium making feasible noninvasive quantitative estimation of myocardial blood flow in mL/g/min. CT attenuation correction is rapid (takes a few seconds) and of excellent quality. SPECT–CT and PET–CT utilize a low-dose x-ray transmission computed tomogram for attenuation correction (CTAC). While of considerable clinical value, alignment of the emission (SPECT or PET) into the transmission (CT) map is critical. If not aligned, the SPECT or PET myocardial perfusion images may be misregistered resulting in artifacts and potentially false-positive interpretation. Accurate registration is critical for improving the diagnostic yield of CT attenuationcorrected MPI (Fig. 27-2A and B).

ch27.indd 518

SPECT–CT MPI has been validated in patients with and without underlying CAD.1,2 Multiple studies of SPECT MPI with radionuclide attenuation correction compared with nonattenuation-corrected images demonstrated that test specificity and normalcy are improved while maintaining sensitivity to detect obstructive CAD.3–6 Of clinical importance, attenuation correction of MPI increases the normalcy rate, a term used to define the percentage of normal studies in a low-risk cohort. Figure 27-3 demonstrates the effect of attenuation correction using a hybrid SPECT–CT system. Prior publications have also demonstrated the improved prognostic capability of attenuation-corrected SPECT MPI.7,8 Due to the distance between the emission data and camera heads (generally 3 feet), PET MPI with attenuation correction has been mandatory clinically. PET MPI without attenuation correction is significantly degraded by attenuation and scatter and cannot be interpreted clinically or for research purposes.9 Thus, all PET studies are interpreted with attenuation-corrected images and generally are free of attenuation artifact. While SPECT studies can be interpreted without AC (although with less specificity), AC is critical for cardiac PET. Multiple studies demonstrated the excellent ­diagnostic10 and prognostic value11 of PET MPI, all using attenuation corrected images. Furthermore, noninvasive quantitative assessment of myocardial blood flow with PET has emerged as a powerful tool to diagnose microvascular dysfunction, follow progression or

15-02-2022 17:17:51

Chapter 27  Hybrid Imaging: SPECT–CT and PET–CT

A

519

B

FIGURE 27-2  (A and B) The top panel demonstrates stress and rest rubidium-82 myocardial perfusion images. The bottom panel shows the overlay of the CT transmission and rubidium-82 emission images. In A, the emission images demonstrate a reversible anterolateral myocardial perfusion defect, and the fusion images demonstrate that the stress transmission and emission images are misregistered. Using software, the transmission and emission images were realigned and the images reconstructed with a new appropriately registered attenuation map, resulting in normal myocardial perfusion images (B).

FIGURE 27-3  SPECT–CT myocardial perfusion images demonstrating a fixed defect involving the entire inferior wall on the nonattenuation-corrected images (NAC, first two rows of images in each view) that resolved with AC (CTAC, third and fourth row of images in each view) suggesting diaphragmatic attenuation on the NAC images.

ch27.indd 519

15-02-2022 17:17:54

520

Section 4  Beyond Perfusion Imaging

regression of CAD, and identify localized ischemia, transplant vasculopathy, and balanced ischemia.9 Absolute myocardial blood flow and coronary flow reserve assessed by PET accurately predicts future adverse cardiovascular outcomes; in particular, a

normal coronary flow reserve offers excellent negative predictive value to exclude high-risk CAD.12–15 Myocardial blood flow with PET is discussed in Chapter 11. Attenuation corrected data are critical to the success of myocardial blood flow (Fig. 27-4).

B

A

C

FIGURE 27-4  Dipyridamole stress and rest rubidium-82 PET–CT myocardial perfusion images demonstrate a large-sized and severe perfusion defect in the entire anteroseptal wall, the mid and apical anterior walls, and LV apex that was reversible, consistent with reversibility in the left anterior descending artery (A). In addition, there was a medium-sized and severe perfusion defect in the entire inferior wall and the basal inferoseptal wall that was reversible consistent with reversibility in posterior descending artery. There was transient ischemic dilation of the left ventricle (T.I.D. ratio = 1.3) and a rest left ventricular ejection fraction of 26% that decreased to 21% during peak stress (high-risk features). Polar plots of perfusion are shown in (B). Coronary angiography demonstrated severe disease in the right coronary artery, left anterior descending artery, and left circumflex artery (C).

ch27.indd 520

15-02-2022 17:17:57

Chapter 27  Hybrid Imaging: SPECT–CT and PET–CT

CORONARY ARTERY CALCIFICATION AND CORONARY CT ANGIOGRAPHY Advances in multidetector row CT scanners have tremendously improved the noninvasive diagnosis of CAD using CAC score, coronary CTA, combined imaging of myocardial perfusion, and anatomy,16 as well as CT-derived estimates of fractional flow reserve (CT-FFR).17 For attenuation correction, lowresolution CT (non-diagnostic CT), two-slice CT or ≥4-slice multidetector-row CT-based hybrid scanners can be used; however, if CAC scoring is desired, at least four-slice CT is required. For coronary CTA, at least a 64-slice multidetector-row CT is recommended with imaging capability for slice width of 0.4 to 0.6 mm and temporal resolution of 500 ms or less (≤350 ms is preferred).18 Coronary artery calcification is a specific marker of coronary atherosclerosis and a powerful indicator of increased cardiovascular risk.19 CAC score in addition to MPI can be of great value both to the clinician and the patient and stimulate further discussions regarding patient’s risk factor management. Likewise, extensive literature supports the role of coronary CTA in the diagnosis and management of individuals with known or suspected CAD.20,21 Extensive coronary artery calcification, high or irregular heart rates, and high body mass index may limit the diagnostic accuracy of coronary CTA; evaluation for ischemia may provide incremental diagnostic value in those instances. Furthermore, revascularization decisions guided by functional testing with SPECT MPI22 or invasive FFR23 may improve clinical outcomes.

HYBRID CORONARY COMPUTED TOMOGRAPHY AND RADIONUCLIDE IMAGING

▶▶Hybrid CAC Score and MPI The CAC score is a robust tool that has been shown in multiple studies to predict the risk of major cardiovascular outcomes, especially in patients with no known CAD (primary prevention), and as a result help guide clinical decision making and preventative medical therapy.24 For MPI studies performed using

ch27.indd 521

521

hybrid SPECT/CT and PET/CT systems, a low-dose CT is obtained, along with the MPI, for the purposes of attenuation correction. This low-dose CT can be used to qualitatively visually assess coronary artery calcifications; however, as this CT is nongated, can be acquired during different points of the respiratory cycle and can be reconstructed at various slice thicknesses (per center protocol), it is generally not suitable for quantitative calcium score measurement. With that being said, qualitative CAC assessment correlates relatively well with severity as assessed by quantitative calcium score measurements.25,26 In a recent observational study, 5528 patients, in a single hospital system, without a history of CAD and normal troponin, underwent PET MPI. The presence of coronary calcium was visually assessed using the lowdose attenuation correction CT and the patients were divided based on the absence of CAC (N = 3018) and presence of CAC (N = 2510). The CAC absent group was less likely to undergo coronary angiography (3.4% vs. 10.2%), have high-grade CAD (0.5% vs. 6.5%), and undergo revascularization (0.4% vs. 5.8%) within 90 days of presentation in comparison to the CAC present group. In patients with more than 10% ischemic burden on MPI, the patients in the CAC absent group had less high-grade CAD (24% vs. 73.2%) and lower rates of revascularization (22% vs. 67.3%) compared to the CAC present group. In addition, the CAC absent group had lower longer-term MACE (2.4% vs. 6.9%).27 This study highlights the incremental prognostic value provided by CAC in patients with no history of CAD. For a more robust, quantitative assessment of CAC, an additional CT acquisition can be performed during the MPI, using a coronary calcium score protocol (prospective ECG gating, breath hold, 3-mm slice reconstruction). Several studies wherein subjects underwent both a CAC score study and MPI (at the same setting or at different settings) have demonstrated that subjects with normal MPI may have extensive underlying calcified atherosclerosis, and this finding may influence physicians to prescribe aspirin and lipid-lowering agents.28 The frequency of ischemia in subjects with Agatston calcium score of greater than 400 is high (>20%);29–33 a myocardial perfusion study is considered appropriate among individuals with CAC score greater than 400 or among individuals with high CHD risk and CAC score 100 to 400 independent of

15-02-2022 17:17:57

522

Section 4  Beyond Perfusion Imaging

symptoms.34 As with CAC score, investigators35 have evaluated the diagnostic and prognostic value of a zero CAC score in conjunction with stress SPECT MPI in patients presenting to the emergency room with chest pain. In that study, 0.8% of patients had an abnormal MPI (5/625 patients, four of whom had no CAD on subsequent invasive angiography) and 0.3 event rate (mildly elevated troponin, no cardiac death) over a mean follow up of 7 months. Nabi et al.35 concluded that most of the patients with chest pain in the ED have a calcium score of 0, which predicts both a normal stress SPECT result and an excellent short-term outcome. A meta-analysis by Bavishi et al. (Table 27-1) ref 36 as well as research by Chang et al. (Table 27-2) ref 39 confirmed that zero to low CAC scores were infrequently associated with ischemia, but there was a wide variance in the frequency of ischemia among patients with intermediate-to-high CAC scores. The broad range of CAC scores29–33 among patients with both normal and abnormal perfusion scans in multiple studies support the concept that presence of calcified coronary atherosclerosis does not necessarily predict ischemia. However, in subjects without overt CAD, the degree of CAC showed an inverse relationship to peak hyperemic myocardial blood flow and coronary flow reserve and direct relation to coronary vascular resistance.55–58 Based on the aforementioned information, although the presence of coronary calcium does not appear to predict ischemia, the degree of CAC does appear to have a hemodynamic effect on stress myocardial blood flow and coronary flow reserve. In patients with normal MPI

and abnormal stress myocardial blood flow/coronary flow reserve, the presence of significant coronary calcifications is likely the cause of reduced blood flow. Conversely, in patients with normal perfusion and no or minimal coronary artery calcifications, abnormal myocardial blood flow would most likely be due to microvascular dysfunction, although, noncalcified coronary artery atherosclerosis, not detected on calcium scores, could also be a contributing factor. The results of these studies suggest that CAC score and coronary microvascular function may provide biologically different information and could be complementary for risk assessment. When combined with SPECT or PET MPI, CAC score provides independent and complementary information about risk of death or myocardial infarction.32,33 In a previous SPECT study, including a lower-risk cohort with some asymptomatic subjects, and normal MPI, calcium scores did not show an incremental prognostic value over MPI over a mean follow up of 32 months.48 In contrast, with a longer follow up (mean follow up of 6.9 years), Chang et al.32 demonstrated that individuals with high calcium score had greater annualized cardiac event rates (3%), despite a normal MPI (Fig. 27-5). The addition of a CAC score to 82Rb perfusion data provided incremental prognostic information in patients with both ischemic and nonischemic perfusion studies.50 These combined data support the concept that while a normal relative MPI may indicate excellent short-term prognosis, a high CAC score may indicate a worse intermediate-term prognosis despite a normal MPI.

Table 27-1 Pooled Prevalence and Odds Ratio for Ischemia by CAC Categories CAC Categories

Patients (n)

Pooled Prevalence of Ischemia (%)

Range of Ischemia (%)

Pooled Odds Ratio (95% CI)

0

487

6.6

0.0–24.1

Reference

1–100

529

8.5

2.1–50.0

1.7 (1.04–8.2)

101–399

513

10.5

4.0–63.6

3.3 (1.4–8.2)

≥400

594

23.6

12.4–57.1

6.9 (3.5–13.4)

Reproduced with permission Bavishi C, Argulian E, Chatterjee S, et al. CACS and the frequency of stress-induced myocardial ischemia during MPI: A meta-analysis. JACC Cardiovasc Imaging. 2016;9(5):580–589.

ch27.indd 522

15-02-2022 17:17:57

ch27.indd 523

2003 2005 2008 2007

Moser45

Nishida46

Piers47

Ramakrishna33

2007 2008

Schepis51

52

2004

Yao54 73

35

100

77

621

126

1153

835

531

 83

102

703

411

462

157

 80

946

260

180

220

58 ± 10

55 ± 10

55 ± 11

68

NR

59 ± 9

53 ± 8

63 ± 11

64 ± 10

62 ± 15

58± 9

51 ± 8

53 ± 8

Suspected CAD

Renal transplant

Asymptomatic, type II DM

Suspected CAD patients

Suspected CAD patients

53 ± 11

37 ± 11 67

53 ± 10

66 ± 9

61 ± 11

Asymptomatic, suspected CAD 59 ± 11

Suspected CAD

Suspected CAD

Suspected CAD

Symptomatic, suspected CAD

Asymptomatic, suspected CAD

Suspected CAD

Suspected CAD

Known or suspected CAD

Suspected CAD

Symptomatic, suspected CAD

Suspected CAD

Siblings of premature CAD

Asymptomatic, type II DM

Asymptomatic, suspected CAD 56 ± 11

Age (yrs)

NR

67

65

62

40

67

74

77

57

66

NR

79

69

68

60

39

75

38

61

72

NR

81

51

73

74

39

38

42

57

63

NR

48

39

53

18

80

50

60

22

35

0, >0

NR NR 0, 1−10, 11−100, 101−399, ≥400

6 NR 101−400, ≥400

100 53 0, 1−10, 11−100, 101−400, 401−1000, ≥1000

18 NR ≤10, 11−100, 101−400, 401−1000, ≥1000

28 54 0, 1−399, ≥400

10 72 0, 1−99, 100−399, 400−999, ≥, 1000

8 65 0, 1−9, 10−99, 100−399, 400−999, ≥1,000

14 67 0, 1−10, 11−100, 101−400, >400

12 41 0

NR NR 0−100, 101−400, ≥400

7 69 0−10, 11−100, 101−400, 401−1000, >1000

6 54 0, 1−10, 11−100, 101−399, ≥400

17 40

17 16 0, >10

30 38 0, >0

10 57 0−10, 11−100, 101−400, ≥400

14 38 0, 1−10, 11−100, 101−399, ≥400

100 62 ≤10, 11−100, 101−400, 401−1000, >1000

30 30 100−399, ≥400

Male HTN DM HL (%) (%) (%) (%) CAC Categories

42.5

17.1

23

28.8

18.3

5.6

8.3

13.9

42.2

18.6

7.4

19.7

22.9

31.8

15.5

10.9

18.8

31.7

30

Ischemia Prevalence (%)

Reproduced with permission from Bavishi C, Argulian E, Chatterjee S, et al. CACS and the frequency of stress-induced myocardial ischemia during MPI: a meta-analysis. JACC Cardiovasc Imaging. 2016;9:580–589.

2011

Seyahi53

Scholte

2008

2006

Schenker50

Rosman

49

2007

2007

Ho

Rozanski

2000

44

He

48

2013

43

2011

Ghadri42

2008

Fathala41

2015

2006

Esteves40

Chang

39

Blumenthal

2006 38

2004

Anand37

Year of Sample Publication Size Patient Characteristics

Anand31

First Author

A Listing of Studies that Reported on the Prevalence of CAC Score and Myocardial Ischemia

Table 27-2

Chapter 27  Hybrid Imaging: SPECT–CT and PET–CT

523

15-02-2022 17:17:58

Section 4  Beyond Perfusion Imaging

Annualized event rate (%)

8

Total cardiac events

8

All-cause death/MI

Normal SPECT

Normal SPECT

Abnormal SPECT

Abnormal SPECT

6

6.08

p = 0.01 for increasing 5.35 CACS (normal SPECT) 4

p < 0.001 for CACS >100 abnormal vs. normal SPECT

2.97

2

Annualized event rate (%)

524

6

p = 0.01 for increasing CACS (normal SPECT) 4

3.89

p < 0.001 for CACS >100 abnormal vs. normal SPECT 2.44

2

2.05 1.25

0.7

0 A CACS 0–10

0.97

11–100

1.1

1.3

101–400

0.59

>400

0 B CACS 0–10

11–100

101–400

>400

FIGURE 27-5  Adjusted annualized total cardiac death, MI, and coronary revascularization (A) and all-cause death/MI (B) event rates based on CACS and SPECT results. CACS, coronary artery calcium score; MI, myocardial infarction. (Reproduced with permission from Chang SM, Nabi F, Xu J, et al. The coronary artery calcium score and stress myocardial perfusion imaging provide independent and complementary prediction of cardiac risk. J Am Coll Cardiol. 2009;54(20):1872–1882.)

▶▶Hybrid Coronary CTA and MPI Imaging coronary atherosclerosis and its functional consequences with hybrid SPECT/CT and PET/CT devices in a single or sequential study is now a reality. When performed on the same day, coronary CTA is typically performed after completion of stress MPI, so that beta-blockers can be administered to slow the heart rate for the coronary CTA without affecting ischemia assessment on MPI. Radionuclide and coronary CTA images can be interpreted independently or together using one of several software options to fuse the images. The relationship between the anatomic assessment of CAD and the functional assessment of perfusion is complex. Lesion severity as assessed by coronary angiography does not account for the degree of endothelial dysfunction or the effect of serial stenoses on vascular resistance. Multiple single-center studies with coronary CTA and MPI have demonstrated the relatively poor ability of coronary CTA to predict ischemia on perfusion imaging, with a modest PPV of approximately 30%.59 Similarly, a normal myocardial perfusion scan is a relatively poor discriminator for the presence or absence of nonobstructive CAD.60 In addition, the extent of CAD can often be underestimated due to reduced heterogeneity of flow in patients with underlying CAD and concomitant endothelial dysfunction. Given the imperfect

ch27.indd 524

relationship between anatomic lesion severity and the degree of ischemia, a hybrid approach of radionuclide MPI and coronary CTA may allow for a more comprehensive characterization of CAD burden. However, both coronary CTA and MPI may not be indicated in all patients. A strategy of sequential imaging with initial MPI followed by coronary CTA (if MPI is not normal or severely abnormal) may be considered in subjects with an intermediate-to-high pretest likelihood. In contrast, an initial coronary CTA followed by MPI (unless the coronary CTA demonstrates normal arteries or critical CAD) may be a better strategy in subjects with low or low intermediate pretest likelihood of CAD. Others have proposed a coronary CTA along with stress MPI. An example of how this hybrid approach can provide useful clinical information is demonstrated in Figure 27-6. Several studies to date support the notion that both CAC score and coronary CTA offer incremental diagnostic and prognostic information to MPI. A few clinical scenarios wherein a hybrid approach of combined MPI along with coronary CTA may be helpful are: (1) to detect severe multivessel CAD in the setting of mild perfusion abnormalities when the clinical suspicion is high (balanced ischemia); (2) to diagnose microvascular dysfunction in subjects with abnormal MPI by excluding atherosclerosis; and (3) to evaluate patients with structural abnormalities

15-02-2022 17:17:59

Chapter 27  Hybrid Imaging: SPECT–CT and PET–CT

525

FIGURE 27-6  A 42-year-old female with hypertension and family history of premature coronary atherosclerosis presented with chest pain. Exercise myocardial perfusion SPECT images demonstrated a small defect of moderate intensity involving the apical septum and true apex that is completely reversible. Coronary CT angiogram reveals a noncalcified plaque involving the proximal left anterior descending artery (LAD; black arrow in axial plane, white arrow on short axis of LAD) with a severe stenosis of more than 70%. Invasive coronary angiography confirmed severe mid-LAD stenosis (bottom-left panel shows long-axis views and the right panel shows short-axis views of the mid-LAD lesion) that was stented successfully.

ch27.indd 525

15-02-2022 17:18:07

526

Section 4  Beyond Perfusion Imaging

of the coronary arteries.61 Discordant findings on MPI and coronary CTA can result from microvascular dysfunction (abnormal blood flow without obstructive epicardial CAD), calcified and nonobstructive CAD with normal perfusion or obstructive CAD that is not flow-limiting (due to hemodynamic or collateral changes), and imaging artifacts. Therefore, combined MPI and CTA can provide better characterization of the extent and severity of underlying CAD and potential benefit from revascularization than does either technique alone.18 Indeed, one recent study showed that combining stress-only

SPECT with coronary CTA in individuals presenting to the emergency room offers the added advantage of feasibility (no contraindications to SPECT), lower radiation dose when stress-only MPI was used, and higher prognostic value.62 However, the coronary CTA approach was less costly, and none of the individuals with a zero calcium score had significant CAD or cardiac event during follow up. There is also emerging evidence that diagnostic performance of SPECT or PET MPI and coronary CTA is superior to SPECT/ PET MPI or coronary CTA alone (Table 27-3).64,66,74–77 The EVINCI study,

Table 27-3 Diagnostic Accuracy of Integrated MPI and Coronary CTA: Vessel-Based Analysis in Identifying Obstructive CAD on Invasive Angiography First Author/ Year

N

Gold Standard (Definition of Significant CAD)

Sensitivity

Specificity

PPV

NPV

Hybrid Technique

Namdar63 2005

25

Flow limiting coronary stenoses requiring revascularization (ICA+PET)

90

98

82

99

13

Rispler64 2007

56

Flow limiting coronary stenoses (>50% stenosis on ICA+SPECT+)

96

95

77

99

99m Tc SPECT/ 16 slice CT

Groves65 2009

33

>50% stenosis on ICA

88

100

97

99

82

Sato66 2010

130

>50% stenosis on ICA

94

92

85

97

99m

Tc SPECT/ 64 slice CT¥

Kajandar67 2010

107

Flow limiting coronary stenoses (>50% stenosis on ICA+FFR)

93

99

96

99

15 O-water PET/ 64 slice CT

Danad68 2013

120

Flow limiting coronary stenoses, CFR (>50% stenosis on ICA+FFR)

72

89

69

90

15

O-water PET/ 64 slice CT

N-ammonia PET/4 slice CT

Rb PET/ 64 slice CT

Thomassen69 2013

44

ICA, QCA, 50%. sMBF

88

97

85

97

15 O-water PET/ 64 slice CT

Schaap70 2013µ

98

Flow limiting coronary stenoses (>50% stenosis on ICA+FFR)

96

95

96

95

99m Tc SPECT/ 64 slice CT

Dong71 2014

78

ICA/CTA≥50% + SPECT defect

89

92

81

96

99m

Tc SPECT/ 16 slice CT

Winther72 * 2015

138

Flow limiting coronary stenoses (>50% stenosis on ICA)

67

86

57

90

99m

Liga73 2016

252

Flow limiting coronary stenoses [(>50% stenosis on ICA) or (30–50% stenosis on ICA & FFR+) and perfusion defect]

83

68

NA

NA

SPECT/PET/ CTA

Tc SPECT/ Dual source CT

*

Pre-renal transplant patients, per patient analysis. Hybrid SPECT/coronary CTA only applied for nonevaluable arteries on coronary CTA. µ Patient-based analysis. Data from Gaemperli O, Bengel FM, Kaufmann PA. Cardiac hybrid imaging. Eur Heart J. 2011;32(17):2100-2108. ¥

ch27.indd 526

15-02-2022 17:18:07

Chapter 27  Hybrid Imaging: SPECT–CT and PET–CT

a multicenter study, included 292 symptomatic individuals with at least intermediate pretest likelihood of CAD, who underwent coronary CTA and at least one form of ischemia testing and were referred to invasive coronary angiography with an intention of evaluating FFR in intermediate lesions. These patients were followed up for 30 days and coronary revascularization was documented. Invasive coronary angiograms by QCA were considered obstructive with more than 50% stenosis in the left main or more than 70% stenosis in any of the other coronary arteries or 30% to 70% stenosis with an FFR ≤ 0.8. Majority of the individuals (70%) in the EVINCI study underwent SPECT MPI and the rest underwent PET MPI. Overall, about 41% of the patients had normal hybrid imaging (MPI and coronary CTA normal) and 24% had a hybrid match (perfusion defect in the territory of a stenotic artery). As in prior-single center studies,70,78–80 rate of coronary revascularization was highest in the matched group (70%), intermediate in the mismatch group (36%), and least in the normal group (10%), P < 0.001. However, radiation dose with the hybrid imaging approach was high (PET/coronary CTA: 9.4 mSv and SPECT/coronary CTA: 18.5 mSv [range 6–31 mSv].73 Before advocating a combined imaging approach, larger studies with longer-term follow up are necessary to determine the optimal strategy for hybrid imaging. Whether a combined imaging strategy improves patient outcomes by identification of patients who will benefit from revascularization, avoidance of invasive angiography, or improved adherence to optimal medical therapy is still being determined.

▶▶Hybrid Cardiac CT and Radionuclide Imaging Cardiac Inflammation and Infection Hybrid imaging is uniquely suited for localizing radiotracer uptake in radiopharmaceutically avid imaging and for molecular imaging (Fig. 27-7).95 Using 18F-FDG PET, inflammation and infection are detected as increased radiotracer uptake; however, with only PET imaging, it may be difficult to distinguish nonspecific FDG uptake from infection or inflammation. The addition of a low-dose CT allows for coregistration of PET and CT images, which makes it possible to accurately localize FDG

ch27.indd 527

527

FIGURE 27-7  Positron emission tomography (PET)–computed tomographic (CT) imaging of morphology and biology. In a model of regional adenoviral transfer of the VEGF (121) gene to myocardium of healthy pigs, PET–CT using multiple moleculardirected radiotracers was employed. Representative short-axis tomographic images are shown. On the left is a short-axis image of contrast-enhanced multislice CT showing the location of titanium clip markings. On the right is PET image showing significant accumulation of the reporter probe [18F]fluorohydroxymethylbutyl-guanine (FHBG). The middle image (overlay of PET and CT) shows that the FHBG accumulation colocalizes with clip markings in areas expressing the herpes simplex virus 1-sr39tk receptor gene. (Reproduced with permission from Wagner B, Anton M, Nekolla SG, et al. Noninvasive characterization of myocardial molecular interventions by integrated positron emission tomography and computed tomography. J Am Coll Cardiol. 2006;48(10):2107–2115.)

uptake to specific anatomic structures and diagnose pathologic inflammation or infection. As discussed in detail in Chapter 26,18F-FDG imaging is emerging as a valuable technique for the diagnosis and management of cardiac sarcoidosis and prosthetic valve or cardiac device infection, and provides incremental value for diagnosing infective endocarditis in prosthetic valves and intracardiac devices wherein echocardiography, cardiac CT, and cardiac magnetic resonance imaging may be inconclusive. Without hybrid imaging (PET−CT), the utility of radiopharmaceutically avid imaging would be greatly diminished in evaluating the above disease processes.

Complex Congenital Heart Disease Hybrid imaging is well suited for imaging individuals with complex congenital heart disease.81,82 In addition to improved image quality, and attenuation correction, the CT portion of the PET or SPECT MPI helps distinguish prosthetic material related perfusion defects from fibrosis.

15-02-2022 17:18:08

528

Section 4  Beyond Perfusion Imaging

RADIATION EXPOSURE Increased utilization of ionizing radiation in medical imaging, in large part attributable to CT scans and nuclear medicine studies, has led to increased scrutiny on the risk of radiation exposure from medical imaging.73 Therefore, a chapter on hybrid SPECT–CT and PET–CT imaging must include a brief discussion on radiation risk and exposure, which is more completely addressed in Chapters 2 and 6. The estimated effective radiation doses for cardiac imaging studies are: coronary calcium CT 1–2 mSv, coronary CTA 2–5 mSv, SPECT MPI 12.8 mSv, PET MPI 3.7 mSv, and CT for attenuation correction 0.5–1 mSv.83–86 The addition of a CAC score CT or a coronary CTA to an MPI study will further increase radiation dose.73 The risk of this radiation exposure must be counterbalanced against the incremental diagnostic and prognostic information gained from the hybrid approach. Recently, measuring the cellular effects of radiation on DNA has emerged as an additional method of assessing radiation risk.63,87 More research is needed to develop new agents to protect patients from the potential adverse effects of radiation and to decrease the radiation exposure.

FUTURE DIRECTIONS

▶▶PET-MR Hybrid Imaging Hybrid imaging using positron emission tomography magnetic resonance imaging (PET-MR) represents an emerging technology that may prove useful in the evaluation of multiple cardiovascular disease processes including ischemic heart disease, cardiac masses, and inflammatory cardiomyopathies. PET-MR combines the targeted molecular imaging of PET with the excellent anatomic, functional, and tissue characterization of cardiac MR. In the past, hybrid PET-MR imaging required separate scanners, as PET detectors could not function within a strong magnetic field. However, with recent advances in technologies, current PET-MR systems combine PET and MR scanners within the same gantry, and as a result allows for simultaneous acquisition of PET and MR data.88 PET-MR hybrid scanners have

ch27.indd 528

the potential to offer cardio-respiratory motion correction and partial volume error correction, as provided with CT, without additional radiation,89 and potentially improving quantitation of radionuclide images.90 In the assessment of ischemic cardiomyopathies, PET and MR have individually been validated as useful tools for the assessment of myocardial viability. Cardiac MR provides information on left ventricular regional wall motion and scar burden using late gadolinium enhancement, while measurement of myocardial blood flow and the presence of hibernating myocardium can be determined using NH3 and 18 F-FDG PET.91,92 Given the differing methods for assessing myocardial viability, PET-MR hybrid imaging may increase the accuracy for assessing myocardial scar and determining the likelihood of functional improvement with revascularization. Inflammatory conditions, such as sarcoidosis and myocarditis, and amyloidosis,93 may also benefit from PET-MR hybrid imaging. In cardiac sarcoidosis, coregistration of PET-MR data allows for the assessment of both scar and active inflammation, which is necessary in differentiating between active cardiac sarcoidosis and prior cardiac sarcoidosis. For the evaluation of masses, cardiac MR provides important information regarding location, sizing, and tissue characterization of the mass, and at times, may be able to diagnose the mass type. However, frequently 18F-FDG PET is needed to assess tumor vascularity to differentiate between malignant and benign tumors and to make a diagnosis. In a small prospective study, 20 patients (16 with unknown tumor type, 4 with cardiac sarcoma after surgical resection) underwent scanning with 18F-FDG PET/MR hybrid imaging. Using PET data and all available MR sequences, the combined imaging had a sensitivity of 100% and a specificity of 92% for the diagnosis of tumor type, supporting the incremental benefit of hybrid imaging in tumor diagnosis.94 PET/MR scanners allow for simultaneous acquisition of the PET and MR data providing accurate coregistration of images when imaging small structures (e.g., coronary arteries or carotid arteries) compared to hybrid SPECT or PET/CT scanners, which provide sequential radionuclide and CT images. Limitations of PET-MR include limited access to technology, complexity of performing accurate attenuation correction, small size of the

15-02-2022 17:18:08

Chapter 27  Hybrid Imaging: SPECT–CT and PET–CT

scanner bore with increased likelihood of exacerbating patient claustrophobia, and high cost of the imaging technology. However, given the rapid development of the technology, these challenges may be overcome in the near future.

SUMMARY Hybrid SPECT–CT and PET–CT systems offer attenuation-corrected MPI that improves the image quality and diagnostic accuracy compared to nonattenuation-corrected MPI. In addition to attenuation correction, the CT component of hybrid scanners can provide an accurate measure of calcified atherosclerotic burden and additional diagnostic and prognostic information to guide risk factor management, particularly when MPI is normal. Incorporation of coronary CTA with relative stress MPI and quantitative myocardial blood flow imaging is providing an intricate anatomic and physiologic assessment of CAD in each individual patient and offers the potential to guide personalized management of CAD. Hybrid PET–CT imaging with 18F-FDG PET–CT is breakthrough for imaging cardiac sarcoidosis, prosthetic valve and intracardiac device endocarditis, and molecular cardiology research. The ultimate transition to hybrid imaging with stress MPI and coronary CT angiography in clinical practice will be further defined and clarified with large, carefully designed and conducted clinical trials. In addition, hybrid PET-MR imaging is a promising technology that may prove useful in the evaluation of multiple cardiovascular disorders in the near future.

REFERENCES 1. Fricke E, Fricke H, Weise R, et al. Attenuation correction of myocardial SPECT perfusion images with low-dose CT: evaluation of the method by comparison with perfusion PET. J Nucl Med. 2005;46:736–744. 2. Masood Y, Liu YH, Depuey G, et al. Clinical validation of SPECT attenuation correction using x-ray computed tomography-derived attenuation maps: multicenter clinical trial with angiographic correlation. J Nucl Cardiol. 2005;12: 676–686. 3. Ficaro EP, Fessler JA, Shreve PD, Kritzman JN, Rose PA, Corbett JR. Simultaneous transmission/emission myocardial perfusion tomography. Diagnostic accuracy of attenuation-corrected

ch27.indd 529

529

99mTc-sestamibi single-photon emission computed tomography. Circulation. 1996;93:463–473. 4. Hendel RC, Berman DS, Cullom SJ, et al. Multicenter clinical trial to evaluate the efficacy of correction for photon attenuation and scatter in SPECT myocardial perfusion imaging. Circulation. 1999;99:2742–2749. 5. Kluge R, Sattler B, Seese A, Knapp WH. Attenuation correction by simultaneous emission-transmission myocardial single-photon emission tomography using a technetium99m-labelled radiotracer: impact on diagnostic accuracy. Eur J Nucl Med. 1997;24:1107–1114. 6. Links JM, DePuey EG, Taillefer R, Becker LC. Attenuation correction and gating synergistically improve the diagnostic accuracy of myocardial perfusion SPECT. J Nucl Cardiol. 2002;9:183–187. 7. Baghdasarian SB, Noble GL, Ahlberg AW, Katten D, Heller GV. Risk stratification with attenuation corrected stress Tc99m sestamibi SPECT myocardial perfusion imaging in the absence of ECG-gating due to arrhythmias. J Nucl Cardiol. 2009;16:533–539. 8. Garcia EV, Esteves FP. Attenuation corrected myocardial perfusion SPECT provides powerful risk stratification in patients with coronary artery disease. J Nucl Cardiol. 2009;16:490–492. 9. Dilsizian V, Bacharach SL, Beanlands RS, et al. ASNC imaging guidelines/SNMMI procedure standard for positron emission tomography (PET) nuclear cardiology procedures. J Nucl Cardiol. 2016;23(5):1187–1226. 10. Di Carli MF, Hachamovitch R. New technology for noninvasive evaluation of coronary artery disease. Circulation. 2007;115:1464–1480. 11. Dorbala S, Di Carli MF. Cardiac PET perfusion: prognosis, risk stratification, and clinical management. Semin Nucl Med. 2014;44:344–357. 12. Herzog BA, Husmann L, Valenta I, et al. Long-term prognostic value of 13N-ammonia myocardial perfusion positron emission tomography added value of coronary flow reserve. J Am Coll Cardiol. 2009;54:150–156. 13. Murthy VL, Naya M, Foster CR, et al. Association between coronary vascular dysfunction and cardiac mortality in patients with and without diabetes mellitus. Circulation. 2012;126:1858–1868. 14. Fukushima K, Javadi MS, Higuchi T, et al. Impaired global myocardial flow dynamics despite normal left ventricular function and regional perfusion in chronic kidney disease: a quantitative analysis of clinical 82Rb PET/CT studies. J Nucl Med. 2012;53:887–893. 15. Ziadi MC, Dekemp RA, Williams KA, et al. Impaired myocardial flow reserve on rubidium-82 positron emission tomography imaging predicts adverse outcomes in patients assessed for myocardial ischemia. J Am Coll Cardiol. 2011;58:740–748. 16. Hulten E, Ahmadi A, Blankstein R. CT Assessment of Myocardial Perfusion and Fractional Flow Reserve. Prog Cardiovasc Dis. 2015;57:623–631. 17. Min JK, Leipsic J, Pencina MJ, et al. Diagnostic accuracy of fractional flow reserve from anatomic CT angiography. JAMA. 2012;308:1237–1245. 18. Dorbala S, Di Carli MF, Delbeke D, et al. SNMMI/ASNC/ SCCT guideline for cardiac SPECT/CT and PET/CT 1.0. J Nucl Med. 2013;54:1485–1507.

15-02-2022 17:18:08

530

Section 4  Beyond Perfusion Imaging

19. Pletcher MJ, Tice JA, Pignone M, Browner WS. Using the coronary artery calcium score to predict coronary heart disease events: a systematic review and meta-analysis. Arch Intern Med. 2004;164:1285–1292. 20 Athappan G, Habib M, Ponniah T, Jeyaseelan L. Multi-detector computerized tomography angiography for evaluation of acute chest pain--a meta analysis and systematic review of literature. Int J Cardiol. 2010;141:132–4021. 21. Newby DE, Adamson PD, Berry C, et al. Coronary CT angiography and 5-year risk of myocardial infarction. N Engl J Med. 2018;379:924–933. 22. Shaw LJ, Berman DS, Maron DJ, et al. Optimal medical therapy with or without percutaneous coronary intervention to reduce ischemic burden: results from the Clinical Outcomes Utilizing Revascularization and Aggressive Drug Evaluation (COURAGE) trial nuclear substudy. Circulation. 2008;117:1283–1291. 23. De Bruyne B, Pijls NH, Kalesan B, et al. Fractional flow reserve-guided PCI versus medical therapy in stable coronary disease. N Engl J Med. 2012;367:991–1001. 24. Greenland P, Blaha MJ, Budoff MJ, Erbel R, Watson KE. Coronary calcium score and cardiovascular risk. J Am Coll Cardiol. 2018;72(4):434–447. 25. Einstein AJ, Johnson LL, Bokhari S, et al. Agreement of visual estimation of coronary artery calcium from low-dose CT attenuation correction scans in hybrid PET/CT and SPECT/CT with standard Agatston score. J Am Coll Cardiol. 2010;56:1914–1921. 26. Patchett ND, Pawar S, Miller EJ. Visual identification of coronary calcifications on attenuation correction CT improves diagnostic accuracy of SPECT/CT myocardial perfusion imaging. J Nucl Cardiol. 2016. 27. Le VT, Knight S, Min DB, et al. Absence of coronary artery calcium during positron emission tomography stress testing in patients without known coronary artery disease identifies individuals with very low risk of cardiac events. Circ Cardiovasc Imaging. 2020;13(1):e009907. 28. Thompson RC, McGhie AI, Moser KW, et al. Clinical utility of coronary calcium scoring after nonischemic myocardial perfusion imaging. J Nucl Cardiol. 2005;12:392–400. 29. Berman DS, Wong ND, Gransar H, et al. Relationship between stress-induced myocardial ischemia and atherosclerosis measured by coronary calcium tomography. J Am Coll Cardiol. 2004;44:923–930. 30. He ZX, Hedrick TD, Pratt CM, et al. Severity of coronary artery calcification by electron beam computed tomography predicts silent myocardial ischemia. Circulation. 2000;101:244–251. 31. Anand DV, Lim E, Raval U, Lipkin D, Lahiri A. Prevalence of silent myocardial ischemia in asymptomatic individuals with subclinical atherosclerosis detected by electron beam tomography. J Nucl Cardiol. 2004;11:450–457. 32. Chang SM, Nabi F, Xu J, et al. The coronary artery calcium score and stress myocardial perfusion imaging provide independent and complementary prediction of cardiac risk. J Am Coll Cardiol. 2009;54:1872–1882. 33. Ramakrishna G, Miller TD, Breen JF, Araoz PA, Hodge DO, Gibbons RJ. Relationship and prognostic value of coronary artery calcification by electron beam computed tomography to stress-induced ischemia by single photon emission computed tomography. Am Heart J. 2007;153:807–814.

ch27.indd 530

34. Hendel RC, Berman DS, Di Carli MF, et al. ACCF/ASNC/ ACR/AHA/ASE/SCCT/SCMR/SNM 2009 Appropriate Use Criteria for Cardiac Radionuclide Imaging: A Report of the American College of Cardiology Foundation Appropriate Use Criteria Task Force, the American Society of Nuclear Cardiology, the American College of Radiology, the American Heart Association, the American Society of Echocardiography, the Society of Cardiovascular Computed Tomography, the S­ ociety for Cardiovascular Magnetic Resonance, and the Society of Nuclear Medicine. Endorsed by the American College of Emergency Physicians. J Am Coll Cardiol. 2009;53:2201–2229. 35. Nabi F, Chang SM, Pratt CM, et al. Coronary artery calcium scoring in the emergency department: identifying which patients with chest pain can be safely discharged home. Ann Emerg Med. 2010;56(3):220–229. 36. Bavishi C, Argulian E, Chatterjee S, Rozanski A. CACS and the frequency of stress-induced myocardial ischemia during MPI: a meta-analysis. JACC Cardiovasc Imaging. 2016;9: 580–589. 37. Anand DV, Lim E, Hopkins D, et al. Risk stratificationin uncomplicated type 2 diabetes: prospective evaluation of the combined use of coronary artery calcium imaging and selective myocardial perfusion scintigraphy. Eur Heart J. 2006;27:713–721. 38. Blumenthal RS, Becker DM, Yanek LR, et al. Comparison of coronary calcium and stressmyocardial perfusion imaging in apparently healthy siblings of individuals with premature coronary artery disease. Am J Cardiol. 2006;97:328–333. 39. Chang SM, Nabi F, Xu J, et al. Value of CACS compared with ETT and myocardial perfusion imaging for predicting longterm cardiac outcome in asymptomatic and symptomatic patients at low risk for coronary disease: clinical implications in a multimodality imaging world. J Am Coll Cardiol Img. 2015;8:134–144. 40. Esteves FP, Sanyal R, Santana CA, Shaw L, Raggi P. Potential impact of noncontrast computed tomography as gatekeeper for myocardial perfusion positron emission tomography in patients admitted to the chest pain unit. Am J Cardiol. 2008;101:149–152. 41. Fathala A, Aljefri A, Alsugair A, Abouzied M. Coronary artery calcification detected by PET/CT scan as a marker of myocardial ischemia/coronary artery disease. Nucl Med Commun. 2011;32:273–278. 42. Ghadri JR, Fiechter M, Fuchs TA, et al. Registryfor the Evaluation of the PROgnostic value of a novel integrated imaging approach combining Single Photon Emission Computed Tomography with coronary calcification imaging (REPROSPECT). Eur Heart J Cardiovasc Imaging. 2013;14:374–380. 43. He ZX, Hedrick TD, Pratt CM, et al. Severity of coronary artery calcification by electron beam computed tomography predicts silent myocardial ischemia. Circulation. 2000;101:244–251. 44. Ho J, FitzGerald S, Stolfus L, Cannaday J, Radford N. Severe coronary artery calcifications are associated with ischemia in patients undergoingmedical therapy. J Nucl Cardiol. 2007;14: 341–346. 45. Moser KW, O’Keefe JH Jr, Bateman TM, McGhie IA. Coronary calcium screening in asymptomatic patients as a guide to risk factor modification and stress myocardial perfusion imaging. J Nucl Cardiol. 2003;10:590–598.

15-02-2022 17:18:08

Chapter 27  Hybrid Imaging: SPECT–CT and PET–CT 46. Nishida C, Okajima K, Kudo T, Yamamoto T, Hattori R, Nishimura Y. The relationship between coronary artery calcification detected by non gated multi-detector CT in patients with suspected ischemic heart disease and myocardialischemia detected by thallium exercise stresstesting. Ann Nucl Med. 2005;19:647–653. 47. Piers LH, Salachova F, Slart RH, et al. The role of coronary artery calcification score in clinical practice. BMC Cardiovasc Disord. 2008;8:38. 48. Rozanski A, Gransar H, Wong ND, et al. Clinical outcomes after both coronary calcium scanning and exercise myocardial perfusion scintigraphy. J Am Coll Cardiol. 2007;49:1352–1361. 49. Rosman J, Shapiro M, Pandey A, VanTosh A, Bergmann SR. Lack of correlation between coronaryartery calcium and myocardial perfusion imaging. J Nucl Cardiol. 2006;13:333–337. 50. Schenker MP, Dorbala S, Hong EC, et al. Interrelation of coronary calcification, myocardial ischemia, and outcomes in patients with intermediate likelihood of coronary artery disease: a combined positron emission tomography/computed tomography study. Circulation. 2008;117:1693–1700. 51. Schepis T, Gaemperli O, Koepfli P, et al. Added value of coronary artery calcium score as an adjunct to gated SPECT for the evaluation of coronary artery disease in an intermediate-risk population. J Nucl Med. 2007;48:1424–1430. 52. Scholte AJ, Schuijf JD, Kharagjitsingh AV, et al. Different manifestations of coronary artery diseaseby stress SPECT myocardial perfusion imaging, coronary calcium scoring, and multislice CT coronary angiography in asymptomatic patients with type 2 diabetes mellitus. J Nucl Cardiol. 2008;15:503–509. 53. Seyahi N, Kahveci A, Cebi D, et al. Coronary artery calcification and coronary ischaemia in renal transplant recipients. Nephrol Dial Transplant. 2011;26:720–726. 54. Yao ZM, Li W, Qu WY, Zhou C, He Q, Ji FS. Comparison of (99)mTc-methoxyisobutylisonitrile myocardial single-photon emission computed tomography and electron beam computed tomography for detecting coronary artery disease in patients with no myocardial infarction. Chin Med J (Engl). 2004;117:700–705. 55. Curillova Z, Yaman BF, Dorbala S, et al. Quantitative relationship between coronary calcium content and coronary flow reserve as assessed by integrated PET/CT imaging. Eur J Nucl Med Mol Imaging. 2009; 36(10):1603–1610. 56. Naya M, Murthy VL, Foster CR, et al. Prognostic interplay of coronary artery calcification and underlying vascular dysfunction in patients with suspected coronary artery disease. J Am Coll Cardiol. 2013;61:2098–2106. 57. Naya M, Murthy VL, Blankstein R, et al. Quantitative relationship between the extent and morphology of coronary atherosclerotic plaque and downstream myocardial perfusion. J Am Coll Cardiol. 2011;58:1807–1816. 58. Assante R, Zampella E, Arumugam P, et al. Quantitative relationship between coronary artery calcium and m ­ yocardial blood flow by hybrid rubidium-82 PET/ CT imaging in patients with suspected coronary artery disease. J Nucl ­Cardiol. 2017;24:494–501. 59. Dorbala S, Hachamovitch R, Di Carli MF. Myocardial perfusion imaging and multidetector computed tomographic coronary angiography: appropriate for all patients with suspected coronary artery disease? J Am Coll Cardiol. 2006;48:2515–2517.

ch27.indd 531

531

60. Di Carli MF, Dorbala S, Curillova Z, et al. Relationship between CT coronary angiography and stress perfusion imaging in patients with suspected ischemic heart disease assessed by integrated ET-CT imaging. J Nucl Cardiol. 2007;14: 799–809. 61. Di Carli MF. Hybrid imaging: integration of nuclear imaging and cardiac CT. Cardiol Clin. 2009;27:257–263. 62. Nabi F, Kassi M, Muhyieddeen K, et al. Optimizing evaluation of patients with low-to-intermediate-risk acute chest pain: a randomized study comparing stress myocardial perfusion tomography incorporating stress-only imaging versus cardiac CT. J Nucl Med. 2016;57:378–384. 63. Namdar M, Hany TF, Koepfli P, et al. Integrated PET/CT for the assessment of coronary artery disease: a feasibility study. J Nucl Med. 2005;46:930–935. 64. Rispler S, Keidar Z, Ghersin E, et al. Integrated single-photon emission computed tomography and computed tomography coronary angiography for the assessment of hemodynamically significant coronary artery lesions. J Am Coll Cardiol. 2007;49:1059–1067. 65. Groves AM, Speechly-Dick ME, Kayani I, et al. First experience of combined cardiac PET/64-detector CT angiography with invasive angiographic validation. Eur J Nucl Med Mol Imaging. 2009;36:2027–2033. 66. Slomka PJ, Cheng VY, Dey D, et al. Quantitative analysis of myocardial perfusion SPECT anatomically guided by coregistered 64-slice coronary CT angiography. J Nucl Med. 2009;50:1621–1630. 67. Kajander S, Joutsiniemi E, Saraste M, et al. Cardiac positron emission tomography/computed tomography imaging accurately detects anatomically and functionally significant coronary artery disease. Circulation. 2010;122(6):603–613. 68. Danad I, Raijmakers PG, Appelman YE, et al. Hybrid imaging using quantitative H215O PET and CT-based coronary angiography for the detection of coronary artery disease. J Nucl Med. 2013;54:55–63. 69. Thomassen A, Petersen H, Diederichsen AC, et al. Hybrid CT angiography and quantitative 15O-water PET for assessment of coronary artery disease: comparison with q ­ uantitative ­coronary angiography. Eur J Nucl Med Mol Imaging. 2013;40: 1894–1904. 70. Schaap J, Kauling RM, Boekholdt SM, et al. Incremental diagnostic accuracy of hybrid SPECT/CT coronary angiography in a population with an intermediate to high pre-test likelihood of coronary artery disease. Eur Heart J Cardiovasc ­Imaging. 2013;14:642–649. 71. Dong W, Wang Q, Gu S, Su H, Jiao J, Fu Y. Cardiac hybrid SPECT/CTA imaging to detect “functionally relevant coronary artery lesion”: a potential gatekeeper for coronary revascularization? Ann Nucl Med. 2014;28:88–93. 72. Winther S, Svensson M, Jorgensen HS, et al. Diagnostic performance of coronary CT angiography and myocardial perfusion imaging in kidney transplantation candidates. JACC Cardiovasc Imaging. 2015;8:553–562. 73. Liga R, Vontobel J, Rovai D, et al. Multicentre multi-device hybrid imaging study of coronary artery disease: results from the EValuation of INtegrated Cardiac Imaging for the Detection and Characterization of Ischaemic Heart Disease (EVINCI) hybrid imaging population. Eur Heart J Cardiovasc Imaging. 2016;17:951–960.

15-02-2022 17:18:08

532

Section 4  Beyond Perfusion Imaging

74. Gaemperli O, Schepis T, Valenta I, et al. Cardiac image fusion from stand-alone SPECT and CT: clinical experience. J Nucl Med. 2007;48:696–703. 75. Santana CA, Garcia EV, Faber TL, et al. Diagnostic performance of fusion of myocardial perfusion imaging (MPI) and computed tomography coronary angiography. J Nucl Cardiol. 2009;16:201–211. 76. Sato A, Nozato T, Hikita H, et al. Incremental value of combining 64-slice computed tomography angiography with stress nuclear myocardial perfusion imaging to improve noninvasive detection of coronary artery disease. J Nucl Cardiol. 2010;17:19–26. 77. Schuijf JD, Wijns W, Jukema JW, et al. Relationship between noninvasive coronary angiography with multi-slice computed tomography and myocardial perfusion imaging. J Am Coll Cardiol. 2006;48:2508–2514. 78. Pazhenkottil AP, Nkoulou RN, Ghadri JR, et al. Prognostic value of cardiac hybrid imaging integrating single-photon emission computed tomography with coronary computed tomography angiography. Eur Heart J. 2011;32:1465–1471. 79. Pazhenkottil AP, Nkoulou RN, Ghadri JR, et al. Impact of cardiac hybrid single-photon emission computed tomography/computed tomography imaging on choice of treatment strategy in coronary artery disease. Eur Heart J. 2011;32: 2824–2829. 80. Schaap J, de Groot JA, Nieman K, et al. Hybrid myocardial perfusion SPECT/CT coronary angiography and invasive coronary angiography in patients with stable angina pectoris lead to similar treatment decisions. Heart. 2013;99:188–194. 81. Partington SL, Valente AM, Landzberg M, Grant F, Di Carli MF, Dorbala S. Clinical applications of radionuclide imaging in the evaluation and management of patients with congenital heart disease. J Nucl Cardiol. 2016;23:45–63. 82. Grani C, Benz DC, Possner M, et al. Fused cardiac hybrid imaging with coronary computed tomography angiography and positron emission tomography in patients with complex coronary artery anomalies. Congenit Heart Dis. 2016. 83. Desiderio MC, Lundbye JB, Baker WL, Farrell MB, Jerome SD, Heller GV. Current status of patient radiation exposure of cardiac positron emission tomography and single-photon emission computed tomographic myocardial perfusion imaging. Circ Cardiovasc Imaging. 2018;11(12): e007565. 84. Stocker TJ, Deseive S, Leipsic J, et al. Reduction in radiation exposure in cardiovascular computed tomography imaging: results from the prospective multicenter registry on

ch27.indd 532

radiation dose estimates of cardiac CT angiography in daily practice in 2017 (PROTECTION VI). Eur Heart J. 2018;39: 3715–3723. 85. Patel AA, Fine J, Naghavi M, Budoff MJ. Radiation exposure and coronary artery calcium scans in the society for heart attack prevention and eradication cohort. Int J Cardiovasc Imaging. 2019;35(1):179–183. 86. Israel O, Pellet O, Biassoni L, et al. Two decades of SPECT/ CT – the coming of age of a technology: An updated review of literature evidence. Eur J Nucl Med Mol Imaging. 2019;46: 1990–2012. 87. Groves AM, Goh V, Rajasekharan S, et al. CT coronary angiography: quantitative assessment of myocardial perfusion using test bolus data-initial experience. Eur Radiol. 2008;18:2155–2163. 88. Krizsan AK, Lajtos I, Dahlbom M, et al. A promising future: comparable imaging capability of MRI-compatible silicon photomultiplier and conventional photosensor preclinical PET systems. J Nucl Med. 2015;56:1948–1953. 89. Munoz C, Kolbitsch C, Reader AJ, Marsden P, Schaeffter T, Prieto C. MR-based cardiac and respiratory motion-­ compensation techniques for PET-MR imaging. PET Clin. 2016;11:179–191. 90. Karakatsanis NA, Abgral R, Trivieri MG, et al. Hybrid PET- and MR-driven attenuation correction for enhanced (18)F-NaF and (18)F-FDG quantification in cardiovascular PET/MR imaging. J Nucl Cardiol. 2020;27: 1126–1141. 91. Eitzman D, al-Aouar Z, Kanter HL, et al. Clinical outcome of patients with advanced coronary artery disease after viability studies with positron emission tomography. J Am Coll Cardiol. 1992;20:559–565. 92. Kim RJ, Wu E, Rafael A, Chen EL, et al. The use of contrastenhanced magnetic resonance imaging to identify reversible myocardial dysfunction. N Engl J Med. 2000;343:1445–1453. 93. Andrews JPM, Trivieri MG, Everett R, et al. 18F-fluoride PET/ MR in cardiac amyloid: A comparison study with aortic stenosis and age- and sex-matched controls. J Nucl Cardiol. 2020. 94. Nensa F, Tezgah E, Poeppel TD, et al. Integrated 18F-FDG PET/MR imaging in the assessment of cardiac masses: a pilot study. J Nucl Med. 2015;56:255–260. 95. Wagner B, Anton M, Nekolla SG, et al. Noninvasive characterization of myocardial molecular interventions by integrated positron emission tomography and computed tomography. J Am Coll Cardiol. 2006;48:2107–2115.

15-02-2022 17:18:08

Review Questions

1. The radiations typically used to image 201Tl are polyenergetic because: A. electron capture creates multiple gamma ray emissions. B. 201Hg K- and L-shell electrons have different binding energies. C. there are multiple gamma ray emissions for 201 Hg. D. 201Tl gamma ray emissions are more abundant than x-ray emissions. 2. If a nucleus decays by isomeric transition, which of the following might be emitted in the process? A. A positron B. An alpha article C. A conversion electron D. A neutron 3. Photons undergoing Compton scatter in ­tissue: A. can be identified by their energy. B. are less abundant than the photoelectrons. C. are not considered in radiation safety. D. are more abundant than Compton ­electrons. 4. As the thickness of tissue overlying the heart increases: A. the percentage of transmitted photons increases. B. the number of photoelectrons decreases. C. the amount of characteristic x-rays decreases. D. the number of energy-degraded photons increases.

reviewquestions.indd 533

S­EC­TI­ON

5

5. The most probable interaction of 511 keV annihilation photons in bone is: A. photoelectric absorption. B. compton scatter. C. pair production. D. complete transmission. 6. Which of the following is true of isotopes? A. They have different energy states. B. They have the same number of neutrons. C. They have the same number of protons. D. They have the same number of nucleons. 7. Which of the following types of radiation has the smallest linear energy transfer (LET)? A. Alpha B. Beta C. Positron D. Gamma 8. What determines the stability of a nucleus? A. The number of neutrons B. The number of protons C. The proton-to-neutron ratio D. The K-shell to L-shell vacancy ratio 9. If you received a 100-mCi dose of FDG at 9:00 am and injected a patient at 1 pm, how much activity would be available for this ­injection (T½ of 18F is ~2 hours)? A. 50 mCi B. 25 mCi C. 12 mCi D. 6 mCi 10. The international unit for describing dose equivalent is the following: A. Coulomb/kg B. Air Kerma C. Gray D. Sievert

15-02-2022 14:30:07

534

Section 5  Review Questions

11. Approximately how much annual radiation from natural background does the US ­population receive? A. 3.1 mSv B. 6.2 mSv C. 9.3 mSv D. 12.4 mSv 12. The occupational worker annual whole body radiation dose limit is the following: A. 0.05 Sv B. 0.15 Sv C. 0.5 Sv D. 5 Sv 13. The Nuclear Regulatory Commission-­ recommended whole body ALARA II ­investigational level is the following: A. 1.25 mSv per calendar quarter B. 3.75 mSv per calendar quarter C. 12.5 mSv per calendar quarter D. 37.5 mSv per calendar quarter 14. Effects from radiation that increase in ­severity after a threshold is exceeded are termed: A. stochastic. B. deterministic. C. linear non-threshold. D. carcinogenic. 15. A radiation source is measured at 20 mR/hr at a distance of 2 m. What is the calculated measure of the source at 3 m? A. 2.2 mR/hr B. 6.7 mR/hr C. 8.9 mR/hr D. 10.0 mR/hr

reviewquestions.indd 534

16. Occupational workers are required to be issued a radiation monitoring device under the following condition: A. The employee is a full-time occupational radiation worker. B. The employee is an undeclared pregnant worker. C. The employee is likely to receive 5% of the annual radiation dose limit. D. The employee is likely to receive 10% of the annual radiation dose limit. 17. Disposal of waste using the decay-in-storage method requires the following condition prior to disposal: A. Waste is held for a minimum of 5 half lives. B. Waste is held for a minimum of 10 half lives. C. Waste is held for a minimum of 10 half lives and is indistinguishable from background radiation. D. Waste must be held for 10 half lives and is less than two times background radiation. 18. A radioactive package containing 10-mCi of 99 mTc with a surface exposure reading of 0.4 mR/hr and a wipe test of 6600 dpm/300 cm2 should be returned with the following package label: A. Excepted Package Limited Quantity B. Radioactive I White C. Radioactive II Yellow D. Radioactive III Yellow 19. A radioactive package containing 10 mCi of 99 mTc with a surface exposure reading of 1.0 mR/hr and a wipe test of 6600 dpm/300 cm2 should be returned with the following package label: A. Excepted Package Limited Quantity B. Radioactive I White C. Radioactive II Yellow D. Radioactive III Yellow

15-02-2022 14:30:08

Section 5  Review Questions

20. What are the respective corresponding physical half-lives of rubidium-82, N-13 ammonia, and O-15 water? A. 76 seconds, 9.8 minutes, and 2 minutes, respectively B. 25.5 days, 2 minutes, and 110 minutes, respectively C. 76 seconds, 2 minutes, and 110 minutes, respectively D. 2 minutes, 9.8 minutes, and 6 hours, respectively 21. Which of the following statements on PET imaging agent is FALSE? A. Rubidium-82 is the only clinical PET radiotracer produced by a generator. B. N-13 ammonia has a lower first-pass extraction fraction than rubidium-82 C. O-15 water is considered the gold standard for clinical noninvasive myocardial blood flow measurements. D. Rubidium-82 has the lowest-image spatial resolution among the other available PET myocardial perfusion imaging agents. 22. Thallium-201 is compromised as a routine SPECT perfusion imaging agent because of the following, except: A. lower energy emission. B. long half-life. C. suboptimal linearity. D. suboptimal radiation exposure. 23. Ideal characteristics for perfusion imaging tracer include the following, except: A. high linearity with blood flow. B. very short half-life for short protocol. C. minimal gut/liver uptake. D. ability to quantitate myocardial blood flow. 24. Technetium-99m imaging tracers are now routinely used for SPECT rather than ­thallium-201. These agents have the following advantages over Tl-201, except: A. better image quality. B. lower radiation exposure. C. less gut, liver activity. D. shorter protocols.

reviewquestions.indd 535

535

25. The OSEM iterative reconstruction is a: A. stepwise algorithm for estimating the source activity from a set of projection data based on a model. B. process for correcting attenuation ­correction. C. reconstruction approach that uses Fourier transforms to produce a 3D reconstruction from a set of projection data. D. process for correcting resolution recovery. 26. Which of these attenuation artifacts is specific to line-source attenuation correction? A. Misregistration artifact B. Downscatter into the photopeak window C. Implanted metal artifact D. Breathing artifact 27. 3D imaging in PET is used to: A. improve the resolution of a PET scan. B. allow for tomographic imaging in PET. C. enable flow imaging with PET. D. improve the sensitivity of the PET system. 28. SPECT camera systems are generally older but reconstruction software programs can improve all the following, except: A. image quality. B. reduced radiation exposure. C. attenuation artifact by attenuation correction. D. shorten acquisition time. 29. CZT SPECT cameras are becoming more commonplace and offer the following, except: A. lower radiation dose. B. higher image quality. C. attenuation correction. D. shorter acquisition protocols. 30. At what frequency should a SPECT uniformity flood be routinely performed? A. Daily B. weekly C. Monthly D. Quarterly

15-02-2022 14:30:08

536

Section 5  Review Questions

31. At what frequency should a SPECT system resolution and linearity be performed? A. Daily B. Weekly C. Monthly D. Quarterly 32. At what frequency should a PET system blank scan be performed? A. Daily B. Weekly C. Monthly or quarterly D. Annually

37. Patient motion is a common artifact in all myocardial perfusion imaging studies. Which of the following techniques should be ­considered to reduce patient motion? A. Make the patient as comfortable as ­possible with a blanket, arm, and knee supports. B. When a patient is unable to place his or her arms above his or her head, image the patient with the arms down at the side. C. If scan motion is severe, the acquisition should be repeated. D. All of the above.

33. At what frequency should a PET system ­normalization be performed? A. Daily B. Weekly C. Monthly or quarterly D. Annually

38. The following are commonly used tracers in nuclear cardiology: A. F-18 FDG B. Thallium-201 C. Technetium-99m sestamibi D. Rubidium-82

34. When performing attenuation correction using gadolinium-153 or germanium-68 sealed sources, at what frequency should a reference scan be performed? A. Daily B. Weekly C. Monthly D. Quarterly

Please list in order of radiation exposure to patients (mSv), highest to lowest: A. a, c, d, b B. c, d, a, b C. b, c, a, d D. a, b, d, c

35. When performing a PET/CT tube warm-up procedure, which of the following statements are true? A. Tube warm-ups are typically required at the start of the day before scanning patients. B. Proper use of tube warm-ups may extend the life of the CT tube. C. Tube warm-ups reduce the possibility of artifacts. D. All of the above. 36. At what frequency should a PET/CT system fast calibration be performed? A. Every 24 hours B. Weekly C. Monthly or quarterly D. Annually

reviewquestions.indd 536

39. Utilizing which of the following techniques would NOT incur lower radiation exposure compared to standard rest/stress Tc-99m sestamibi? A. Ordered subset expectation minimization (OSEM) algorithms B. Anger camera C. Stress-first protocols D. Careful screening of patients prior to testing 40. The American Society of Nuclear ­Cardiology has recommended a maximum radiation exposure to patients during a nuclear cardiology study be less than 9 mSv per patient. Latest data demonstrate what percentage of US laboratories are following this recommendation? A. 23% B. 75% C. 10% D. 2.5%

15-02-2022 14:30:08

Section 5  Review Questions

41. Stress-only (or stress-first) imaging protocols are advantageous to reduce radiation exposure because data indicate the percentage of all nuclear cardiology studies which demonstrate either infarction or ischemia is estimated to be: A. 36% to 55%. B. 20% to 35%. C. 8% to 15%. D. 1% to 7%. 42. Cardiac PET perfusion imaging has become more popular in recent years. Recent data from the IAC database demonstrate which of the following for per patient radiation exposure undergoing cardiac PET studies in the year 2015, the latest available data? A. 14.7 mSv B. 6.7 mSv C. 3.7 mSv D. 25.1 mSv 43. Radiation exposure is a concern for patient exposure. The American Society of Nuclear Cardiology has recommended the following to reduce overall radiation exposure, except: A. stress-only imaging in patients with no known CAD. B. PET imaging rather than SPECT when available. C. other forms of testing in low likelihood patients (for suspected CAD). D. Angar camera technology rather than CZT. 44. Which of the following is true regarding physician certification through CBNC? A. Certification is for cardiologists only B. Requires a separate, specialized fellowship C. Requires 4 months of dedicated nuclear cardiology training with 700 hours input D. Can easily be transferred from non-US- to US-based certification

reviewquestions.indd 537

537

45. All of the following are true regarding physician certification, except for: A. it is a process to achieve professional acknowledgment of dedicated understanding of the field of nuclear cardiology. B. the physician certification exam is composed of 160 questions. which are a test of competency and knowledge. C. it provides for confidence in the patient population by providing quality care for nuclear cardiology practice. D. it was established to encourage physicians to maintain prestige and ownership in nuclear cardiology. 46. Following are true regarding the accreditation of a nuclear laboratory in the United States, except: A. medicare reimbursement requires a lab to have compliant accreditation. B. it provides for quality assurance in reporting and outcome. C. it is done every 5 years. D. it is an ongoing quality improvement ­process. 47. Quality assurance and improvement programs strive for all of the following, except: A. appropriate patient selection. B. imaging testing protocols and procedures. C. imaging quality and reporting. D. comparison with other nuclear ­laboratories. 48. What are the minimum work experience qualifications required to apply for US ­physician certification? A. 700 hours of work experience with 6 months of training B. 70 hours of direct nuclear technology education and 300 case interpretations C. At least 80 hours of radiopharmaceuticals, radiation physics, instrumentation, and safety D. 700 hours and 4 months of training.

15-02-2022 14:30:08

538

Section 5  Review Questions

49. Lab accreditation metric Interpretive Quality Review and Correlation evaluates what aspect of quality outcomes? A. Requires quarterly assessment comparisons with neighboring nuclear laboratories B. Is a continuing quality improvement process requiring annual foreign expert assessment with established reference standards C. Focuses on self-assessment with concordance exercises and clinical correlation D. All of the above 50. Which of the following is NOT an absolute contraindication to exercise stress testing? A. Atrial fibrillation B. MI within the past 48 hours C. Acute pericarditis D. Severe pulmonary hypertension 51. A 66-year-old woman with 3-week history of dyspnea on exertion presents to clinic. ­Medical history is significant for COPD, hypertension, and coronary artery disease. She underwent stenting of the mid-right coronary artery 5 years ago. Due to her lung disease, she lives a predominantly sedentary lifestyle and has limited exercise ability. On physical examination, blood pressure is 136/72 mm Hg, pulse rate 88 beats per minute, and respiration rate is 16 breaths per minute. BMI is 36. There is inspiratory and expiratory wheezing bilaterally. Electrocardiogram reveals left ventricular hypertrophy and repolarization abnormalities. Which of the following is the most appropriate diagnostic test to perform next? A. Adenosine pharmacologic stress test B. Coronary catherization C. Exercise treadmill stress test D. Dobutamine stress echocardiogram

reviewquestions.indd 538

52. A 52-year-old man with 2-month history of chest pain presents to clinic. He has a history of hypertension, hyperlipidemia, gastroesophageal reflux disease, and tobacco use. He undergoes exercise treadmill stress testing. He is able to exercise for 3 minutes to a heart rate of 83% of the maximum predicted. The study is discontinued due to fatigue. Testing did not reproduce his symptoms and there were no significant electrocardiographic changes with exercise. Which of the following would be the most appropriate next test? A. Pharmacologic stress testing B. Cardiac catherization C. Clinical observation D. Starting a beta blocker 53. Shortly after dipyridamole administration for pharmacologic stress test, the patient begins to have chest discomfort, severe hypotension, and shortness of breath with wheezing. The infusion is held; however, the patient continues to be symptomatic. Which of the following is NOT an appropriate reversal agent? A. IV aminophylline 125 mg B. PO aminophylline 125 mg C. IV theophylline 50 mg D. Caffeinated beverage 54. Which of the following statements is true regarding adjunctive low-level exercise with vasodilator stress myocardial perfusion ­imaging? A. Symptoms, such as dyspnea, are lessened with the use of adjunctive exercise. B. Image quality is improved with the use of adjunctive exercise. C. Diagnostic accuracy of SPECT MPI is improved when exercise is combined with pharmacologic stress. D. Most individuals referred for pharmacologic stress testing cannot perform lowlevel exercise.

15-02-2022 14:30:08

Section 5  Review Questions

55. The Duke treadmill score does not depend on which of the following? A. Exercise time B. Severity of angina C. Blood pressure changes D. Magnitude of ST depression 56. Which of the following pharmacologic stress test protcols is not endorsed by current guidelines? A. Regadenoson 400 mcg given over 10 seconds B. Graded dobutamine infusion up to 50 mcg/kg/minute C. 0.56 mg/kg of dipyridamole given IV over 4 minutes D. 3-minute adenosine infusion 57. Which of the following is correct with regards to Regadenoson?: A. may be safely used in patients with COPD or ESRD. B. is administered by an infusion pump. C. rarely causes headaches. D. has not been shown to cause complete heart block. 58. A 59-year-old man presents to clinic complaining of substernal chest tightness. He reports chest tightness with exertion, which typically resolves with rest. He takes no medications. His BP in the office is 165/92, HR 85, and physical exam is unremarkable. EKG shows normal sinus rhythm with T-wave flattening in the V5 and V6. Which of the following is the best next test to evaluate this patient’s symptoms? A. EKG treadmill stress test B. SPECT myocardial perfusion imaging, stress followed by rest imaging C. SPECT myocardial perfusion imaging, rest followed by stress imaging D. Left heart catheterization

reviewquestions.indd 539

539

59. A 65-year-old woman presents to your office for evaluation of her cardiac complaints, consistent with “typical angina.” She takes losartan, amlodipine, and atorvastatin. Her body mass index is 47 kg/m2. You decide to order a SPECT myocardial perfusion imaging study. Which of the following is the best imaging protocol to use for this patient? A. One-day Tc-99m stress/rest study B. One-day Tc-99m rest/stress study C. Two-day Tc-99m stress/rest study D. Two-day Tc-99m rest/stress study 60. Which of the following best practices is more utilized in the United States compared to the rest of the world? A. Stress-first SPECT myocardial perfusion imaging B. Advanced post-processing software C. Solid-state SPECT cameras D. PET for myocardial perfusion imaging 61. The American Society of Nuclear Cardiology has set a goal for reducing radiation from nuclear cardiology of: A. all nuclear cardiology studies should be performed with effective dose ≤9 mSv per patient. B. all nuclear cardiology studies should be performed with no more than 10% of studies with effective dose >20 mSv. C. all nuclear cardiology studies should be performed with median effective dose ≤9 mSv. D. all nuclear cardiology studies should be performed with an average effective dose ≤9 mSv. 62. Stress-first SPECT myocardial perfusion imaging should be performed where ­appropriate: A. for all patients. B. only for patients without a history of ­coronary artery disease. C. only for patients with abnormal left ventricular function. D. for patients for whom there is a reasonable chance that stress imaging will be normal.

15-02-2022 14:30:08

540

Section 5  Review Questions

63. The INCAPS study has recommended “best practices” for SPECT myocardial imaging, which include all of the following, except: A. performing stress-only imaging, when ­possible. B. using camera-based dose reduction ­strategies. C. giving higher than recommended radionuclide doses to improve image quality. D. using Tc-99m perfusion agents. 64. A 69-year-old man with atypical chest pain, diabetes, and hypertension who recently developed back pain undergoes pharmacologic stress Rb-82 imaging. These images represent: A. three-vessel ischemia with TID. B. false positive due to misregistration. C. good prognosis. D. single-vessel disease, medical therapy.

reviewquestions.indd 540

65. PET imaging is emerging as a very useful diagnostic tool, and offers benefit in comparison to standard SPECT. Which of the following is NOT an example of an advantage PET imaging? A. PET imaging is able to detect reversible wall motion abnormalities at peak ­hyperemiA. B. Exercise PET is currently available and routinely performed. C. PET perfusion imaging protocols are shorter. D. PET imaging demonstrates higher spatial resolution when compared to SPECT ­imaging.

15-02-2022 14:30:09

Section 5  Review Questions

66. Due to the advantages of PET, many patients may be more recommended to undergo a PET study instead of a SPECT study. All of the following indications are appropriate for PET, except: A. a 54-year-old woman with hypertension/ chest pain and previous equivocal SPECT study. B. a 68-year-old woman with a history of CABG, new chest pain, and unable to exercise. C. a 75-year-old man with a history of smoking presents for preoperative evaluation prior to a fem-pop bypass surgery. D. a 35-year-old woman with HTN, a 12 pack/ year history of tobacco use, and normal resting ECG with atypical chest pain. 67. There are two PET perfusion tracers available commercially. Of these, the most often used is Rb-82. Of the following, which is not a characteristic of Rb-82? A. Provides rapid sequence between stress and rest imaging B. On-site cyclotron is necessary C. Longer positron range than N-13 ­Ammonia D. Radiation exposure is 3 to 5 mSv per patient 68. Myocardial blood flow (MBF) offers additional information beyond perfusion imaging by estimating flow reserve. MBF can provide additional information for all of the following conditions, except: A. reduce likelihood of obstructive CAD with normal MPI and MBF reserve. B. endothelial dysfunction by the presence of regional reduction of MBF reserve. C. indicate lack of pharmacologic stress hyperemia by no augmentation of MBF. D. identify more severe disease with abnormal MBF reserve and single territory MPI

reviewquestions.indd 541

541

69. N-13 ammonia is an established PET MPI tracer for detecting functionally significant CAD. Ammonia has all the following attributes, except: A. is cyclotron produced. B. has lower relative uptake in the lateral wall. C. first pass extraction fraction is lower than rubidium-82. D. exercise stress is feasible. 70. Cardiac PET tracers have an established role in patients with suspected or known CAD. Other than ischemia detection, PET tracers are used in the following clinical conditions, except: A. detecting cardiac amyloid infiltration using FDG. B. active myocardial inflammation due to sarcoidosis using Rb-82/N-13 ammonia and FDG. C. confirming myocardial viability using ­Rb-82/N-13 ammonia and FDG. D. suspected prosthetic valve endocarditis using FDG. 71. SPECT has an established role in both diagnosis and prognosis of patients with suspected or known CAD. PET is a relatively newer imaging modality compared to SPECT. Which of the following statement is NOT true regarding PET? A. A normal PET scan confers excellent ­prognosis B. Better diagnostic accuracy than SPECT in patients with multivessel disease C. Attenuation artefacts are more common affecting diagnostic accuracy D. Offers better diagnostic accuracy than SPECT irrespective of BMI

15-02-2022 14:30:09

542

Section 5  Review Questions

72. An 89-year-old man presents with chest pain to his cardiologist. He is limited in ability to exercise and chest pain has been stable for 1 month. Risk factors are age, hyperlipidemia, and hypertension. The patient undergoes a rest/stress cardiac PET study with rubidium-82 with no symptoms or ECG changes. The TID ratio was 1:23. The study is presented below: This study represents which of the following? A. Single vessel ischemia with TID B. Multivessel ischemia with TID C. Normal study D. Inferior/lateral reversible defect

reviewquestions.indd 542

73. Which of the following patterns is consistent with single-vessel obstructive disease? A. Normal perfusion, global MBFR 2­.6­ B. Inferior reversible perfusion defect, inferior territory MBFR 1.6, global MFR 2.2 C. Inferior reversible perfusion defect, global MBFR 1.6 D. Normal perfusion, global MBFR 1.2 74. Which of the following patterns is consistent with lack of vasodilator response? A. Normal perfusion, global MBFR 2.6 B. Inferior reversible perfusion defect, global MBFR 1.1 C. Inferior reversible perfusion defect, global MBFR 1.6 D. Normal perfusion, global MBFR 0.9

15-02-2022 14:30:09

Section 5  Review Questions

75. A 67-year-old woman with a history of hypertension, obesity, and coronary calcium score 800 underwent regadenoson PET MPI for anginal symptoms which showed anterior, anteroseptal reversible perfusion defect, rest LVEF 50%, stress LVEF 50%, and global MBFR 1.2. What is the next best step in ­management? A. Repeat study with dobutamine B. Referral to catheterization C. Medical optimization D. Referral to CCTA 76. Myocardial blood flow assessment is less ­useful and should be reported with caution in patients with all of the following, except: A. known chronic total occlusion. B. chronic kidney disease (CKD). C. history of coronary artery bypass surgery. D. history of large anterior myocardial ­infarction. 77. PET would be indicated as an initial test for all of the following patients, except: A. a 55-year-old woman with exertional dyspnea with a recent normal coronary CT angiogram. B. a 69-year-old morbidly obese man with multiple CV risk factors presenting with exertional dyspnea. C. a 68-year-old active man with a family history of CAD presenting with worsening chest pain noticed during daily jogging. D. a 48-year-old man with MI and LAD stenting a year ago presenting with atypical chest pain.

543

79. RNA depends on effective labeling of red blood cells. Which statement is correct? A. In-vivo method has the highest labeling efficiency. B. In-vitro is the current standard labeling method. C. Stannous chloride facilitates binding of the radiopharmaceutical to RBCs. D. Reduced RBC labeling may be noted with warfarin. 80. Radionuclide angiography: A. has less than 15% inter- and intraobserver variability. B. permits quantitative assessment of right ventricular function. C. changes of more than 3% (EF units) are considered abnormal on serial imaging. D. has been supplanted by echocardiography for guiding chemotherapy. 81. Which of the following statements is true regarding determination of global LV dyssychrony with gated SPECT? A. Only histogram bandwidth may be used to diagnose dyssychrony. B. Methodology and normal ranges are the same across all software platforms. C. It is advisable to perform 16-frame (bin)gated SPECT for dyssychrony assessment. D. Gated SPECT is the current standard to guide resynchronization therapy.

78. Which of the following statements about gated SPECT imaging is correct? A. This technique relies in the derivation of a time-activity curve. B. Flashing noted on gated SPECT is due to patient motion. C. 16-bin (or frame) ECG gating is recommended for acquisition. D. Once data acquisition is complete, the gated SPECT study cannot be altered.

reviewquestions.indd 543

15-02-2022 14:30:09

544

Section 5  Review Questions

82. A 56-year-old male smoker with a pastmedical history of hypertension and diabetes mellitus type 2 presents to the emergency department with a 2-day history of intermittent nonexertional chest pain. Vital signs and physical examination at presentation are within normal limits and electrocardiogram demonstrates left ventricular hypertrophy with repolarization changes. Basic labs, including troponin levels repeated 4 hours apart, were within normal limits. Given his multiple risk factors for coronary artery ­disease, an exercise SPECT-CT is obtained which was negative for inducible ischemia. However, on review of the attenuation c­ orrection CT, the following was noted (arrow):

Which of the following incidental noncardiac finding does this likely represent? A. Liver cyst B. Cholelithiasis C. Choledocholithiasis D. Calcified lymph node

reviewquestions.indd 544

83. During the evaluation of the attenuation ­correction images of a myocardial perfusion scan obtained for a 78-year-old smoker with a history of colon cancer, now in remission, a 7-mm solitary nodule is noted in the ­lingula of the left lung. Based on the Fleischner ­S ociety Guidelines, which of the following should be recommended for this patient? A. No follow up needed B. Obtain CT scan within next 2 weeks C. Repeat CT scan in 2 to 4 months D. Dedicated CT at 6 to 12 months followed by repeat at 18 to 24 months 84. A 48-year-old woman with a history of ­hypertension is evaluated for sticking chest discomfort with activity and undergoes an exercise SPECT CT. The following image is obtained from her attenuation correction CT:

Which of the following abnormalities can be noted in the obtained image? A. Abnormal density in the right breast B. Large pericardial effusion C. Moderate size bilateral pleural effusions D. Calcified descending aorta

15-02-2022 14:30:09

Section 5  Review Questions

85. During the review of the planar images of a SPECT myocardial perfusion scan using 9mTc sestamibi, which was obtained for the evaluation of atypical chest pain, uptake of 99mTc is noted in the large intestine. Which of the following is not a site of normal 99mTc uptake? A. Liver B. Kidney C. Large intestine D. Lung 86. Bilateral diffuse lung radiotracer uptake noted on a SPECT MPI might be suggestive of which of the following? A. Chronic obstructive pulmonary disease B. Elevated pulmonary capillary wedge ­pressure C. Bilateral pleural effusions D. Interstitial lung disease 87. A 65-year-old man with a long-standing history of tobacco use presents to the ED with exertional chest pain for the last 2 weeks. Patient reports a history of weight loss and night sweats. A PET myocardial perfusion scan is performed with the fusion image noted below. Which of the following is the likely cause of this finding?

545

88. A large perfusion defect may be defined by which of the following? A. A summed score of 4 B. More than 17% of the left ventricle C. Proximal LAD distribution D. Six segments 89. Which of the following is NOT true regarding the raw, rotating planar images? A. Should be reviewed only when a technical artifact is suspected B. May demonstrate an unsuspected ­neoplasm C. Vertical patient motion may be readily detected D. Assist in defining soft tissue attenuation 90. Which of the following statements might be included in a high-quality SPECT report? A. Clinical correlation is suggested. B. There is a subtle area of questionable ­significance noted in the inferior wall. C. A small area of ischemia cannot be entirely excluded. D. Based on the current study, the risk for a perioperative cardiac event is not increased.

A. Enlarged lymph nodes concerning for malignancy B. Normal retrocardiac tracer activity C. Lung abscess D. Granuloma concerning for tuberculosis

reviewquestions.indd 545

15-02-2022 14:30:09

546

Section 5  Review Questions

91. A 76-year-old male patient with typical chest pain undergoes a rest/stress regadenoson PET study. The patient had no symptoms, ECG changes and the following images were obtained: Which of the following answers is appropriate for this study?

A. Medium reversible anterior defect C/W diagonal CAD B. Medium/large reversible anteroseptal and anterior defect C/W LAD CAD C. Normal PET perfusion D. Normal PET perfusion with stress motion artifact inconsistent with CAD

92. A 77-year-old female patient undergoes pharmacologic rest/stress cardiac PET perfusion imaging with RB-82 tracer due to recurrent chest pain. The patient was unable to complete exercise stress. There were no symptoms or ECG changes during the procedure. The ­following images were obtained:    Interpretation of this study: A. Differential breast attenuation artifact B. Motion artifact C. Misregistration artifact D. Medium reversible anterior, anteroseptal, and apical defect

reviewquestions.indd 546

15-02-2022 14:30:10

Section 5  Review Questions

93. With either SPECT or PET interpretation, the following are considered vital, except: A. quality control evaluation. B. display including SA, VLA, and HLA on same page. C. bull’s eye/polar plot display only. D. ECG-gated display. 94. Attenuation correction (AC) is available for both SPECT and PET. The net impact of AC includes the following, except: A. improved specificity. B. reduced equivocal studies. C. improved sensitivity. D. improved reader confidence. 95. Which of the following statements most accurately describes the current standard process for quantification of relative myocardial perfusion? A. Comparison of rest and stress perfusion images in a single patient to detect areas of reversibility B. Grading severity of stress perfusion abnormalities for each myocardial segment C. Comparison of each pixel of a stress myocardial perfusion polar map to a normal limit database to determine the extent and severity of myocardial perfusion abnormal D. Comparing radiotracer activity for each pixel of a stress myocardial perfusion polar map compared to the highest activity pixel 96. Which of the following statements best compares the quantitative assessment of myocardial perfusion with expert visual interpretation? A. Quantitative assessment has similar accuracy, but lower variability, compared to expert visual interpretation. B. Quantitative assessment has higher accuracy and higher variability compared to expert visual interpretation. C. Quantitative assessment has lower accuracy and lower variability compared to expert visual interpretation. D. Quantitative assessment has lower accuracy and higher variability compared to expert visual interpretation.

reviewquestions.indd 547

547

97. Change analysis can be performed with several different datasets. Which of the following does not accurately describe an application of change analysis? A. Comparison or rest and stress images within a single patient to determine burden of ischemia B. Quantification of stress and rest images compared to a normal limit database C. Comparison of stress images in a single patient before and after revascularization to assess the impact of revascularization D. Comparison of stress images in a patient obtained several years apart to assess for progression of coronary artery disease. 98. Artificial intelligence describes computer alogrithms which perform tasks normally associated with human intelligence, such as interpreting images. These algorithms have been applied in nuclear cardiology in order to: A. improve image segmentation and ­reconstruction. B. identify obstructive coronary artery disease automatically from polar map images. C. combine imaging and clinical information to provide more precise estimates of cardiovascular risk. D. all of the above. 99. The “black box” nature of AI is a major barrier to clinical implementation. Which statement best describes the ability of AI algorithms that combines many variables and features to explain predictions? A. Cannot explain the importance of the features for the individual patient but can show the relative importance of features as learned by the algorithm. B. Can explain which specific features contribute to the decision but not their relative weights C. Can explain which features contribute to the AI decision or prediction along with their relative importance D. Will always have “black box” nature and thus will not be used clinically

15-02-2022 14:30:10

548

Section 5  Review Questions

100. Appropriate use criteria (AUC): A. define the optimal use for all nuclear cardiology indications. B. are used by payers to determine ­reimbursement. C. are based strictly on guidelines. D. allow for clinical judgment and recognize the absence of a rarely appropriate ­threshold. 101. According to the 2013 multimodality AUC, which one of the following would be considered an appropriate indication post revascularization for SPECT myocardial imaging? A. Evaluation of atypical chest pain 1 month after CABG B. Asymptomatic patient 3 years post-CABG C. Asymptomatic patient who underwent incomplete revascularization D. Asymptomatic patient 1 year following PCI 102. The selection of a category of appropriateness is based on which of the following? A. The “most appropriate” category may be selected, leading to a high level of categorization of appropriate use. B. A computer algorithm imputes the data and provides for the appropriate use category. C. Third-party payers provide for criteria of appropriate use of a test. D. The use of a hierarchal approach provides the correct category and pathway for appropriate use selection. 103. When calculating the “value” of a diagnostic test, which of the following factors has most weight? A. Quality B. Appropriateness C. Cost D. Both A and C

reviewquestions.indd 548

104. A 55-year-old woman presents to the nearest emergency room with hypertensive emergency. After treatment, she is found to have an ejection fraction of 35% on echocardiography. Which of the following option would be considered appropriate to determine the cause of her cardiomyopathy? A. SPECT myocardial perfusion imaging B. Invasive coronary angiography C. Coronary CT angiography D. All would be considered appropriate 105. A 64-year-old woman is scheduled to undergo endoscopy for persistent gastroesophageal reflux symptoms. She has a history of a prior myocardial infarction but has had no symptoms since this event 3 years ago. Overall, she is quite sedentary, largely due to osteoarthritis. She is referred to you for pre-procedural evaluation. Which of the following statements is true? A. No testing is necessary and she should undergo the EGD as planned. B. Pharmacologic stress SPECT myocardial perfusion imaging is needed for risk assessment. C. As she has had a prior MI, cardiac catheterization should be performed. 106. A 50-year-old morbidly obese woman with a history of MI s/p PCI to mid-LAD 7 years ago presents with atypical chest pain symptoms. She has a left bundle branch block on EKG. Which of the following would be an appropriate study to rule out an ischemic etiology of her chest pain? A. Vasodilator SPECT myocardial perfusion imaging B. Coronary CT angiography C. Dobutamine Stress echocardiography D. Coronary Angiography

15-02-2022 14:30:10

Section 5  Review Questions

107. Which of the following patients would be most appropriate for an exercise SPECT MPI? A. A 55-year-old male athlete with pre-excitation (WPW) on ECG B. A 45-year-old male smoker with new onset chest pain and ST elevation on ECG C. A 66-year-old asymptomatic man with a coronary calcium score of 80 and a normal ECG D. A 49-year-old runner with exertional chest pain and a normal ECG 108. A 65-year-old woman presents to clinic with several months’ history of exercise-induced chest discomfort associated with dyspnea. Baseline ECG demonstrates nonspecific ST/T changes in the lateral leads. Which one of the following is the most appropriate modality to diagnose coronary artery disease? A. No testing indicated B. Exercise tolerance testing C. Vasodilator Tc-99m SPECT D. Exercise SPECT myocardial perfusion imaging 109. The overall diagnostic accuracy is best with which modality? A. Exercise tolerance testing (ETT) B. Exercise SPECT myocardial perfusion imaging C. Vasodilator PET D. Vasodilator Tc-99m SPECT 110. The following are advantages of PET MPI over SPECT MPI, except: A. increased diagnostic accuracy. B. improved laboratory efficiency. C. reduced radiation exposure. D. best suited for exercise PET. 111. An asymptomatic 55-year-old diabetic patient presents to your clinic for evaluation. Knowing the history of diabetes, which testing procedure has been found to be the most ­valuable? A. Exercise tolerance testing B. ETT with SPECT MPI C. Pharmacologic stress PET MPI D. Calcium scoring

reviewquestions.indd 549

549

112. Coronary CT angiography (CCTA) has been recommended as a first-line test in patients with suspected CAD because: A. it has high sensitivity. B. it has high negative predictive value. C. it has high specificity. D. both A and B. 113. A 45-year-old asymptomatic male cyclist undergoes coronary artery calcium scoring for cardiac risk stratification purposes. He has an Agatston calcium score of 120. The next step in the management of this individual would be: A. no intervention required. B. lifestyle changes alone. C. lifestyle changes, aspirin, and statin. D. stress testing with myocardial perfusion imaging. 114. A 70-year-old diabetic male with chest pain undergoes an exercise stress MPI. He achieves 85% maximal predicted heart rate with no ECG changes. His myocardial perfusion imaging is normal. The next step in his management would be: A. refer for CCTA since stress MPI results in diabetics may not be reliable. B. no intervention required. C. risk factor modification. D. refer for cardiac catheterization since stress MPI results in diabetics may not be ­reliable. 115. According to 2013 multimodality AUC, in which one of the following cases would nuclear stress MPI be considered ­APPROPRIATE post-revascularization? A. Evaluation of chest pain B. Asymptomatic patient ≥5 years post-CABG C. Asymptomatic patient 65 C. Chronic stable angina D. Inability to exercise 121. In which of the following is SPECT MPI considered “rarely appropriate?” A. Asymptomatic, < years post-CABG B. Asymptomatic with incomplete revascularization C. Prior left main stent D. Asymptomatic, ≥2years post-PCI 122. Which of the following trials determined that, among stable post-MI patients who did not undergo coronary revascularization, SPECT MPI not only defines risk of future cardiac events, but may also help guide management strategies? A. ISCHEMIA trial B. INSPIRE trial C. COURAGE trial D. FAME trial 123. A low risk for cardiac events: A. implies an annual event rate of less than 1% per year. B. is often associated with fixed perfusion defect. C. is an indication for repeat testing in 1 year. D. is present with normal perfusion images but 1.5-mm ST depression with adenosine. 124. Markers of patients at high risk for myocardial infarction do not include: A. transient cavity dilation. B. reversible perfusion defect. C. large, severe fixed perfusion defect. D. ischemia in multivessel distribution.

reviewquestions.indd 551

551

125. Appropriate use criteria (AUC) for cardiac imaging: A. define optimal imaging protocol for each clinical scenario. B. often deviate from practice guidelines. C. are based solely on medical literature. D. allow for clinical judgment and recognize the absence of 0% rarely appropriate threshold. 126. Which of the following statements is true related to cavity dilation? A. Transient ischemic dilation (TID) is seen only in multivessel CAD. B. The presence of persistent cavity enlargement is a marker for increased risk of MI. C. Transient ischemic dilation after dipyridamole reflects a temporary increase in left ventricular dimension. D. Transient cavity dilation may be seen in hypertensive patients without significant obstructive CAD. 127. Which of the following is true regarding PETderived coronary flow reserve (CFR)? A. Provides incremental prognostic value only when MPI is abnormal B. Provides incremental prognostic value only when MPI is normal C. Provides incremental prognostic value regardless of MPI findings D. A value ≥ 2 predicts high risk of adverse events 128. Which of the following is true regarding impaired LVEF reserve with vasodilator stress Rb-82 PET MPI? A. Refers to increase in LVEF during myocardial hyperemia B. Associated with increased risk of cardiovascular events C. Refers to decline or failure to augment LVEF on post-stress gated MPI D. Significant only when associated with perfusion abnormalities

15-02-2022 14:30:10

552

Section 5  Review Questions

129. A 62-year-old man with past medical history of type 2 diabetes mellitus, hypertension, dyslipidemia, and obesity presents to his primary care physician (PCP) with complaints of nonexertional chest pain for 6 weeks. The patient is unable to undergo adequate exercise and therefore receives pharmacologic stress. The resulting images are normal. What should the referring healthcare provider tell the diabetic patient regarding his cardiovascular outcomes and further management? A. Low risk of myocardial infarction or death (60% B. Normal exercise nuclear stress testing with EF >60% C. Normal exercise nuclear stress testing with EF